0% found this document useful (0 votes)
59 views304 pages

Vol Ii - 2024

Uploaded by

Aditya Ganvir
Copyright
© © All Rights Reserved
We take content rights seriously. If you suspect this is your content, claim it here.
Available Formats
Download as PDF, TXT or read online on Scribd
0% found this document useful (0 votes)
59 views304 pages

Vol Ii - 2024

Uploaded by

Aditya Ganvir
Copyright
© © All Rights Reserved
We take content rights seriously. If you suspect this is your content, claim it here.
Available Formats
Download as PDF, TXT or read online on Scribd

29th Annual

COMPREHENSIVE REVIEW
for INFECTIOUS DISEASE
BOARD PREPARATION

COURSE CO-DIRECTORS:
VOLUME 2 COURSE DIRECTORS:
John E. Bennett, MD Barbara D. Alexander, MD, MHS
Henry Masur, MD Paul Auwaerter, MD
David N. Gilbert, MD
Roy M. Gulick, MD, MPH
Robin Patel, MD
Andrew Pavia, MD
Richard J. Whitley, MD

www.IDBoardReview.com
2
TABLE OF CONTENTS
Course Overview ........................................................................................................................................... 7

Guide to Online Materials App .................................................................................................................... 9

Accreditation, Evaluation & CME Claim Information-Physicians ..........................................................11

Live Course .............................................................................................................................................. 13

Online Materials.................................................................................................................................................................15

Faculty Listing………………………………………………………………………………………………………………………….19

Faculty Disclosures and Resolutions…………………………..……………………………………………………………….21

3
4
Page left blank intentionally.

5
Page left blank intentionally.

6
COURSE OVERVIEW
ABOUT THE COURSE
This course is designed specifically for physicians planning to certify or recertify in the Infectious
Disease Subspecialty of the American Board of Internal Medicine and is also suitable for physicians
planning to take Infectious Disease sections of the internal medicine board examination. As the latest
information is not on these examinations, the course does not intend to be an update, though
speakers may choose to include some of that information in their talks.

The Infectious Disease Board Review Course is designed not only to expand your knowledge, but also to
help you find areas in which you need to increase your knowledge. Neither the talks nor the questions
cover all the topics that may be on the ABIM exam. The questions during the live course and online
should give you a better idea of the format and depth of detail you can expect from the ABIM exam.
You can compare your scores with other registrants. Now that the MOC exam allows access to “Up-to-
date” during the entire exam, registrants who have access to “Up-to-date” through their institution
could experiment ahead of the exam, accessing IDBR online questions and “Up-to-date"
simultaneously, perhaps using different browsers. After answering an IDBR online question, the
correct answer and rationale are provided, so users will know if their search produced the needed
information. As the exam is time-limited, we anticipate that searching “Up-to-date” will need to be
focused and limited. The certifying exam does not provide “Up-to-date” access.

The lectures, board review sessions, and web-based material will be available for one year
following the course so that registrants can access the material as often as desired. The faculty
are all experts in their content area, and are experienced educators. Most have extensive
experience writing ABIM-style questions, although all adhere to the ABIM pledge not to divulge
specific questions they may have read while taking their own examinations, or while
previously working on ABIM committees.

EDUCATIONAL OBJECTIVES
1. Review the core infectious disease information that would prepare a physician to take the American
Board of Internal Medicine Certification or Recertification Examination in infectious disease.
2. Answer questions written in the format used by the ABIM for the certification and
recertification examinations.
3. Provide a comparison of knowledge and test-taking experience with colleagues likely to be taking
the certification or recertification tests in infectious diseases.
4. Review state of the art clinical practice for the specialty of infectious diseases.

7
8
GUIDE TO COURSE MATERIALS APP
This course offers a mobile app and website for course attendees to access the syllabus and other
course features.

With the App you can:

 Draw on presentation slides, highlight text, and take notes


 Access the full course schedule and create a personal schedule by starring the sessions you plan to
attend
 Message other app users
 Receive alerts and updates for the meeting
 Access supplementalresources

To Access the App via Mobile Device:

1. Search for "eventScribe" in the Apple App Store or Google PlayStore.


2. Install and open the eventScribe app.
3. Search for your event app by entering “IDBR 2024.”
4. To start using the app, please log in with the email and password emailed to you prior to your
arrival.

Please Note:

 You will need internet access to download the app and any slides.
 After you have downloaded the slides to the app, you can access them anywhere on
your tablet or smartphone, even without an internet connection.
 If you are experiencing difficulties with the App please go to the Registration Desk where we will
be happy to assist you.

9
10
ACCREDITATION, CME & MOC CLAIM INFORMATION -
PHYSICIANS

TYPES OF CREDIT
There are two types of CME credit for Live Course participants:

1. Attending the Live Course - 43 credits


2. Completing the Online Materials - 75 credits

Please note that there are separate evaluation and credit claim processes for each type of CME
credit, which is described in further detail in the subsequent pages.

LIVE COURSE

Accreditation
This activity has been planned and implemented in accordance with the Essential Areas and policies
of the Accreditation Council for Continuing Medical Education through the joint providership of The
George Washington University School of Medicine and Health Sciences and the Infectious Disease
Board Review, LLC. The George Washington University School of Medicine and Health Sciences is
accredited by the ACCME to provide continuing medical education for physicians.

CME Credit for Physicians


The George Washington University School of Medicine and Health Sciences designates this live
activity for a maximum of 43 AMA PRA Category 1 Credit(s) ™. Physicians should claim only the credit
commensurate with the extent of their participation in the activity.

Claiming MOC Points


Successful completion of this CME activity enables the participant to earn up to 43 MOC points in the
American Board of Internal Medicine’s (ABIM) Maintenance of Certification (MOC) program.

Participants will earn MOC points equivalent to the amount of CME credits claimed for the activity. It is
the CME activity provider’s responsibility to submit participant completion information to ACCME for the
purpose of granting ABIM MOC credit.

Deadline for Claiming MOC Points


ABIM Board Certified physicians need to claim MOC points for this course by December 31, 2024
in order for the MOC points to count toward any MOC requirements that are due by the end
of 2024.

11
CEHP will continue to submit participant completion data for the course until August 20, 2025. No
ABIM MOC credit will be awarded for this activity after August 20, 2025.

12
OVERVIEW AND INSTRUCTIONS FOR CLAIMING CME CREDIT
AND MOC POINTS

LIVE MATERIALS
Live Lectures

 Participants can receive CME credits and MOC points by listening to the live lectures, participating in the
daily ARS questions, and completing the course evaluation.

 In addition, the archived recordings of these lectures will be available on or before September 8th and will
be organized chronologically by day. You have the option to view them online with the slides with
streaming audio, or you can download the MP3 audio file onto your personal computer or mobile device.

To Claim CME Credit:


1. Complete the five (5) daily session/speaker evaluations (emailed at the end of each
CME Hours: day).
2. Complete the final course evaluation (emailed on the final day of the course).
43 3. Upon completing the final course evaluation, you will be redirected to the link to claim
CME credit where you will be asked to check the Attestation Statement box and enter
the number of CME credits commensurate with the extent of your participation in the
activity.

To Claim MOC Points:

MOC Points: 1. You must pass the Pre- and Post-Test and claim CME credit prior to claiming MOC points.
2. After claiming your CME hours, you will be asked to attest whether you want your
43 participation in the live course to be reported to the ABIM.
3. If you select yes, you will be asked to input your name, ABIM number, and date of
birth.

13
14
ONLINE MATERIALS
Credit
The George Washington University School of Medicine and Health Sciences designates this enduring
material for a maximum of 75 AMA PRA Category 1 Credit(s)™. Physicians should claim only the credit
commensurate with the extent of their participation in the activity.

MOC Points
Successful completion of this CME activity enables the participant to earn up to 75 MOC points in the
American Board of Internal Medicine’s (ABIM) Maintenance of Certification (MOC) program.

Participants will earn MOC points equivalent to the amount of CME credits claimed for the activity. It is
the CME activity provider’s responsibility to submit participant completion information to ACCME for
the purpose of granting ABIM MOC credit.

Claiming Credit and MOC


Participants can earn up to 75 hours of CME credit and MOC points by completing the below online
activities associated with the course.

After the completion of each set of activities, participants will be asked to attest to the number of CME
hours and MOC points that they wish to claim. Please note that you do not have to complete the online
activity in its entirety and you may claim partial CME/MOC credit.

Deadlines for Claiming MOC Points


ABIM Board Certified physicians need to claim MOC points for this course by December 31, 2024
in order for the MOC points to count toward any MOC requirements that are due by the end
of 2024.

CEHP will continue to submit participant completion data for the course until August 21, 2025. No
ABIM MOC credit will be awarded for this activity after August 21, 2025.

15
OVERVIEW OF ONLINE MATERIALS AND INSTRUCTIONS FOR
CLAIMING CREDIT AND MOC

Online‐Only Lectures CME Hours: 9 MOC Points: 9

• These lectures feature topics that were not covered in the live course.

Board Prep Questions CME Hours: 56 MOC Points: 56

• There are four (4) sets of 100 board prep questions.


• There is one (1) set of 100 photo opportunity questions.
• There is one (1) set of 30 questions on HIV.
• You will see the correct answer and rationale after submitting each question.
• You can only go in the forward direction when answering questions.
• You cannot go backwards, but you can retake each set of questions as many times as you like.
Online Primers and
CME Hours: 10 MOC Points: 10
Study Guides

 There are eight (7) study guides and primers that present core material for you to review.
 This PDF reviews information that summarizes important topics in photos, tables and short
summaries.

16
13
GUIDE TO ONLINE MATERIALS ACCESS
Initial Notification
 If you registered on or before June 14, you will receive an email from [email protected]
before or on June 15 with information on accessing the online materials.
 If you registered after June 14, you will receive the access information in 2-3 business days
after your registration date.

Current Access
Instructions for accessing the Online Materials

 Please login to your account at https://cme.smhs.gwu.edu with your username and password
(created when you originally registered for the course)

 Course Page: https://cme.smhs.gwu.edu/idbr24/homepage

Important Links
Please note that you must be logged in to access.

 Main Course Link:


https://cme.smhs.gwu.edu/idbr24/homepage

 To Edit Your User Profile:


https://cme.smhs.gwu.edu/user/login?destination=my/edit/profile

 To View/Download Your CME Certificate After Completing the Course:


https://cme.smhs.gwu.edu/user/login?destination=my/activities

 To Access Your Receipt of Payment:


Click on link to “Already Registered?”
https://cvent.me/2ka4L0

17
18
17
FACULTY LISTING
COURSE DIRECTORS
John E. Bennett, MD* Karen Bloch, MD
Henry Masur, MD* Vanderbilt University Medical Center
Nashville, Tennessee
CO-DIRECTORS Helen Boucher, MD
Tufts University School of Medicine
Barbara D. Alexander, MD, MHS
Boston, Massachusetts
Duke University
Durham, North Carolina
Henry F. Chambers, MD
University of California San Francisco
Paul G. Auwaerter, MD
San Francisco, California
Johns Hopkins University
Baltimore, Maryland
Shireesha Dhanireddy, MD
University of Washington
David N. Gilbert, MD
Seattle, Washington
Oregon Health and Science University
Portland, Oregon
Susan Dorman, MD
Medical University of South Carolina
Roy M. Gulick, MD, MPH
Charleston, South Carolina
Weill Cornell Medical College
New York, New York
Rajesh T. Gandhi, MD
Harvard Medical School
Robin Patel, MD
Boston, Massachusetts
Mayo Clinic
Rochester, Minnesota
Khalil G. Ghanem, MD, PhD
Johns Hopkins University
Andrew T. Pavia, MD
Baltimore, Maryland
University of Utah
Salt Lake City, Utah
Steven M. Holland, MD*
Bethesda, Maryland
Richard J. Whitley, MD
University of Alabama at Birmingham
Michael Klompas, MD
Birmingham, Alabama
Harvard Pilgrim Health Care Institute
Boston, Massachusetts

Camille Kotton, MD
FACULTY Harvard Medical School
Boston, Massachusetts
David M. Aronoff, MD, FIDSA
Indiana University School of Medicine Frank Maldarelli, MD, PhD*
Indianapolis, Indiana Bethesda, Marylan

Taison Bell, MD Edward Mitre, MD*


University of Virginia Bethesda, Maryland
Charlottesville, Virginia

19
15
Sandra Nelson, MD
Massachusetts General Hospital
Boston, Massachusetts

James Platts‐Mills, MD
University of Virginia School of Medicine
Charlottesville, Virginia

Stacey Rubin Rose, MD, FACP, FIDSA


Baylor College of Medicine
Houston, Texas

Michael S. Saag, MD
University of Alabama at Birmingham
Birmingham, Alabama

Jennifer L. Saullo, MD, PharmD


Duke University School of Medicine
Durham, North Carolina

Pranita D. Tamma, MD, MPH


Johns Hopkins University
Baltimore, Maryland

David L. Thomas, MD, MPH


Johns Hopkins University
Baltimore, Maryland

Barbara W. Trautner, MD, PhD


Baylor College of Medicine
Houston, Texas

Allan R. Tunkel, MD, PhD


Brown University
Providence, Rhode Island

Kevin Winthrop, MD, MPH


Oregon Health & Science University
Portland, Oregon

*Individual employees of the National Institutes of Health (NIH) have participated in the planning and
development of the course, although the NIH is not an official sponsor. The views expressed by the participants
do not necessarily represent the opinions of the NIH, DHHS, or the Federal Government.

20
FACULTY DISCLOSURES AND RESOLUTIONS
In accordance with the Accreditation Council for Continuing Medical Education’s Standards for Commercial
Support, The George Washington University Office of CEHP requires that all individuals involved in the
development of activity content disclose their relevant financial relationships and that all conflicts of
interest be identified, resolved, and communicated to learners prior to delivery of the activity. The following
faculty and CME staff members, upon submission of a disclosure form, made no disclosures of commercial
relationships:

FACULTY (SPEAKERS) PLANNERS


 John E. Bennett, MD
• David Aronoff, MD
• Taison Bell, MD  Henry Masur, MD
• Karen C. Bloch, MD, MPH, FIDSA, FACP
• Shireesha Dhanireddy, MD Both planners also resolved
• Susan Dorman, MD financial disclosures
• Rajesh Gandhi, MD
• Khalil G. Ghanem, MD STAFF
• David Gilbert, MD  Kelly Byrne
• Roy M. Gulick, MD, MPH  Lisa Krueger
• Steven M. Holland, MD  Naomi Loughlin
• Frank Maldarelli, MD  Dorothy Martinez
• Edward Mitre, MD
• Sandra Nelson, MD
• James Platts-Mills, MD
• Stacey R. Rose, MD, FACP
• Michael Saag, MD
• Pranita Tamma, MD
• Allan R. Tunkel, MD, PhD

21
The following faculty members (speakers) disclosed commercial relationships:

FACULTY MEMBER FINANCIAL DISCLOSURE(S)


(Speaker)

Paul G. Auwaerter, MD  Consulting: Gilead, Shionogi


 Ownership Interest: Johnson & Johnson
 Research: Pfizer

Barbara D. Alexander, MD,  Consulting: Scynexis, GSK, Astellas, Merck, HealthTrackRx,


MHS Basilea
 Research Grant (Institution): Karius
 Clinical Trials (Site PI/Study PI): Scynexis, F2G
 Royalties (Chapter Author): UpToDate

Helen Boucher, MD  Editor: ID Clinics of North America, Antimicrobial Agents


and Chemotherapy, Sanford Guide

Henry F. Chambers, MD  Equity: Moderna, Merck


 Data Monitoring Committee: Merck

Michael Klompas, MD  Grant Funding: Centers for Disease Control and Prevention,
Agency for Healthcare Research and Quality, Massachusetts
Department of Public Health
 Royalties: UpToDate

Camille Kotton, MD  Consulting: Evrys, Kamada Biotest, Merck, QIAGEN,


Shire/Takeda
 Adjudication Committee: Roche Diagnostics, ResTORBio,
Evrys
 Data Monitoring Committee: Merck
 Research Funding: Kamada Biotest, QIAGEN, Roche
Diagnostics
 Speaker: Merck

 Grants: MicuRx Pharmaceuticals, BioFire


 Consultant: PhAST, Day Zero Diagnostics, Abbott
Laboratories, Sysmex, DEEPULL DIAGNOSTICS, S.L., Netflix,
Robin Patel, MD Oxford Nanopore Technologies, HealthTrackRx, CARB-X
 Patent: Bordetella pertussis/parapertussis PCR issued;
Device/method for sonication with royalties paid by
Samsung to Mayo Clinic; Anti-biofilm substance issued
 Honoraria: Up-to-Date

22
Andrew T. Pavia, MD  Commercial Interests: Antimicrobial Therapy Inc, WebMD,
Sanofi

David L. Thomas, MD, MPH  Data and Safety Monitoring Board: Merck
 Advisory Board: Merck, Excision Bio

Barbara W. Trautner, MD  Research Funding: Genentech and Peptilogics, STRIVE


(Shionogi arm)
 Ownership interest: Abbott Laboratories, Bristol-Myers
Squibb, Abbvie, Pfizer (past)
 Past Advisory Board: Phiogen

Richard J. Whitley, MD  Steering Committee: NIAID Covid-19 Recovery Study, NIAID


Recover VITAL Study
 Past Chairperson: NIAID Covid-19 Vaccine DSMB, Merck
Letermovir DMC and GSK IDMC (Zoster)
 Scientific Advisory Board: Treovir, LLC, Altesa Biosciences
 Member of the Board of Directors: Evrys Bio, Virios
Therapeutics

Kevin L. Winthrop, MD  Research: Insmed


 Consulting: Insmed, Spero, Paratek, AN2

23
24
Agenda Day 1: Saturday, August 17, 2024

AM Moderators: Henry Masur and John Bennett, MD


# Start End Presentation Faculty
8:00 AM 8:30 AM John Bennett, MD
1 - Introduction
EDT EDT and Henry Masur, MD
QP1 8:30 AM - 9:00 AM Daily Question Preview: Day 1 Henry Masur, MD
Core Concepts: Microbiology: What You
2 9:00 AM - 10:00 AM Robin Patel, MD
Need to Know for the Exam
AM Moderator: Andrew Pavia, MD
Drs. Pavia (Moderator),
FC1 10:00 AM - 10:15 AM Faculty Q&A
Bennett, and Patel
3 10:15 AM - 11:15 AM Clinical Immunology and Host Defense Steve Holland, MD
4 11:15 AM - 12:00 PM Core Concepts: Antifungal Drugs Barbara Alexander, MD
12:00 PM - 12:30 PM Lunch Break
Drs. Pavia (Moderator),
BR1 12:30 PM - 1:15 PM Board Review Day 1 Alexander, Aronoff, Patel,
and Thomas
PM Moderator: Robin Patel, MD
5 1:15 PM - 1:45 PM Core Concepts: Antiviral Drugs Andrew Pavia, MD
Drs. Patel (Moderator),
FC2 1:45 PM - 2:00 PM Faculty Q&A Alexander, Aronoff, and
Pavia
Respiratory Viral Infections Including
6 2:00 PM - 3:00 PM Influenza, Immunocompetent, and Andrew Pavia, MD
Immunocompromised Patients
Nocardia, Actinomycosis, Rhodococcus,
7 3:00 PM - 3:30 PM David Aronoff, MD
and Melioidosis
8 3:30 PM - 4:15 PM Acute Hepatitis David Thomas, MD
9 4:15 PM - 5:00 PM Zoonoses David Aronoff, MD
10 5:00 PM - 5:45 PM Chronic Hepatitis David Thomas, MD
11 5:45 PM - 6:30 PM Helicobacter and Clostridium Difficile David Aronoff, MD

End of the Day Drs. Alexander, Aronoff,


FC3 6:30 PM - 6:45 PM
Faculty Q&A Pavia, and Thomas

25
Agenda Day 2: Sunday, August 18, 2024

AM Moderator: Henry Masur, MD


# Start End Presentation Faculty
8:00 AM 8:30 AM
QP2 - Daily Question Preview Day 2 Henry Masur, MD
EDT EDT
How to Prepare for the Certification and
12 8:30 AM - 8:45 AM Helen Boucher, MD
Recertification, Including the LKA
Core Concepts: Antibacterial Drugs I
13 8:45 AM - 9:45 AM Pranita Tamma, MD
Gram Negative Organisms
Core Concepts: Antibacterial Drugs II
14 9:45 AM - 10:45 AM Helen Boucher MD
Gram Positive Organisms
Drs. Bennett (Moderator),
FC4 10:45 AM - 11:00 AM Faculty Q&A
Boucher, Tamma
CMV, EBV, HHV6 and HHV8 in
15 11:00 AM - 11:45 AM Immunocompetent and Camille Kotton, MD
Immunocompromised Patients
11:45 AM - 12:15 PM Lunch Break
Drs. Alexander
(Moderator),
BR2 12:15 PM - 1:00 PM Board Review Day 2 Boucher, Kotton, Platts-
Mills, Saullo, Tamma,
Trautner, and Whitley
PM Moderator: Barbara Alexander, MD
Infections in the Neutropenic Cancer Patient
16 1:00 PM - 2:00 PM Jennifer Saullo, MD
and Hematopoietic Stem Cell Recipients
17 2:00 PM - 3:00 PM Infections in Solid Organ Transplantation Barbara Alexander, MD
Drs. Alexander (Moderator)
FC5 3:00 PM - 3:15 PM Faculty Q&A
Kotton, and Saullo
18 3:15 PM - 3:45 PM GI Infections Part 1 James Platts-Mills, MD
19 3:45 PM - 4:30 PM Skin and Soft Tissue Infections Helen Boucher, MD
20 4:30 PM - 5:00 PM GI Infections Part 2 James Platts-Mills, MD
Infections of Upper and Lower Urinary Tract
21 5:00 PM - 5:45 PM Barbara Trautner, MD
Infections
HSV and VZV in Immuno-competent and
22 5:45 PM - 6:15 PM Richard Whitley, MD
Immunocompromised Hosts
Drs. Alexander
End of the Day (Moderator), Boucher,
FC6 6:15 PM - 6:30 PM
Faculty Q&A Platts-Mills, Trautner, and
Whitley

26
Agenda Day 3: Monday, August 19, 2024

AM Moderator: Paul Auwaerter, MD


# Start End Presentation Faculty
8:00 AM 8:30 AM
QP3 - Daily Question Preview Day 3 Paul Auwaerter, MD
EDT EDT
Sexually Transmitted Infections: Genital
23 8:30 AM - 9:00 AM Khalil Ghanem, MD
Ulcers Diseases (GUD)
Fungal Diseases in Normal and Abnormal
24 9:00 AM - 9:45 AM John Bennett, MD
Hosts
Drs. Auwaerter (Moderator),
FC7 9:45 AM - 10:00 AM Faculty Q&A
Bennett, and Ghanem
Sexually Transmitted Infections: Other
25 10:00 AM - 11:00 AM Khalil Ghanem, MD
Diseases and Syndromes
Nontuberculous Mycobacteria in Normal
26 11:00 AM - 11:45 AM Kevin Winthrop, MD
and Abnormal Hosts
11:45 AM - 12:15 PM Lunch Break
Drs. Auwaerter (Moderator),
Bell, Bennett, Dhanireddy,
BR3 12:15 PM - 1:00 PM Board Review Day 3
Dorman, Ghanem, Klompas,
and Winthrop
PM Moderator: Paul Auwaerter MD
Ticks, Mites, Lice, and the Diseases They
27 1:00 PM - 1:45 PM Paul Auwaerter, MD
Transmit
Immunizations: Domestic, Travel, and
28 1:45 PM - 2:30 PM Shireesha Dhanireddy, MD
Occupational
Tuberculosis in Immunocompetent and
29 2:30 PM - 3:15 PM Susan Dorman, MD
Immunosuppressed Hosts
Drs. Auwaerter (Moderator),
FC8 3:15 PM - 3:30 PM Faculty Q&A
Dhanireddy, and Dorman

30 3:30 PM - 4:00 PM Lyme Disease Paul Auwaerter, MD


31 4:00 PM - 5:00 PM Hospital Epidemiology Michael Klompas, MD
Syndromes in the ICU that ID Physicians
32 5:00 PM - 5:45 PM Taison Bell, MD
Should Know
33 5:45 PM - 6:15 PM Pneumonia Paul Auwaerter, MD

End of the Day Drs. Auwaerter, Bell, and


FC9 6:15 PM - 6:30 PM
Faculty Q&A Klompas

27
Agenda Day 4: Tuesday, August 20, 2024

AM Moderator: Roy Gulick, MD


# Start End Presentation Faculty
8:00 AM 8:30 AM
QP4 - Daily Question Preview Day 4 Roy Gulick, MD
EDT EDT
Clinical Manifestations of Human Retroviral
34 8:30 AM - 9:15 AM Frank Maldarelli, MD
Diseases and Slow Viruses
35 9:15 AM - 10:00 AM HIV-Associated Opportunistic Infections I Henry Masur, MD
36 10:00 AM - 10:15 AM HIV Diagnosis Frank Maldarelli, MD
Drs. Gulick (Moderator),
FC10 10:15 AM - 10:30 AM Faculty Q&A
Maldarelli, and Masur
37 10:30 AM - 11:15 AM Antiretroviral Therapy Roy Gulick, MD
38 11:15 AM - 11:30 AM HIV Drug Resistance Michael Saag, MD
39 11:30 AM - 12:15 PM Antiretroviral Therapy for Special Populations Roy Gulick, MD
12:15 PM - 12:45 PM Lunch Break
Drs. Gulick (Moderator),
BR4 12:45 PM - 1:30 PM Board Review Day 4 Bloch, Gandhi, Maldarelli,
Masur, Saag, and Tunkel
PM Moderator: Roy Gulick, MD
Pharyngitis Syndromes Including Group A
40 1:30 PM - 1:45 PM Karen Bloch, MD
Strep Pharyngitis
41 1:45 PM - 2:30 PM HIV-Associated Opportunistic Infections II Rajesh Gandhi, MD
42 2:30 PM - 3:15 PM Syndromes Masquerading as Infections Karen Bloch, MD

Drs. Gulick (Moderator),


FC11 3:15 PM - 3:30 PM Faculty Q&A
Bloch, and Gandhi

43 3:30 PM - 4:15 PM Non-AIDS-Defining Complications of HIV/AIDS Mike Saag, MD


44 4:15 PM - 5:00 PM Encephalitis including West Nile and Rabies Allan Tunkel, MD
Photo Opportunity I: Photos and Questions to
45 5:00 PM - 5:45 PM Rajesh Gandhi, MD
Test Your Board Preparation
46 5:45 PM - 6:10 PM What Could Be on the Exam About COVID Roy Gulick, MD

End of the Day Drs. Gandhi, Gulick, Saag,


FC12 6:10 PM - 6:25 PM
Faculty Q&A and Tunkel

28
Agenda Day 5: Wednesday, August 21, 2024

AM Moderator: John Bennett, MD


# Start End Presentation Faculty
Endocarditis of Native and Prosthetic Devices,
8:00 AM 9:00 AM
47 - and Infections of Pacers and Ventricular Assist Henry Chambers, MD
EDT EDT
Devices
Photo Opportunities II You Should Know for
48 9:00 AM - 9:45 AM John Bennett, MD
Exam
Drs. Bennett (Moderator)
FC13 9:45 AM - 10:00 AM Faculty Q&A
and Chambers
49 10:00 AM - 10:45 AM Staphylococcus aureus Henry Chambers, MD
50 10:45 AM - 11:30 AM Bone and Joint Infections Sandra Nelson, MD
11:30 AM - 11:45 AM Lunch Break
PM Moderator: Henry Masur, MD
Drs. Masur (Moderator),
BR5 11:45 AM - 12:30 PM Board Review Day 5 Bennett, Chambers, Mitre,
Nelson, and Rose
51 12:30 PM - 1:30 PM Lots of Protozoa Edward Mitre, MD

Drs. Masur (Moderator),


FC14 1:30 PM - 1:45 PM Faculty Q&A
Mitre, Nelson, and Rose

52 1:45 PM - 2:15 PM Worms That Could Be on The Exam Edward Mitre, MD


53 2:15 PM - 2:30 PM Penicillin Allergies Sandra Nelson, MD
Kitchen Sink: Syndromes Not Covered
54 2:30 PM - 3:15 PM Stacey Rose, MD
Elsewhere

29
Online Materials

Online Only Lectures


# Duration Title Faculty
OL – 1 40 Mins Bootcamp: HIV Roy Gulick, MD
OL – 2 50 Mins Bootcamp: Transplant Camille Kotton, MD
OL – 3 45 Mins Brain Abscess, Cavernous Sinus Thrombosis, and Allan Tunkel, MD
Subdural and Epidural Empyema
OL – 4 40 Mins Viral and Bacterial Meningitis Allan Tunkel, MD
OL – 5 33 Mins Other Antibacterial Drugs (Macrolides, TMP, SMX, etc) Pranita Tamma, MD
OL – 6 45 Mins HIV-Associated Opportunistic Infections III Rajesh Gandhi, MD
OL – 7 45 Mins Even More Worms Edward Mitre, MD
OL – 8 25 Mins Statistics Khalil Ghanem, MD
OL – 9 45 min Epididymitis, Orchitis, and Prostatitis Barbara Trautner, MD
Primers and Study Guides
# Title Faculty
P–1 Microbiology Primer Robin Patel, MD
P–2 Antibacterial Resistance Primer Robin Patel, MD
P–3 Antifungal Resistance Primer Barbara Alexander, MD
John Bennett, MD
P–4 Antiviral Resistance Primer Richard Whitley, MD
Andrew Pavia, MD
P–5 HIV Drug Resistance Primer Roy Gulick, MD
P–6 Rickettsia Primer Paul Auwaerter, MD
John Bennett, MD
P–7 Differential Diagnosis of Diseases presenting as Skin David Gilbert, MD
Nodules, Ulcers, or Ulceronodular Skin Lesion
Board Review Question Sets
Title # Questions
Question Set A 100
Question Set B 100
Question Set C 100
Question Set D 100
Question Set E: Short HIV Therapy Questions 30
You Should Know For An Exam
Photo Opportunities 100

30
Page left blank intentionally.

31
Page left blank intentionally.

32
Agenda Day 4: Tuesday, August 20, 2024

AM Moderator: Roy Gulick, MD


# Start End Presentation Faculty
8:00 AM 8:30 AM
QP4 - Daily Question Preview Day 4 Roy Gulick, MD
EDT EDT
Clinical Manifestations of Human Retroviral
34 8:30 AM - 9:15 AM Frank Maldarelli, MD
Diseases and Slow Viruses
35 9:15 AM - 10:00 AM HIV-Associated Opportunistic Infections I Henry Masur, MD
36 10:00 AM - 10:15 AM HIV Diagnosis Frank Maldarelli, MD
Drs. Gulick (Moderator),
FC10 10:15 AM - 10:30 AM Faculty Q&A
Maldarelli, and Masur
37 10:30 AM - 11:15 AM Antiretroviral Therapy Roy Gulick, MD
38 11:15 AM - 11:30 AM HIV Drug Resistance Michael Saag, MD
39 11:30 AM - 12:15 PM Antiretroviral Therapy for Special Populations Roy Gulick, MD
12:15 PM - 12:45 PM Lunch Break
Drs. Gulick (Moderator),
BR4 12:45 PM - 1:30 PM Board Review Day 4 Bloch, Gandhi, Maldarelli,
Masur, Saag, and Tunkel
PM Moderator: Roy Gulick, MD
Pharyngitis Syndromes Including Group A
40 1:30 PM - 1:45 PM Karen Bloch, MD
Strep Pharyngitis
41 1:45 PM - 2:30 PM HIV-Associated Opportunistic Infections II Rajesh Gandhi, MD
42 2:30 PM - 3:15 PM Syndromes Masquerading as Infections Karen Bloch, MD

Drs. Gulick (Moderator),


FC11 3:15 PM - 3:30 PM Faculty Q&A
Bloch, and Gandhi

43 3:30 PM - 4:15 PM Non-AIDS-Defining Complications of HIV/AIDS Mike Saag, MD


44 4:15 PM - 5:00 PM Encephalitis including West Nile and Rabies Allan Tunkel, MD
Photo Opportunity I: Photos and Questions to
45 5:00 PM - 5:45 PM Rajesh Gandhi, MD
Test Your Board Preparation
46 5:45 PM - 6:10 PM What Could Be on the Exam About COVID Roy Gulick, MD

End of the Day Drs. Gandhi, Gulick, Saag,


FC12 6:10 PM - 6:25 PM
Faculty Q&A and Tunkel

33
34
Tuesday, August 20, 2024

QP4

Daily Question Preview 4


Dr. Roy Gulick (Moderator)

©2024 Infectious Disease Board Review, LLC


COPYRIGHT NOTICE: The Copyright Act (Title 17 of U.S. Code) governs the rights attributed to owners of
copyrighted work. Under certain circumstances, educational institutions may provide copies of
copyrighted works to continuing education participants. The copies may not be copied nor used for any
other purpose besides private study, scholarship, or research. Participants should not provide electronic
or print copies of ant materials provided by the university to unauthorized users. If a participant fails to
comply with these restrictions, the participant may be held liable for copyright infringement. No further
35
transmission or electronic distribution is permitted.
36
QP4 – Question Preview: Day 4
Moderator: Roy Gulick, MD

BOARD
PREVIEW
REVIEW
QUESTION
DAY 1 2024
4.1 The patient whose photo is shown is HIV positive (CD4=10
cells/uL, VL=2 mil copies) and has noted these lesions
developing on his trunk, face and extremities over the past 8
months.

He has had low grade


fevers for several months.

Daily Question Preview: Day 4


Moderator: Roy Gulick, MD

7/1/2024 1 of 3

BOARD
PREVIEW
REVIEW
QUESTION
DAY 1 2024 BOARD
PREVIEW
REVIEW
QUESTION
DAY 1 2024
4.1 For your differential diagnosis, what besides Kaposi 4.2 28-year-old man with HIV on TDF/emtricitabine +
sarcoma would be the most likely cause of these lesions and atazanavir/ritonavir for 2 years with HIV RNA <50 cps/ml and
their associated fever? CD4 200s→300s presents for routine follow-up; labs reveal
HIV RNA 68 cps/ml and CD4 352.

A) HHV-6 What do you recommend?


B) CMV A) Obtain genotype
C) Cryptococcus neoformans B) Obtain genotype and phenotype
D) Bartonella C) Repeat HIV RNA at next visit
D) Change regimen to TAF/emtricitabine/bictegravir to
E) Rhodococcus
improve adherence
2 of 3 1 of 2

BOARD
PREVIEW
REVIEW
QUESTION
DAY 1 2024 BOARD
PREVIEW
REVIEW
QUESTION
DAY 1 2024
4.3 You have been monitoring a 36-year-old man with HIV, CD4 4.4 •34 yo MSM receiving CAB IM q 2 months for
~350, VL 636,000 who is now ready to start ART, but wants
the “simplest regimen possible.”
pre-exposure prophylaxis for last 6 months

Which of these regimens do you recommend? •Asymptomatic


A) IM cabotegravir/rilpivirine •HIV Ag/Ab test negative
B) dolutegravir/rilpivirine
C) tenofovir alafenamide/emtricitabine/rilpivirine •Routine screening: HIV RNA 6.1 c/ml
D) dolutegravir/lamivudine
E) tenofovir alafenamide/emtricitabine/bictegravir

1 of 2 1 of 3

©2024 Infectious Disease Board Review, LLC


37
QP4 – Question Preview: Day 4
Moderator: Roy Gulick, MD

BOARD
PREVIEW
REVIEW
QUESTION
DAY 1 2024 BOARD
PREVIEW
REVIEW
QUESTION
DAY 1 2024
4.4 Which of the following ARV resistance 4.5 A 22-year-old man presents with fever, mouth pain, and
skin rash. PE reveals 3 small oral ulcers and diffuse
mutations is most likely in this setting?
macular rash. Labs show WBC 3K, platelets 89K,
A) S147G monospot negative, RPR NR, HIV antibody negative, HIV
RNA 1,876,000 cps/ml.
B) N155H
Which statement is correct?
C) Y143R
A) ART should not be offered
D) E92Q B) ART would decrease his symptoms
E) K65R C) ART has long-term virologic benefits in this setting
D) ART has long-term clinical benefits in this setting
2 of 3 1 of 2

BOARD
PREVIEW
REVIEW
QUESTION
DAY 1 2024 BOARD
PREVIEW
REVIEW
QUESTION
DAY 1 2024
4.6 A 52-year-old woman is admitted for progressive SOB, is 4.7 38yo female with 1 day of sore throat and fever.
intubated, undergoes BAL and is found to have PCP. HIV
Childhood history of anaphylaxis to penicillin.
Ab test is positive, CD4 103, HIV RNA 135,000 copies/ml.
She is day 4 of IV trimethoprim-sulfa and corticosteroids Physical exam:
and still intubated.
T=102.3
When should she start ART? HEENT-tonsillar erythema & petechiae
Neck-Tender bilateral anterior LAN
A) Immediately
B) In the next 2 weeks Labs:
C) After completing 21 days of trimethoprim-sulfa Rapid strep antigen test negative
D) At her first outpatient clinic visit
1 of 2 1 of 3

BOARD
PREVIEW
REVIEW
QUESTION
DAY 1 2024 BOARD
PREVIEW
REVIEW
QUESTION
DAY 1 2024
4.7 What is the most appropriate antimicrobial treatment? 4.8 • 50 yo M with HIV (CD4 40, HIV
RNA 600,000 not on antiretroviral
therapy) presents with fever,
A) Cephalexin headache

B) None • Northeast US, no travel; no animal


or TB exposures
C) Doxycycline
• MRI: ring enhancing lesions; no
D) Clindamycin midline shift

E) Levofloxacin • Serum toxoplasma IgG + CSF: no


WBC, normal protein, toxoplasma
(toxo) PCR pending
2 of 3 1 of 3

©2024 Infectious Disease Board Review, LLC


38
QP4 – Question Preview: Day 4
Moderator: Roy Gulick, MD

BOARD
PREVIEW
REVIEW
QUESTION
DAY 1 2024 BOARD
PREVIEW
REVIEW
QUESTION
DAY 1 2024
4.8 You recommend: 4.9 50-yo woman with HIV (CD4 20, HIV RNA 500,000) presents with
fever and headache. Not on antiretroviral therapy (ART).
A) Brain biopsy Diagnosed with cryptococcal meningitis.
Started on induction therapy (liposomal amphotericin plus 5FC).
B) Meningeal biopsy
When should she be started on ART?
C) Initiate anti-toxo therapy; if no response in 2 A) Start ART at the same time as anti-fungal therapy
weeks, brain biopsy B) About 4 weeks after starting anti-fungal therapy
C) 6 months after starting anti-fungal therapy
D) Vancomycin, cefepime, metronidazole
D) After completing a full course of maintenance anti-fungal
therapy
2 of 3 1 of 2

BOARD
PREVIEW
REVIEW
QUESTION
DAY 1 2024 BOARD
PREVIEW
REVIEW
QUESTION
DAY 1 2024
4.10 A 39-year-old woman is admitted for fever for 3 weeks, 4.10 A rash is present on the trunk and extremities, most
associated with diffuse arthralgias involving the knees, prominently under the breasts and in the area of her underwear
wrists and ankles. waistband.
A severe sore throat was present during the first week of the Labs:
illness but has resolved. Ferritin 3600 ng/ml (nl 40-200)
WBC 32,200 (89% neutrophils)
T=104.2⁰F.
AST and ALT 3x normal
Tender cervical LAN appreciated. ESR and CRP 5x normal
Spleen tip is palpable. ANA and RF negative
Both knees are swollen & painful. Throat and blood cultures are so far negative
On afternoon rounds with the attending, the fever has resolved
with Tylenol and the rash is no longer present.
1 of 4 2 of 4

BOARD
PREVIEW
REVIEW
QUESTION
DAY 1 2024 BOARD
PREVIEW
REVIEW
QUESTION
DAY 1 2024
4.10 The most likely diagnosis is? 4.11 A 24-year-old man was referred by the ED for
evaluation of ulcers of the mouth and penis. He
A) Lymphoma was born in Japan and is in the U.S. to attend
graduate school.
B) Adult Still’s Disease
He has a history of recurrent painful oral ulcers for
C) Acute Rheumatic Fever
3-4 years. Four days ago, he developed a painful
D) Cryoglobulinemia ulcer on the penile shaft. He takes no medicines
E) Kikuchi Disease and denies sexual contact for the past 5 years.

3 of 4 1 of 4

©2024 Infectious Disease Board Review, LLC


39
QP4 – Question Preview: Day 4
Moderator: Roy Gulick, MD

BOARD
PREVIEW
REVIEW
QUESTION
DAY 1 2024 BOARD
PREVIEW
REVIEW
QUESTION
DAY 1 2024
• A 6mm papulo-pustular
4.11• Left eye is inflamed and
there is a hypopyon. lesion is present in the right
4.11 The most likely diagnosis is?
• Numerous painful ulcers on antecubital fossa where they
the oral mucosa. drew blood yesterday in the
ED. A. Syphilis
• There is a 0.5cm ulcer on the
penis. B. Behçet’s disease
C. Herpes simplex virus infection
D. Sarcoidosis
E. Cytomegalovirus infection

2 of 4 3 of 4

BOARD
PREVIEW
REVIEW
QUESTION
DAY 1 2024 BOARD
PREVIEW
REVIEW
QUESTION
DAY 1 2024
4.12 • 55 year old man presents with R hip pain 4.12 Which of the following is the most likely
• H/o COPD requiring steroids frequently underlying cause of his hip pain?
• HIV diagnosed 17 years ago
• On TDF / FTC / EFV for 10 years; originally on IND / AZT / 3TC
A) Osteonecrosis of Femoral Head
• Initial HIV RNA 340,000; CD4 43 cells/ul
B) Fanconi’s syndrome
• Now HIV RNA < 50 c/ml; CD4 385 cells/ul
• Electrolytes NL; Creat 1.3; Phos 3.5 Ca 8.5 C) Vitamin D deficiency
• Mg 2.1, alk phos 130; U/A neg D) Tenofovir bone disease
• R Hip film unremarkable E) Hypogonadism
1 of 3 2 of 3

BOARD
PREVIEW
REVIEW
QUESTION
DAY 1 2024 BOARD
PREVIEW
REVIEW
QUESTION
DAY 1 2024
4.13 • 50-year-old man presents with a several day history 4.13 • Acyclovir is initiated
of fever, headache, and personality change with • MRI with gadolinium reveals enhancement in the left
progression to confusion temporal lobe
• On exam, temperature is 101oF; he is disoriented • Results of initial cerebrospinal fluid (CSF)
and unable to follow commands polymerase chain reaction (PCR) for HSV-1 and
• CT scan of the head without contrast is negative HSV-2 return negative
• CSF analysis reveals a WBC of 80/mm3 (95% • After 3 days, the patient is now oriented to name
lymphs), glucose 70 mg/dL (serum 100 mg/dL), and follows simple commands
protein 120 mg/dL; Gram stain is negative
1 of 4 2 of 4

©2024 Infectious Disease Board Review, LLC


40
QP4 – Question Preview: Day 4
Moderator: Roy Gulick, MD

BOARD
PREVIEW
REVIEW
QUESTION
DAY 1 2024 BOARD
PREVIEW
REVIEW
QUESTION
DAY 1 2024
4.13 What is the next step in the management of this 4.14 What’s the strongest risk factor for progression of
patient? COVID-19 to severe disease?

A) Perform a brain biopsy of the left temporal lobe A) Older age


B) Obtain new CSF for HSV PCR testing B) Diabetes, heart disease, or other comorbidities
C) Send serum for HSV IgG antibodies C) Race/ethnicity
D) Repeat brain MRI D) Vaccine status
E) Discontinue acyclovir E) Being infected with an omicron variant
3 of 4 1 of 2

BOARD
PREVIEW
REVIEW
QUESTION
DAY 1 2024
4.15 What’s the treatment of choice for COVID-19 with
hypoxia?

A) Nirmatrelvir-ritonavir
B) Remdesivir
C) Dexamethasone
D) A and B
E) B and C
1 of 2

©2024 Infectious Disease Board Review, LLC


41
42
Tuesday, August 20, 2024

34

Clinical Manifestations of Human Retroviral


Diseases and Slow Viruses

Dr. Frank Maldarelli

©2024 Infectious Disease Board Review, LLC


COPYRIGHT NOTICE: The Copyright Act (Title 17 of U.S. Code) governs the rights attributed to owners of
copyrighted work. Under certain circumstances, educational institutions may provide copies of
copyrighted works to continuing education participants. The copies may not be copied nor used for any
other purpose besides private study, scholarship, or research. Participants should not provide electronic
or print copies of ant materials provided by the university to unauthorized users. If a participant fails to
comply with these restrictions, the participant may be held liable for copyright infringement. No further
transmission or electronic distribution is permitted.

43
44
34– ClinicalManifestationsofHumanRetroviralDiseasesandSlowViruses
Speaker:FrankMaldarelli,MD

Disclosures of Financial Relationships with Relevant


Clinical Manifestations of Human Retroviral Commercial Interests
Diseases and Slow Viruses
• None

Frank Maldarelli, MD
Bethesda, MD

7/1/2024

Adults and Children Estimated to be Living with HIV 2022 Current US Epidemic
Prevalence: 1.1 Million
Eastern Europe
& Central Asia
North America and Western and Central
1.7 million
Europe 2.3 million [1.5 million – 1.8 million]
[1.9 million – 2.6 million]
Middle East & North Africa
180 000
[150 000 – 210 000]
Asia and the Pacific
Latin America and Western and Central Africa
5 million 6 million
the Caribbean [4.5 million-5.6 million [4.9 million – 7.2 million]
2.2 million Eastern and Southern
[1.5 million – 2.8 million]
Africa
20.6 million
[18.9 million –23 million]

>75% access to antiretroviral therapy 75% Male Male to Male Sexual Contact (MMSC): 68%
HIV incidence: 1.3 million new infections/year 70% Persons of Color Heterosexual Contact 22%
HIV prevalence: 39 million persons living with HIV IDU 10%
Diagnoses: 31,800 in 2022
UNAIDS CDC, 2022

2030 US HIV Public Health Goals: Treatment Cascade 2030 US HIV Public Health Goals: Transmission
95 % Diagnosed 90% Reduction in Incidence by 2030
95% Undergoing Antiretroviral Therapy Trends in HIV Incidence
95% Viral RNA Suppressed Total 37,981 Diagnoses
2018 2022

New
Diagnoses
(N)

>50% Diagnoses are <35


>50% Living with HIV are
>50 years 13-24 25-34 35-44 45-54 55-64 65+
CDC Age (years) CDC

©2024InfectiousDiseaseBoardReview,LLC

45
34– ClinicalManifestationsofHumanRetroviralDiseasesandSlowViruses
Speaker:FrankMaldarelli,MD

PrEP-to-Need Ratio:
Trends StatesandPuertoRico
in the US Epidemic:
EstimatedHIVIncidenceamongPersonsAgedш13Years,byAreaofResidence2019—United
PrEP using/HIV Diagnoses
Geography: Shift South and† Out of Metro Areas
Total=34,800

43% all new


Diagnoses are in Southeast

Note.2019Data.EstimateswerederivedfromaCD4depletionmodelusingHIVsurveillancedata.Estimatesroundedtothenearest 100forestimates>1,000andtothe AIDSvu.org


nearest10forestimatesч1,000toreflectmodeluncertainty.
†TotalestimatefortheUnitedStatesdoesnotincludedataforPuertoRico.

HIV INFECTION COURSE


Acute HIV Syndrome
Percent Reporting
Sign/symptom NEJM
Review
Kenyan sex
workers HIVNET
Fever >80-90 53 55
Fatigue >70-90 26 56
Rash >40-80 9 16
Headache 32-70 44 33
Lymphadenopathy 40-70 7 35
Pharyngitis 50-70 15 43
Myalgia or arthralgia 50-70 24 39
Nausea, vomiting or
diarrhea 30-60 18 12-27
Night sweats 50 nd nd
Aseptic meningitis 24 nd nd
Oral ulcers 10-20 nd 6
Genital ulcers 5-15 3 nd
Thrombocytopenia 45 nd nd
Leukopenia 40 nd nd
Elevated LFTs 2 nd nd
Too ill to work nd
1 44 58

Primary HIV-1 Infection Syndrome HIV Presentation: Question #1


Maculopapular Trunk > Extremities A 23 year old man presents with a history of unprotected
receptive anal sex with known HIV-infected man, and one
week of fever, adenopathy. HIV-1/2 ELISA is reactive, viral
RNA level 500,000 c/ml. He is started immediately on
antiretrovirals. His supplemental assay is negative, and
repeat assays sent 3 weeks, 3 months, and one year after
starting antiretrovirals are also negative.
ELISA remains reactive. HIV-2 assay is negative.
Viral RNA on therapy is <40 c/ml.

Enanthem Aphthous ulcer

©2024InfectiousDiseaseBoardReview,LLC

46
34– ClinicalManifestationsofHumanRetroviralDiseasesandSlowViruses
Speaker:FrankMaldarelli,MD

HIV Presentation: Question #1 (Cont.) Early Antiretroviral Therapy


Which of the following is correct explanation for the absence of
positive results with the supplementary HIV test: • Prompt reduction in HIV-1 RNA
• Potential blunting of humoral immune response
A. The patient was infected with a strain of HIV-1 that was not
• Confirmatory assay may remain negative
detected by the confirmatory assay
B. The patient is HIV-infected but did not develop a positive • HIV-1 DNA PCR has been useful in documenting infection
results with the supplementary assay because of the early
antiretroviral therapy intervention
C. The patient never had HIV infection.
D. The patient had HIV but is now cured of HIV and antiretrovirals
can safely be stopped

HIV Presentation Question #2 Long Acting Early Viral Inhibition (LEVI) Syndrome
A 30 year old individual who is completely adherent with long-acting cabotegravir
as PrEP presents in January to your ED with low grade fever, fatigue, and mild • True breakthrough infection
myalgias. 4th generation HIV testing is non-reactive, rapid Flu A testing is
negative. The ER physician asks whether this patient may have breakthrough HIV • Smoldering presentation- symptoms may be present
infection in the setting of PrEP, and whether further evaluation for HIV infection
should be arranged. • Serologic testing: seroconversion, seroreversion,
A. The patient does not have breakthrough infections, because 4th generation “serowaffling” may persist for months
assays are always reactive in the setting of breakthrough infection.
B. The patient does not have breakthrough infections, because breakthrough • Drug resistance to integrase inhibitor can emerge
infections are always asymptomatic.
C. The patient may have breakthrough HIV infection, and further evaluation for
HIV infection should be arranged.
D. The patient does not have breakthrough infections because breakthrough
infections have never been reported with individuals completely adherent
with long acting cabotegravir.

HIV Clinical Presentation: Question #3


A 49 year old woman from Guinea-Bissau has a reactive HIV-1/2
ELISA and a HIV Geenius positive for HIV-2 and negative for HIV-
1. CD4 cell count is 350 cells/μl.
Which of the following is correct?

A. HIV-2 is less pathogenic than HIV-1 so she only needs therapy


with one antiretroviral drug 1986

B. She should not be treated with protease inhibitors because HIV-2


is naturally resistant to PIs. <250 Geographic
C. She should not be treated with NNRTI therapy because HIV-2 is HIV-2 cases Distribution
naturally resistant to NNRTIs.
D. Use of routine HIV-1 viral load assays is useful in patient reported in of HIV-2
management
US Infection

©2024InfectiousDiseaseBoardReview,LLC

47
34– ClinicalManifestationsofHumanRetroviralDiseasesandSlowViruses
Speaker:FrankMaldarelli,MD

HIV-1 and HIV-2


Question #4
Characteristic HIV-2 HIV-1
Epidemiology A 42 year old man from the Haiti presents with fever, moderate respiratory
Geography West /Central Africa Worldwide distress, and nonproductive cough. HIV-1/2 ELISA is reactive and
Local Distribution Urban=rural Urban>rural discriminatory test is positive for HIV-1. A PCR test of the induced sputum is
Age-Specific Prevalence positive for Pneumocystis jiroveci. On evaluation the lymphocyte count is
Stable or Decreasing Increasing 2,000 cells/μl; the CD4 count is 750 cells/μl and the hematology technician
Pathogenesis remarks that some of the lymphocytes are “flower cells”. Which of the
Average age at diagnosis 45-55 20-34 following is most correct in explaining these findings:
Maternal-fetal (without RX) 0-4% 20-35%
Kaposi Sarcoma Less common (10X) More common
A. The patient has HIV and B cell lymphoma
Therapy NRTI, PI, INSTI, Corec NRTI, PI, NNRTI B. The patient has HIV infection and the elevated CD4 count is due to
Diagnosis NOT NNRTI, Fusion,(Capsid) INSTI, Corec, Fusion steroids used in the treatment of the Pneumocystis pneumonia
Screening HIV1/2 ELISA HIV1/2 ELISA C. The patient has HTLV-1 infection only the HIV test is a false positive
Confirmatory Supplemental Supplemental D. The patient has both HIV infection and HTLV-1 infection
(e.g., Geenius) Qual. HIV RNA)
Monitoring HIV-2 RNA Assay HIV-1 RNA assay

HTLV DISTRIBUTION
Question #5
A 25 year old pregnant woman immigrant from southern Japan was
referred to you for evaluation of a positive HTLV-I western blot.
Which of the following statements is true: US: HTLV-I c. 5/100,000

A. The risk of HTLV-I transmission can be entirely eliminated by


caesarean section.
B. The risk of HTLV-I transmission will be entirely eliminated by not
breastfeeding.
C. Breastfeeding will provide sufficient immunity to prevent infection
with HTLV-I. HTLV-I 40% in Remote
D. The risk of HTLV-I transmission will be significantly decreased but Indigenous
not entirely eliminated by avoiding breastfeeding.
E. There is no risk of HTLV-I disease. In this ethnic group, the HTLV-I 5-10 million cases worldwide
test was likely a false positive.
Sporadic
High endemicity HTLV-II Endemic Amerindian and Central African tribes

Question #6
HTLV-I Transmission, Pathogenesis, Diagnostics
37 year old Jamaican female
• Treansmission with diffuse pruritic rash (right),
ƒ Breastfeeding bone pain with lytic bone
o Prolonged duration: 20-30% seroconvert if breastfed >12 mos lesions.
o High maternal HTLV proviral load in breastmilk:
28.7 infections/1000 person months with 1.5% HTLV+ lymphs WBC: 50,000, 90% lymphocytes
ƒ Sexual
ƒ Transfusion
ƒ Risk of seroconversion: 40-60% Which is most likely cause of
• Pathogenesis her presentation?
ƒ Spread to CD4+ T cells A. HTLV-I
o 1-4% of all CD4 cells become infected - multilobed nuclei “flower cells”
o Spread is NOT continuous, but controlled shortly after infection takes place B. HTLV-II
o Infection maintained in CD4 by persistence and clonal expansion C. HIV-1
• Laboratory diagnosis by sequential testing ELISA/Western blot FDA approved
ƒ Can distinguish HTLV-I from HTLV-II
D. HTLV-IV

©2024InfectiousDiseaseBoardReview,LLC

48
34– ClinicalManifestationsofHumanRetroviralDiseasesandSlowViruses
Speaker:FrankMaldarelli,MD

HTLV-I Acute T cell Leukemia (ATL)


Question #7
• Disease Onset • Therapy
ƒ Long Latency (>30 years)
38 year old woman from Jamaica presents with weakness,
ƒ Cytotoxic chemotherapy
ƒ Small pediatric series in South America
unsteadiness of several months duration and has recently
ƒ AZT+Ifn
• Epidemiology ƒ Transplant
developed incontinence. Neurologic exam notes
ƒ Approximately 1% of HTLV- I infected adults ƒ Mogamulizumab (Poteligeo, anti-
hyperreflexia ankle clonus, and positive Babinski reflex
ƒ M>F (Japan); M=F (Jamaica) CCR4 monoclonal) WBC = 7500 cells/μl
• Associated syndromes o APPROVED in Japan for ATL
ƒ Infectious o In US FDA approved for CD4 T cell = 1000 cells/μl
o TB, MAC, Leprosy relapsed or refractory Sezary or
o PCP mycosis fungoides CSF cell count: 10 cells/mm3 (lymphocytes )
ƒ Lenalidamide in trials
o Recurrent Strongyloides CSF protein: 75 mg/dl
o Scabies esp. Norwegian scabies
ƒ Noninfectious-hypercalcemia+lytic bone lesions

HTLV-I Tropical Spastic Paraparesis /HTLV-1


Question #7 Continued Associated Myelopathy
The etiologic agent associated with this illness
is also associated with: • Epidemiology
ƒ<1% of HTLV-I develop HAM/TSP
A. Acute T cell leukemia ƒThe second most common neurologic
B. Multiple sclerosis syndrome in Jamaica after stroke
C. Variant Creutzfeldt-Jacob ƒLatency may be short--several years
D. Hemorrhagic cystitis ƒFemale predominance

HTLV-I TSP/HAM Therapy of HTLV-I TSP/HAM


• Corticosteroids
• Presentation • Differential Diagnosis
• May slow progression and reduce disability
ƒ Spastic paraparesis ƒ Cord compression
• Mogamulizumab (Poteligeo, anti-CCR4
oLower>upper ƒ B12 deficiency monoclonal)
oProximal>distal ƒ Syphilis • Teriflunomide in trials (FDA- Approved for MS;
ƒ Bladder disturbance ƒ HIV-1 myelopathy pyrimidine synthesis inhib)
ƒ Hyperreflexia ƒ Multiple sclerosis • Antiretroviral therapy is NOT effective
ƒ Positive Babinski
reflex

©2024InfectiousDiseaseBoardReview,LLC

49
34– ClinicalManifestationsofHumanRetroviralDiseasesandSlowViruses
Speaker:FrankMaldarelli,MD

Question #8
Question #8
Which of the following is most correct:
62 year old man from Jamaica with rheumatoid arthritis has not
responded to several antirheumatic drugs including the A. He at risk for the development of HTLV-I drug resistance with this
methotrexate that he is currently taking. He is now being two drug combination. He should receive an additional reverse
considered for treatment with rituximab. He is hepatitis B positive transcriptase inhibitor like doravirine.
(surface antibody positive, surface antigen negative) and HTLV-1 B. He at risk for the development of HTLV-I drug resistance with this
positive (antibody and PCR). He will continue to receive Tenofovir two drug combination. He should receive an integrase inhibitor
like dolutegravir
+ FTC to prevent HBV reactivation, and you are consulted
regarding the development of HTLV-I drug resistance. C. He at risk for the development of HTLV-I drug resistance with this
two drug combination. He should also receive a protease inhibitor
like darunavir.
D. He is not at risk for the development of HTLV-I drug resistance.

Question #9 Pearls
A 56 year-old HTLV-I infected woman is diagnosed with multiple myeloma. She
has never had complications from HTLV-I infection and is otherwise eligible for HTLV-1 Infection Associated Infections
autologous bone marrow transplant. You are consulted regarding her eligibility • Asymptomatic -95%
for chemotherapy vs. chemotherapy and autologous bone marrow transplant • Acute T cell Leukemia • Strongyloides hyperinfection
• HAM/TSP • Norwegian Scabies
Which of the following is most correct:
• But also
A. She should not undergo autologous BMT because of reduced overall ƒ Bronchiectasis • Pneumocystis
survival from ATL or other secondary malignancy in the post transplant ƒ Uveitis • MAC
ƒ Rheumatologic syndromes
period ƒ Lymphocytic pneumonitis
B. She should not undergo autologous BMT because of the high risk of graft Infective Dermatitis (pediatric)
HTLV-II
ƒ
failure • “Flower” cells
ƒ Lymphocytes with HTLV provirus present
C. She can undergo autologous BMT, but she should be treated with ƒ Frequency in HIGHER in ATL and HAM/TSP Not a cause of disease
antiretroviral therapy from induction, until she recovers her counts ƒ NOT an indication for specific therapy
A distractor
(WBC>500 cells/μl)
D. She can undergo autologous BMT; her 3 year survival is equivalent to
Thanks to Tamara Nawar, Ying
individuals withough HTLV-I infection. Taur, Anna Kaltsas (SKMC, NYC)

Prion Disease Question #1


68 y. o. butcher who is an avid hunter presents with dementia progressing over
4 months, myoclonus, MRI below, periodic sharp waves on EEG.
Acquisition of this illness was most likely due to:
A. Contact with elk brains C. Contact with pork brains
B. Contact with sheep brains D. A spontaneous event

SLOW VIRUSES

T2 Flair

©2024InfectiousDiseaseBoardReview,LLC

50
34– ClinicalManifestationsofHumanRetroviralDiseasesandSlowViruses
Speaker:FrankMaldarelli,MD

Prion Diseases: Prion Disease Pathogenesis


Transmissible Spongiform Encephalopathies A. Initiation
• Spontaneous (N=~6000 worldwide per year)
ƒ Sporadic Creutzfeldt-Jakob disease (sCJD)
The prion protein is a host protein with a normal and abnormal
• Associated with specific exposure conformation
ƒ Ingestion of beef from cows with Bovine Spongiform Encephalopathy
o Denoted “Variant CJD”, “vCJD” (N ~ 220 total cases)
ƒ Blood transfusion from individual with vCJD (4 cases)
ƒ Human brains
o Kuru (N= ~2700 total cases)
• Associated with a medical procedure (N ~ 450 total cases)
ƒ Iatrogenic
ƒ Denoted “iCJD”
• Hereditary (N ~600-900 worldwide per year) NORMAL ABNORMAL
ƒ Familial (fCJD)
ƒ Gerstmann-Straussler-Sheinker (GSS)
Transition to abnormal conformation is rare but essentially irreversible
ƒ Fatal Familial Insomnia (FFI)
ƒ Fatal Sporadic Insomnia (FSI) Naturally occurring mutations favor interconversion

Prion Disease Pathogenesis


B. Propagation Spontaneous Creutzfeldt-Jacob Disease (sCJD)
Protein-Protein Contacts recruit normal proteins into abnormal conformation Epidemiology
• Most common human Transmissible
Spongiform Encephalopathy (TSE)
Direct contact ƒ 95% cases

• Incidence estimated 1 per million


Prion Protein
Mutant conformation ƒ US: 0.1/million in <55 yo, 5.3/million >55 yo
ƒ Mean age of onset is 60 years
Disaggregase chaperone proteins
Prion Protein may scavenge these proteins
Mutant conformation with mutant conformation

Dementia Comparison Prion Disease Question #2


Type Protein Clinical Course Path MRI A 68 year old man with dementia progressing over the last
sCJD Prion Myoclonus <2 y Spongif. Caudate
6 months undergoes evaluation. Which of the following
Degen. Striatum CSF results is most consistent with Creutzfeldt Jakob
Thalamus
Disease:
Alzheimer Apo E4, Tau Memory >4 y Neurofib. Hippocampus
Language tangles White matter A. 14-3-3 protein: Positive
B. RT-QuIC: Positive
Lewy Body D- Synuclein Parkinsonian >4 y Lewy Bodies Less common
Visual hallucin. C. T-tau protein: 3000 pg/ml (normal 0-1150 pg/mL)
Multi-infarct Atheroma Focal Incremental Vascular Caudate,Pons
Thalamus D. Aȕ42: 1250 pg/mL (normal >1026 pg/mL)
Ovoid Nuc

©2024InfectiousDiseaseBoardReview,LLC

51
34– ClinicalManifestationsofHumanRetroviralDiseasesandSlowViruses
Speaker:FrankMaldarelli,MD

Abnormal Prion Detection


Spontaneous Creutzfeldt-Jacob Disease (sCJD)
RT-QuIC
Typical Clinical Presentation
ƒ Rapid progression
Direct ƒ RT-QuIC elevated abnormal prion protein
contact ƒ 14-3-3 not specific for sCJD
ƒ Classic Clinical Triad
Patient sample
Mutant conformation ƒDementia
Normal
CSF Mutant ƒMyoclonus
Blood Conformation
conformation
Urine “Quaking” ƒEEG: periodic sharp waves
Sonication

Sensitivity 92% Specificity >99% Herran, BMC Neurology 2018

Prion Disease Question #3


Meat and Bone derived
Meal (MDM)
A 35 year old man presents with dementia progressing over the last year.
He was born in rural Indonesia, lived in London from 1985 – 2010, then
moved to Philadelphia.

Which of the following diseases is most likely the cause of his symptoms: WWII era, larg
A. Kuru 1732 Scrapie
Chronic wasting Disease
processing 䇿re
䇿 of sheep car
B. Variant Creutzfeldt-Jacob Disease
Debilitating Neurologic Symptoms
C. Familial Creutzfeldt-Jacob Disease Occurs in a fraction of large herds
D. Rabies

1995 First cases of variant CJD (vCJD) reported in UK 40000.0 -


Many RELATIVELY YOUNG patients
No typical EEG 30000.0 -
Progress SLOWER than sCJD
BSE 20000.0 -
Predominantly in countries consuming UK beef
10000.0
-
0.0
1989 1992 1995 1998 2001 2004 2007 2010

©2024InfectiousDiseaseBoardReview,LLC

52
34– ClinicalManifestationsofHumanRetroviralDiseasesandSlowViruses
Speaker:FrankMaldarelli,MD

Numbers of vCJD Cases Worldwide sCJD vCJD


Sporadic CJD Variant CJD
• United Kingdom: 177
• France: 26
• Spain: 5
• US: 4
ƒ (ALL infections acquired OUTSIDE of US)
• Ireland: 4
T2 axial proton weighted T2 axial proton weighted
• Netherlands, Italy: 3 Putamen and caudate head 䇾Pulvinar Sign䇿
• Portugal, Canada, Italy: 2 each 䇾Double hockey stick sign䇿 present >75% cases
“Abnormal
• Saudi Arabia, Japan, Taiwan: 1 each
Prion “florid plaques”
(https://www.ecdc.europa.eu/en/vcjd/ 2024) accumulation”

Prion Diseases Question #4 Iatrogenic CJD ~450 cases


Definite Causes No Link
A 49 year old man recently emigrated from Japan presents ƒ Pituitary extracts ƒ Vaccines
with rapidly progressing dementia over the course of months. ƒ Human Growth Hormone
ƒ Feces
ƒ Delay may be >30 y
He underwent a meningioma resection with dura mater graft ƒ Saliva
ƒ Dura mater grafts
in Japan 35 years ago. He is an avid deer hunter and ƒ Mostly Lyodura brand ƒ Sputum
consumes venison. ƒ Transplants (RARE)
ƒ Bovine insulin
ƒ Corneal
What is the most likely cause of his dementia:
ƒ Pericardium ƒ Semen, vaginal secretions
ƒ Liver
A. Iatrogenic CJD from the dura mater graft ƒ Instrumentation/Laboratory accident
ƒ NeurosurgeonsImplantable Neurosurgical-
B. CJD from eating deer.
implanted EEG, stereotactic procedures
C. HTLV-I
D. Alzheimer’s disease

Summary
CJD and Recommendations
sCJD iCJD vCJD

•Patient •Family members


Human growth hormone
Source Spontaneous event Ingested beef
Dura mater graft
Linked to Beef
Human growth hormone:
ƒ Detailed history ƒ Detailed history/Detailed Distribution Worldwide US, Europe
originating largely in
UK. US cases all have
ƒ Blood/urine testing for discussion Dura mater graft: Japan
travel history
presence of prions RT-QuIC ƒ No role for RT-QuIC routine Median Age (y) 68 51 28

ƒ Referrals screening for presence of prions Progression SHORTER shorter LONGER

ƒ Resources
in blood or urine
NOT Typically
EEG Typically abnormal few data but abnormal
ƒ Genetic testing for prion variants abnormal
may be useful MRI Basal Few Data, Double Hckey
“Double Hockey Stick” “Pulvinar sign”
ganglia Stick
ƒ Referrals
Abnormal Prion Protein Abnormal Prion Protein
ƒ Resources Pathology “Florid Plaques”
deposits deposits

©2024InfectiousDiseaseBoardReview,LLC

53
34– ClinicalManifestationsofHumanRetroviralDiseasesandSlowViruses
Speaker:FrankMaldarelli,MD

Transmissible Spongiform
Prions Reference Material Encephalopathy:Time and Place
Modeof Geographic RiskWindow
transmission Region
Beef ingestion UK, France, Europe 1980-present
Human growth France 1963-1985
hormone
Dura mater graft Japan 1969-1987

CJD and Blood Supply


Kuru 䇾shivering, trembling䇿
• Fore tribe Papua New Guinea ƒ Transfusion-associated vCJD rarely documented (N=4, UK)
• Ritual mourning w/cannibalism
ƒ NO documented transfusion-associated sCJD
• Older females, children (especially
female) ƒ No FDA approved tests to detect transmission
• Progressive Ataxia w/dementia ƒ Deferral
ƒ Ambulant, leaning (pictured)
ƒ Dura mater graft or human growth hormone
ƒ Sedentary
ƒ Donors with CJD or family history of CJD
ƒ Terminal 䇾laughing death䇿
ƒ “Florid plaques” (inset) on H+E ƒ Residence in Europe after 1980
• No maternal/fetal transmission ƒ Transfusion in Europe after 1980
• New cases would have been infected ƒ Bovine insulin after 1980 unless certain that insulin was not from UK
as children
• No cases <40 y.o. since 1991

Transmissible Spongiform Encephalopathy Transmissible Spongiform Encephalopathy


Infection Control Issues Multiple trials BUT NO FDA Approved Therapy
ƒ Universal precautions
ƒ No confirmed occupational transmissions
ƒ CJD in health care workers occurs, occupational links have been suggested
PRN100 Antibody Under Study
ƒ Incinerate single use instruments Anti-Prion antibody/G4 isotype
ƒ Inactivate other instruments and materials UK /J. Collinge/N=6
Achieved antibody levels in
ƒ 1N NaOH
CSF
ƒ autoclave 121o C, 15 psi 30 min No disease reversal
ƒ Formic acid for tissue sections ?stabilization of rating scales
ƒ Alternatives include hypochlorite (20,000 ppm chlorine) + autoclave
ƒ REMEMBER: Infectivity is STABLIZED by alcohol, formalin, or glutaraldehyde
ƒ WHO infection control guidelines Future: Disaggregase induction
ƒ http://www.who.int/csr/resources/publications/bse/whocdscsraph2003.pdf?ua=1

Zerr, Lancet Neurology 2022

©2024InfectiousDiseaseBoardReview,LLC

54
34– ClinicalManifestationsofHumanRetroviralDiseasesandSlowViruses
Speaker:FrankMaldarelli,MD

Resources
• RT-QuIC: Case Western
– https://case.edu/medicine/pathology/divisions/national-prion-disease-pathology-
surveillance-center/resources-professionals/contact-and-shipping-information

• Epidemiology
– https://www.cdc.gov/prions/cjd/resources.html

• Patient support
– https://cjdfoundation.org/other-resources

[email protected]

©2024InfectiousDiseaseBoardReview,LLC

55
56
34 – Clinical Manifestations of Human Retroviral Diseases and Slow Viruses
Speaker: Frank Maldarelli, MD

Transmissible Spongiform Encephalopathy


Multiple trials BUT NO FDA Approved Therap

PRN100 Antibody
Under Study
Anti-Prion antibody/G4 isotype
UK /J. Collinge/N=6
Achieved antibody levels in
CSF
No disease reversal
?stabilization of rating scales

Future: Disaggregase
* Zerr, Lancet Neurology 2022

©2024 Infectious Disease Board Review, LLC


57
58
Tuesday, August 20, 2024

35

HIV‐Associated Opportunistic Infections I

Dr. Henry Masur

©2024 Infectious Disease Board Review, LLC


COPYRIGHT NOTICE: The Copyright Act (Title 17 of U.S. Code) governs the rights attributed to owners of
copyrighted work. Under certain circumstances, educational institutions may provide copies of
copyrighted works to continuing education participants. The copies may not be copied nor used for any
other purpose besides private study, scholarship, or research. Participants should not provide electronic
or print copies of ant materials provided by the university to unauthorized users. If a participant fails to
comply with these restrictions, the participant may be held liable for copyright infringement. No further
transmission or electronic distribution is permitted.

59
60
35– HIVAssociatedOpportunisticInfectionsI
Speaker:HenryMasur,MD

Disclosures of Financial Relationships with Relevant


Commercial Interests
HIV-Associated Opportunistic Infections I
• None

Henry Masur, MD, FIDSA, MACP


Bethesda, Maryland

7/1/2024

Question #1 Question #2
For which of the following infections would life long suppressive therapy The patient whose photo is shown is HIV positive (CD4=10 cells/uL, VL=2
be indicated for a patient with a CD4 count <50 cells and a high viral load, mil copies) and has noted these lesions developing on his trunk, face and
regardless of subsequent success of ART regimen in terms of CD4 count extremities over the past 8 months.
and viral load?
He has had low grade fevers for several months.
1. Disseminated histoplasmosis
2. Cryptococcal meningitis For your differential diagnosis, what besides Kaposi sarcoma would be
3. Coccidiodes meningitis the most likely cause of these lesions and their associated fever?
4. Miliary tuberculosis
5. Disseminated Mycobacterium avium complex

Question #2 Question #2

The most likely cause of these skin lesions, if they are not Kaposi sarcoma, is:

A. HHV-6
B. CMV
C. Cryptococcus neoformans
D. Bartonella
E. Rhodococcus

©2024InfectiousDiseaseBoardReview,LLC 61
35– HIVAssociatedOpportunisticInfectionsI
Speaker:HenryMasur,MD

Why Does Anyone in US Develop an HIV Associated


Opportunistic Infection in Current Era?

Clinical Indicators of Immunosuppression Cardinal AIDS-Defining Illnesses


• Pneumocystis pneumonia

• Cryptococcus

• Toxoplasma encephalitis

• CMV Retinitis

• Disseminated Mycobacterium avium complex/Tuberculosis

• Chronic cryptosporidiosis/microsporidiosis

• Kaposi Sarcoma

Susceptibility to Opportunistic Infections


Patients with HIV

• CD4 Count
— Current count is most important
— Prior nadir count is much less important At What CD4 Counts Do
Opportunistic Infections Occur?
• Viral Load
— Independent risk factor for OIs

©2024InfectiousDiseaseBoardReview,LLC 62
35– HIVAssociatedOpportunisticInfectionsI
Speaker:HenryMasur,MD

Scatterplot of CD4 Number vs CD4 Percent CD4+ Lymphocyte Counts Are Excellent Predictor of the
Within 6 Months of HIV-Associated PCP Occurrence of Opportunistic Infections for HIV/AIDS
50

CD4+ T-Lymphocyte Count (/PL)


N=12 (5%) N=44 (18%)
600
40
500
30 400
CD4%

300
20
200
10 100

N=167 (70%) N=15 (6%)


0
0 MAC CMV CMV HISTO TOXO Crypt Cocci Cand PCP Crypt Diss HSV Strep Pulm Cervical
Ret Other Esoph Spor TB Pneumo TB CA

0 100 200 300 400 500 600


CD4 Count (cells/PL)

Warning for Utility of CD4 Counts in Non HIV % Non HIV Patients With PCP When CD4>200

• 100
90 80%
80
70
60%
CD4 Count Are Not A Sensitive Indicator of PCP 60
52%
% Patients%

50%
50 48%
40 40%
30
20
10
0
2007 1994 2008 2011 2002 2008 2000 2012 2005
Overgaard Godeau Monnet Fily Roblot Su Mansharamani Martin-Garrido De Castro
n=50 n=34 n=27 n=46 n=103 n=34 n=22 n=30 n=13

Messiaen. Transpl Infect Dis. 2017;19:e12651.

What is the Most Effective Intervention to Prevent


Opportunistic Infections and Neoplasms?

What is the Most Effective Intervention to Prevent Antiretroviral Therapy


Opportunistic Infections and Neoplasms?
CD4 Count

Viral Load

©2024InfectiousDiseaseBoardReview,LLC 63
35– HIVAssociatedOpportunisticInfectionsI
Speaker:HenryMasur,MD

When to Start ART Following Opportunistic Infection When to Start ART Following Opportunistic Infection

• Most OIs
—Within 2 weeks of diagnosis

ART Initiation Following HIV Related Opportunistic Infections


Early Initiation (<2 weeks) Favors Survival
When to Start ART : Exceptions to Two Week “Rule”
• Tuberculosis: 2-8 weeks after initiation RX*
Early Initiation
— CD4<50 or Pregnant-within 2 weeks of diagnosis
— CD4>50-within 8 weeks of diagnosis
Later Initiation

Survival Without • Cryptococcal Meningitis: 4-6 weeks after initiation of RX


Additional OI — Sooner if mild and if CD4<50
— Later if severe

• “Untreatable” OIs, i.e., PML, Cryptosporidiosis


— Start immediately
Zolopa PLoS One 2009;4:e5575 *For TB meningitis: potentially longer

Primary and Secondary OI Prophylaxis Discontinue Prophylaxis/Chronic Maintenance


These Are Guidelines But They Are Based on 1980-1990 ART Board might consider this a 䇾look up䇿
• Primary Prophylaxis
— PCP (CD4 <200, oral candida, prior AIDS Defining) Primary Prophylaxis CD4 Count Due to ART
— Toxo (CD4 <100, old or new positive anti Toxo IgG)
— Cocci (CD4<250, IgG or new positive cocci IgM — PCP or Toxo >200 x 3 months
— MAC (CD4 < 50) ---NIH/CDC/IDSA guideline has eliminated this except patients whose VL cant be suppressed and
have CD4 less than 50 — PCP (>100 and VL<50)
— *
• Secondary Prophylaxis /Chronic Suppression


PCP
Toxo
Secondary Prophylaxis/Chronic Maintenance
— MAC — PCP >200 x 3 months
— CMV
— Cryptococcus — Toxo >200 x 6 months
— Histoplasma
— Coccidio — Crypt >200 x 6 months
*Some experts would give Histo primary prophylaxis with itraconazole in high risk situations if CD4<150/200 and would not
— MAC >100 x 6 months + 12 m Rx
use histo serology in decision (not reliable)
— CMV >100 x 3-6 months*

©2024InfectiousDiseaseBoardReview,LLC 64
35– HIVAssociatedOpportunisticInfectionsI
Speaker:HenryMasur,MD

Discontinue Prophylaxis/Chronic Maintenance Primary Coccidiomycosis Prophylaxis


Board might consider this a 䇾look up䇿 2024 OI Guideline
Many of “Rules” About Primary and Secondary Prophylaxis Are
Primary
Based Prophylaxis
on Studies CD4
from the 1980-2000 Count
Time Due to ART
Period
SerologicTesting
— PCP or Toxo >200 x 3 months • Once or twice yearly testing for seronegative patients
— PCP (>100 and VL<50)
• For Exam: These Recommendations Are Current Guideline Primary Prophylaxis
Secondary
• Are Prophylaxis/Chronic
they still relevant Maintenance
for patient who durably suppressed by
• Do not administer in endemic area if serology negative
— PCP
ART? >200 x 3 months • Within the endemic area, administer if…..
— Toxo >200 x 6 months — New positive IgM or IgG serology and
— Crypt >200 x 6 months — CD4 count is <250 cells (BIII) and
— No Active Disease
— MAC >100 x 6 months + 12 m Rx
— CMV >100 x 3-6 months* • Regimen
— Fluconazole 400mg qd until CD4>250 and fully suppressed viral load

This is All Oversimplified, But for the Exam


• Avoid live vaccines at CD4 counts < 200 or Uncontrolled Viral Replication
¾ MMR, Varicella, Yellow Fever, Oral typhoid, *Intranasal Influenza
¾ Mpox Jynneos live vaccine is safe because it is non replicating
• Administer
• HAV, HBV, Meningococcus ACWY, Pneumococcus, COVID
¾ All higher incidence or more severe in HIV than non HIV
• RZV (Shingrix) age >18 years
• Pneumococcus, when in doubt use PCV 20
• (or PCV 15 plus 23 valent polysaccharide)
• Administer Mpox if possibly exposed or likely to be exposed
• Assess Post vaccine titers for HBV (and HAV if CD4<200)

https://clinicalinfo.hiv.gov/en/guidelines/adult-and-adolescent-opportunistic-infection Slide 28 https://clinicalinfo.hiv.gov/en/guidelines/adult-and-adolescent-opportunistic-infection

Who Should be Vaccinated for HBV HBV Non-Responders


• People without chronic HBV infection and without immunity to • Definition
HBV infection (anti-HBs <10 mIU/mL) — Anti-HBs <10 international units/mL 1 month after vaccination series
• Options: Not testable
— Switch to another HBV vaccine
— The specific regimens are too granular and changing to likely be on exam
— Double dose of recombinant vaccine (if that was not the initial regimen)
• Preferred by some: two dose regimen
— Vaccine conjugated to HepBCpG (Heplisav-B®) IM at 0 and 1 months
— Four dose recombinant regimen

— NIH/IDSA perspective re assessing post vaccine titers


• 1-2 months post vaccine and then some experts would test annually
• Boost responders when annual level <10mIU/ml

©2024InfectiousDiseaseBoardReview,LLC 65
35– HIVAssociatedOpportunisticInfectionsI
Speaker:HenryMasur,MD

HIV Associated Pulmonary Disease


HBV Immunization for Persons with Isolated Anti HBc
• Recommend one standard dose of HepB vaccine followed by
checking anti-HBs level at 1–2 months.
— If the titer is >100 mIU/mL, no further vaccination is needed,
— If the titer is <100 mIU/mL, a complete series of HepB vaccine should be
completed, followed by anti-HBs testing

• If the anti-HBs quantitative titer is not available


— Recommend complete HepB vaccine series

Respiratory Disease in Patients with HIV Respiratory Disease in Patients with HIV
Do Not Focus Only on OIs! Do Not Focus Only on OIs!
• Non-Infectious
• Non-Infectious — Congest Heart Failure (Age, cocaine, pulm hypert)
— Pulmonary emboli (Increased risk)
— Congestive Heart Failure (Age, cocaine, pulm hypertension)
— Drug toxicity (Abacavir, Lactic acidosis, dapsone)
— Neoplastic (Kaposi sarcoma, Lymphoma, Lung
— Pulmonary emboli (Increased risk) CA)

— Drug toxicity (Abacavir, Lactic acidosis, dapsone) • Non-Opportunistic Infections


— Community acquired (Influenza and MRSA)
— Neoplastic (KS, Lymphoma, Lung CA) — Aspiration (Opioid related, nosocomial)
— Septic Emboli (IV catheters, endocarditis)

Etiology of HIV Associated


Approach to Diagnosis and Therapy of Pneumonia in PWH
Pulmonary Disorders
Parameter Example

• Rapidity of Onset > 3 days: PCP, TB,


Common Less Common Rare
<3 days: Bacteria, viral • Pneumococcus • Histo/Cocci • CMV
• Temperature Afebrile: Neoplasm, PE, CHF • Pneumocystis • Toxoplasma • MAC
• Sputum Scant: PCP, Virus, TB
Purulent: Bacteria • Tuberculosis • Lymphoma • HSV

• Physical Exam Normal: PCP • Kaposi sarcoma • Asperg


Consolidation: Bacteria

• Xray Suggestive But Never Diagnostic

©2024InfectiousDiseaseBoardReview,LLC 66
35– HIVAssociatedOpportunisticInfectionsI
Speaker:HenryMasur,MD

Pneumococcal Disease in Persons with HIV Infection


Internal Medicine Question
• CD4<200
— Enhanced Frequency, Severity, Extrapulmonary Complications
• CD4>350
— Frequency enhanced but NOT severity
Are There Strategies for Reducing Bacterial
• Comorbidities Predisposing to Pneumococci Over- Pneumonias in Patients with HIV Infection?
Represented in HIV
— Opioid Use Disorder, Etoh, Tobacco, Lack of Immunization
— COPD, CHF, Obesity, MRSA colonization, Liver Disease

Strategies to Reduce Incidence of Pneumonia


for Patients with HIV
HIV and Covid
• No increased susceptibility
• Patient Focused Strategies
— Antiretroviral Therapy
• Probably increased severity
— Pneumococcal vaccine
— May be primarily linked to other co-morbidities
— Influenza vaccine
— Tobacco cessation
• Drug interactions
• Environmental Strategies — Integrase inhibitors and Cobicistat and Ritonavir contain regimens
likely OK with Paxlovid
— Immunize contacts and community (esp children)
• Pneumococcal and Hemophilus vaccines
— ART and Remdesivir no interactions
• Influenza vaccine

Question #3 HIV Patient with Shortness of Breath

• A 28-year-old male with HIV (CD4 count = 10 cells) presents to


the ER 4 weeks of malaise and mild cough, and now has
bilateral interstitial infiltrates and a right sided pneumothorax.

• The patient lives in Chicago, works in an office and has never


left the Midwest and no unusual exposures.

• The most likely INFECTIOUS cause of this pneumothorax is:

©2024InfectiousDiseaseBoardReview,LLC 67
35– HIVAssociatedOpportunisticInfectionsI
Speaker:HenryMasur,MD

Pneumocystis Jirovecii
Question #3 (Formerly P. carinii)(PCP or PjP)
A 28-year-old male with HIV (CD4 count = 10 cells) presents to the ER 4 weeks
of malaise and mild cough, and now has bilateral interstitial infiltrates and a • Taxonomy
right sided pneumothorax. — Fungus (no longer Protozoan)
The patient lives in Chicago, works in an office and has never left the Midwest
and no unusual exposures.
• Epidemiology
The most likely INFECTIOUS cause of this pneumothorax is: — Environmental source unknown
• Life Cycle
A. Mycobacterium avium complex — Unknown
B. Blastomycosis • Transmission
C. PCP — Respiratory
D. CMV
E. Aspergillosis

PCP is More Subacute in Persons With HIV


Host Susceptibility to PCP
Than Other Immunosuppressed Persons
Sign or Symptom HIV Non-HIV
• CD4 < 200 cells/μL --(90% of cases) (n=48) (n=38)
Symptom
• CD4% <14 Fever 81% 87%
Cough 81% 71%
Shortness of breath 68% 66%
Duration of symptoms, 28 days 5 days
Temp> 38oC 76% 92%
PaO2 69 mm Hg 52 mm Hg
A-a gradient 41 mm Hg 59 mm Hg
% with normal ABG 5-20% Kovacs et al. Ann Intern Med 1984

Uncommon Manifestations of PCP HIV Related PCP

©2024InfectiousDiseaseBoardReview,LLC 68
35– HIVAssociatedOpportunisticInfectionsI
Speaker:HenryMasur,MD

Development of Pneumatocoeles Radiologic Patterns Associated with Documented


Pneumocystis Pneumonia
• Most Frequent
— Diffuse symmetric interstitial infiltrates progressing to
diffuse alveolar process
• Butterfly pattern radiating from hilum

May 23 June 13

Radiologic Patterns Associated with Documented Diagnosis of Pneumocystis Pneumonia


Pneumocystis Pneumonia
• Other Patterns Recognized 1957
— (Other concomitant infectious or neoplastic disease processes?) Specimen Acquisition Organism Detection
— Lobar infiltrates
Open lung biopsy Methenamine silver
— Upper lobe infiltrates
— Pneumothorax Transbronchial biopsy Immunofluorescence
— Solitary nodules Bronchoalveolar lavage Giemsa / Diff Quik
— Cavitating lesions
— Infiltrates with effusions Induced sputum PCR
— Asymmetric or unilateral processes
— Normal chest x-ray

2024

Pneumocystitis
PCR
Diagnosis of Pneumocystis
Bronchoalveolar Lavage or Sputum
MethenamineSilver Giemsa/DiffQuick
• Highly sensitive in BAL
— Not useful in blood/serum/plasma

• High biologic specificity


— Positive = infection or disease
— Cycle number (copy number ) helpful but not definitive

Immunofluorescence Biopsy:HandE

©2024InfectiousDiseaseBoardReview,LLC 69
35– HIVAssociatedOpportunisticInfectionsI
Speaker:HenryMasur,MD

PCR Don’t Use Beta Glucan Test for Diagnosis of PCP!!


500
For Diagnosis of Pneumocystis in Bronchoalveolar Lavage
400
• Highly sensitive in BAL

Beta-glucan
— Not usefulNegative
in blood/serum/plasma
BAL PCR rules out PCP 300

• High biologic specificity


Positive BAL PCR might be PCP 200
— Positive result might be infection or disease
• Colonization vs Disease
— Cycle number (copy number )helpful but not definitive
100
Positive = 80

31 Sax PE. Clin Infect Dis 2011

PCP No PCP DelCorpo. Clini Microb and Infection 2020

Question #4 CMV and Lungs

• A 45-year-old woman with HIV (CD4 = 50 cells/uL, HIV viral load = 500,000 copies/uL)
presents with fever, shortness of breath, room air P02 =80mm Hg) and diffuse bilateral CMV almost never causes pneumonia
infiltrates and is started on TMP-SMX. In PWH
• The bronchoalveolar lavage is positive for pneumocystis by direct fluorescent antibody
test.
• The microbiology lab also reports the BAL positive by PCR for CMV
CMV in pulmonary secretions or blood is
The best course of action in addition to considering antiretroviral therapy would be: a marker of more severe
immunosuppression but not usually the
A. To add ganciclovir to the TMP-SMX regimen Eosinophilic Intranuclear Inclusion and cause of pneumonia…in this population
B. To add prednisone to the TMP-SMX regimen Basophilic Cytoplasmic Inclusions
C. To add ganciclovir plus prednisone to the TMP-SMX regimen
D. To add ganciclovir plus IVIG to the regimen
E. To add nothing, ie continue TMP-SMX alone

Question #5 Two Pharmacologic Issues To Watch For


A patient with oral thrush and newly diagnosed HIV infection (CD4=10, VL= 200,000
copies/uL) was started on the following medications: dolutegravir, emtricitabine, • Methemoglobinemia (>8-10%)
tenofovir, dapsone, fluconazole. — Most common antimicrobial causes: dapsone and tafenoquine, primaquine
(and occasionally chloroquine, quinolones and sulfa)
Ten days later the patient returns with headache, exercise intolerance, shortness of — O2 Saturation low compared to pO2 and does not improve with O2 (stays at 85%)
breath, a normal chest CT • Cyanosis out of proportion to pulse oximetry
• Specifically detected by co-oximetry but NOT routine pulse oximetry
Pulse oximetry shows an O2 saturation of 85% which does not increase with — Rx Methylene blue
supplemental oxygen

• Glucose-6-Phosphate Deficiency
The most likely cause of this patient’s syndrome is: — Genetic
A. Covid-19 — Hemolysis
B. Pneumocystis pneumonia unmasking — Trigger: Dapsone, quinolones, primaquine/tafenoquine
C. Fluconazole interaction with another drug • Sulfa and trimethoprim probably not important
D. Dapsone • Even trigger drugs can be safe to give for life threatening diseases
E. Dolutegravir

©2024InfectiousDiseaseBoardReview,LLC 70
35– HIVAssociatedOpportunisticInfectionsI
Speaker:HenryMasur,MD

Therapy for HIV Related Likelihood of Death in Patients with Moderate-Severe PCP
Pneumocystis Pneumonia Receiving Corticosteroids (n=251)

• Specific Therapy
— First Choice 40%
• Trimethoprim-Sulfamethoxazole
— Alternatives 30%
• Parenteral Pentamidine Probability
• Atovaquone Of Death 20%
Standard Rx
• Clindamycin-Primaquine

10% Adjunctive
• Adjunctive Corticosteroid Therapy Steroids
— Moderate to Severe PCP 0
0 7 14 21 28
• Room air p02 less than 70mmHg or A-a gradient >35mm Hg Days on Therapy
( Bozette, NEJM 5/90)

Reasons to Deteriorate
How to Manage Patients Who Are Failing TMP-SMX
During Treatment for PCP
• Fluid overload
— Iatrogenic, cardiogenic, renal failure (Sulfa or Pentamidine related)
• Deterioration common first 1-2 days (steroids)
• Anemia
• Methemoglobinemia
• Average Time to Clinical Improvement — Dapsone, primaquine
— 4-8 Days
• Pneumothorax
• Unrecognized concurrent infection
• Radiologic Improvement
• Immune Reconstitution Syndrome (IRIS)
— Lags clinical improvement

Reasons to Deteriorate
During Treatment for PCP
• Fluid overload
Patients
— Iatrogenic, cardiogenic, Failing
renal TMP-SMX
failure (Sulfa or Pentamidine
related) Not Testable! Can Pneumocystis Jiroveci Become
• Anemia Resistant to TMP-SMX?
• Whether to Switch
• Methemoglobinemia
• When to Switch
— Dapsone, primaquine
• Pneumothorax• What to Switch To
• Unrecognized concurrent infection
• How to Manage Steroid Dosing
• Immune Reconstitution Syndrome (IRIS)

©2024InfectiousDiseaseBoardReview,LLC 71
35– HIVAssociatedOpportunisticInfectionsI
Speaker:HenryMasur,MD

Toxicity and Other Considerations


Toxicities of TMP-SMX and Pyrimethamine-Sulfadiazine
Regarding Antipneumocystis Therapy
Drug Issues
Drug Toxicities
TMP-SMX pWBC, pPlat, nLFT, nCreat,
nAmylase, rash, fever, pruritus,
Pentamidine - IV Hypotension-rate related
䇾Sepsis䇿 syndrome-distributive shock nCreatinine, nAmylase, pWBC
Hyperkalemia and increased serum creatinine n Early and then pGlucose
(TMP competes with K and creat for excretion) Associated withnCreatinine
Cross reactivity: dapsone (r 50%) May occur days-wks post therapy
Torsade de Pointes
Pyrimethamine- Similar to TMP-SMX
Sulfadiazine Folinic acid necessary (not folate) to prevent Atovaquone Poor absorption if low fat diet
cytopenias
Rash, N + V, diarrhea, LFT

Without ART or Chemoprophylaxis


Second Episodes of HIV Associated PCP Are Amazingly Common Indications for Primary and Secondary PCP Prophylaxis

Start CD4 < 200 cells/uL (14%)


80
% with Second PCP

Oral candidiasis
60 AIDS Defining Illness
Prior PCP
40
Stop CD4 >200 cells/μL x 3 M
20 (Consider Stoppin: CD4 100-200 and VL<50 x 3M)

0 Restart CD4<200 cells/μL


1 2 3 4 5 6 7 8 9 10 11 12 13 14 15 16 17 18

Months
Fischl/ACTG 002, 10/88

Non HIV---What Are Risk Factors and Timeline of Risk Primary or Secondary Prophylaxis for
Pneumocystis Pneumonia
• Long List of Immunosuppressive Diseases and Drugs
— Risk Factor is cell mediated immunity (lymphocytes) not neutrophils • First Choice
— Severe hypoglobulinemia also risk factor — TMP-SMX (dose not testable)
• CD4 Count
— <200 cells indicates susceptibility • Other Options
— >200 cells is not necessarily protective — Aerosol pentamidine OR
• Duration of risk not well established — Atovaquone OR
— e.g. Dose of drug, number of weeks after dose — (Monthly IV pentamidine-poor data in adults) OR
• Prophylaxis is effective — (Dapsone)
— TMP-SMX is optimal but often stopped arbitrarily or after perceived toxicity,
ie cytopenia, renal dysfunction, transaminitis

©2024InfectiousDiseaseBoardReview,LLC 72
35– HIVAssociatedOpportunisticInfectionsI
Speaker:HenryMasur,MD

Thank You!

©2024InfectiousDiseaseBoardReview,LLC 73
74
Tuesday, August 20, 2024

36

HIV Diagnosis

Dr. Frank Maldarelli

©2024 Infectious Disease Board Review, LLC


COPYRIGHT NOTICE: The Copyright Act (Title 17 of U.S. Code) governs the rights attributed to owners of
copyrighted work. Under certain circumstances, educational institutions may provide copies of
copyrighted works to continuing education participants. The copies may not be copied nor used for any
other purpose besides private study, scholarship, or research. Participants should not provide electronic
or print copies of ant materials provided by the university to unauthorized users. If a participant fails to
comply with these restrictions, the participant may be held liable for copyright infringement. No further
transmission or electronic distribution is permitted.

75
76
36– HIVDiagnosis
Speaker:FrankMaldarelli,MD

Disclosures of Financial Relationships with Relevant


Commercial Interests
HIV Diagnosis

• None

Frank Maldarelli, MD
Bethesda, MD

7/1/2024

HIV Diagnosis:
Question #1
New Modalities and New Terminology
A 26 year old otherwise healthy gay white man has his first HIV test as
part of a new health plan. The fourth generation test is antibody
Old Limitations Persist
reactive and antigen non-reactive. A supplemental third generation
HIV-1/2 ELISA is non-reactive, and an HIV RNA test does not detect HIV • HIV Diagnosis
RNA. The most likely explanation for these results is • History
• Physical
A. This person HIV-infected and is an elite controller • Laboratory testing
B. This person is HIV-infected but is in the window period for HIV
infection • Two Step Diagnostic Approach
C. This person is infected with an HIV variant that is not detected by • No Laboratory Test is Perfect
the supplemental test
D. This person is not HIV-infected
• False positive results require resolution

EvolutionoftheHIVDiagnosticApproach EvolutionoftheHIVDiagnosticApproach
Screening+Supplemental/ Screening+Supplemental/
discriminatorytesting discriminatorytesting
4th Gen.Ag/Ab 4th Gen.Ag/Ab
Screening+Confirmation Screening+Confirmation
ELISA/WB ELISA/WB
Urine,SalivaELISA Urine,SalivaELISA
Home Home
Testing Testing
HIVͲ1/2ELISA HIVRNAtesting HIVͲ1/2ELISA HIVRNAtesting

Clinical/ Clinical/
Immune Immune

DiagnosticsandGuidelinesLaggedBEHINDtheEpidemic

U=U U=U
PrEP PrEP
90Ͳ90Ͳ90 90Ͳ90Ͳ90
CDC:Opt outTesting CDC:Opt outTesting
CDC:GuidelinesforDiagnosis CDC:GuidelinesforDiagnosis

©2024InfectiousDiseaseBoardReview,LLC
77
36– HIVDiagnosis
Speaker:FrankMaldarelli,MD

EvolutionoftheHIVDiagnosticApproach Question #2
Screening+Supplemental/
discriminatorytesting 27 year old female commercial sex worker working in Washington DC
4th Gen.Ag/Ab visits your clinic and requests PrEP. She shows you her home HIV test,
Screening+Confirmation which she took yesterday, and which is non-reactive. She has normal
ELISA/WB
Urine,SalivaELISA
laboratory results and a negative pregnancy test. Which of the following
Home is most appropriate next step?
Testing
HIVͲ1/2ELISA HIVRNAtesting

Clinical/
A. She can immediately initiate PrEP with tenofovir-FTC with no
Immune additional testing
B. She requires additional testing with fourth generation Ag/Ab HIV
DiagnosticsandGuidelinesLaggedBEHINDtheEpidemic test to determine whether she is infected with a non-B subtype of
HIV-1 that is not detected by the home HIV test.
ButareAHEADoftheEradication
U=U C. She requires additional testing with fourth generation HIV test to
PrEP determine whether she has early HIV infection not detected by the
90Ͳ90Ͳ90 home HIV test.
CDC:Opt outTesting
D. She should not initiate PrEP because PrEP does not work well in
CDC:GuidelinesforDiagnosis
women

HIVDetection:ThereisalwaysaWindowPeriod Detecting HIV Infection TWO STEPS


8Ͳ30XMORE • ScreeningͲ HighestSensitivity
Infectious AntiͲHIV • 4th genELISAforHIVantibody+p24antigen
Relative Antibody detection
Level HIVRNA • QualitativeHIVRNA
• Supplemental/DiscriminatoryͲ HighestSpecificity
p24antigen • GEENIUS
TimePostͲInfection
• ConfirmsHIVͲ1orHIVͲ2
(Hometestingш3months)

21days3rd GenerationassayAntibodyonlydetection
16daysHIVp244th GenerationassayAntibodyandp24detection

11daysHIVNucleicacidtesting(NAT)detection
CDC2014

DiagnosisofEarlyHIVInfection
• HISTORY,PHYSICAL,LABORATORYTESTING Evaluation for HIV Infection during PrEP
• MostsensitiveModalities
•4th Generation • Every three months
•HIVRNA:APTIMA • Includes detailed history and physical examination
• LessSensitiveModalities • Ag/Ab (4th generation) testing preferred
•Oralorurinetesting • Viral RNA
•Hometesting(3monthwindow) • Qualitative assay – FDA approved
• Quantitative assay
•GEENIUSisLESSsensitiveforEARLYinfectioncomparedwith • >3000 copies/ml plasma cutoff
4th gentesting • DELAYED antibody emergence POSSIBLE in individuals
• FOLLOWUPandREPEATtesting infected during PreP with extended release cabotegravir
• Antiretroviraltherapymaybluntserologicimmuneresponse
frommaturing

©2024InfectiousDiseaseBoardReview,LLC
78
36– HIVDiagnosis
Speaker:FrankMaldarelli,MD

Question #3
HIV Serologic Testing Pregnancy
You are following a couple who have had a planned pregnancy. The
man is HIV positive and 100% adherent with first line therapy with
Tenofovir+3TC+Dolutegravir; The woman has had monthly fourth • False positive results with antibody testing are possible in pregnancy
generation HIV testing, which has been non-reactive throughout the
first two trimesters; on the most recent visit the man has an HIV RNA • May be specific for individuals tests and persist during pregnancy
was <20 c/ml, but the woman has shows HIV antigen negative and • Testing with viral RNA testing can resolve most issues
HIV antibody positive. The most appropriate next step is: • Qualitative tests (e.g., APTIMA) ARE FDA-APPROVED for testing
• Expensive and generally longer turn around
• Quantitative testing are NOT FDA-APPROVED for diagnosis
A. Obtain the HIV viral RNA test to find out how high the viral load is,
• Rapid turnaround but low level results are possible
and begin antiretroviral therapy immediately
• Rapid screening reactive during labor in previously untested
B. Consider laboratory error, repeat the same 4th generation test
• Initiate therapy
C. Perform supplemental testing with third generation discriminatory • Do not wait for supplemental results
testing
D. Reassure the couple that the woman is not infected and the test is
just a false positive

Question #4 HIV-1 Long Term Non-Progressors


A 65 yo American male has had unprotected sex with men for many
years. The HIV-1/2 ELISA is reactive and supplemental testing is
positive for HIV-1. Viral RNA level is <50 copies/ml and CD4 count is • Represents authentic HIV infection
700 cells/μl. He has never been on antiretroviral therapy and has no
history of travel outside the US. Which of the following is most likely: • ELISA REACTIVE
• SUPPLEMENTAL POSITIVE
A. The patient is in the window period of HIV-1 infection. • HIV RNA may not be detectable
B. The patient is chronically infected with HIV-1 and has a viral load too
low to be detected because he is a long term non progressor. • Slow disease progression
D. The patient is not infected with HIV-1 or -2, all tests are false positive. • Associated with specific HLA subtypes
E. The patient is infected with non-B subtype of HIV-1

HIV Diagnostic Algorithm Question#5


HIVͲ1/2AntigenAntibodyAssay
A68yearoldmanundergoingPrEP (cabotegravir)comesforroutinePrEP visit.Hereports
Reactive Nonreactive multiplepartners(maleandfemale)andengagesinreceptiveanalsexwithpartnerswhodo
notusecondoms.Hisprior4th generationtestwas6monthsagoandwasnonreactive.He
admitsthathehasbeengoingouttoclubsmorefrequentlyafterCOVIDrestrictionseased.He
o
HIVͲ1/2AntibodyDifferentiationAssay doesnotusecondoms.Tendaysago,hedevelopedfever101 F,cough.Acovidtestwas
positive.Hefeelsbetterbutnotbacktohisusualstateofhealth.The4th generationtestis
nowreactive.Hisotherlaboratoryresultsinclude
HIVͲ1Positive HIVͲ1Negative HIVͲ1Positive HIVͲ1Negative
HIVUninfected
HIVͲ2Negative HIVͲ2Positive HIVͲ2Positive HIVͲ2Negative CD4:250cells/μl(14%;priorCD4was1000cells/μl;55%)

HIVͲ1NAT Whichofthefollowingismostcorrect?

HIVͲ1NATPositive HIVͲ1NATNegative
A. TellhimtheCovidtestwasafalsepositive,hehasHIV,andshouldstartTDF+FTC+
Rilpivirine
B. TellhimtheHIVtestisafalsepositiveandcontinuePrEP
HIVInfected C. TellhimhemayhaveHIVinfection,sendsupplementaltestingandcontinuePrEP
D. TellhimhemayhaveHIVinfection,sendsupplementaltestingandswitchtoTDF+FTC+
Rilpivirine

©2024InfectiousDiseaseBoardReview,LLC
79
36– HIVDiagnosis
Speaker:FrankMaldarelli,MD

Question #6 HIVTestingandFalsePositives
A 42 year old woman has a reactive 4th generation test for HIV infection. She is
7 months pregnant, and had COVID-19 infection one month ago despite • Numerousrecentexamplesforfalsepositiveresults
vaccination with Moderna COVID vaccine four months prior to testing. She
had a nonreactive 4th generation screen 7 months ago at the beginning of her • Acuteinfection
pregnancy, she denies any HIV exposures. Subsequent qualitative HIV RNA • Africantrypanosomiasis
testing is negative. The most likely explanation for these results is:
• Heterophileantibodies
• Workersinporkprocessingplant
A. False positive 4th generation test for HIV infection due to pregnancy • Rheumatologicdiseases
B. False positive 4th generation test for HIV infection due to COVID • Metastaticcancer
vaccination
• Pregnancy
C. False positive 4th generation test for HIV infection due to COVID infection
• COVIDinfection
D. False negative HIV RNA testing in the setting of recent HIV infection
•…

Pearls for Board Exam


HIV Testing • HIVTestingisComprehensive • Notestisperfect
• NonͲBSubtypesarealldetectable • 4thGenlesssensitive
• Opt-out testing is Recommended by IDSA and CDC • HIVͲ2hasanapproveddiagnosis • Acute
• Patients are informed that an HIV test will be conducted unless they
explicitly decline to be tested. • LongtermNonͲProgressor • PEP/PrEP
• Written consent in this setting is incorporated into intake • ELISAreactive/
• Counseling is available
• EarlyAntiretroviral
SupplementalPositive therapy
• Opt-in: NOT Recommended by IDSA and CDC
• Patients need to initiate the request for HIV infection • FalsePositives
• Requirements for testing:FIVE C’s: • Pregnancy
• Counseling
• Consent • Mindthegap
• Confidentiality • LonggapforHometesting
• Correct test results • Resources:
• Connection to prevention care and treatment
• https://www.cdc.gov/hiv/guidelines/testing.html
• Boardexamisn’tperfect
[email protected] either
• Referenceslidesfollow
Sodon’toverthinkit

A. 4th generation
Ag/AbELISA
REFERENCE: Sample Added REFERENCE:
Sample 4th Gen ELISA Strategies for HIV Detection Supplementary Discriminatory HIV-1/2 assay ELISA
Added

Lateral flow Lateral flow Lateral flow


HIV-1
Protein A conjugated to colloidal gold
Positive: Any 2 bands of the 4 HIV-1 test lines with at least 1 ENV - gp160
Lateralflow Lateralflow Lateralflow (Band 4) or gp41 (Band 6)
Patient derived anti-HIV antibody
HIV-2 gp36 Negative: NO Bands
Patientderivedp24antigen HIV-2 gp140 Indeterminate:1ENV (Band 4 or 6) 1GAG (Band 5) 1POL (Band 3) 1GAG and
PatientderivedantibodytoHIVproteins HIV-1 p31
HIV-1 gp160 1POL
MonoclonalantiͲp24antibodyconjugatedtostreptavidin
HIV-1 p24 (Bands 5 and 3)
HIVͲ1gp41conjugatedtoselenium HIV-1 gp41
HIV-2:
HIVͲ2gp36conjugatedtoselenium
Positive: 2 HIV-2 bands must be present: gp36 and gp140 (Band 1 and 2)
Monoclonalantip24antibodyconjugatedtoselenium Negative: No bands

©2024InfectiousDiseaseBoardReview,LLC
80
Tuesday, August 20, 2024

37

Antiretroviral Therapy

Dr. Roy Gulick

©2024 Infectious Disease Board Review, LLC


COPYRIGHT NOTICE: The Copyright Act (Title 17 of U.S. Code) governs the rights attributed to owners of
copyrighted work. Under certain circumstances, educational institutions may provide copies of
copyrighted works to continuing education participants. The copies may not be copied nor used for any
other purpose besides private study, scholarship, or research. Participants should not provide electronic
or print copies of ant materials provided by the university to unauthorized users. If a participant fails to
comply with these restrictions, the participant may be held liable for copyright infringement. No further
transmission or electronic distribution is permitted.

81
82
37– AntiretroviralTherapy
Speaker:RoyGulick,MD

Lecture Title

• Disclosures of Financial Relationships with Relevant


Commercial Interests
Antiretroviral Therapy (ART)
- None

Roy M. Gulick, MD, MPH


Rochelle Belfer Professor in Medicine
Chief, Division of Infectious Diseases
Weill Cornell Medicine
7/1/2024

ID Boards – Medical Content: 15% HIV ID Boards – Medical Content: 15% HIV
• Lab testing (<2%) • Other complications of HIV (2%)
• Epidemiology (<2%) • Opportunistic Infections (5%)
• Diagnostic evaluation • Heme, endocrine, GI, renal
• Transmission • Prevention
• Baseline evaluation (including HIVAN), cardiac,
• Testing and counseling • When to start ART with an OI pulmonary, HEENT,
• Initial laboratory evaluation • IRIS musculoskeletal, neuro, psych,
• HIV Treatment Regimens (4.5%)
• Prevention • ART drug classes • Bacteria; Mycobacteria; Fungi; derm
• Adverse effects of treatment Parasites; Viruses • Related issues (<2%)
• Pathogenesis (<2%)
• Drug-drug interactions • Malignancies (<2%) • Substance use
• Virology • When to start therapy • Kaposi sarcoma (KS) • Organ transplantation
• Immunopathogenesis
• Selection of optimal initial regimen • Lymphoma • Primary care
• Acute HIV infection
• Laboratory monitoring
• Cervical cancer • Misc non-HIV complications
• Treatment-experienced patients
• Anal cancer • Pregnancy

Antiretroviral Therapy (ART)


• Questions
• When to start?

WHEN TO START?
• What to start?
• When to change?
• What to change to?

• Treatment as Prevention
• HIV Drug Resistance / Case Scenarios
• ART for Special Populations

©2024InfectiousDiseaseBoardReview,LLC
83
37– AntiretroviralTherapy
Speaker:RoyGulick,MD

Question #1 When to Start?: Chronic Infection


A 43-year-old man with HIV has CD4 900-1200 and HIV RNA AIDS/ Asymptomatic
consistently <200 copies over the last 11 years. symptoms
CD4 CD4 CD4 CD4
Do you recommend starting ART? <200 200-350 350-500 >500

A. Yes, all current guidelines recommend starting. US DHHS 2024 recommended


www.clinicalinfo.hiv.gov
B. No, he’s a long-term non-progressor and doesn’t need ART.
C. No, he should wait until his viral load level is confirmed >200
IAS-USA 2023 recommended
copies/ml. Gandhi JAMA 2023;329:63-84
D. No, he should wait until CD4 is confirmed <500 cells/uL.

Antiretroviral Drug Approval: 1987 - 2024


Goal of Antiretroviral Therapy BIC
CAB LEN
IBA FTR
35
• To suppress HIV RNA (viral load level) DOR

ETR DTG TAF


30
as low as possible, for as long as possible ENF RAL
EVG
RPV
ATV MVC
25 DRV
FTC
NFV FPV TPV
20 DLV APV TDF
• To preserve or enhance immune function RTV EFV LPV/r
15 IDV ABC
NVP
3TC
10 SQV
• To delay clinical progression of HIV disease ddC
d4T
5 ddI
AZT
(and prolong healthy life)
0
1987 1990 1993 1996 1999 2002 2005 2008 2011 2014 2017 2020 2023

Life Cycle of HIV Life Cycle of HIV

CD4+ T cell CD4+ T cell

reverse
transcriptase
inhibitors
https://scienceofhiv.org/wp/animations/ nucleosides non-nucleosides https://scienceofhiv.org/wp/animations/

©2024InfectiousDiseaseBoardReview,LLC
84
37– AntiretroviralTherapy
Speaker:RoyGulick,MD

Life Cycle of HIV Life Cycle of HIV


protease protease
inhibitors entry inhibitors inhibitors
fusion
inhibitors

CD4+ T cell CD4+ T cell


CD4 chemokine
attachment receptor
inhibitors inhibitors

reverse reverse
transcriptase transcriptase
inhibitors inhibitors
nucleosides non-nucleosides https://scienceofhiv.org/wp/animations/ nucleosides non-nucleosides https://scienceofhiv.org/wp/animations/

Life Cycle of HIV Life Cycle of HIV


protease protease
entry inhibitors inhibitors entry inhibitors inhibitors
fusion fusion
inhibitors inhibitors

CD4+ T cell CD4+ T cell


CD4 chemokine integrase CD4 chemokine integrase
attachment receptor inhibitors attachment receptor inhibitors
inhibitors inhibitors inhibitors inhibitors

reverse reverse
transcriptase transcriptase
inhibitors inhibitors
nucleosides non-nucleosides https://scienceofhiv.org/wp/animations/ nucleosides non-nucleosides https://scienceofhiv.org/wp/animations/
capsid
inhibitors

Approved ART: 2024*


nucleoside/tide RTIs protease inhibitors (PIs) entry inhibitors (EIs)
(NRTIs) • saquinavir (SQV) • enfuvirtide (T-20, fusion
• ritonavir (RTV)
inhibitor)
• zidovudine (ZDV, AZT)
• maraviroc (MVC, CCR5
• lamivudine (3TC) • indinavir (IDV)
antagonist)

WHAT TO START?
• nelfinavir (NFV) • ibalizumab (IBA, CD4
• abacavir (ABC)
• lopinavir/r (LPV/r) post-attachment inhibitor)
• emtricitabine (FTC)
• atazanavir (ATV) • fostemsavir (FTR, CD4
• tenofovir (TAF, TDF) attachment inhibitor)
• tipranavir (TPV)
• darunavir (DRV)
NNRTIs
capsid inhibitors (CIs)
• nevirapine (NVP) integrase inhibitors (IIs)
• lenacapavir (LEN)
• efavirenz (EFV) • raltegravir (RAL)
• elvitegravir (EVG)
• etravirine (ETR)
• dolutegravir (DTG)
• rilpivirine (RPV)
• bictegravir (BIC)
• doravirine (DOR)
• cabotegravir (CAB)
*ddI, ddC, d4T, DLV, APV, and FPV discontinued from market

©2024InfectiousDiseaseBoardReview,LLC
85
37– AntiretroviralTherapy
Speaker:RoyGulick,MD

First ART Regimen: Individual Factors


Question #2
• convenience (number of
• antiretroviral activity (VL,
You have been monitoring a 36-year-old man with HIV, CD4 ~350,
CD4, clinical responses) pills, dosing interval,
VL 636,000 who is now ready to start ART, but wants the “simplest food/fasting requirements)
regimen possible.” Which of these regimens do you recommend? • durability of responses
• preserving future treatment
A. IM cabotegravir/rilpivirine • baseline drug resistance
B. dolutegravir/rilpivirine
options
• tolerability
C. tenofovir alafenamide/emtricitabine/rilpivirine • stage of HIV disease,
• acute side effects concomitant illnesses and
D. dolutegravir/lamivudine
E. tenofovir alafenamide/emtricitabine/bictegravir • chronic side effects medications (drug-drug
interactions)
• access and cost

Recommended Regimens (for most people) Alternative Regimens (Certain Situations) (1)
(1-2 NRTI + integrase inhibitor) • Integrase inhibitor-based (INSTI + 2 NRTI)
• elvitegravir/cobicistat/tenofovir (TAF or TDF)/emtricitabine
• Integrase inhibitor-based • raltegravir + tenofovir (TAF or TDF) + (lamivudine or emtricitabine)
• bictegravir/tenofovir alafenamide (TAF)/emtricitabine
• dolutegravir/abacavir/lamivudine (if HLA-B*5701 negative)
• Protease inhibitor-based (Boosted PI + 2 NRTI)
• dolutegravir + tenofovir (TAF or TDF) + (emtricitabine or lamivudine) • In general, boosted darunavir preferred over boosted atazanavir
• dolutegravir/lamivudine (except HIV RNA >500,000 cps/ml, HBV • darunavir/(ritonavir or cobicistat) + tenofovir (TDF or TAF) + (lamivudine or
surface antigen +, or no resistance results) emtricitabine)
• darunavir/(ritonavir or cobicistat) + abacavir*/lamivudine
• atazanavir/(ritonavir or cobicistat) + tenofovir (TDF or TAF) + (lamivudine or
U.S. DHHS Guidelines 2/27/24 clinicalinfo.hiv.gov
emtricitabine)
U.S. DHHS Guidelines 2/27/24 www.clinicalinfo.hiv.gov

Alternative Regimens (Certain Situations) (2) Alternative Regimens (Certain Situations) (3)
• NNRTI-based (NNRTI + 2 NRTI) • Options when ABC, TAF, and TDF cannot be used
• doravirine/TDF/lamivudine or doravirine + TAF/emtricitabine • dolutegravir + lamivudine (except HIV RNA >500,000
• efavirenz + tenofovir (TAF or TDF) + (emtricitabine or cps/ml, HBV surface antigen +, or no resistance results)
lamivudine) • darunavir/ritonavir + lamivudine
• efavirenz 600 + TDF + (emtricitabine or lamivudine)
• efavirenz 600 + TAF/emtricitabine
• darunavir/ritonavir + raltegravir BID
• efavirenz 400/TDF/lamivudine (if HIV RNA <100,000 cps/ml and CD4 >200)
• rilpivirine + tenofovir (TAF or TDF)/emtricitabine
(if VL <100,000 cps/ml and CD4 >200) U.S. DHHS Guidelines 2/27/24 www.clinicalinfo.hiv.gov

U.S. DHHS Guidelines 2/27/24 www.clinicalinfo.hiv.gov

©2024InfectiousDiseaseBoardReview,LLC
86
37– AntiretroviralTherapy
Speaker:RoyGulick,MD

Choice of NRTIs Choice of NNRTIs


Drug DHHS GL Dose Toxicities Considerations
Combination DHHS GL Dosing Toxicities Considerations
doravirine alternative qd Ļ CNS toxicity than TDF/FTC/DOR
tenofovir recommended 1 tab qd renal, bone 1-pill, once-daily (DOR) EFV; Ļ lipids (1 pill, once-daily)
(TAF or TDF)/ (with TDF); formulations efavirenz alternative qd (600 CNS toxicity TDF/FTC/EFV
emtricitabine Ļ toxicity with available (EFV) or 400 (50%), rash (10%), (1 pill, once-daily)
(FTC) TAF mg) suicidality (rare)
abacavir/ recommended 1 tab qd HSR (5-8%) ABC/3TC/DTG rilpivirine alternative qd not well absorbed (TAF or TDF)/FTC/RPV
lamivudine (with dolutegravir (do HLA- available; less (RPV) with PPI (1 pill, once-daily with a
(ABC/3TC) only) / alternative B*5701 test) effective with VL meal);
>100K; ??ĹMI NOT for HIV RNA
zidovudine/ not 1 tab bid GI, anemia, toxicity >100K or CD4 <200
lamivudine recommended lipoatrophy nevirapine not qd or bid hepatotoxicity, toxicity
(ZDV/3TC) (NVP) recommended hypersensitivity
Based on DHHS Guidelines 2/27/24 Based on DHHS Guidelines 2/27/24

Choice of PIs Choice of Integrase Inhibitors


Drug DHHS GL Dose Toxicities Considerations Drug DHHS GL Dosing Toxicities Considerations
darunavir alternative; in qd (if no prior skin rash active against PI- bictegravir recommended 1 coform- few, Ĺcreat, TAF/FTC/BIC (1 pill, qd);
/(ritonavir or general, prefer- PI resistance) (rare); resistant viral strains (BIC) with TAF/FTC ulated pill wt gain Ĺ barrier to resistance
cobicistat) red over ATV or bid dolutegravir recommended 50 mg qd few, Ĺcreat, ABC/3TC/DTG (1 pill,
(DRV/r or c) (DTG) with (TAF or (bid with II CNS, wt gain qd);
atazanavir alternative qd Ĺ indirect avoid PPI; TDF)/(FTC or resistance) Ĺ barrier to resistance
/(ritonavir or bilirubin, GI kidney stones 3TC) or ABC/3TC
cobicistat) (uncommon) elvitegravir alternative with 1 coform- mild GI (TAF or TDF)/FTC/
(ATV/r or c) (EVG) (TAF or TDF) ulated pill EVG/cobicistat (1 pill,
lopinavir/ not bid or qd diarrhea, co-formulated /FTC/cobicistat qd); drug interactions
ritonavir recommended Ĺlipids raltegravir alternative with 400 mg few twice-daily dosing;
(LPV/r) (RAL) (TAF or bid; 600 no co-formulations
TDF)/FTC mg X 2 qd
Based on DHHS Guidelines 2/27/24 Based on DHHS Guidelines 2/27/24

Selected Drug Interactions (1) Selected Drug Interactions (2)


• Cytochrome P450 3A4 effects
• Cytochrome P450 3A4 effects
• PIs are inhibitors; ritonavir is the most potent inhibitor ever
• Most NNRTI (EFV, ETR, NVP, RPV – NOT DOR) are
described; cobicistat is a potent inhibitor
inducers • In general, Ĺ levels of other metabolized drugs
• In general, Ļ levels of other metabolized drugs
• Concern with: rifampin – cannot be used/(rifabutin),
• Concern with: rifampin/(rifabutin),
ketoconazole/itraconazole, anticonvulsants,
ketoconazole/itraconazole, anticonvulsants, simvastatin/lovastatin, midazolam/triazolam, ergotamines,
simvastatin/lovastatin, midazolam/triazolam, ergotamines St. John’s Wort
• HIV protease inhibitors
• HIV NNRTI
• maraviroc
• maraviroc
• Some HCV drugs
• HCV drugs

©2024InfectiousDiseaseBoardReview,LLC
87
37– AntiretroviralTherapy
Speaker:RoyGulick,MD

ART: What NOT to use as Initial therapy


• Monotherapy • Protease Inhibitors (PI)
• Nucleosides (NRTI) • unboosted PIs
• 3 or 4 all-NRTI combination • older drugs (fosamprenavir,
regimens indinavir, lopinavir, nelfinavir,
• older drugs ritonavir [except as a
(e.g. zidovudine, didanosine) booster], saquinavir
tipranavir)
• Non-nucleosides (NNRTI)
• Entry inhibitors (EI)
• older drugs
(e.g. nevirapine) • Some 2-drug regimens
• IM CAB/RPV or DTG/RPV
• etravirine
Based on DHHS Guidelines 2/27/24

ART: Side Effects (1)


ART Side Effects (2)
• Life threatening
Ł hepatitis (NNRTIs, PIs) • Acute/early
x nevirapine – women with CD4 >250; men with CD4 >400; • gastrointestinal (zidovudine, TDF, PIs, ?all ART)
Ł hypersensitivity reaction (HSR) (abacavir, nevirapine, etravirine) • anemia, neutropenia (zidovudine)
x abacavir HSR greatly reduced by HLA-B*5701 screening • bone mineral density Ļ (TDF)
x stop nevirapine or etravirine for rash with constitutional symptoms • central nervous system (efavirenz, integrase inhibitors[?])

Ł Stevens-Johnson syndrome (nevirapine, etravirine) • fatigue (zidovudine)


• indirect hyperbilirubinemia (atazanavir, indinavir)
Ł teratogenicity
• injection site reactions (enfuvirtide)
Ł efavirenz = pregnancy category D
• rash (NNRTIs)
Ł dolutegravir during conception/very early pregnancy
Æ neural tube defects – RARE, not significantly Ĺ vs. other ART

ART Side Effects (3)


• Chronic/longer term
• cardiovascular (abacavir??, PIs except atazanavir)
• kidney stones (indinavir > atazanavir)
• metabolic – glucose, lactate, lipids (older PIs) WHEN TO CHANGE?
• morphologic –
• fat loss – lipoatrophy (stavudine, zidovudine)
• fat gain – lipohypertrophy (older PIs)
• proximal renal tubular dysfunction (TDF)
• weight gain (bictegravir, dolutegravir, TAF)

©2024InfectiousDiseaseBoardReview,LLC
88
37– AntiretroviralTherapy
Speaker:RoyGulick,MD

ART Change Why Does Treatment Fail Patients?


• Reasons: adverse events, drug-drug or drug-food interactions, pill burden,
pregnancy, cost, simplification • ADHERENCE
• Fundamental principle: maintain virologic suppression • Baseline resistance or cross-resistance
• Review ART history, prior ART-associated toxicities, cumulative drug • Prior use of antiretroviral therapy
resistance testing results
• Less potent antiretroviral regimens
• Within-class or between-class ¨ usually works if no resistance
• Specific regimens:
• Drug levels and drug interactions
• DTG/3TC; DTG/RPV; Boosted PI (ATV, DRV) + [3TC or FTC]; • Tissue reservoir penetration
Boosted PI + II (e.g. DRV/r + DTG); IM CAB + RPV
• Not recommended: monotherapy, boosted ATV + RAL, MVC-based
• Provider inexperience
• Consideration: concomitant HBV infection • Other, unknown reasons
DHHS Guidelines 2/27/24

Question #3
When to change therapy?
Virologic failure Immunologic failure
28-year-old man with HIV on TDF/emtricitabine +
• VL undetectable – drug resistance • Associated factors:
atazanavir/ritonavir for 2 years with HIV RNA <50 cps/ml and unlikely • CD4 <200 at ART initiation
CD4 200sÆ300s presents for routine follow-up; labs reveal HIV
• VL <200 cps/ml (low-level viremia) • older age
RNA 68 cps/ml and CD4 352. – risk of resistance believed to be • co-infections
What do you recommend? relatively low
• meds
• VL persistently >200 cps/ml – drug
A. Obtain genotype. • persistent immune activation
resistance often associated
B. Obtain genotype and phenotype. (particularly >500 cps/ml) • loss of regenerative potential
C. Repeat HIV RNA at next visit. • Caution with change to newer VL • other reasons
assays and blips • No consensus on definition
D. Change regimen to TAF/emtricitabine/bictegravir to improve
or treatment
adherence DHHS Guidelines 2/27/24

What to change to?: U.S. DHHS Guidelines


• Review goal of therapy:
• Maximal virologic suppression (HIV RNA below detection)
• Review ART history
• Assess adherence, tolerability, and PK
WHAT TO CHANGE TO? • Perform resistance testing while on drugs
(or within 4 weeks of d/c of ART)
• Identify susceptible drugs/drug classes (e.g. fostemsavir, lenacapavir)
• Do not add a single active drug to a failing regimen
• Goal:
Design a regimen with 2 fully active drugs (one with a high barrier to
resistance: boosted darunavir, dolutegravir, [bictegravir]), or if no
high-barrier drug available, 3 fully active drugs
DHHS Guidelines 2/27/24

©2024InfectiousDiseaseBoardReview,LLC
89
37– AntiretroviralTherapy
Speaker:RoyGulick,MD

Treatment = Prevention

TREATMENT =
• Pregnant women with HIV Fowler NEJM 2016;375:1726
• 3-drug ART Ļ transmission risk to child to 0.5%

PREVENTION
• Men and women with HIV Cohen NEJM 2016;375:830
• Suppressive ART Ļ transmission to sexual partners by 93%

• HIV- post-exposure prophylaxis (PEP) CDC Guidelines


• 3-drug integrase inhibitor-based ART recommended for 4 weeks (e.g. TDF/FTC + DTG)
• At-risk men and women without HIV
Molina NEJM 2015, McCormack Lancet 2016, Landovitz NEJM 2021,
Delany-Moretlwe Lancet 2022; Choopanya Lancet 2013
• PrEP Ļ HIV acquisition by sex >75-85% (TDF/FTC ƃƂ; TAF/FTC ƃ only; IM CAB ƃƂ)
• PrEP Ļ HIV acquisition by injection drug use ~50% (TDF/FTC)

HIV Cure (N=1)


/ 5!
Cure #2
London
Gupta, et al
Nature 2019;568: 244-248

CURE HIV-1
RNA
Cure #3
NYC
Hsu, et al
Cell 2023;186:1115-1126

Cure #4
CD4 Dusseldorf
cell Jensen, et al
count Nat Med 2023;29:583-587

Cure #5
City of Hope
Dickter, et al
NEJM 2024;39;669-671
Hutter NEJM 2009;360:692

ART Controversies: Conclusions Acknowledgements


• When to start? Any viral load or CD4 count and 䇾when
• Cornell HIV Clinical Trials Unit
the patient is ready.䇿 (CCTU)
• What to start? Excellent options; integrase inhibitor- • Division of Infectious Diseases
based regimens for most people. • Weill Cornell Medicine
• When to change? Evaluate virologic response; try to • AIDS Clinical Trials Group
prevent emergence of resistance. • HIV Prevention Trials Network
• Division of AIDS/NIAID/NIH
• What to change to? Use treatment history and drug
• The patient volunteers!
resistance testing to design new regimen with 2 active
drugs (1 with Ĺ barrier to resistance) or 3 active drugs. [email protected]
• Treatment = Prevention Treat HIV, offer PEP and PrEP

©2024InfectiousDiseaseBoardReview,LLC
90
Tuesday, August 20, 2024

38

HIV Drug Resistance

Dr. Michael Saag

©2024 Infectious Disease Board Review, LLC


COPYRIGHT NOTICE: The Copyright Act (Title 17 of U.S. Code) governs the rights attributed to owners of
copyrighted work. Under certain circumstances, educational institutions may provide copies of
copyrighted works to continuing education participants. The copies may not be copied nor used for any
other purpose besides private study, scholarship, or research. Participants should not provide electronic
or print copies of ant materials provided by the university to unauthorized users. If a participant fails to
comply with these restrictions, the participant may be held liable for copyright infringement. No further
transmission or electronic distribution is permitted.

91
92
38– HIVDrugResistance
Speaker:MichaelSaag,MD

Lecture Title

• Disclosures of Financial Relationships with Relevant


HIV Drug Resistance Commercial Interests

- None

Michael S. Saag, MD
Professor of Medicine
University of Alabama at Birmingham

7/1/2024

How does resistance happen? Latently Infected


CD4+ Lymphocytes

HIV Infected
6
10

Cells
5
10

HIV virions
Viral Load
4
10
3
10
2
10

T1/2 = 1.1 days


1
10

0 2 4 6

Weeks
8 10 12
Antiretroviral Rx

Uninfected Activated
CD4+ Lymphocytes

Uninfected Resting
CD4+ Lymphocytes
M Saag, UAB

©2024InfectiousDiseaseBoardReview,LLC
93
38– HIVDrugResistance
Speaker:MichaelSaag,MD

Resistance Testing Key Issues in HIV Resistance


y Genotypic resistance test Easily Tested Tough to Test
Ń Perform test that gives mutations in viral genes y Specific Mutations y Definition of Phenotypes
y Phenotypic resistance test y Complex resistance
y Cross – resistance
Ń Perform test that describes growth of virus in the patterns
y Prevalence of y Genetic Barrier
presence of anti-HIV drugs
resistance at y Nuances of Resistance
y Limitations:
baseline y Relationship between Pk
Ń Cannot detect minority species (< 10% of viral
population) and Pd
12

©2024InfectiousDiseaseBoardReview,LLC
94
38– HIVDrugResistance
Speaker:MichaelSaag,MD

HIV Drug Resistance Testing HIV-1 Genome


y Current guidelines recommend an HIV genotype as part of screening
BEFORE ART is started.
y Following failure of 1st or 2nd regimens, HIV genotype is recommended to
use with the history to choose the optimal next regimen.
y Following failure of 3rd and subsequent regimens, both HIV genotype AND
HIV phenotype should be sent.
y If there is discordance between genotype and phenotype results, use the
geno result (more sensitive).

y NOTE WELL: Resistance mutations accrued from an earlier regimen MAY


NOT be detected by tests obtained at the time of the current failing
regimen

Mutation Nomenclature Mutation Nomenclature


Codon (position) Codon (position)
PR = 1-99 amino acids PR = 1-99 amino acids
RT = 1-560 amino acids RT = 1-560 amino acids

M184V M184V
Wild-type Mutant
amino acid amino acid
(consensus)

Everything You Need to Know About Nucleoside Analog


Resistance in One Slide!
Mutation Selected by Effects on other NRTIs
CASE 1
184V 3TC, FTC - Loss of susceptibility to 3TC, FTC y 25 year old man presents with newly
- Ļ susceptibility to ABC, ddI (clinically insignificant)
- Delayed TAMS and Ĺ susceptibility to AZT, d4T, TDF diagnosed HIV
y Had an episode c/w acute seroconversion
TAMs AZT, d4T - Ļ susceptibility to all NRTIs based on number of TAMs
- More resistance with 41/210/215 than 67/70/219 pathway

151M, 69ins AZT/ddI, ddI/d4T - Resistance to all NRTIs


syndrome 4 months ago
- T69ins: TDF resistance
K65R TDF, ABC, ddI - Variable Ļ susceptibility to TDF, ABC, ddI (and 3TC, FTC) y Initial HIV RNA 40,000; CD4 443 cells/ul
- Ĺ susceptibility to AZT

74V ABC, ddI - Ļ susceptibility to ABC, ddI


y He wants to start ARV therapy
- Ĺ susceptibility to AZT,TDF
44D, 118I AZT, d4T -Increase NRTI resistance (with 41/210/215 pathway) 18

©2024InfectiousDiseaseBoardReview,LLC
95
38– HIVDrugResistance
Speaker:MichaelSaag,MD

Question #1
A baseline genotype is ordered that shows an
M184V mutation. Which of the following drugs will
have reduced susceptibility with this mutation?
A. Efavirenz
B. Zidovudine
C. Tenofovir
D. Etravirene
E. Emtricitabine
19
*

CASE 2 Question #2
y 34 yo woman diagnosed with HIV 10 years The genotype shows an M184V and K65R
ago mutations. Which nRTI drugs would you
y Initially presented with PJP include?
y Initial Lab values
A. ZDV
Ń CD4 82 cells/uL
Ń VL 106,000 c/mL B. TDF
y Started on TDF / FTC / EFV (FDC) C. ddI
y Did well for a while, then the regimen failed D. ABC

©2024InfectiousDiseaseBoardReview,LLC
96
38– HIVDrugResistance
Speaker:MichaelSaag,MD

Non-nucleoside Reverse Transcriptase (NNRTI) Mutations


y K103N is the signature mutation for efavirenz (EFV).
CASE 3
y Y181C is the signature mutation for nevirapine (NVP). y 34 yo woman diagnosed with HIV three years ago
y Older NNRTIs, efavirenz and nevirapine, have low genetic y Initially presented with PJP
barriers (require only 1 mutation for resistance) and are
COMPLETELY cross-resistant to one another. y Initial Lab values
y Newer NNRTIs, etravirine (ETR), rilpivirine (RPV), and Ń CD4 82 cells/uL
doravirine (DOR) have higher barriers to resistance (require Ń VL 106,000 c/mL
>1 mutation for resistance).
y She was treated with TDF / FTC / ELV/ Cobi (FDC)
y K103N has no effect on etravirine susceptibility.
y Rilpivirine failure is associated with E138K, K101E, and/or y The regimen failed after 12 months
Y181C and consequently, resistance to ALL NNRTIs.

Question #3 InSTI Resistance Mutations


Which of the following mutations indicate
high level resistance to elvategravir ?
A. Q148R
B. L68I
C. L68V
D. K67N
E. K65R

Lenacapavir Resistance Mutations Case #4


y 34 yo MSM receiving CAB IM q 2 months
for pre-exposure prophylaxis for last 6
months
y Asymptomatic
y HIV Ag/Ab test negative
y Routine screening: HIV RNA 6.1 c/ml

©2024InfectiousDiseaseBoardReview,LLC
97
38– HIVDrugResistance
Speaker:MichaelSaag,MD

Question #4
Which of the following ARV resistance
mutations is most likely in this setting?
A. S147G
B. N155H
C. Y143R
D. E92Q
E. K65R

InSTI Resistance Mutations CASE 4


y 34 yo woman diagnosed with HIV 22
years ago
y Initially presented with PJP
y Initial Lab values
Ń CD4 82 cells/uL
Ń VL 106,000 c/mL
y Has been on multiple regimens over the
years

Prevalence of Patients with Limited


Question #5
Treatment Options
What is the likelihood she has high level 20%

resistance (< 2 active drugs available) ?


18%

16%

A. < 1 % 14%

12%
Percentage LTO

B. 1 - 5 % 10%
514 LTO

C. 5 -10%
of 6,857
8%
151 LTO
of 8,348
6%

D. 10 - 20% 4%
107 LTO

E. > 20%
of 13,350
2%

0%
2000 2002 2004 2006 2008 2010 2012 2014 2016
Crane et al, IAS 2019
Year

©2024InfectiousDiseaseBoardReview,LLC
98
38– HIVDrugResistance
Speaker:MichaelSaag,MD

Virologic Success in Those with or Common Mutations To Memorize


without LTO y
y
M184V/I
M41L, D67N, K70R,L210W, T215Y, K219Q
3TC and FTC
䇿TAMS䇿
4 or more thymidine-analog mutations (TAMS) affect all approved nucleosides
y K65R tenofovir
y Q151M, 69SSS multi-NRTI

y K103N EFV (and NVP)


retains susceptibility to etravirine
y Y181C NVP and other NNRTI
y E138K, K101E RPV and other NNRTI

y I50L ATV

y N155H, Q148H/R/K RAL and EVG


y Y143C RAL
Crane et al, IAS 2019 y R263K DTG

Summary
y High concern about resistance testing on Board
Exams • [email protected]
y Difficult to create test questions that do not
require complex interpretation, have a single
best answer, or are not ‘multiple true-false’
y Knowing common mutations and their role is a
good way to prepare for the exam

39

©2024InfectiousDiseaseBoardReview,LLC
99
100
38 – HIV Drug Resistance
Speaker: Michael Saag, MD
Enlarged Slide: 22

*
*

©2024 Infectious Disease Board Review, LLC


101
102
Tuesday, August 20, 2024

39

Antiretroviral Therapy for Special


Populations

Dr. Roy Gulick

©2024 Infectious Disease Board Review, LLC


COPYRIGHT NOTICE: The Copyright Act (Title 17 of U.S. Code) governs the rights attributed to owners of
copyrighted work. Under certain circumstances, educational institutions may provide copies of
copyrighted works to continuing education participants. The copies may not be copied nor used for any
other purpose besides private study, scholarship, or research. Participants should not provide electronic
or print copies of ant materials provided by the university to unauthorized users. If a participant fails to
comply with these restrictions, the participant may be held liable for copyright infringement. No further
transmission or electronic distribution is permitted.

103
104
39– AntiretroviralTherapyforSpecialPopulations
Speaker:RoyGulick,MD

Lecture Title

• Disclosures of Financial Relationships with Relevant


Antiretroviral Therapy (ART) for Special Commercial Interests
Populations
- None

Roy M. Gulick, MD, MPH


Rochelle Belfer Professor in Medicine
Chief, Division of Infectious Diseases
Weill Cornell Medicine
7/1/2024

Special Populations Question #1


• acute/recent HIV infection A 22-year-old man presents with fever, mouth pain, and skin
• acute opportunistic infection rash. PE reveals 3 small oral ulcers and diffuse macular rash.
• tuberculosis Labs show WBC 3K, platelets 89K, monospot negative, RPR
• HIV-HBV co-infection NR, HIV antibody negative, HIV RNA 1,876,000 cps/ml.
• HIV-HCV co-infection
Which statement is correct?
• pregnancy
A. ART should not be offered.
• post-HIV exposure (PEP)
• occupational
B. ART would decrease his symptoms.
• non-occupational C. ART has long-term virologic benefits in this setting.
• pre-HIV exposure (PrEP) D. ART has long-term clinical benefits in this setting.

Acute or Recent HIV


• ART is RECOMMENDED. Question #2
• ART reduces symptoms and signs and reduces transmission. A 52-year-old woman is admitted for progressive SOB, is
• No long-term virologic, immunologic, or clinical data available. intubated, undergoes BAL and is found to have PCP. HIV Ab
• Goal is full virologic suppression. test is positive, CD4 103, HIV RNA 135,000 copies/ml. She is
day 4 of IV trimethoprim-sulfa and corticosteroids and still
• Obtain genotype prior to ART.
intubated.
• If ART is started prior to genotype results, use bictegravir,
When should she start ART?
dolutegravir, or boosted darunavir, together with tenofovir (TAF
A. Immediately
or TDF) + emtricitabine.
B. In the next 2 weeks
• If patient was on IM cabotegravir for PrEP, use boosted
darunavir-based regimen (rather than integrase inhibitor-based). C. After completing 21 days of trimethoprim-sulfa

• Can modify regimen, if needed, when genotype results return.


D. At her first outpatient clinic visit
DHHS Guidelines 2/27/24

©2024InfectiousDiseaseBoardReview,LLC
105
39– AntiretroviralTherapyforSpecialPopulations
Speaker:RoyGulick,MD

ACTG 5164: Immediate vs Delayed ART HIV-TB Co-infection


with an Acute OI • Treat active TB the same with or without HIV.
• 282 patients with treatable OI
diagnosed within 14 days Time to AIDS/death
• All PWH with TB should start TB meds immediately.

death/new AIDS defining event


Probability of surviving without
randomized to start ART within 48
hours vs. after 4 weeks
• In PWH with TB, timing of starting ART depends on
• most common OI: PCP (63%) CD4 count:
• For CD4 <50, start ART ASAP, within 2 weeks of TB rx
• AIDS progression/death:
immediate rx (14%) vs • For CD4 >50, start ART within 8 weeks of TB rx
delayed rx (24%)
• Start pregnant women with HIV and TB on ART as
• No differences in safety/toxicity,
IRIS, or week 48 responses
Time to Death/new AIDS defining illness (weeks) early as feasible.
• Caution with CNS OI (e.g.
Zolopa PLoS One 2009;4:e5575
• For TB meningitis, monitor closely.
cryptococcus, TB) DHHS Guidelines 2/27/24

Question #3 HIV-TB Co-infection (2)


• Include a rifamycin in the regimen.
A 39-year-old man with HIV, CD4 298, HIV RNA 23,000 cps/ml, • rifampin
never on ART is diagnosed with pulmonary TB. The plan is to start • significantly Ļ TAF – current FDA label: not recommended
INH, RIF, PZA, and ETH pending susceptibilities. He agrees to • significantly Ļ ALL PIs – do not use
start ART and genotype is wild-type. • Ļ dolutegravir (DTG) (need to Ĺ DTG to 50 mg bid)
• significantly Ļ bictegravir (BIC) – do not use (conflicting data)
Which of the following ART regimens do you recommend?
• Ļ NNRTI concentrations: efavirenz (EFV) 600 mg daily is recommended
A. TDF/emtricitabine/efavirenz • rifabutin: preferred; more manageable drug interactions with protease inhibitors

B. TAF/emtricitabine + atazanavir (boosted) • For IRIS, continue both ART and TB meds while managing the
syndrome.
C. TDF/emtricitabine + atazanavir (unboosted)
• Treatment support, including directly observed therapy (DOT) of TB rx
D. TAF/emtricitabine + darunavir (boosted)
is strongly recommended.
DHHS Guidelines 2/27/24

Question #4 HIV-HBV Co-infection


A 55-year-old with HIV not previously on rx, CD4 320 and HIV RNA • Some ART has activity against HBV
67,000 cps/ml • lamivudine (3TC), emtricitabine (FTC), tenofovir (TDF and TAF)
Lab testing reveals: toxoplasma Ab+; CMV Ab+; HAV total Ab+; • Some HBV drugs have activity against HIV
HBV surface Ag+, core Ab+, surface Ab-; HCV Ab-; RPR NR • entecavir (can select M184V) McMahon NEJM 2007;356:2614
Of the following, which ART regimen would you recommend?
• If treatment started, treat both optimally
A. abacavir/lamivudine/dolutegravir • 2 active agents for HBV (TAF or TDF) + (3TC or FTC)
B. dolutegravir/lamivudine • + 3rd drug for HIV (preferred = BIC or DTG)
• If tenofovir cannot be used, start a fully suppressive regimen and
C. tenofovir (TAF or TDF) + atazanavir (boosted)
add entecavir
D. tenofovir (TAF or TDF)/emtricitabine + darunavir (boosted) DHHS Guidelines 2/27/24

©2024InfectiousDiseaseBoardReview,LLC
106
39– AntiretroviralTherapyforSpecialPopulations
Speaker:RoyGulick,MD

HIV-HCV Co-Infection
• Anyone with HCV should be screened for HIV.
Question #5
• High-risk HIV+ patients should be screened for HCV A 26-year-old woman with HIV on abacavir/lamivudine +
annually. efavirenz with CD4 630 and VL suppressed below
• ART should be started in those with concomitant HCV. detection becomes pregnant.
• Same initial regimens recommended, but caution with drug-drug
interactions and overlapping toxicities. What do you recommend regarding ART?
• Patients with HIV and HCV should be evaluated for HCV A. Discontinue ART until 2nd trimester.
therapy (including assessing liver fibrosis stage). B. Change abacavir to zidovudine.
• Also evaluate for HBV co-infection.
C. Change efavirenz to bictegravir.
• HCV direct-acting antiviral regimens Æ high cure rates
D. Continue current regimen.
DHHS Guidelines 2/27/24

Antiretrovirals in Pregnancy ART in Pregnancy: NRTI


• ART recommended for all pregnant people, as early as possible,
regardless of CD4 or VL level (rx and prevention of MTCT) • Preferred:
• Perform drug-resistance testing if VL >500-1000 cps/ml • abacavir/lamivudine

• Start (or continue if safe/tolerated) standard 3-drug ART as early • tenofovir (TAF or TDF)/(emtricitabine or lamivudine)
as possible (while awaiting drug resistance testing):
• 2-drug regimens can be continued, if virologically suppressed • Alternative:
• Modify regimen when drug resistance testing results available
• zidovudine/lamivudine
• ART does NOT increase the risk of birth defects
• Near delivery, if HIV RNA >1000 (or unknown), use intravenous
zidovudine, and recommend Cesarean section at 38 weeks • IV zidovudine recommended close to delivery if VL >1000

DHHS Perinatal Guidelines 1/31/24 <www.clinicalinfo.hiv.gov>


DHHS Perinatal Guidelines 1/31/24 <www.clinicalinfo.hiv.gov>

ART in Pregnancy: NNRTI ART in Pregnancy: PI


• Alternative: • Preferred:
• efavirenz (birth defects reported in primate studies, NO evidence in human • darunavir/ritonavir (need to use bid)
studies and extensive experience; screen for depression)
• rilpivirine (NOT with baseline VL >100K or CD4 <200 or PPIs)
• Alternative:
• atzanavir/ritonavir
• Insufficient data: doravirine

• Not recommended:
• Not recommended (could continue if on):
• cobicistat (Ļ drug concentrations, limited experience)
• etravirine (not for treatment-naïve pts)
• lopinavir/ritonavir (side effects, need to use bid; could continue if on;
• nevirapine (toxicity, need for lead-in dosing, low barrier to resistance)
may need to Ĺ dose)
DHHS Perinatal Guidelines 1/31/24 <www.clinicalinfo.hiv.gov> DHHS Perinatal Guidelines 1/31/24 <www.clinicalinfo.hiv.gov>

©2024InfectiousDiseaseBoardReview,LLC
107
39– AntiretroviralTherapyforSpecialPopulations
Speaker:RoyGulick,MD

ART in Pregnancy: INSTI ART in Pregnancy: Other


• Preferred: • Not recommended:
• dolutegravir (neural tube defects not significantly Ĺ vs. other ART) • 2-drug regimens (e.g. dolutegravir/lamivudine,
dolutegravir/rilpivirine; could continue if on)
• Alternative: • cobicistat as a booster (for EVG or PIs)
• bictegravir
• enfuvirtide (limited data; could continue if on)
• raltegravir (need to use bid)
• fostemsavir (limited data; could continue if on)
• ibalizumab (limited data; could continue if on)
• Not recommended:
• elvitegravir/cobicistat (Ļ drug concentrations) • lenacapavir (limited data; could continue if on)
• IM cabotegravir + rilpivirine • maraviroc (need tropism testing; limited data, could
DHHS Perinatal Guidelines 1/31/24 <www.clinicalinfo.hiv.gov> continue if on) DHHS Perinatal Guidelines 1/31/24 <www.clinicalinfo.hiv.gov>

Question #6 Antiretrovirals for PEP (1)


Postexposure prophylaxis (PEP) for occupational
A 34-year-old nurse without HIV sustains a needlestick from a exposure:
patient with HIV who has not taken ART for 2 years. • Assess nature of exposure:
source fluid, volume of fluid, type of exposure, timing
Which of these post-exposure (PEP) regimens do you • Assess exposure source; HIV and hepatitis testing
recommend? • Testing (baseline, 6 + 12 wks + 6 months with standard HIV Ab or 6 wks + 4
months if new HIV Ab/p24 test used) and counseling
A. tenofovir (TDF)/emtricitabine
• Offer 4 weeks of rx for recognized transmission risk
B. tenofovir (TDF)/emtricitabine + integrase inhibitor • start ASAP (within 72 hours)
C. tenofovir (TAF)/emtricitabine + integrase inhibitor • tenofovir (TDF)/emtricitabine + dolutegravir (not in women in early
D. tenofovir (TDF)/emtricitabine + protease inhibitor pregnancy or sexually active and not on birth control) or raltegravir
• adjust regimen for possibility of resistance in source patient
• f/u within 72 hours PHS Guidelines updated 5/23/18

Antiretrovirals for PEP (2) Question #7


PEP for non-occupational exposure:
• Presentation <72 hours with substantial risk exposure from HIV+ 23 year old man without HIV with a partner with HIV on ART with HIV RNA
or likely to be HIV+ – recommended suppressed below detection asks about starting pre-exposure prophylaxis
• Presentation >72 hours or no substantial risk of exposure – (PrEP).
not recommended
In addition to safer sex counseling, which of these do you recommend?
• Testing: Do rapid HIV (Ag)/Ab test or if results not available, start PEP
A. Nothing – PrEP is not indicated.
• Prior to PEP: BUN/creatinine, LFTs, STI testing (CT, GC, syphilis),
HBV/HCV testing, pregnancy testing B. PrEP with tenofovir (TDF)/emtricitabine daily.
C. PrEP with tenofovir (TAF)/emtricitabine “on demand”.
• Treatment: 4 weeks of D. PrEP with bictegravir/tenofovir (TAF)/emtricitabine daily.
• Preferred: TDF/FTC + [dolutegravir or raltegravir]
• Alternative: TDF/FTC + darunavir/ritonavir
https://www.cdc.gov/hiv/clinicians/prevention/prescribe-pep.html#regimens

©2024InfectiousDiseaseBoardReview,LLC
108
39– AntiretroviralTherapyforSpecialPopulations
Speaker:RoyGulick,MD

CDC Guidance for PrEP: Conclusions


https://www.cdc.gov/hiv/pdf/risk/prep/cdc-hiv-prep-guidelines-2021.pdf
• Inform all sexually active adults and adolescents about PrEP 1. Acute (and recent) HIV – ART recommended.
• Before starting: 2. Acute OI – ART within 2 weeks of diagnosis reduces mortality; caution
• exclude acute and chronic HIV infection (by HIV testing and symptoms) with CNS opportunistic infections.
• assess baseline CrCl, screen for STIs and HBV infection 3. TB – Early ART prolongs survival; caution with rifamycin drug
• Prescribe PrEP for people with ongoing risk from sex or injecting drugs: interactions.
• tenofovir (TDF)/emtricitabine for ƃ and Ƃ 4. Hepatitis B and C co-infection – Consider antiviral activity, drug-drug
• tenofovir (TAF)/emtricitabine for ƃ ONLY interactions, drug toxicities.
• IM cabotegravir for ƃ and Ƃ 5. Pregnancy – Treat and reduce MTCT; modify ART recommendations
• provide risk reduction, adherence counseling, condoms based on safety and experience.
• On PrEP: 6. Post-exposure prophylaxis (PEP) – ART within 72 hours; give for 4
• HIV testing every 3-4 months, monitor CrCl every 6 (age >50 or CrCl <90) or 12 months weeks; adjust for known drug resistance.
• risk reduction, condoms, STI assessments/treatment 7. Pre-exposure prophylaxis (PrEP) – TDF/FTC (䗢+䗠), TAF/FTC (䗢), IM
• evaluate the need to continue PrEP CAB (䗢+䗠)

Acknowledgments
• Cornell HIV Clinical Trials Unit
(CCTU)
• Division of Infectious Diseases
• Weill Cornell Medicine
• AIDS Clinical Trials Group (ACTG)
• HIV Prevention Trials Network
• Division of AIDS/NIAID/NIH
• The patient volunteers!

[email protected]

©2024InfectiousDiseaseBoardReview,LLC
109
110
Tuesday, August 20, 2024

BR4

Board Review Session 4

Drs. Gulick (Moderator), Bloch, Gandhi,


Maldarelli, Masur, Saag, and Tunkel

©2024 Infectious Disease Board Review, LLC


COPYRIGHT NOTICE: The Copyright Act (Title 17 of U.S. Code) governs the rights attributed to owners of
copyrighted work. Under certain circumstances, educational institutions may provide copies of
copyrighted works to continuing education participants. The copies may not be copied nor used for any
other purpose besides private study, scholarship, or research. Participants should not provide electronic
or print copies of ant materials provided by the university to unauthorized users. If a participant fails to
comply with these restrictions, the participant may be held liable for copyright infringement. No further
111
transmission or electronic distribution is permitted.
112
BR4 –Board Review: Day 4
Moderator: Roy Gulick, MD

BOARD REVIEW DAY 41 2024


#40 A 30-year-old woman presented with newly diagnosed
HIV infection 9 months ago. She was 6 weeks
pregnant.
Initial: HIV RNA 28,000 c/ml
CD4 count 650 cells/ul

Board Review: Day 4 She was started on DTG + TAF/ FTC. Viral load
became below level of detection and remained so
Moderator: Roy Gulick, MD, MPH throughout pregnancy and delivery.
Faculty: Drs. Bloch, Gandhi, Maldarelli, Masur, A healthy baby girl was delivered 2 days ago.
Saag, and Tamma 7/1/2024 1 of 3

BOARD REVIEW DAY 41 2024 BOARD REVIEW DAY 41 2024


#40 Mom is in the US and wants to breastfeed. You tell #41 A 40-year-old apple-grower from Eastern
her? Washington State presented to the Emergency
A) Yes, she should feel free to breast feed her infant Department with the acute onset of diplopia and
exertional dyspnea which started evolving over 12
B) No, it is unsafe to breast feed in any situation
hours.
C) No, it’s unsafe to breast feed because of her viral
Over a few hours, the muscle weakness extended
load when she presented early in pregnancy
to all 4 extremities with concomitant decreases in
D) Breastfeeding is a possible option: Discuss pros his oxygen saturation.
and cons of breastfeeding with her and let her
decide He required intubation.
2 of 3 1 of 6

BOARD REVIEW DAY 41 2024 BOARD REVIEW DAY 41 2024


#41 His last meals in the 24 hours prior to the onset of #41 His vital signs were normal including a normal
symptoms were breakfast that included eggs, and temperature with the exception of an oxygen
toast with locally grown peaches and lunch in saturation of 89% with a respiratory rate of 20 per
which he had a venison sandwich with minute.
mayonnaise with home canned corn; he had shot,
Skin examination was unremarkable.
butchered, and frozen the deer meat 6 months
previously. A full beard and very long hair were noted.
One day before he developed diplopia and He is unable to move his eyes laterally. There is
dyspnea, he sprayed 10 acres of his apple trees decreased strength in both arms.
with a potent insecticide on a windy day.
2 of 6 3 of 6

©2024 Infectious Disease Board Review, LLC


113
BR4 –Board Review: Day 4
Moderator: Roy Gulick, MD

BOARD REVIEW DAY 41 2024 BOARD REVIEW DAY 41 2024


#41 Strength of his leg muscles did not decrease with #41 Which one of the following is the most likely
repetitive contractions. etiology of his paralytic clinical syndrome?
Basic laboratory work and chest X-Ray were A) Tick paralysis
unremarkable.
B) Guillain-Barre
A head CT scan with contrast was unremarkable.
C) Organophosphate poisoning
Lumbar puncture: opening pressure, cell count, D) Botulism
glucose and protein were normal.
E) Myasthenia gravis

4 of 6 5 of 6

BOARD REVIEW DAY 41 2024 BOARD REVIEW DAY 41 2024


#42 48-year-old asymptomatic man presents with #42 Hepatitis panel reveals:
newly diagnosed HIV infection. • HBVsAg neg
• HBsAb neg
His initial HIV RNA is 280,000 c/ml and CD4 count • HBcAb +
65 cells/ul. • HBV DNA neg (<1000)
Other labs are normal; Genotype is Wild-type
4 months ago, he started on DTG + TAF/FTC;
virus.
He did well: with HIV RNA <20 and CD4 Count 270 cells/ul.
He has heard about injectable ARV therapy on TV and
would like to try such a regimen.
1 of 4 2 of 4

BOARD REVIEW DAY 41 2024 BOARD REVIEW DAY 41 2024


#42 What would you recommend? #43 A 28-year-old man is newly found to have HIV
infection. Initial work-up reveals he’s
asymptomatic with a normal physical exam.
A) Cabotegravir alone
Labs demonstrate:
B) Rilpivirine alone
• Normal CBC, electrolytes, and LFTs
C) Cabotegravir-rilpivirine
• HIV RNA 23,000
D) Stay on current regimen: this patient should not
be given a long-acting regimen with the drugs • CD4 count 379 cells/uL
currently available • Genotype: reverse transcriptase (RT) M184V
3 of 4 1 of 3

©2024 Infectious Disease Board Review, LLC


114
BR4 –Board Review: Day 4
Moderator: Roy Gulick, MD

BOARD REVIEW DAY 41 2024 BOARD REVIEW DAY 41 2024


#43 He prefers a one-pill, once-daily oral regimen. Which #44 A 37-year-old man with a history of intravenous
regimen do you recommend starting? drug use and HIV infection appeared in the
emergency room with fever and pulmonary
infiltrates.
A) Abacavir/lamivudine/dolutegravir
He is diagnosed with tuberculosis by sputum smear
B) Tenofovir AF/emtricitabine/bictegravir microscopy and started on conventional 4 drug
C) Tenofovir AF/emtricitabine/darunavir/ritonavir antituberculosis therapy.
D) Tenofovir AF/emtricitabine/elvitegravir/cobicistat Two weeks later, he was started by another
E) Tenofovir DF/lamivudine/efavirenz physician on abacavir-lamivudine- and double-dose
dolutegravir.
2 of 3 1 of 5

BOARD REVIEW DAY 41 2024 BOARD REVIEW DAY 41 2024


#44 His CD4 was 60 cells/µL, and his viral load was #44 The sputum specimens are negative for AFB.
100,000 copies/µL at the time ART was started. Bronchoscopy shows no pneumocystis, fungus,
Eight weeks after starting ART (10 weeks after or bacteria on direct smear, but the GeneXpert
starting anti-TB therapy), he returns with new MTB/RIF remains positive for TB (rifampin
resistance not detected).
fever.
Chest X-ray shows more extensive infiltrates, a The original culture has now been reported as
positive for M. tuberculosis; phenotypic
new pleural effusion, and new mediastinal
susceptibility testing results are pending.
adenopathy.

2 of 5 3 of 5

BOARD REVIEW DAY 41 2024 BOARD REVIEW DAY 41 2024


#44 This worsening clinical syndrome most likely #45 47-year-old woman started BIC/FTC/TAF 12
represents: months ago as her first regimen (Bictegravir,
emtricitabine, Tenofovir disoproxil fumarate).
A) Drug-resistant tuberculosis Initial: HIV RNA 28,000 c/ml (Wild-type virus).
B) Abacavir hypersensitivity syndrome
CD4 count 450 cells/ul.
C) BAL negative pneumocystis pneumonia
Current: HIV RNA <20 c/mL / CD4+ count 930 /uL.
D) Immune reconstitution syndrome
E) A drug interaction between INH and abacavir
4 of 5 1 of 4

©2024 Infectious Disease Board Review, LLC


115
BR4 –Board Review: Day 4
Moderator: Roy Gulick, MD

BOARD REVIEW DAY 41 2024 BOARD REVIEW DAY 41 2024


#45 Since starting her current regimen her weight has #45 In addition to diet and exercise, you recommend:
increased from 145 lbs to 171 lbs.
Fasting glucose 101 mg/dl. HbA1c 5.9. A) No other interventions at this time
Diet and exercise have not been effective. B) Changing ARV to non-TAF, non-InSTI regimen
She is bothered by the weight gain and wants C) Start Metformin 500 mg twice daily
something done to reduce her weight.
D) Start Semaglutide, ramp up dose to 1.0 mg SQ
weekly
2 of 4 3 of 4

BOARD REVIEW DAY 41 2024 BOARD REVIEW DAY 41 2024


#46 A 40-year-old man with no significant past medical history #46 On examination, his temperature is 101°F. He is awake, alert,
presents in December with complaints of fever, headache, and oriented. He has meningismus and shotty cervical
and stiff neck. adenopathy.
His symptoms started 10 days ago and has not responded to Genital examination reveals some pain on palpation of his
analgesic therapy – in fact, his headache has worsened over left testes. Abdominal examination is normal.
the last several days.
Laboratory studies reveal a WBC count of 3,000/mm3 and his
He lives alone in a mobile home and has recently seen a platelet count is 80,000/mm3. Lumbar puncture shows an
number of mice and rats in his home, but he denies any bites opening pressure of 210 mm H2O, WBC count of 200/mm3
from these rodents. (95% lymphocytes), glucose of 45 mg/dL, and protein of 250
mg/dL.
He takes no medications and has received all of his
vaccinations. CSF Gram stain is negative.
1 of 4 2 of 4

BOARD REVIEW DAY 41 2024 BOARD REVIEW DAY 41 2024


#46 Which of the following is the most likely cause of #47 A 25-year-old man without HIV infection was
this patient’s meningitis? receiving every other month injections of
cabotegravir for HIV pre-exposure prophylaxis
(PrEP).
A) Mumps virus
B) Measles virus
He missed 2 consecutive injections due to work
C) Lymphocytic choriomeningitis virus travels and was evaluated showing HIV
D) Leptospira interrogans antigen/antibody test +, HIV-1 immunoblot +, HIV
RNA 120,000, and HIV genotype is pending.

3 of 4 1 of 3

©2024 Infectious Disease Board Review, LLC


116
BR4 –Board Review: Day 4
Moderator: Roy Gulick, MD

BOARD REVIEW DAY 41 2024 BOARD REVIEW DAY 41 2024


#47 What do you recommend? #48 A 50-year-old man with untreated HCV presented
with a 6-week history of ulcerating skin lesions.
A) Restart cabotegravir PrEP He relates a history of injection drug use of both
B) Change to tenofovir DF/emtricitabine PrEP cocaine and fentanyl over this time period.
C) Start tenofovir AF/emtricitabine/bictegravir On physical exam, he is afebrile.
D) Start tenofovir AF/emtricitabine + darunavir/ritonavir

2 of 3 1 of 4

BOARD REVIEW DAY 41 2024 BOARD REVIEW DAY 41 2024


#48 Skin exam reveals multiple small, painful #48 What is the most likely cause of these ulcers?
ulcerations on his chest, neck, arms, and legs,
most but not all of which are adjacent to areas A) Pyoderma gangrenosum
where he has injected various street drugs.
B) Polyarteritis nodosum
There is no purulence, odor, or surrounding
erythema. C) Xylazine
D) Porphyria cutanea tarda
Punch biopsy showed nonspecific inflammation
and subcutaneous necrosis, without vasculopathy. E) Cryoglobulinemia

2 of 4 3 of 4

BOARD REVIEW DAY 41 2024 BOARD REVIEW DAY 41 2024


#49 A 71-year-old man with HIV transfers care to you with #49 In addition to optimizing his antiretroviral regimen,
a history of taking and failing “nearly all HIV you recommend:
medications including T20 (enfuvirtide).”
He currently takes tenofovir alafenamide A) Adding maraviroc
(TAF)/emtricitabine (FTC) + etravirine + darunavir + B) Adding double dose maraviroc
ritonavir with a CD4 15 and HIV RNA 233,140
C) Adding enfuvirtide
copies/ml.
D) Adding fostemsavir
You send an HIV genotype, phenotype, and tropism
test. The tropism test returns “dual/mixed virus.”
1 of 3 2 of 3

©2024 Infectious Disease Board Review, LLC


117
BR4 –Board Review: Day 4
Moderator: Roy Gulick, MD

BOARD REVIEW DAY 41 2024 BOARD REVIEW DAY 41 2024


#50 A 35-year-old sexually active heterosexual man wants to reduce #51 A 44-year-old man was diagnosed with Pneumocystis
his risk of HIV and asks about taking HIV pre-exposure pneumonia as his AIDS-defining illness and begun on
prophylaxis (PrEP) “only when needed.”
antiretroviral therapy with 2 nucleosides and an
Which do you recommend? integrase inhibitor during his hospitalization.

A) None, PrEP not recommended He stabilizes and follows up for repeated outpatient
B) Daily tenofovir disoproxil fumarate (TDF)/emtricitabine visits with an HIV RNA consistently <20 copies/ml and
a CD4 cell count of 44 that increased to 163 (at 3
C) TDF/emtricitabine “on demand” (2 pills 24 hours before sex, months), 232 (at 6 months), 242 (at 9 months), and was
then one 24 hours later and one 48 hours later)
repeated at 243 (at 12 months).
D) TAF/emtricitabine “on demand”
E) Cabotegravir “on demand” 1 of 2 1 of 4

BOARD REVIEW DAY 41 2024 BOARD REVIEW DAY 41 2024


#51 His current medications are: tenofovir #51 What do you recommend?
alafenamide/emtricitabine, dolutegravir, trimethoprim-
sulfa double strength daily, and azithromycin 1200 mg
A) Stop tenofovir alafenamide/emtricitabine
once weekly.
B) Stop trimethoprim-sulfa
He says he’s tired of taking pills and would like to stop
some of them. C) Stop azithromycin
D) Stop trimethoprim-sulfa and azithromycin
E) Continue the current regimen

2 of 4 3 of 4

BOARD REVIEW DAY 41 2024 BOARD REVIEW DAY 41 2024


#52 A 30-year-old woman is admitted to the hospital #52 She was diagnosed with herpes simplex encephalitis
with seizures and hallucinations. (HSE) and was discharged to a skilled nursing
facility to complete a 3-week course of intravenous
Two weeks prior to this admission, she was
acyclovir (10 mg/kg every 8 hours).
hospitalized with fever, confusion, and headaches.
She initially did well, with resolution of fever and
A CSF analysis at that time showed 160 normalization of mentation.
WBCs/mm3 with 89% lymphocytes and HSV-1 PCR
was positive. On the day prior to re-admission, she was noted to
be paranoid (believed the nurses were poisoning
MRI showed a T2-weighted lesion in the right her) and on the day of admission had a generalized
temporal lobe. seizure.
1 of 6 2 of 6

©2024 Infectious Disease Board Review, LLC


118
BR4 –Board Review: Day 4
Moderator: Roy Gulick, MD

BOARD REVIEW DAY 41 2024 BOARD REVIEW DAY 41 2024


#52 On exam, she is afebrile, and her neck is supple. #52 MRI showed slight improvement in the right
temporal lobe with no new lesions, Lumbar
Choreoathetoid movements of both hands are noted.
puncture is performed with 27 WBC/mm3, 66%
She is oriented only to person. lymphocytes.

Routine laboratory testing including chemistry panel CSF protein and glucose are normal. PCR for HSV
and CBC are within normal limits. was negative.

3 of 6 4 of 6

BOARD REVIEW DAY 41 2024


#52 Which of the following is the most likely
diagnosis?
A) Acyclovir neurotoxicity
B) Anti-N-methyl-D-aspartate receptor (NMDAR)
encephalitis
C) Acute disseminated encephalomyelitis (ADEM)
D) Relapsed HSV encephalitis
E) CNS vasculitis
5 of 6

©2024 Infectious Disease Board Review, LLC


119
120
Tuesday, August 20, 2024

40

Pharyngitis Syndromes Including Group A


Strep Pharyngitis

Dr. Karen Bloch

©2024 Infectious Disease Board Review, LLC


COPYRIGHT NOTICE: The Copyright Act (Title 17 of U.S. Code) governs the rights attributed to owners of
copyrighted work. Under certain circumstances, educational institutions may provide copies of
copyrighted works to continuing education participants. The copies may not be copied nor used for any
other purpose besides private study, scholarship, or research. Participants should not provide electronic
or print copies of ant materials provided by the university to unauthorized users. If a participant fails to
comply with these restrictions, the participant may be held liable for copyright infringement. No further
transmission or electronic distribution is permitted.

121
122
40Ǧ PharyngitisSyndromesandGroupAStrep
Speaker:KarenC.Bloch,MD,MPH,FIDSA,FACP

Disclosures of Financial Relationships with Relevant


Pharyngitis Syndromes Including Group A Strep
Commercial Interests
Pharyngitis

• None
Karen C. Bloch, MD, MPH, FIDSA, FACP
Professor, Division of Infectious Diseases
Vanderbilt University Medical Center
7/1/2024

Think Like A Realtor


Think Like a Realtor Location
Location
Location

Pharyngitis Case 1
38yo female with 1 day of sore throat and fever.
• Micro-neighborhoods Childhood history of anaphylaxis to penicillin.
Physical exam
T=102.3
• Regional differences
HEENT-tonsillar erythema & petechiae
Neck-Tender bilateral anterior LAN
Labs:
Rapid strep antigen test negative

©2024InfectiousDiseaseBoardReview,LLC
123
40Ǧ PharyngitisSyndromesandGroupAStrep
Speaker:KarenC.Bloch,MD,MPH,FIDSA,FACP

Question 1 Group A streptococcus


What is the most appropriate antimicrobial
treatment? • AKA Streptococcus pyogenes
A. Cephalexin
B. None • 5-15% sore throats in adults
C. Doxycycline
D. Clindamycin • Usually self-limited infection in adults
E. Levofloxacin (even untreated)

Differentiating Pharyngitis Differentiating Pharyngitis


GAS Viral pharyngitis GAS Viral pharyngitis
• Sudden onset • The 3 C’s
• Fever – Conjunctivitis
• Lymphadenopathy – Coryza
– Cough
• Exposure to contact with
streptococcal pharyngitis • Other symptoms VS
– Diarrhea
– Ulcerative stomatitis
– Hoarseness

How Specific are Clinical Findings? Streptococcal Clues


• Modified CENTOR score Points Strep probability
0 or 1 < 10% • Palatal petechia • Scarletina
• Can’t cough
2 11 -17%
• Exudate
3 28 -35%
• Nodes
4 or 5 35-50%
• Temperature
• OR age <15 yr (+1) or >44 years (-1)

IDSA guidelines recommend antibiotics only following a RADT positive testing.

©2024InfectiousDiseaseBoardReview,LLC
124
40Ǧ PharyngitisSyndromesandGroupAStrep
Speaker:KarenC.Bloch,MD,MPH,FIDSA,FACP

Laboratory Diagnosis Treatment for GAS Pharyngitis


• First line: PCN Allergic:
• Adults:
– Oral Penicillin or – cephalosporin,
– RADT screen, if negative, culture optional amoxicillin x 10 days clindamycin,
• ASO titer or Anti-DNAse B antibodies macrolides (+/-)
– Not recommended:
– helpful in diagnosis of rheumatic fever and tetracyclines,
post-streptococcal glomerulonephritis, but not sulfonamides,
for strep pharyngitis. fluoroquinolones

Secondary Complications Pharyngitis and….


• Infectious complications • Immunologic complications

Pharyngitis & Rash Pharyngitis & Rash


• Young adult with fever, sore throat, tonsillar exudate, scarletinoform
rash BUT…Negative RADT and culture • Acute HIV • Secondary syphilis

• Gram positive rod

• Rash in >50%, mimics strep

• Rarely life-threatening sequelae

©2024InfectiousDiseaseBoardReview,LLC
125
40Ǧ PharyngitisSyndromesandGroupAStrep
Speaker:KarenC.Bloch,MD,MPH,FIDSA,FACP

Pharyngitis after Receptive Oral Intercourse Pharyngitis & Conjunctivitis


Neisseria gonorrhoeae Herpes simplex virus • College freshman with sore
throat, fever, and conjunctivitis.
• Highest risk MSM • HSV-1 or HSV-2 • Roommate and 3 others in her
• Diagnose by nucleic acid • Usually with initial dorm with similar syndrome
amplification test of infection
pharyngeal swab • Tonsillar vesicles
• Labial or genital ulcers
variably present
Epidemics in group living situations—barracks, dorms, camps, etc

Pharyngitis and Vesicles Case 2


• A 62 yo man presents with 24hr of fever, chills, and
• 35 yo man with sore throat, low grade fever, and
odynophagia
lesions on palms & soles. His 3 yo son is sick
• He works at a vineyard in Napa Valley, and last week
with a similar illness. participated in the grape harvest. He admits to sampling
the grape must.
• His cat recently had kittens

• Caused by enteroviruses (most common Coxsackie virus)


• More common in kids (often serve as vector)

Case 2 Question 2
• PE: What is the most likely cause of this patient’s illness?
Ill appearing,
T=102.4, HR=122, BP=97/52
A. Toxoplasmosis
left tonsil swollen and erythematous
B. Bartonellosis (Cat Scratch Fever)
Left suppurative lymph node tender to
palpation C. Tularemia
D. Epstein Barr virus
CMAJ 2014;186:E62
E. Scrofula (mycobacterial lymphadenitis)

©2024InfectiousDiseaseBoardReview,LLC
126
40Ǧ PharyngitisSyndromesandGroupAStrep
Speaker:KarenC.Bloch,MD,MPH,FIDSA,FACP

Oropharyngeal Tularemia Pharyngitis and Chest Pain


• Uncommon in the US • 20 yo college student with sore throat, fever and chills. Despite oral
amoxicillin, develops new onset of cough and pleuritic CP; CT below
• Transmission through ingestion (or rarely inhalation)
– Inadequately cooked game
– Contaminated water
– Rodent contamination • Septic phlebitis of internal jugular vein
• Exudative tonsillitis, suppurative LAN • Often follows GAS pharyngitis or mono (EBV)
• Treatment: streptomycin, doxycycline or quinolone • Classic cause is Fusobacterium necrophorum
• Causes septic pulmonary emboli

Pharyngitis & TNF-alpha inhibitors Extra-Tonsillar Infections: 1


• 69yo man on infliximab presents with • Epiglottitis
2 months of painful oral ulcer and 20 – Fever, sore throat
lb wt loss – Hoarseness, drooling, muffled voice, stridor
– Examine with care!
Oropharyngeal Histoplasmosis
– Lateral neck x-ray: Thumb sign
– H. influenzae type B, pneumococcus
– Can mimic oral malignancy
– Denotes disseminated disease

Extra-Tonsillar Infections: 2 Extra-Tonsillar Infections: 3


• Vincent Angina • Ludwig Angina
– AKA Trench mouth – Cellulitis of floor of the mouth
– Often starts with infected molar
– AKA acute necrotizing ulcerative
– Rapid spread with potential for airway
gingivitis obstruction
– Bad breath (mixed anaerobes) – Fevers, chills, drooling, dysphagia,
– Painful muffled voice, woody induration of
neck
– Sloughing of gingiva – Mixed oral organisms

©2024InfectiousDiseaseBoardReview,LLC
127
40Ǧ PharyngitisSyndromesandGroupAStrep
Speaker:KarenC.Bloch,MD,MPH,FIDSA,FACP

Case 3 • T 100.2F; P 126; BP 118/74.


• A 32-year-old woman is seen for a bad • HEENT: Submandibular swelling with gray
exudate coating posterior pharynx.
sore throat for 4 days
An S3 gallop is heard.
• Recently returned from her sister’s
wedding in Kazakhstan
• She c/o odynophagia, and a low-grade
fever. Today, she noted a choking
• EKG shows 1st degree AV nodal block,
sensation, prompting medical evaluation. QT prolongation, and ST-T wave changes.

Question 3 Buzz words and Visual Associations


Bull neck:
The most likely diagnosis is?
A. Streptococcal pharyngitis
B. Kawasaki disease
C. Vincent angina Grey pseudomembrane: extends onto palate or
uvula; bleeds when scraped
D. Diphtheria
E. Candida

Other clues Noninfectious Mimics


• Location, location, location • PFAPA (periodic fever, aphthous
– Almost unheard of in developed countries stomatitis, pharyngitis, and adenitis)
(vaccination) • Still’s disease
– Still an issue (high mortality) in developing world • Lymphoma
• Sore throat and myocarditis (~25%).
• Kawasaki disease
• Sore throat and neuropathies (~5%).
• Behçet disease’s
• Sore throat and cutaneous ulcer

©2024InfectiousDiseaseBoardReview,LLC
128
40Ǧ PharyngitisSyndromesandGroupAStrep
Speaker:KarenC.Bloch,MD,MPH,FIDSA,FACP

[email protected]

©2024InfectiousDiseaseBoardReview,LLC
129
130
Tuesday, August 20, 2024

41

HIV-Associated
Opportunistic Infections II

Dr. Rajesh Gandhi

©2024 Infectious Disease Board Review, LLC


COPYRIGHT NOTICE: The Copyright Act (Title 17 of U.S. Code) governs the rights attributed to owners of
copyrighted work. Under certain circumstances, educational institutions may provide copies of
copyrighted works to continuing education participants. The copies may not be copied nor used for any
other purpose besides private study, scholarship, or research. Participants should not provide electronic
or print copies of ant materials provided by the university to unauthorized users. If a participant fails to
comply with these restrictions, the participant may be held liable for copyright infringement. No further
transmission or electronic distribution is permitted.
131
132
41– HIVǦAssociatedOpportunisticInfections
Speaker:RajeshGandhi,MD

Disclosures of Financial Relationships with Relevant


HIV-Associated Opportunistic Infections II Commercial Interests

• None
Rajesh T. Gandhi, MD
Massachusetts General Hospital
Professor of Medicine, Harvard Medical School • Acknowledgement: Dr. Henry Masur for slides
7/1/2024

HIVAssociatedOpportunisticInfections:Part2 Question#1
• 50yo MwithHIV(CD440,HIVRNA600,000noton
OpportunisticCNSInfections:BrainLesions antiretroviraltherapy)presentswithfever,headache.
• NortheastUS,notravel;noanimalorTBexposures
• MRI:ringenhancinglesions;nomidlineshift
OpportunisticCNSInfections:Cryptococcal Meningitis • SerumtoxoplasmaIgG+.CSF:noWBC,normal
protein,toxoplasma(toxo)PCRpending
• Yourecommend
Mycobacterial Infections A. Brainbiopsy
B. Meningealbiopsy
C. InitiateantiͲtoxo therapy;ifnoresponsein2
weeks,brainbiopsy
ImmuneReconstitutionInflammatorySyndrome D. Vancomycin,cefepime,metronidazole

BrainLesionsinPeoplewithHIV(PWH) ToxoplasmaEncephalitis PrimaryCNSLymphoma

• MRIwithcontrastfavoredoverCT(CTwithoutcontrastmaymisslesions)
• Clues:
• Toxoplasma:multipleringenhancinglesions,ofteninvolvingbasalganglia;
serumtoxoplasmaIgGpositive(reactivation)
• PrimaryCNSlymphoma:largesolitaryfocalbrainlesion;maycrosscorpus
callosum;increasedFDGPETuptake;Bcelllymphoma;CSFEBVPCR+.CD4
cellcount<50
• Tuberculoma:considerinpersonfromendemicareawithcontrast
enhancinglesions,basilarmeningitis
• Progressivemultifocalleukoencephalopathy(PML):asymmetricnonͲ
enhancinglesionsinsubcorticalwhitematterwithoutmasseffect
Siripurapu RandOtaY,Neuroimag ClinNAm,2023 www.idimages.org Siripurapu RandOtaY,Neuroimag ClinNAm,2023

©2024InfectiousDiseaseBoardReview,LLC
133
41– HIVǦAssociatedOpportunisticInfections
Speaker:RajeshGandhi,MD

PML:Asymmetricwhitematter HIVEncephalitis:bilateral EvaluationofCNSMassLesionsinPeople


changesadjacenttocorticalribbon,
nomasseffect
symmetricwhitematterchanges withHIV/AIDS
Toxoplasmosis RadiologicResults
Lymphoma NonͲspecificalthoughcertainfeaturessuggestive
LookforExtraCNSlesionsforbiopsy
Tuberculosis
Fungus LaboratoryStudiestoPerform
Nocardia Serology:Toxo IgG
Bacterial SerumCryptococcalAgandHistoplasmaAg
BloodcultureͲ AFB
Syphilis CSFͲ CryptococcalAg
Kaposi PCR(EBV,CMV,Toxoplasma,JCvirus)
Chagoma
Glioblastoma ResponsetoEmpiricTherapy
www.idimages.org.ContributedbyDr.VinceMarconi

ToxoplasmaEncephalitis(TE) TimetoNeurologicResponseforToxoplasmaEncephalitis
35PatientswithTETreatedwithClindamycinͲ Pyrimethamine
• Causedbyprotozoan,Toxoplasmagondii
• Reactivationoflatenttissuecysts 90%

• HighestriskisinPWHwithCD4count<100 50%
• Maypresentwithheadache,confusion,weakness,fever
• Diagnosis: Response
• SerumtoxoplasmaIgGusuallypositive;negativeserologymakesTEunlikely %Pts
• MRI:ringͲenhancinglesions,ofteninvolvingbasalganglia
• CSFtoxoplasmaPCR:highspecificity(96Ͳ100%);sensitivity50Ͳ60%(negativePCR
doesnotruleoutTE)
• Empiricdiagnosis:clinical,radiographicimprovementwithantiͲtoxoplasmatherapy;
ifnoresponsebyabout2weeks,considerbrainbiopsy
https://clinicalinfo.hiv.gov/en/guidelines/hivͲclinicalͲguidelinesͲadultͲandͲadolescentͲopportunisticͲinfections/toxoplasmaͲgondii?view=full Days LuftBJ,NEJM1993

ComparedwithSulfaͲPyrimethamine,TrimͲsulfahas
TherapyforToxoplasmaEncephalitis
similarresponserate,lowertoxicity
• PreferredRegimen
• Sulfadiazinepluspyrimethamineplusleucovorin(POonly)
• Maybeunavailableorexcessivelyexpensive
• TrimethoprimͲsulfamethoxazole(POorIV)
• Inpatientswithsulfaallergy,sulfadesensitizationshouldbeattempted

• AlternativeRegimens– forthosewhocannottoleratesulfonamides
• Clindamycinpluspyrimethamine(andleucovorin)
• Atovaquone+/Ͳ Pyrimethamine(andleucovorin)

Note:InitiateantiretroviraltherapywhenpatientistoleratingantiͲtoxoplasma
therapy(usuallywithinaweekortwoafterstartingantiͲtoxoplasmatherapy)
https://clinicalinfo.hiv.gov/en/guidelines/hivͲclinicalͲguidelinesͲadultͲandͲadolescentͲopportunisticͲinfections/toxoplasmaͲgondii?view=full Prosty C,CID,2023

©2024InfectiousDiseaseBoardReview,LLC
134
41– HIVǦAssociatedOpportunisticInfections
Speaker:RajeshGandhi,MD

AdjunctiveTherapiesforToxoplasma PrimaryPreventionofToxoplasmosis
Encephalitis inPeoplewithHIV
• Corticosteroids • Indication
• Notroutine • PositiveToxoplasmaIgGandCD4<100cells/uL
• Onlyifmasseffect,increasedintracranialpressure/symptoms/signs • Drugs
• FirstChoice
• TMPͲSMX(onedoublestrengthtabletdaily)
• Anticonvulsants
• Alternatives
• Shouldnotbegivenprophylactically
• OtherdosingregimensforTMP/SMX
• Onlyifpatientshaveseizures • DapsoneͲPyrimethamine(withleucovorin)
• Atovaquone+/Ͳ Pyrimethamine(withleucovorin)

https://clinicalinfo.hiv.gov/en/guidelines/hivͲclinicalͲguidelinesͲadultͲandͲadolescentͲopportunisticͲinfections/toxoplasmaͲgondii?view=full

PrimaryPreventionofToxoplasmosisinPWH Case
• ForpatientswithCD4<200whoareonTMPͲSMXoratovaquoneforPCP • A39ͲyearͲoldfemalefromBrazil
prophylaxis presentstoEDwithaseizure.
• Nothingmoreisneeded • HIVAg/Abispositive
• CD4=20/μL
• ForpatientonAerosolPentamidineorDapsoneforPCPprophylaxis • VL=100,000copies/μL
• Ifondapsone:addpyrimethamine(plusleucovorin) • Sheisstartedonsulfadiazineand
• IfonAerosolpentamidinebecausecannottakeTMPͲSMX:not pyrimethamine.
protectedͲ • After10days,shehasnotimproved,
• Considerswitchingtoatovaquoneifseropositivefortoxo andabrainbiopsyisperformed

https://clinicalinfo.hiv.gov/en/guidelines/hivͲclinicalͲguidelinesͲadultͲandͲadolescentͲopportunisticͲinfections/toxoplasmaͲgondii?view=full

Trypanosomacruzi inBloodSmearandCSF
(Chagasic EncephalitisinPWH)

Flagellum Baderoetal,AIDSTHERAPY,4thEd
DiazGranados C,LancetID,2009

©2024InfectiousDiseaseBoardReview,LLC
135
41– HIVǦAssociatedOpportunisticInfections
Speaker:RajeshGandhi,MD

HIVAssociatedOpportunisticInfections:Part2 Question#2
• 50ͲyowomanwithHIV(CD420,HIVRNA500,000)presentswithfeverand
headache.Notonantiretroviraltherapy(ART).Diagnosedwith
OpportunisticCNSInfections:Cryptococcal Meningitis cryptococcalmeningitis
• Startedoninductiontherapy(liposomalamphotericinplus5FC)
• WhenshouldshebestartedonART?
A. StartARTatthesametimeasantiͲfungaltherapy
B. About4weeksafterstartingantiͲfungaltherapy
C. 6monthsafterstartingantiͲfungaltherapy
D. AftercompletingafullcourseofmaintenanceantiͲfungaltherapy

HIVͲAssociatedCryptococcalMeningitis
• Usuallypresentswithsubacuteonsetofconfusion,lethargy
• Neckstiffnessandphotophobiaonlyoccurin25%
• MaybeaccompaniedbynonͲCNSmanifestations:pneumonia,skinlesions,
prostateinfection
• CD4Count<100cells/uL in90%ofpatients
• CSF:minimalabnormalitiesorlymphocyticpleocytosiswithelevatedprotein.
• Openingpressure>25cmH20in60Ͳ80%ofpatients(besuretomeasure)
• SerumandCSFcryptococcalantigenpositiveinalmostallpatients.
• Bloodculturespositiveforcryptococcusin60%
https://clinicalinfo.hiv.gov/en/guidelines/hivͲclinicalͲguidelinesͲ
https://clinicalinfo.hiv.gov/en/guidelines/hivͲclinicalͲguidelinesͲadultͲandͲadolescentͲopportunisticͲinfections/cryptococcosis?view=full adultͲandͲadolescentͲopportunisticͲinfections/whatsͲnew

TherapyofCryptococcalMeningitis SingleͲdoseLiposomalAmB withFluconazole/5FC


AMBITIONTrial(n=814participants)
LiposomalAmphoB3Ͳ4mg/kgdaily Allcausemortality,week10:
plus Nodifferencebetween
2weeks Induction groups
Flucytosine*25mg/kgQID

Fluconazole800 mgpoqd** 8weeks Consolidation

Fluconazole200mgpodaily*** ш52weeksMaintenance
*5FCAssociatedwithearlier **Forclinicallystablepatientswith ***Stopafter12mtotaltherapyif
sterilizationCSF,fewerrelapses, negativeCSFcultures,dosecanbe CD4>100Ͳ 150x>3m,asymptomatic, Adverseeventslessfrequentin
improvedsurvival reducedto400mgdaily VL<50copies JarvisJNetal,NEJM,2022 singleͲdoseAmB group

©2024InfectiousDiseaseBoardReview,LLC
136
41– HIVǦAssociatedOpportunisticInfections
Speaker:RajeshGandhi,MD

DexamethasoneDidNotReduceMortalityandWas
ManagementofCryptococcalMeningitis AssociatedwithMoreAdverseEventsandDisability
• Forflucytosine,therapeuticdrugmonitoringindicated.Toxicities:marrow
suppression,hepatitis,diarrhea.Renalelimination:monitorkidneyfunction
• Successfulinductiontherapy=clinicalimprovementandnegativeCSFculture
• IndiainkandCSFCrAg frequentlypositiveatWeek2:notindicativeoffailure
• Monitoringofcryptococcalantigentitersnotrecommended
• Inpatientswithsymptomsofelevatedintracranialpressureandopening
pressure>25cm:removeCSFtoreducepressurebyhalfor<20cmH20
• LumbardrainorVPshuntmaybeneededifpressuresremainelevated
• Notroutinelyrecommended:Corticosteroids,Mannitol,Acetazolamide
NEJM,2016
https://clinicalinfo.hiv.gov/en/guidelines/hivͲclinicalͲguidelinesͲadultͲandͲadolescentͲopportunisticͲinfections/cryptococcosis?view=full

WhentoStartARTforCryptococcalMeningitis PreventingDisease
(PreͲemptiveTherapyforCryptococcalAg+/LowCD4)
COATtrial:earlyART(1Ͳ2wks)associatedwith • Recommendation:
• DHHSOIGuidelinesrecommend
highermortalitythandelayedART(5wk) • ScreenpatientswithCD4count<100withserumcryptococcalantigen
ARTinitiation4Ͳ6weeksafter
initiationofantifungaltherapy • Frequency:2.9%ifCD4<100,4.3%ifCD4<50
• PositiveserumCrAg predictsdevelopmentofdisease
Survival

• SomeexpertsstartARTearlier(at
2Ͳ4weeksafterinitiationofantiͲ
• IfPositive:PerformLPandBloodCulturestodetermineRx
fungaltherapy)basedonevolving
• IfCSFpositiveorserumLFAis>=640
datawithclosemonitoring
• Treatlikecryptococcalmeningitis/disseminated(Ampho/5FC)
• IfCSFnegative
• Treatwithfluconazole400mgor800mgx6months
BoulwareDetal,NEJM,2014
https://clinicalinfo.hiv.gov/en/guidelines/hivͲclinicalͲguidelinesͲadultͲandͲadolescentͲopportunisticͲinfections/cryptococcosis?view=full
GandhiRTetal,IASUSAGuidelines,JAMA2022 https://clinicalinfo.hiv.gov/en/guidelines/hivͲclinicalͲguidelinesͲadultͲandͲadolescentͲopportunisticͲinfections/cryptococcosis?view=full

HIVAssociatedOpportunisticInfections:Part2 TuberculosisinPWH:Highlights
• HighriskofTBreactivationinPWH:у5Ͳ10%peryear;mayoccurevenwhenCD4count>200

• ScreenPWHforlatentTB(tuberculinskintest,TST,orIGRA);ifCD4countlow,repeatTB
Mycobacterial Infections screeningafterimmunereconstitutiononART

• TBprophylaxis:positiveTST(>5mm)orIGRA;closecontactofpersonwithinfectiousTB

• WhentostartARTinpeoplewithHIVandTB
• CD4count<50:startwithin2weeksofTBtherapy
• CD4count>50:startwithin2Ͳ8weeksofTBtherapy(mostwouldstartsooner)
• TBMeningitis:highmortality;startARTonceTBmeningitisundercontrolandatleast2
weeksafterinitiatingTBtreatment;closemonitoringneeded

• PrednisonemaypreventparadoxicalTBimmunereconstitutioninflammatorysyndrome
https://clinicalinfo.hiv.gov/en/guidelines/hivͲclinicalͲguidelinesͲadultͲandͲadolescentͲopportunisticͲinfections/mycobacterium?view=fullTorok etal,CID,2011;Meintjes NEJM,2018

©2024InfectiousDiseaseBoardReview,LLC
137
41– HIVǦAssociatedOpportunisticInfections
Speaker:RajeshGandhi,MD

ExtrapulmonaryTBandHighOrganismLoadMore Question#3
CommoninPWHwithLowCD4Count • 45ͲyomanwithHIV(CD411,HIVRNA300,000)
presentswithfever,diarrheaandweightloss.
• Heisinitiatedondolutegravir+
tenofovir/emtricitabine
• Twoweekslater,hedevelopsmarkedlyenlarged
supraclavicularlymphnode
• BiopsyshowsnecrotizinggranulomasandAFB;
culturesgrowMAC
• Yourecommend:
A. StopARTandinitiatetreatmentforMAC
0 0/+ + ++ +++ B. ContinueART;initiatetreatmentforMAC ImagefromRiddellJ,JTranslational
AFBinTissue C. Startsteroidsandstopallothertreatments
Med,2007
Jonesetal,AmRevRespirDis,1993;Perlmanetal,CID,1997

MycobacteriumAvium Intracellulare Complex Diagnosis


• Epidemiology
• Ubiquitousintheenvironment • CompatiblesymptomsandsignsalongwithisolationofMACfrom
• Transmission culturesofblood,lymphnodeorothernormallysterilesites
• Inhalation,ingestion
• Riskfactors • MACmaybedetectedinrespiratoryorGItractbutroutinescreening
• CD4<50,HIVRNA>1000 ofthesesitesandpreͲemptivetherapyforMACisnotrecommended
• ClinicalManifestationsofDisseminatedMAC
• Fever,sweats,wasting,diarrhea,lymphadenopathy,hepatosplenomegaly
• Rareascauseoflungdisease
• Labs:elevatedalkalinephosphatase,anemia
https://clinicalinfo.hiv.gov/en/guidelines/hivͲclinicalͲguidelinesͲadultͲandͲadolescentͲopportunisticͲinfections/disseminated?view=full https://clinicalinfo.hiv.gov/en/guidelines/hivͲclinicalͲguidelinesͲadultͲandͲadolescentͲopportunisticͲinfections/disseminated?view=full

TreatmentforMAC
PrimaryMACProphylaxis
• SpecificTherapy
• ClarithromycinorAzithromycin+Ethambutol • PrimaryprophylaxisagainstdisseminatedMACdiseaseisNOT
recommendedifARTinitiatedimmediately
• Rifabutin,fluoroquinoloneoramikacinasa3rd or4thdrug,
particularlyifseveredisease(“highburdenoforganisms”)
• Bewaredruginteractionswithclarithromycinorrifabutin
(azithromycinhasfewerdruginteractions)
• PerformsusceptibilitytestingonMACisolate
• AntiretroviralTherapy
• Startassoonaspossibleafterdiagnosis,preferablyatthesametimeor
withinafewdaysofinitiationofantiͲmycobacterialtherapy
https://clinicalinfo.hiv.gov/en/guidelines/hivͲclinicalͲguidelinesͲadultͲandͲadolescentͲopportunisticͲinfections/disseminated?view=full https://clinicalinfo.hiv.gov/en/guidelines/hivͲclinicalͲguidelinesͲadultͲandͲadolescentͲopportunisticͲinfections/disseminated?view=full

©2024InfectiousDiseaseBoardReview,LLC
138
41– HIVǦAssociatedOpportunisticInfections
Speaker:RajeshGandhi,MD

HIVAssociatedOpportunisticInfections:Part2
ImmuneReconstitutionInflammatorySyndrome
• Definition
• Worseningmanifestationsorabrupt/atypicalpresentationofinfection
ImmuneReconstitutionInflammatorySyndrome ortumorwhenARTstarted
• Paradoxical:exacerbationofpreͲexistinginfectionortumor
• Unmasking:exacerbationofpreviouslyoccultinfection/tumor

• Timing
• Fewdaysto6monthsafterARTinitiated
• ViralloaddropmorerelevantthanCD4rise
• (betterlymphocytefunction>number)

ImmuneReconstitutionInflammatorySyndrome PathogensCommonlyAssociatedwithIRIS
• Predictors • Mycobacteriumaviumcomplex
• PretherapylowCD4orhighVL
• PriorOIorrecentinitiationoftherapyforOI
• Mycobacteriumtuberculosis
• Highpathogenload

• ClinicalFeatures • Cryptococcusneoformans
• CharacterizedbyfeversandworseningoftheunderlyingOIortumor
• May”unmask”diseaseatpreviouslyunrecognizedsiteorleadto • Reportedwithvirtuallyallopportunisticinfectionsandtumors
paradoxicalworseningofaknownOI
• Usuallyoccurs4Ͳ8weeksafterARTinitiationbutmaymanifestearlier
orlater

MycobacterialIRIS ExamplesofIRIS
PATHOGEN TYPICAL/CHARACTERISTICSOFTHEDISEASE
Cryptococcusneoformans Worseningmeningitis(mayhavebriskCSFpleocytosis)
PATHOGEN TYPICAL/CHARACTERISTICSOFTHEDISEASE
Pneumocystisjiroveci Exacerbationofpneumonia
Mycobacterium Worseninglunginfiltrates,lymphadenitis,CNS Cytomegalovirus(CMV) Vitritis
tuberculosis tuberculomas JCpolyomavirus/PML Worseningwhitematterchanges;enhancement,edema
Humanherpesvirus8/Kaposi
MAC Lymphadenitis;pulmonaryandabdominaldisease Rapidprogressionofexistingand/ornewKSlesions
Sarcoma

VaricellaͲzostervirus Dermatomalormultidermatomal zoster;rarelymyelitis

CecilTextbook(FrenchandMeintjes)
CecilTextbook(FrenchandMeintjes)

©2024InfectiousDiseaseBoardReview,LLC
139
41– HIVǦAssociatedOpportunisticInfections
Speaker:RajeshGandhi,MD

ImmuneReconstitutionInflammatorySyndrome MACIRISinPatientwithHIV
(Mycobacteriumaviumcomplex)

PreͲART PostͲART

Sereti I,IASUSATopicsinAntiviralMedicine,2019 Sereti I,IASUSATopicsinAntiviralMedicine,2019

ManagementofIRIS Summary
MultiplecausesofbrainlesionsinpeoplewithadvancedHIV;
• ReassessDiagnosis responsetoempirictherapymakesdxoftoxoplasmaencephalitis
• Evaluateforconcurrent,additionalOIsandtumors
Newguidelinesforinduction,consolidation andmaintenance
• TreatIRIS therapyforcryptococcalmeningitis;deferringARTforabout4
weeksappropriate
• ContinueART
• Continuetreatmentofidentifiedpathogen TBreactivationmay occurevenwhenCD4count>200;MAC
• NSAIDSorCorticosteroids ProphylaxisnolongerrecommendedwhenARTstartedquickly
• Prednisone20Ͳ40mgqd x4Ͳ8weeks
ImmuneReconstitutionInflammatorySyndromemayoccurafter
almostallopportunistic infectionsortumors:paradoxical
worseningorunmaskingofsubclinicaldisease

©2024InfectiousDiseaseBoardReview,LLC
140
Tuesday, August 20, 2024

42

Syndromes that Masquerade as


Infections

Dr. Karen Bloch

©2024 Infectious Disease Board Review, LLC


COPYRIGHT NOTICE: The Copyright Act (Title 17 of U.S. Code) governs the rights attributed to owners of
copyrighted work. Under certain circumstances, educational institutions may provide copies of
copyrighted works to continuing education participants. The copies may not be copied nor used for any
other purpose besides private study, scholarship, or research. Participants should not provide electronic
or print copies of ant materials provided by the university to unauthorized users. If a participant fails to
comply with these restrictions, the participant may be held liable for copyright infringement. No further
transmission or electronic distribution is permitted.

141
142
42– SyndromesthatMasqueradeasInfections
Speaker:KarenCBloch,MD,MPH,FIDSA,FACP

Disclosures of Financial Relationships with Relevant


Syndromes that Masquerade as Infections Commercial Interests

• None
Karen C. Bloch, MD, MPH, FIDSA, FACP
Professor, Division of Infectious Diseases
Vanderbilt University Medical Center
7/1/2024

ID Board Content Mimics


Medical Content Category % of exam

Bacterial Diseases 27%


HIV Infection 15%
• Many conditions masquerade as infections.
Antimicrobial therapy 9% – Fever almost universally present
Viral Diseases 7%
– Sometimes focal abnormality
Travel and Tropical Medicine 5%
Fungi 5% • Cellulitis vs stasis dermatitis
Immunocompromised Host (non HIV) 5% • Viral vs Organizing Pneumonia
Vaccinations 4% • Lymphadenitis vs Lymphoma
Infection Prevention and Control 5% VS
General Internal Medicine, Critical Care & Surgery 18%
Total 100%

Test taking tip Test taking tip


• Just as for infections, look for “buzz words” • For infections:
and “hooks” If I say “rabbit”, you say…..

• For infections:
If I say “skinned rabbit”, you say…..
(pulmonary) TULAREMIA

©2024InfectiousDiseaseBoardReview,LLC
143
42– SyndromesthatMasqueradeasInfections
Speaker:KarenCBloch,MD,MPH,FIDSA,FACP

Test taking tip Test taking tip


I say “Chitterlings” (aka chitlins, aka hog I say “chitterlings”
intestines)
You say…..
You say…..

YERSINIA (gastroenteritis)

Test taking tip Test taking tip


I say ”Bull’s-eye rash” I say ”Bull’s-eye rash”

You say….. You say…..


Lyme disease
(or Erythema migrans or STARI)

My Approach to Mimics Examples


• Think like an Internist
• The key is recognition, not treatment
• This talk will emphasize illustrative cases
• Goal is to cover lots of non-infectious
diseases rather than in-depth discussion
using buzz words for easy recognition!

©2024InfectiousDiseaseBoardReview,LLC
144
42– SyndromesthatMasqueradeasInfections
Speaker:KarenCBloch,MD,MPH,FIDSA,FACP

Question 1 Question 2
A young man has oral and genital ulcers. You Sweet Syndrome is most likely to occur in a
suspect Behçet’s disease. Which of the following patient with which of the following illnesses?
is most consistent with that diagnosis?
A. Evanescent, salmon-colored rash A. Ulcerative colitis
B. High ferritin B. Adult-onset Still’s Disease
C. Saddle nose deformity C. Acute leukemia
D. Pustule at site of venipuncture D. Systemic lupus
E. Posterior cervical adenopathy
E. Ankylosing spondylitis

Question 3 But this being boards…..


A patient has a slowly enlarging ulcerated skin lesion on
his shin after being hit by a soccer ball. Which of the
following is the most likely diagnosis?

A. Pyoderma gangrenosum
B. Ecthyma gangrenosum
C. Erythema nodosum
D. Sweet Syndrome
E. Behçet’s disease
To optimize learning : CLOSE THE SYLLABUS

Case 4 Case 4
• 26yo man presents with a 1-month h/o fever, • Exam: • Labs
night sweats and fatigue. He was evaluated by – Vitals: – CBC
his PCP 2 weeks ago with a positive monospot. • T=38.4๦C, HR=118 bpm • WBC=2.7, plt=53
– No lymphadenopathy • Normal H/H
• But fevers have persisted, and he has lost 10 lbs
– Palpable spleen tip – Normal Cr
since the positive test.
– No rash – AST/ALT=120/200
• He lives in Indiana with his wife and 2 yo son,
– Alk phos=494, bili=1.9
who are healthy. They have 2 cats.
– Ferritin=35,148 mg/ml

©2024InfectiousDiseaseBoardReview,LLC
145
42– SyndromesthatMasqueradeasInfections
Speaker:KarenCBloch,MD,MPH,FIDSA,FACP

Question 4 Hemophagocytic Lymphohistiocytosis

• AKA HLH
• What is the most appropriate next study?
• Immune activation syndrome
A. Flow cytometry of whole blood – Primary (Peds): Familial due to genetic mutation
B. ANA profile – Secondary (Adult or peds):
C. CMV PCR • Infections (EBV or other herpes group viruses, HIV,
histoplasmosis, Ehrlichia, COVID-19 etc)
D. Soluble IL-2 receptor level
• Malignancy (lymphoma, leukemia)
E. Toxoplasma titer

HLH: Diagnostic Criteria HLH Clues


• At least 5 of the following: • EBV or other infection with progressive
– Fever
symptoms
– Splenomegaly
– Cytopenias (any line)
– Hypertriglyceridemia (>3mmol/L)
– Ferritin >500 mcg/mL
• Massively elevated ferritin
– Elevated soluble IL-2 receptor (aka CD25)
– Low NK cell activity
– Hemophagocytosis on pathology • Cytopenia with negative ID evaluation

Case 5 Physical Exam


• T=104.2๦ F.
• A 39-year-old woman is admitted for fever for 3 • Tender cervical LAN appreciated.
weeks, associated with diffuse arthralgias • Spleen tip is palpable.
involving the knees, wrists and ankles. • Both knees are swollen & painful.
• A severe sore throat was present during the first • A rash is present on the trunk and
week of the illness but has resolved. extremities, most prominently
under the breasts and in the area
of her underwear waistband.

©2024InfectiousDiseaseBoardReview,LLC
146
42– SyndromesthatMasqueradeasInfections
Speaker:KarenCBloch,MD,MPH,FIDSA,FACP

• Labs:
Ferritin 3600 ng/ml (nl 40-200)
Question 5
WBC 32,200 (89% neutrophils)
AST and ALT 3x normal • The most likely diagnosis is?
ESR and CRP 5x normal A. Lymphoma
ANA and RF negative B. Adult Still’s Disease
Throat and blood cultures are so far negative
C. Acute Rheumatic Fever
• On afternoon rounds with the attending, the fever
D. Cryoglobulinemia
has resolved with Tylenol and the rash is no
longer present. E. Kikuchi Disease

Adult Still’s Disease (Adult Onset JRA) Adult Still’s Disease


Yamaguchi Criteria: (5 features with 2 major criteria) • Buzz words and associations:
evanescent, salmon-colored rash
Major: Minor:
1. Fever >39๦C for >1week 1. Sore throat
2. Arthritis/arthralgia >2 wks
3. Typical rash (during
2. Lymphadenopathy
3. Lg Liver or spleen
=
febrile episodes) 4. Abnl LFTs
4. Leukocytosis >10K with 5. Negative ANA & RF
>80% PMNs. Koebner phenomenon (rash at pressure sites)

Case 6 • Left eye is inflamed and


there is a hypopyon.
• A 24-year-old man was referred by the ED for evaluation • Numerous painful ulcers • A 6mm papulo-pustular
of ulcers of the mouth and penis. He was born in Japan on the oral mucosa. lesion is present in the
and is in the U.S. to attend graduate school. • There is a 0.5cm ulcer on right antecubital fossa
the penis. where they drew blood
• He has a history of recurrent painful oral ulcers for 3-4 yesterday in the ED.
years. Four days ago, he developed a painful ulcer on
the penile shaft. He takes no medicines and denies
sexual contact for the past 5 years.

©2024InfectiousDiseaseBoardReview,LLC
147
42– SyndromesthatMasqueradeasInfections
Speaker:KarenCBloch,MD,MPH,FIDSA,FACP

Question 6 Behçet’s disease


• The most likely diagnosis is? Pleomorphic vasculitis diagnosed clinically
A. Syphilis • Recurrent oral ulcers (>3 per year) PLUS 2 of the following
1) recurrent genital ulcers
B. Behçet’s disease 2) eye (uveitis, retinitis, hypopyon)
3) skin lesions, esp pathergy (red papule 24- 48 hours after needlestick)
C. Herpes simplex virus infection
D. Sarcoidosis • Less common manifestations (oral ulcers PLUS…)
– GI disease (abdomenal pain, bloody diarrhea)
E. Cytomegalovirus infection – Aseptic meningitis
– Arterial and venous thrombosis

Behçet’s disease Case 7


• Ulcers is the buzz word, but the • A 38-year-old woman with AML is admitted with fever.
She underwent induction chemotherapy 2 weeks prior,
trick is differentiation from complicated by neutropenic fever that resolved with
infectious causes (HSV, marrow recovery.
coxsackie, etc) • She presents with a 1-day history of fever without
VS localizing symptoms.
• Additional Clues • Exam: T 101.4; P 98, Otherwise unremarkable.
Recurrence • CBC showed a white blood cell count of 12,250 with
20% bands.
Ocular findings
Pathergy (needle or IV site)

Hospital Day 2: Hospital Day 3:


Fever persists; some of the papules develop
• Fever persists despite broad a plaque-like appearance
spectrum antibiotics.

• Interval development of raised, Hospital Day 4:


red-purple, tender papules and skin biopsy with dense
nodules on her face, neck and perivascular infiltrates of
the dorsum of her hands. neutrophils without evidence of
vasculitis; stains for organisms
negative.

©2024InfectiousDiseaseBoardReview,LLC
148
42– SyndromesthatMasqueradeasInfections
Speaker:KarenCBloch,MD,MPH,FIDSA,FACP

Question 7 Sweet Syndrome


• AKA acute febrile neutrophilic dermatosis
• Which is the most likely diagnosis? • Three variants:
– Idiopathic or “classical” >50% (IBD, post viral illness, preg, etc)
A. Ecthyma gangrenosum – Malignancy associated~20% (may precede dx, AML most frequent)
– Drug induced-G-CSF most common, antibiotics
B. Pyoderma gangrenosum
• Fever and Rash universally present
C. DRESS • Rarely oral ulcers or extra-cutaneous disease characterized
D. Leukemic infiltrates by neutrophilic infiltrate on path
• Lab tests with leukocytosis with left shift, inc ESR & CRP
E. Sweet syndrome • Path diagnostic—Neutrophilic infiltrate without vasculitis

Skin Lesions in Sweet Syndrome Sweet Syndrome


• Lesions appear abruptly
• Buzz words and associations:
and usually tender. Fever and a rash
• May be single or multiple, Neutrophilia (peripheral and on path)
often involving dorsum of
hand.
• Red, violaceous, or yellow
center
• Nodular or plaque-like • Be suspicious in patients with malignancy (esp
• Central umbilication with
AML), IBD, recent URI, vaccination, pregnancy, or
target appearance colony stimulating factor use in preceding 2 weeks

Case 8 • Exam:
T 100.2
• A 33-year-old recent immigrant from Central
America is seen for a chronic ulcer of the leg. Abdo pain to palpation
• The ulcer has progressively enlarged over 3 months Skin lesion
after he bumped his leg on a table
• There has been no response to oral antibiotics. • Labs:
• For the past year he has been troubled by an “upset WBC 11,150 (2% eos)
stomach”. On further probing, he describes ESR=79, CRP=110
intermittent abdominal cramps, frequent diarrhea;
and, on 2 occasions, blood in the stool. BMP normal
Chest x-ray normal

©2024InfectiousDiseaseBoardReview,LLC
149
42– SyndromesthatMasqueradeasInfections
Speaker:KarenCBloch,MD,MPH,FIDSA,FACP

Question 8 Pyoderma gangrenosum


Which one of the following is the most likely • Another neutrophilic dermatosis
diagnosis? – Indolent, fever rare (vs Sweet)

A. Ulcerative colitis • Papule starts at site of often trivial trauma, progressing to


a painful ulcer with violaceous border and necrotic base
B. Cutaneous leishmaniasis • >50% of cases occur with systemic illness (but may
C. Amebic colitis precede dx, or occur independent of flares)
D. Cutaneous blastomycosis – IBD (Ulcerative colitis>Crohn’s)
E. Squamous cell cancer – Inflammatory arthritis
– Solid organ or heme malignancy

Pyoderma Gangrenosum Case 9


• Buzzwords & Hooks • A 79-year-old woman is seen for 3
– Minor trauma weeks of fever and fatigue.
(Pathergy) frequent • One week earlier she developed
– Painful, progressive jaw discomfort when chewing food
undermined ulcer with and had a brief episode of double
violaceous edges vision.
and necrotic base • One month ago, she attended a
– Associated with IBD, luau and ate roast suckling pork
arthritis, neoplasm prepared over an open fire.

• Exam: Question 9
T 102.2, P 104, BP 124/84
Slight tenderness over left scalp Which of the following is most likely to be diagnostic?
mitral regurgitant murmur A. Anti-neutrophil cytoplasmic antibody (ANCA)
rest of exam normal B. Taenia solium serology
• Labs: C. Blood cultures
Hb 9.8; WBC 9800, normal diff D. Arteriography
UA normal
basic metabolic panel normal E. Temporal artery biopsy
sedimentation rate 147

©2024InfectiousDiseaseBoardReview,LLC
150
42– SyndromesthatMasqueradeasInfections
Speaker:KarenCBloch,MD,MPH,FIDSA,FACP

Giant Cell Arteritis Giant Cell Arteritis


• Extracranial branches of the carotid.
Buzz words & Associations:
• Clinical findings:
– Fever (almost exclusively older adults)
– Scalp or TA tenderness, jaw claudication
– amaurosis fugax or sudden vision loss
• Marked inc ESR/CRP suggestive, FUO in a patient >50 years PLUS
TA biopsy diagnostic – scalp or TA tenderness
• Immediate steroid therapy indicated – Visual symptoms (diplopia or transient visual loss)
if visual changes to prevent – jaw or tongue fatigue or pain while chewing
blindness – ESR >100

Overlap of GCA and PMR Takayasu Arteritis


• Large vessel vasculitis
– Aorta, carotids and pulmonary arteries.
• ~50% patients with GCA have • Buzz words and associations:
concomitant PMR – Young woman (>80%), Asian ancestry
– Subacute onset of fever, weight loss, arthralgias and
• Consider GCA in febrile patient myalgias
with Buzz words for PMR…. – Carotidynia (pain with palpation), decreased pulses
– morning stiffness in proximal – Extremity claudication; visual changes; TIAs
muscles of shoulder and hip girdle • Dx: Arteriography
– Gel phenomenon (stiffness with
inactivity)

Case 10 Exam:
• A 37-year-old female presents with fever and joint T 100.5; Pulse 72; BP 110/70
pain. She is a long-distance runner and in Bilateral synovial thickening of ankles with warmth and
excellent health. tenderness to passive movement
Skin exam with painful pre-tibial nodules
• Three weeks prior she noted R knee pain after a
long run. She was treated with a steroid injection Labs:
with transient improvement, but subsequently WBC 8.8 (76% segs)
developed bilateral ankle pain and redness. She CRP=167
notes subjective chills and sweats. Uric acid=4.4
• She recalls several tick bites in the last 2 months RF <15, Anti-CCP Ab negative

©2024InfectiousDiseaseBoardReview,LLC
151
42– SyndromesthatMasqueradeasInfections
Speaker:KarenCBloch,MD,MPH,FIDSA,FACP

Sarcoidosis
Question 10
Which of the following is most likely to be • Extra-pulmonary disease in ~1/3 of cases
diagnostic? • Lofgren Syndrome
– Only form of sarcoid that is a clinical diagnosis
A. Chest x-ray – Triad of hilar LAN, acute arthritis, EN
– Women, ankles (>90%), fevers common
B. Serology for Borrelia burgdorferi
• BUZZ WORDS
C. Urine Histoplasma antigen – Hilar LAN, EN, uveitis, parotid enlargement
D. Arthrocentesis – Non-caseating granulomas
– Aseptic meningitis with basilar enhancement
E. Skin biopsy

Erythema nodosum Erythema nodosum


• No cause >50% of cases • NO cause >50% of cases
• Drugs: sulfonamides, penicillins • Drugs: sulfonamides, Penicillins
• Oral contraceptives • Oral contraceptives
• Sarcoid (Lofgren’s syndrome) • Sarcoid (Lofgren’s syndrome)
• Ulcerative colitis (or Crohn’s) • Ulcerative colitis (or Crohn’s or Bechet’s)
• Microbes: • Microbes:
• EBV, Hep B/C • EBV, Hep B/C
• Streptococci, Bartonella, TB • Streptococci, Bartonella, TB, Mycoplasma
• Endemic fungi • Endemic fungi

Case 11 • Exam:
• A 19-year-old Iraqi immigrant is hospitalized for T 102.2; pulse 114; no rash
2-day history of fever and abdominal pain Abdominal guarding, rebound
tenderness, hypoactive bowel sounds.
• He has had similar episodes on at least 3 • Labs:
previous occasions over the past 7 years. At WBC 16,650; UA normal
the first episode he underwent appendectomy; BMP & LFTs normal
the appendix path was normal. Subsequent no occult blood in stool
episodes resolved spontaneously after 2-3 days. CT of abdomen and pelvis normal

©2024InfectiousDiseaseBoardReview,LLC
152
42– SyndromesthatMasqueradeasInfections
Speaker:KarenCBloch,MD,MPH,FIDSA,FACP

Question 11 Familial Mediterranean Fever


• Auto-inflammatory disease causing a periodic fever
The most likely diagnosis is: syndrome
– Others: PFAPA, TRAPS, hyperimmunoglobulin D
A. Hereditary angioneurotic edema
• Recurrent attacks of fever & serositis (peritonitis,
B. Familial Mediterranean fever pleuritis, arthritis) manifesting as pain.
C. Systemic lupus erythematosus • Dx: Genetic testing
• Buzz words and associations: -
D. Crohn’s disease – Periodic fever episodes (PLUS…)
E. Acute intermittent porphyria – Serositis
– Mediterranean ancestry

Case 12 • Exam:
T 101.4; unilateral anterior and posterior cervical
• A 26-year-old medical enlarged lymph nodes, firm, and mildly tender.
student presents with fever Otherwise, unremarkable.
and cervical adenopathy.
• Labs:
• She was completely well Hb 13.9; WBC 4,900 (9% atypical lymphocytes)
until 9 days ago when she Basic metabolic panel normal
had the acute onset of fever Chest x-ray normal
and vague neck discomfort. ESR=72
She had no sore throat and
no dental or scalp problems. Monospot: Negative

• Serologic studies:
EBV IgM negative Question 12
CMV, Toxo, Bartonella negative Which one of the following is the most likely
RF, ANA, ds-DNA negative
diagnosis?
• Lymph node pathology: A. Cat Scratch Disease
Necrotizing lymphadenitis with
histiocytic infiltrate and phagocytosed B. Adult Still’s Disease
debris. C. Sarcoidosis
D. Kikuchi Disease
Stains for AFB and fungi negative.
E. Non-Hodgkin Lymphoma

©2024InfectiousDiseaseBoardReview,LLC
153
42– SyndromesthatMasqueradeasInfections
Speaker:KarenCBloch,MD,MPH,FIDSA,FACP

Kikuchi Disease Kikuchi Disease


• AKA acute necrotizing histiocytic lymphadenitis • Diagnosis by pathology:
• Self-limited condition of unknown cause – necrotizing histiocytic infiltrate (not neutrophils)
• Typically occurs in young women and fragments of nuclear debris.
• Fever & cervical LAN (esp posterior, usually unilateral).
• Rarely: morbilliform rash, diffuse LAN, aseptic • Buzz words and associations:
meningitis, uveitis. – Acute onset fever and cervical adenopathy in young
woman
• Leukopenia and atypical lymphocytes in 25% of cases.
– Atypical lymphocytes (mono-like syndrome)
– Path: necrotizing adenitis with histiocytosis

Case 13 • Exam: Temp 101.5; RR 24


• A 41-year-old woman is seen for fever,
worsening respiratory symptoms, and a rash. • Diffuse wheezing; palpable purpura with
• She has long-standing asthma with frequent nodules on elbows and legs.
exacerbations
• She uses an inhaler several times a day and • Labs: WBC 15,230 (22% eosinophils).
was recently placed on a leukotriene receptor
antagonist. She is being tapered off steroids • CT scan: bilateral peripheral infiltrates.
which she has taken for several months.
• Skin nodule biopsy: granulomas

Question 13 EGPA
• AKA Churg-Strauss Syndrome
Which one of the following is the most likely • Multisystem, small vessel vasculitis with allergic rhinitis,
diagnosis? asthma, peripheral and lung eosinophilia.
• Most often involves lung and skin, but can involve heart,
A. Strongyloidiasis
GI tract, and nervous system.
B. Disseminated histoplasmosis • Presence of blood eosinophilia and peripheral pulmonary
C. Sarcoidosis infiltrate in setting of difficult to control asthma.
D. Allergic bronchopulmonary aspergillosis • Tapering of steroids often “unmasks” EGPA
• May be p-ANCA positive.
E. Eosinophilic granulomatosis with polyangiitis

©2024InfectiousDiseaseBoardReview,LLC
154
42– SyndromesthatMasqueradeasInfections
Speaker:KarenCBloch,MD,MPH,FIDSA,FACP

EGPA Case 14
• A 38-year-old man is seen for a 6-week history of
• Buzz words and associations: cough, intermittent fever and night sweats.
– Longstanding asthma • He has had nasal stuffiness for 4-5 months with
– New infiltrates and eosinophilia (>10%) as occasional epistaxis.
steroids tapered. • He lives in Philadelphia, and 6 months ago
– Rash (tender nodules on extensor surfaces, traveled to Cincinnati on business.
purpura, ecchymosis, necrosis) • He has no pets and takes only an OTC
– Fever UNCOMMON (until late) decongestant. He denies use of illicit substances,
including intranasal cocaine.

Exam: Question 14
• T 100.2; RR 18;
Nasal deformity with perforation of septum • The diagnosis will most likely be supported
Lungs clear; rest of exam normal. by which of the following?
A. c-ANCA
• Labs:
B. Anti-glomerular basement membrane Ab
WBC 6,900 with normal differential;
UA 30-50 RBC; BMP normal C. Urine toxicology screen
Chest CT: bilateral nodules with cavitation. D. Angiotensin converting enzyme (ACE)
E. Pulmonary angiogram

Granulomatosis with polyangiitis (GPA) Granulomatosis with polyangiitis


• Dx:
• Systemic vasculitis of medium Suggestive: Positive ANCA (~85% sensitivity)
and small arteries. IFA: c-ANCA.
• Primarily involves upper and ELISA: anti-proteinase 3 (PR3-ANCA)
lower respiratory tracts and Diagnostic: Biopsy
kidneys.
Buzz words and associations:
• Variably involves joints,
Nasal symptoms (Saddle nose and perforation)
cartilage, eyes, skin, and
Lung nodules
nervous system.
Respiratory and renal findings (hematuria)

©2024InfectiousDiseaseBoardReview,LLC
155
42– SyndromesthatMasqueradeasInfections
Speaker:KarenCBloch,MD,MPH,FIDSA,FACP

Case 15 Case 15
• A 42-year-old man is seen for his third episode
of cellulitis of the external ear. Exam:
Afebrile
• Two previous episodes involving the same ear,
Left auricle is inflamed and
2 and 5 months ago, responded very slowly to
tender, ear lobe is spared.
antibiotics.
• He has a several year history of chronic nasal He has a saddle-nose deformity;
stuffiness and had an episode of knee arthritis in the nasal mucosa is normal.
the past year but is otherwise well.
Labs: CBC normal

Question 15 Relapsing Polychondritis


The most likely diagnosis is? --Immune-mediated condition.
A. Malignant otitis externa --Inflammation of cartilaginous
B. Leprosy structures, particularly ears, but
C. Granulomatosis with polyangiitis also nose, eyes, joints, and
D. Relapsing polychondritis airways.
E. Congenital syphilis --Clinical diagnosis.

Saddle-nose Deformity Relapsing Polychondritis


• Buzz words and associations:
– Granulomatosis with polyangiitis
Recurrent “cellulitis” (cartilage inflammation)
– Relapsing polychondritis Saddle-nose
– Lepromatous leprosy Cauliflower ear
– Congenital syphilis Sparing of ear lobe
– Leishmaniasis Parasternal joint involvement
– Cocaine use

©2024InfectiousDiseaseBoardReview,LLC
156
42– SyndromesthatMasqueradeasInfections
Speaker:KarenCBloch,MD,MPH,FIDSA,FACP

[email protected]

©2024InfectiousDiseaseBoardReview,LLC
157
158
Tuesday, August 20, 2024

43

Non‐AIDS‐Defining Complications of
HIV/AIDS

Dr. Michael Saag

©2024 Infectious Disease Board Review, LLC


COPYRIGHT NOTICE: The Copyright Act (Title 17 of U.S. Code) governs the rights attributed to owners of
copyrighted work. Under certain circumstances, educational institutions may provide copies of
copyrighted works to continuing education participants. The copies may not be copied nor used for any
other purpose besides private study, scholarship, or research. Participants should not provide electronic
or print copies of ant materials provided by the university to unauthorized users. If a participant fails to
comply with these restrictions, the participant may be held liable for copyright infringement. No further
transmission or electronic distribution is permitted.

159
160
43Ǧ NonǦAIDSǦDefiningComplicationsofHIV/AIDS
Speaker:MichaelSaag,MD

Lecture Title

• Disclosures of Financial Relationships with Relevant


Non AIDS-Defining Complications of HIV/AIDS Commercial Interests

- None

Michael S. Saag, MD
Professor of Medicine
University of Alabama at Birmingham

7/1/2024

CASE 1 QUESTION #1
` 55 year old man presents with R hip pain Which if the following is the most likely underlying cause
` H/o COPD requiring steroids frequently of his hip pain?
` HIV diagnosed 17 years ago
` On TDF / FTC / EFV for 10 years; originally on IND / AZT / A. Osetonecrosis of Femoral Head
3TC
B. Fanconi䇻s syndrome
` Initial HIV RNA 340,000; CD4 43 cells/ul
` Now HIV RNA < 50 c/ml; CD4 385 cells/ul C. Vitamin D deficiency
` Electrolytes NL; Creat 1.3; Phos 3.5 Ca 8.5 D. Tenofovir bone disease
` Mg 2.1, alk phos 130; U/A neg
` R Hip film unremarkable E. Hypogonadism

Osteonecrosis Avascular necrosis in HIV


` Reported prior to the HAART era; increasing in HAART era.
` Rates of AVN 4.8/1000 person years >> general population.
` Age ~ 35 yrs
` Male predominance
` H/o IDU
` Increased duration of HIV
` Low CD4
` Elevated lipids
This image demonstrates a classic segmental
area of osteonecrosis with a dark line ` Glucocorticoid steroid use
denoting the border between dead bone and
living bone. ` Alcohol use
M. Levine. Ostoenecrosis of the hip- emedicine.com
Monier et al, CID 2000;31:1488-92, Moore et al, AIDS 2003

©2024InfectiousDiseaseBoardReview,LLC
161
43Ǧ NonǦAIDSǦDefiningComplicationsofHIV/AIDS
Speaker:MichaelSaag,MD

CASE 2 CASE 2: Exam


` 46yowf c/o (CD4 582, VL <50 c/ml) c/o 1 week cramps
in calves, tingling in hands, feet ` VS: T 98.2 P 79 BP 112/73
` Today awoke and can䇻t move except hands/feet ` RR 16, O2 sat 97%
` No F/C, chest pain, SOB, incontinence
` Pertinent findings
` + chronic diarrhea 4x/day
` Chronic fatigue, poor appetite ` Neuro: CNII-XII intact, strength 1+ all
` Meds extremities except 4+ hand/wrist and ankles.
` TDF/FTC/EFV (2008), on TDF/FTC/Elv/cobi since 2014
` zoloft, buproprion, norco, prilosec, trazodone, pravachol ` Nl reflexes. Alert, oriented.
ibuprofen

CASE 2: Labs QUESTION #2


137|116|5 Gluc 83 Which of the following is the most likely
1.6 |18 |1.0 AG 3 diagnosis?
A. Cocaine toxicity
Ca 8.3 Phos 1.8 Mg 2.1 B. Nucleoside-induced myopathy (ragged red fiber
Lactate 1.5 CK 186 disease)
UDS +cocaine/benzo/opiate C. Serotonin Syndrome
UA: 1.015 pH 6.5 2+ pro D. Statin toxicity
Neg: gluc/ketones E. Fanconi䇻s syndrome

Fanconi syndrome CASE 3


` Type II RTA ` 35 year old man presents with complaints of increasing
fatigue, headache, SOB / DOE
` Generalized proximal tubule dysfunction
` HIV diagnosed 4 mos ago with PCP; intolerant to TMP/SMX
` Hypophosphotemia, renal glucosuria, ` Now on TAF / FTC / BIC + PCP Prophylaxis with Dapsone
hypouricemia, aminoaciduria ` Claims adherence to all meds;
` Not all have present at once
䇾Doesn't miss a dose!䇿
` Osteomalacia can occur ` Normal PE
` Recovery is the rule; can take months ` Pulse Ox 85%; CXR no abnormalities
` ABG: 7.40 / 38 / 94/ 96% (room air)

©2024InfectiousDiseaseBoardReview,LLC
162
43Ǧ NonǦAIDSǦDefiningComplicationsofHIV/AIDS
Speaker:MichaelSaag,MD

QUESTION #3 Hemoglobin and Methemoglobin


Which of the following is the most likely Hemoglobin Methemoglobin
underlying cause of his symptoms?
O2
A. Recurrent PCP O2 O2 O2
Fe2+ Fe2+ Fe3+ Fe2+
B. IRIS Reaction
O2 O2
C. Drug toxicity Fe2+
O2 O2
Fe2+ Fe2+
Fe3+
D. Pulmonary Embolus
RBC RBC
E. Patent Foramen Ovale

CASE 4: REPRIEVE Study (started in 2015)


In a 40 yo male PWH non-smoker, non-diabetic with LDL ` 7769 HIV+ men and women (30%) age 40 – 70 yo
cholesterol 125 mg/dl, HDL 45 mg/dl, with an ASCVD ` Low to moderate risk for statin use
score of 1.5%, should he be started on a statin ? ` All patients on ARV Rx with CD4 > 100 cells / ul
A. Yes
` Randomized to pitavastatin vs placebo
B. No
` Study stopped by DSMB
C. Not sure
` Findings:
` 35% reduction in CV events

CASE 5 QUESTION #5
` 25 year old black woman presents with fatigue Which of the following is the most likely cause of her
` History of IV Heroin use; intermittently takes TDF/FTC PreP renal failure?
` Exam no edema A. Volume depletion / ATN
` Work up in ER shows creatinine 8.4 BUN 79; mild
B. Heroin Associated Nephropathy
anemia; mild acidemia
` In ER 10 weeks earlier; normal renal function C. HIVAN
` U/A high grade proteinuria D. Membranous glomerulonephritis
` US of kidneys: Normal to increase size; no obstruction E. Tenofovir Toxicity (PrEP)
` Rapid HIV test positive

©2024InfectiousDiseaseBoardReview,LLC
163
43Ǧ NonǦAIDSǦDefiningComplicationsofHIV/AIDS
Speaker:MichaelSaag,MD

Bonus Question #1: CASE 6


In a patient with HIV Associated Nephropathy, which of ` 55 year old man presents with complaints of fever /
the following is the most effective intervention to volume depletion
prevent progression to ESRD? ` HIV diagnosed in ER on rapid test
A. An ACE inhibitor ` Lymphadenopathy / splenomegaly / few petechiae /
B. Corticosteroids Oriented X 3
` HIV RNA 340,000; CD4= 3 cells/ul
C. High Molecular Weight Dextran
` On no medications
D. Antiretroviral Therapy
Hb 8.2 gm/dl; Plt count 21,000; Creatinine 2.0
E. A calcium channel blocker
Rare schizocytes on peripheral blood smear

QUESTION #6 CASE 7
Which of the following is the most effective ` 45 year old recently diagnosed with HIV
intervention to increase the platelet count? ` HIV RNA 140,000; CD4= 230 cells/ul
A. Splenectomy ` Baseline labs:
` Hb 11.2 gm/dl; AST 310 / ALT 120
B. Corticosteroids
140|101 | 5 Gluc 100
C. Plasmapheresis
4.2 | 28 |1.1 eGFR = 65 ml/min
D. Ethambutol + Azithromycin ` Started on TAF/FTC+ Dolutegravir; No other medications
E. Antiretroviral Therapy ` Returns 4 weeks later, labs unchanged except creatinine
now 1.3 mg/dl (eGFR 55)

Tenofovir and COBI Interact with


QUESTION #7 Distinct Renal Transport Pathways
Which of the following is the most likely cause of her Anion Transport Pathway Cation Transport Pathway

increased creatinine / reduced eGFR?


OAT1
A. Glomerular lesion
ATP
MRP2 OCT2 MATE1 H+
OAT3
B. Proximal Tubule damage
Tenofovir Creatinine
COBI

C. Proximal Tubule inhibition Blood


(Basolateral)
Active Tubular Secretion Urine
(Apical)
Blood
(Basolateral)
Active Tubular Secretion Urine
(Apical)

D. Distal Tubule damage


The active tubular secretion of tenofovir and the effect of
E. Distal Tubule inhibition COBI on creatinine are mediated by distinct transport
pathways in renal proximal tubules
Ray A, et al. Antimicro Agents Chemo 2006;3297-3304
Lepist E, et al. ICAAC 2011; Chicago. #A1-1724

©2024InfectiousDiseaseBoardReview,LLC
164
43Ǧ NonǦAIDSǦDefiningComplicationsofHIV/AIDS
Speaker:MichaelSaag,MD

Changes in eGFR CASE 8


` 26 year old presents with cryptococcal meningitis and
newly diagnosed HIV (Rx with AMB +5FC; to fluconazole)
` HIV RNA 740,000; CD4= 23 cells/ul
` Baseline labs:
` CSF: 2 lymphocytes / protein 54 / glu 87 (serum 102)
OP = 430 mm H20
Started on TAF/FTC /Bictegravir at week 2
` Returns 6 weeks later, Fever 103 and a mass in supra-
clavicular region (3 x 4 cm)
Walmsley, et al. N Engl J Med. 2013;369:1807-18.

QUESTION #8 IRIS
Which of the following is the most likely cause of the
` Immune Reconstitution Inflammatory Syndrome
new mass?
` Occurs 4 – 12 weeks after initial ARV
A. B Cell Lymphoma
administration
B. Multicentric Castleman’s Disease
` Most often in patients with advanced HIV infection
C. IRIS reaction to cryptococcus
` High viral load / low CD4 count
D. Mycobacteria Avium Complex
` TB, MAC, crypto, PML, KS are most common OIs
E. Bacterial Abscess from prior PICC line
` Is NOT related to type of ARV therapy

CASE 9 QUESTION # 9
• 48 yo Male presents with newly diagnosed HIV infection Which of the following will most likely be present
• Asymptomatic
on his 3 month visit from use of dolutegravir:
• Initial: HIV RNA 160,000 c/ml
A. Morbilliform skin rash (extremities)
CD4 count 221 cells/ul
• Other labs are normal; Started on ARV Rx with DTG + B. 3 kg weight gain
TAF/FTC C. Mild cognitive impairment
• Returns for a 3 month follow up visit D. Depression
• HIV RNA < 20 c/ml; CD4 390 cells/ul
E. Anemia

©2024InfectiousDiseaseBoardReview,LLC
165
43Ǧ NonǦAIDSǦDefiningComplicationsofHIV/AIDS
Speaker:MichaelSaag,MD

Change in Weight Overtime – NA-ACCORD Change in Weight Overtime – NA-ACCORD


Bourgi et al CROI 2019 Bourgi et al CROI 2019

Change in Weight Overtime – NA-ACCORD


Bourgi et al CROI 2019
CASE 10
• 48 yo Male presents with newly diagnosed HIV
infection
• Asymptomatic except for weight loss / fatigue
• Initial: HIV RNA 160,000 c/ml
CD4 count 221 cells/ul
• Other labs are normal; Started on ARV Rx
• Returns for a 3 month follow up visit
• HIV RNA < 20 c/ml; CD4 390 cells/ul

QUESTION # 10 PARTNERS Study


Assuming he remains undetectable, you tell ` 548 heterosexual and 972 discordant gay couples
followed up to 8 years
him that his risk of transmitting HIV to his
` Seropositive partner had VL < 200 c/ml
seroneg partner via sex is:
` 77,000 sexual acts without condoms
A. Virtually zero risk (< 0.2%) ` Zero transmissions (from seropositive partner)
B. Very low risk (< 2%) ` Upper bound of 95% CI: 0.23 /100 CYFU
C. Possible (<10 %) ` Sexual Transmission from a person with
Undetectable Viral Load is Effectively Zero
D. It depends on which ARV regimen he’s on
Rodger AJ, et al. Lancet 393: 2428-38, 2019

©2024InfectiousDiseaseBoardReview,LLC
166
43Ǧ NonǦAIDSǦDefiningComplicationsofHIV/AIDS
Speaker:MichaelSaag,MD

U=U:Undetectable=Untransmittable CASE 11
• 58 yo MSM Male presents for routine evaluation
• On ARV Rx:
• HIV RNA < 20 c/ml; CD4 590 cells/ul
• He is sexually active with 3 to 4 different partners /
year
• Receptive and insertive anal intercourse
• A routine annual anal PAP is collected and shows
https://www.preventionaccess.org/about, LSIL
https://www.health.ny.gov/diseases/aids/ending_the_epidemic/,
https://www.cdc.gov/hiv/library/dcl/dcl/092717.html

QUESTION # 11 Treatment of HSIL reduces risk of anal cancer by 57%

Which of the following should be performed? ` 30 anal cancers diagnosed in median


A. High Resolution Anoscopy with Biopsy f/u of 25.8 months
` 9 in Treatment arm (173/100,000
B. Digital Rectal Exam; if negative monitor for 1 yr PY)
` 21 in Active Monitoring arm
C. Sigmoidoscopy (402/100,000 PY)
D. Colonoscopy ` 8 study-related serious AEs:
` 7 in treatment arm (3 pain, 3
E. Monitor only; repeat anal PAP in 6 months abscess, 1 skin ulceration)
` 1 in monitoring arm (infection)

Analdysplasia Palefsky J,etal.NEngl JMed2022;386:2273Ͳ2282

Figure1.FollowͲupofAnalCytologicScreeningResults Recommendations:Screening
; CliniciansshouldpromotesmokingcessationforallpatientswithHIV,
especiallythoseatincreasedriskforanalcancer.(A3)

; Forallpatientsagedш35yearswithHIV,cliniciansshouldrecommend
andperformDAREannuallytoscreenforanalpathology(B3)

; CliniciansshouldevaluateanypatientwithHIVwhois<35yearsoldand
presentswithsignsorsymptomsthatsuggestanaldysplasia.(A3)

; CliniciansshouldconductorreferforHRAandhistology(viabiopsy)in
anypatientwithabnormalanalcytology.(A2)
; Cliniciansshouldreferpatientswithsuspectedanalcancerdetermined
byDAREorhistologytoanexperiencedspecialistforevaluationand
management.(A3)
7/31/2024 NYSDOHAIDSInstituteClinicalGuidelinesProgram 7/31/2024 NYSDOH AIDS Institute Clinical Guidelines Program

©2024InfectiousDiseaseBoardReview,LLC
167
43Ǧ NonǦAIDSǦDefiningComplicationsofHIV/AIDS
Speaker:MichaelSaag,MD

CASE 12
• 30 yo Male presents with new lesions on his
buttocks, groin, back, and face
• MSM; reports fever
• Denies sexual activity in the last 12 weeks
• HIV RNA 68,000 c/ml (off ARV now)
CD4 count 250 cells/ul
• UDS + methamphetamine

QUESTION # 12
addition to STI screening and Mpox culture,
In
which of the following would you do? Contact me:
A. Treat for molluscum contagiosum
B. Start tecovirimat at this visit [email protected]
C. Wait for cultures, if positive for mpox, start
tecovirimat
D. No specific mpox Rx; give JYNNEOS vaccine now
instead
E. Administer Benzathine Penicillin

©2024InfectiousDiseaseBoardReview,LLC
168
Tuesday, August 20, 2024

44

Encephalitis Including West Nile and


Rabies

Dr. Allen Tunkel

©2024 Infectious Disease Board Review, LLC


COPYRIGHT NOTICE: The Copyright Act (Title 17 of U.S. Code) governs the rights attributed to owners of
copyrighted work. Under certain circumstances, educational institutions may provide copies of
copyrighted works to continuing education participants. The copies may not be copied nor used for any
other purpose besides private study, scholarship, or research. Participants should not provide electronic
or print copies of ant materials provided by the university to unauthorized users. If a participant fails to
comply with these restrictions, the participant may be held liable for copyright infringement. No further
transmission or electronic distribution is permitted.

169
170
44– EncephalitisincludingWestNileandRabies
Speaker:AllanTunkel,MD

Lecture Title

• Disclosures of Financial Relationships with Relevant


Commercial Interests
Encephalitis Including West Nile and Rabies
- None

Allan R. Tunkel, MD, PhD, MACP


Professor of Medicine and Medical Science
The Warren Alpert Medical School of Brown University
7/1/2024

ENCEPHALITIS ENCEPHALITIS
3
Definitions 4
Epidemiology
Encephalitis ~5 cases/100,000 population annually in US
Inflammation of brain parenchyma with neurologic dysfunction
†
from 1990-2017
† Gold standard is pathologic examination and testing of brain
tissue >1 million cases annually worldwide
† Usually based on clinical, laboratory, and imaging † Rabies
Encephalopathy † Measles
† Altered consciousness (confusion, disorientation, behavioral
† Japanese encephalitis virus
changes, cognitive impairment) + inflammation
† Usually metabolic or toxic conditions

ENCEPHALITIS ENCEPHALITIS
5
Etiology 6
Etiology
California Encephalitis Project (CEP) reviewed 1,570 Australian Childhood Encephalitis Study (CID
cases over 7-year period (CID 2006;43:1565) 2020;70:2517)
Confirmed or probable etiology in 16%
† 69% viral
287 children with confirmed encephalitis
† 20% bacterial 57% infectious (confirmed/probable)
† 7% prion 25% immune-mediated
† 3% parasitic
† 1% fungal 17% unknown
Possible etiology in 13%

©2024InfectiousDiseaseBoardReview,LLC
171
44– EncephalitisincludingWestNileandRabies
Speaker:AllanTunkel,MD

Reasons Etiology not Identified General Approach


7 8

Undiscovered pathogens Can’t test for everything


Uncommon presentation by common pathogens Epidemiologic and clinical clues
Common presentation by uncommon pathogens General diagnostic studies
Wrong test Neuroimaging clues
Consider noninfectious etiologies
Wrong sample
Wrong timing Tunkel et al. Clin Infect Dis 2008;47:303
Not an infection Venkatesan et al. Clin Infect Dis 2013;57:1114
Bloch et al. Clin Infect Dis 2023;doi.org/10.1093/cid/ciad306

CASE #1 CASE #1
9 10

50-year-old man presents with a several day history of Acyclovir is initiated


fever, headache, and personality change with MRI with gadolinium reveals enhancement in the
progression to confusion left temporal lobe
On exam, temperature is 101oF; he is disoriented and Results of initial cerebrospinal fluid (CSF)
unable to follow commands polymerase chain reaction (PCR) for HSV-1 and
CT scan of the head without contrast is negative HSV-2 return negative
CSF analysis reveals a WBC of 80/mm3 (95% lymphs), After 3 days, the patient is now oriented to name
glucose 70 mg/dL (serum 100 mg/dL), protein 120 and follows simple commands
mg/dL; Gram stain is negative

QUESTION #1 CASE #1 (Continued)


11 12

What is the next step in the management of this patient? Repeat CSF analysis on day #4 reveals that the PCR is
now positive for HSV-1
A. Perform a brain biopsy of the left temporal lobe The patient continues to improve and completes a 14-
day course of acyclovir
B. Obtain new CSF for HSV PCR testing One month later, he presents again with fever and
C. Send serum for HSV IgG antibodies confusion
D. Repeat brain MRI CSF analysis reveals a WBC count of 30/mm3 (all
E. Discontinue acyclovir lymphocytes) with normal glucose and mildly elevated
protein; CSF PCR tests for HSV-1 and HSV-2 are
negative

©2024InfectiousDiseaseBoardReview,LLC
172
44– EncephalitisincludingWestNileandRabies
Speaker:AllanTunkel,MD

QUESTION #2 Herpes Simplex Encephalitis


13 14

Which of the following is the most likely reason for his Epidemiology
second presentation of encephalitis? † Among the most severe of all human viral infections of brain;
>70% mortality with no or ineffective therapy
† Accounts for 10-20% of encephalitis viral infections
A. Relapse of herpes simplex encephalitis
† Occurs throughout the year and in patients of all ages
B. Development of acyclovir-resistant herpes simplex
† Described following whole brain irradiation or following a
encephalitis
neurosurgical procedure
C. Development of autoimmune encephalitis
† Majority in adults caused by HSV-1
D. Acyclovir neurotoxicity Clinical features
† Fever, personality change, dysphasia, autonomic dysfunction

Herpes Simplex Encephalitis


15

Electroencephalography
† Sensitivity of ~84%
† Periodic lateralizing epileptiform discharges (PLEDs)
Neuroimaging
† Computed tomography (lesions in 50-75% of patients)
† Magnetic resonance imaging (>90% of cases)
Brain biopsy
† Inflammation with widespread hemorrhagic necrosis
† Intranuclear inclusions (50% of patients)
† Reserve for patients not responding to acyclovir therapy
16

Herpes Simplex Encephalitis Herpes Simplex Encephalitis


17 18

Cerebrospinal fluid (CSF) findings Acyclovir is the antiviral agent of choice


† Lymphocytic pleocytosis (mean of 100 cells/mm3) † Mortality of 19% at 6 months
† Presence of red blood cells (25% never have RBCs) † Mortality of 28% at 18 months
† Elevated protein
† Morbidity ~50%
† Normal in 5-10% of patients on first evaluation
CSF Polymerase Chain Reaction Dosage in adults is 30 mg/kg/day in 3 divided
† Sensitivity 98% dosages (in those with normal renal function) for
† Specificity 94% 14-21 days
† Positive predictive value 95% No added benefit on oral valacyclovir (3-month
† Negative predictive value 98% course) after standard course of acyclovir
† If negative, may need new CSF sample in 3-7 days

©2024InfectiousDiseaseBoardReview,LLC
173
44– EncephalitisincludingWestNileandRabies
Speaker:AllanTunkel,MD

Other Herpesviruses Other Herpesviruses


19 20

Varicella-zoster virus Human herpesvirus 6


† Can occur without rash (zoster sine herpete) † Immunocompromised patients, but seen in children
† Focal neurologic deficits and seizures † CSF PCR (sensitivity >95%); high rate of detection in healthy adults
(PPV only 30%)
† CSF PCR; lower sensitivity in those with vasculopathy so also
† Ganciclovir or foscarnet
check CSF antibodies
† MRI/MRA large vessel vasculitis and ischemia
Cytomegalovirus
† Immunocompromised (especially HIV)
† Acyclovir (however, no controlled studies) + ?corticosteroids
(if vasculopathy) † Evidence of widespread disease
† CSF PCR (sensitivity 82-100%; specificity 86-100%)
Epstein-Barr virus † MRI may reveal subependymal gadolinium enhancement and non-
† Encephalitis and/or transverse myelitis specific white matter changes
† Serologic testing; CSF PCR (may have false-positives) † Ganciclovir + foscarnet

CASE #2 CASE #2
21 22

72-year-old man presents in late August with


complaints of fever, chills, and weakness beginning 1 T 103.1oF, P 110, RR 16, BP 110/70 mmHg
week earlier; on the day of admission, he becomes No rash or petechiae, neck supple, no adenopathy,
confused lungs clear, heart without murmurs, abdomen normal
He lives in central New Jersey, where he and his wife On neurologic exam, he is oriented to person only.
have a horse farm; they often noted mosquito and Cranial nerves intact. Motor strength 4/5 UE, and
tick bites 3/5 LLE and 2/5 RLE. Sensation intact. Reflexes
On presentation, he is somnolent and unable to diminished in LE
provide a complete history, although denies headache
and stiff neck

QUESTION #2 West Nile Virus (WNV) Encephalitis


23 24

Which of the following tests is most likely to establish the


etiology of this patient’s encephalitis? First US cases reported in 1999 in New York City
Birds are main reservoirs
A. Serum IgM Mosquito vector
B. Serum polymerase chain reaction Other modes of transmission
C. Cerebrospinal fluid IgM † Transplanted organs
† Blood transfusions
D. Cerebrospinal fluid polymerase chain reaction
† Breast milk
E. Brain MRI
† Transplacental
† Occupational

©2024InfectiousDiseaseBoardReview,LLC
174
44– EncephalitisincludingWestNileandRabies
Speaker:AllanTunkel,MD

25 26

WNV Human Cases Reported To CDC


Year Total Human Cases Neuroinvasive Deaths

2007 3630 1227 124


2009 720 386 32
2011 712 486 43
2012 5674 2873 286
2014 2122 1283 85
2018 2544 1594 137
2019 971 633 60
2021 2911 2008 227
27 2023 (through 1/9/2024) 2406 1599

West Nile Virus Clinical Syndromes West Nile Virus Encephalitis


29 30

No clinical illness or symptoms (~80%) Diagnosis


West Nile Fever (~20%) † Serum IgM antibody (8-14 days of illness onset)
Severe WNV Disease (1 in 150) † CSF reveals lymphocytic pleocytosis and elevated
protein; glucose is normal
† Meningitis (37%)
† CSF IgM (positive in >90%)
† Encephalitis/Meningoencephalitis (53%)
† CSF PCR (<60% sensitivity)
† Poliomyelitis-like flaccid paralysis (7%)
† Neuroimaging

©2024InfectiousDiseaseBoardReview,LLC
175
44– EncephalitisincludingWestNileandRabies
Speaker:AllanTunkel,MD

West Nile Virus Encephalitis


32

Therapy
† Supportive

† Ribavirin, interferon alpha, and IVIG don’t work

31

Other Arboviruses Other Arboviruses


33 34

St. Louis encephalitis virus Powassan virus


† Mosquito vector; bird reservoir † Tick vector (Ixodes scapularis in NE); rodent reservoir; New England
† Endemic in western US; periodic outbreaks in eastern US † Prevalence among animal hosts and vectors increasing
† Urinary symptoms early; SIADH (one-third of cases) † Parkinsonism, involvement of basal ganglia and thalamus common
† Serology; CSF IgM † Serology; CSF IgM; metagenomic sequencing
Japanese encephalitis virus Tickborne encephalitis virus
† Most common cause of mosquito-borne encephalitis worldwide (SE † Tick vector, rodent reservoir; drinking unpasteurized milk or cheese;
Asia, China, India, Nepal, Korea, Japan) solid organ transplantation; rituximab
† Mainly children; rice fields where vectors breed † Eastern Russia, central Europe
† Seizures and parkinsonian features; poliomyelitis-like flaccid paralysis † Poliomyelitis-like paralysis
† Serology; CSF IgM † Serology; CSF IgM
† Anti-TBE immune globulin for post-exposure prophylaxis

Other Arboviruses Measles Virus


35 36

La Crosse virus Acute disseminated encephalomyelitis


† Mosquito vector; chipmunk and squirrel reservoir † Usually 1-2 weeks after exposure; incidence 1 per 1,000 infections
† Midwest and eastern US; woodlands † Fever, fatigue, headache, nausea, vomiting
† 2nd most common arbovirus in US Inclusion body encephalitis
† Serology; CSF IgM; SIADH (~20%)
† Unvaccinated children and adults; immunocompromised
Eastern equine encephalitis virus
† Symptoms 1-6 months after exposure; decreased consciousness, focal signs, seizures
† Mosquito vector; bird reservoir in North America; organ transplantation
† Primarily Atlantic and Gulf coast states Subacute sclerosing panencephalitis
† Abrupt onset with fulminant course; seizures common † 6-10 years after infection (range 3-35 years)
† High case-fatality rate (50-70%) † Behavioral changes, cognitive impairment at presentation
† Serologic testing
† Myoclonus, seizures, neurologic deterioration (coma and death) later
† High CSF WBC count (>1000 cells/mm3)

©2024InfectiousDiseaseBoardReview,LLC
176
44– EncephalitisincludingWestNileandRabies
Speaker:AllanTunkel,MD

37
BioFire FilmArray Metagenomic Next-Generation Sequencing
Bacteria Viruses Fungi
Consider for encephalitis cases in which no cause
Escherichia coli K1 Cytomegalovirus Cryptococcus identified
neoformans/gatti
Haemophilus influenzae Enterovirus
Allows unbiased or agnostic pan-species molecular
diagnostics
Listeria monocytogenes Herpes simplex virus 1
In one study of 204 patients (58 with meningitis or
Neisseria meningitidis Herpes simplex virus 2 encephalitis), NGS identified an infectious cause in
Streptococcus agalactiae Human herpesvirus 6
22% not identified by clinical testing (Wilson et al.
NEJM 2019;380:2327).
Streptococcus pneumoniae Human parechovirus
Possible role in testing of enigmatic cases
Varicella zoster virus

CASE #3 QUESTION #3
39 40

36-year-old man is on a hiking trip in northern In addition to administration of rabies vaccine, what is the
California and is bitten on his lower leg by a skunk most appropriate management?
Upon presentation, he is afebrile and has several
puncture wounds on his right lower extremity A. Rabies immune globulin at the bite sites
B. Rabies immune globulin in the deltoid muscle
You irrigate with wounds with soap and povidone
C. Rabies immune globulin in the buttocks
iodine, and administer a tetanus booster
D. Rabies immune globulin intraperitoneally
He has never been vaccinated against rabies
E. Nothing further is indicated

Rabies
41

Transmitted by bite of infected animal


† Dogs are principal vector (98% of cases) worldwide
† May be transmitted after unrecognized bites by bats
Rare and sporadic in US – 125 cases from 1960-2018
† 36 (28%) attributed to dog bite during international travel
† 89 acquired in US; 62 (70%) attributed to bats
Worldwide in distribution (50,000-100,000 annual deaths)
Incubation period 20-90 days
42

©2024InfectiousDiseaseBoardReview,LLC
177
44– EncephalitisincludingWestNileandRabies
Speaker:AllanTunkel,MD

Rabies Rabies
43 44

Encephalitic (furious) form (80%) Diagnosis


† Culture and RT-PCR of saliva
† Agitation alternating with lucidity † Immunofluorescent detection of viral antigens and RT-PCR in nuchal
† Hypersalivation biopsy
† Hydrophobia † CSF antibodies and RT-PCR
† Brain biopsy (antigen detection/Negri bodies)
† Bizarre behavior
Therapy
† Disorientation, stupor, coma, death † Supportive
Paralytic (dumb) form † Milwaukee Protocol has failed in 26 cases
† Ascending paralysis; early muscle weakness † Post-exposure prophylaxis (rabies immune globulin at bite site and
vaccine)
† Later cerebral involvement

CASE #4 CASE #4
45 46

22-year-old woman with no significant past medical or EEG reveals diffuse slowing
psychiatric history develops headache and low-grade CSF Gram stain and cultures, and PCR for HSV are
fever followed by confusion and hallucinations negative
On presentation, she is afebrile and disoriented; she has A diagnosis of autoimmune encephalitis is considered,
evidence of abnormal movements of her mouth and face
and appropriate studies sent
CSF analysis reveals a WBC count of 20/mm3, with
CSF returns positive for antibodies to the NR1 subunit
normal glucose and protein
of the N-methyl-D-aspartate receptor
Brain MRI is normal
Corticosteroids and IV immune globulin are initiated

ENCEPHALITIS
QUESTION #4 Noninfectious Etiologies
47 48

Which of the following studies should now be performed? Acute disseminated encephalomyelitis (ADEM)
† 10-15% of encephalitis cases in US
A. CT scan of the chest † Post-infectious
† Symptoms 2-4 weeks after trigger
B. CT scan of the abdomen
† MRI bilateral asymmetric T2 hyperintensity in
C. Carotid ultrasound subcortical and deep white matter
D. Renal ultrasound † Corticosteroids
E. Transvaginal ultrasound Anti-N-methyl-D-aspartate receptor (Anti-NMDAR)
encephalitis

©2024InfectiousDiseaseBoardReview,LLC
178
44– EncephalitisincludingWestNileandRabies
Speaker:AllanTunkel,MD

A
Anti-NMDAR Encephalitis
A 50

Neuronal antibody-associated encephalitis


In California Encephalitis Project, this entity
exceeded that of any single viral entity in children
and was also seen in adults
Female to male ratio of about 8:2
37% of patients younger than 18 years at
presentation

Anti-NMDAR Encephalitis Anti-NMDAR Encephalitis


51 52

Abnormal behavior (psychiatric symptoms) CSF analysis


Cognitive dysfunction † Mild pleocytosis (median WBC 23/mm3); normal
Seizures glucose and protein
Movement disorders (orofacial dyskinesias)
† Specific IgG antibodies to GluN1 subunit of the NMDAR
Decreased level of consciousness
in CSF and serum
Autonomic instability
† Viral causes of encephalitis (e.g., HSV) are associated
May be associated with ovarian teratoma (in ~50% of
patients older than 18 years) with development of NMDAR antibodies

Anti-NMDAR Encephalitis Anti-NMDAR Encephalitis


53 54

Neuroimaging Therapy
† Abnormal in 50%, but nonspecific † First-line
„ Corticosteroids
† T2 and FLAIR hyperintensity (hippocampi, cerebellar or „ Intravenous immunoglobulin
cerebral cortex, frontobasal and insular regions, basal „ Plasma exchange
ganglia, brainstem) † Second-line
Rituximab or cyclophosphamide
EEG †
„
Female patients should be evaluated for ovarian teratoma; if
† Diffuse or focal slowing present, remove
† Occasional superimposed epileptic activity 75% of patients have mild sequelae or fully recover; relapse
in up to 24%

©2024InfectiousDiseaseBoardReview,LLC
179
44– EncephalitisincludingWestNileandRabies
Speaker:AllanTunkel,MD

55 QUESTIONS
Allan R. Tunkel, MD, PhD, MACP
Email: [email protected]

©2024InfectiousDiseaseBoardReview,LLC
180
Tuesday, August 20, 2024

45

Photo Opportunity I:
Photos and Questions to Test Your
Board Preparation

Dr. Rajesh Gandhi

©2024 Infectious Disease Board Review, LLC


COPYRIGHT NOTICE: The Copyright Act (Title 17 of U.S. Code) governs the rights attributed to owners of
copyrighted work. Under certain circumstances, educational institutions may provide copies of
copyrighted works to continuing education participants. The copies may not be copied nor used for any
other purpose besides private study, scholarship, or research. Participants should not provide electronic
or print copies of ant materials provided by the university to unauthorized users. If a participant fails to
comply with these restrictions, the participant may be held liable for copyright infringement. No further
transmission or electronic distribution is permitted.
181
182
45– PhotoOpportunityI:PhotosandQuestionstoTestYourBoardPreparation
Speaker:RajeshGandhi,MD

Disclosures of Financial Relationships with Relevant


Photo Opportunity I: Photos and Questions to Test Your
Commercial Interests
Board Preparation

• None
Rajesh T. Gandhi, MD
Massachusetts General Hospital
Professor of Medicine, Harvard Medical School
7/1/2024

Case1

50yo Fdevelopedulceratedlesiononherleftthumbwhichenlarged
overseveralmonthsdespiteseveralcoursesofantibiotics.She
reportednosorethroat,fever,chills,dyspneaorcough.
• CasesarefromaneducationalwebͲsite:
www.idimages.org
SH:Threemonthsbefore,shetravelledtoEcuador,whereshestayedin
Iacknowledgethecontributorstothesitefortheircase anecotourismhotelnearariver.NoknownfreshͲ orsaltͲwater
submissionsandimages.
exposure.Reportedseeingseveralkindsofinsectsandreceivingseveral
bites.Noknownanimalexposuresortickbites.

ContributedbyRojelio Mejia,MD
4

DifferentialDiagnosis Skinbiopsyshowedamastigote,withkinetoplastinavacuole.
CultureoftissuefromskinbiopsyinSchneider’sMediarevealed
promastigotes.
PE:Patientappearedwell.T98.1. PCRoftissue:Leishmaniaguyanensis.
Raisedulceratedlesiononthumb
withaviolaceousborder

• A.Cutaneousleishmaniasis
• B.Mycobacteriummarinum
• C.Sporotrichosis
• D.Pyodermagangrenosum
Skinbiopsy,HandEstain Cultureofskinbiopsytissuein
• E.Tularemia Schneider’smedium
5\ 6

©2024InfectiousDiseaseBoardReview,LLC
183
45– PhotoOpportunityI:PhotosandQuestionstoTestYourBoardPreparation
Speaker:RajeshGandhi,MD

DifferentialDiagnosis
Treatedwithliposomalamphotericin
• Mycobacteriummarinum:patientdidnothaveknownfreshͲ orsaltͲ
waterexposure;shedidnothavenodularlymphangitis
• Sporotrichosis:noknownexposurestosoilorthorn;shedidnot
havenodularlymphangitis
• Pyodermagangrenosum:patientdidnothaveknowninflammatory
Oneweekaftertreatment
boweldiseaseorotherunderlyingpreͲdisposingcondition;ulcerative
PGusuallyoccursonlowerextremities,trunk
FollowͲupat3months • Tularemia:noanimalortickexposure;nosystemicsymptoms;no
adenopathy
7 8

Case2 Case2(continued)
• Amaninhisfiftiespresentedwithdiarrhea,nausea,andvomitingof • Pastmedicalhistory:WBC12,000(neutrophils43%,bands38%,
threedays'duration. lymphocytes10%).Creatinine1.8
• Hehadrecentlybeendischargedfromanotherhospitalwherehehad • Clinicalcourse:
receivedaoneͲweekcourseofivsteroidsforbackpain.
• Patientreceivedivfluidsbecauseofconcernforacutegastroenteritis
• Pastmedicalhistory:spinalstenosis.Medication:prednisone anddehydration.
• Socialhistory:ImmigratedtotheUSfromtheCaribbeantwodecades • Onhospitalday3,developedlethargyandfever(temp102.4).
ago;returnedtovisitoneyearago. • Shortlythereafter,developedrespiratoryfailureandKlebsiellawas
• PE:Temp98.6.Mildepigastrictenderness.Remainderofexamnormal isolatedfrombloodcultures(4/4bottles)andcerebrospinalfluid

Abdominal CT:
Gram stain of sputum
Strongyloides hyperinfection syndrome
colonic wall inflammation
A. Salmonella • Mayoccurduringimmunosuppression,
bacteremia evenshortcoursesofsteroids IodinestainofstoolshowedStrongyloides
B. Strongyloides
hyperinfection • Acceleratedautoinfection
syndrome
• LarvalmigrationinGItract,lungs,skin
C. Amebicinfection
and,attimes,otherorgans
D. Ascariasis
E. Fascioliasis • Migrationoffilariformlarvamaybe
associatedwithentryofentericbacteria
(eg,gramͲnegativesepsis,meningitis)
• Peripheraleosinophiliaabsent

©2024InfectiousDiseaseBoardReview,LLC
184
45– PhotoOpportunityI:PhotosandQuestionstoTestYourBoardPreparation
Speaker:RajeshGandhi,MD

Larvacurrens:CutaneousStrongyloidiasis Case3
• 30yo womanwithHIV(CD4cellcount20,notontherapy)presented
• Serpiginousurticarial
withgradualonsetofwordͲfindingdifficulties,expressiveaphasiaand
rashcausedbythe
rightupperextremityweaknessover4weeks.
dermalmigrationof
filariformlarvae • Socialhistory:ShelivedinNewEngland.Norecenttravelorknown
insectbites.Notsexuallyactive.
• Rashmaymoverapidly:
5Ͳ10cmperhour • PE:Onexam,shewasafebrile.Shehadoralthrush.Shehaddifficulty
namingobjectsandrightͲsidedweakness.
• Studies:WBCcountof2.2(44%P,45%L)
ContributedbyWendyYeh,M.D.

Themostlikelydiagnosisis: Progressivemultifocalleukoencephalopathy
A. Anarbovirus
• CSFJCviruspositive
B. Apolyomavirus
• Demyelinatingdiseaseofcentralnervoussystemcausedbyreactivationof
C. Aherpesvirus
JCvirus,apolyomavirus
D. Aspirochete
• Immunocompromisedhosts(hememalignancy;HIV,natalizumab,
E. Adematiaceous fungus rituxamab)
• Rapidlyprogressivefocalneurologicdeficits,usuallyduetocerebralwhite
matterdisease.
MRI:AbnormalT2signalinvolvingwhitematter,
• Rx:reversalofimmunodeficiency.InpeoplewithHIV:antiretroviraltherapy
leftfrontoͲparietalregion.Noenhancement,
edema,masseffect

PML Differentialdiagnosis
• Arbovirus,suchasWestNileVirus:Unlikelybecauseofnoconfusion,
headache,meningealsigns,paralysis.

• Herpesvirus,suchasHSV:temporallobe.

• Spirochetalinfection,suchassyphilis:centralnervoussystemgumma or
strokeͲlikesyndrome(meningovasculardisease).

• Dematiaceous fungus:noriskfactors(e.g.adjacentparanasalsinusinfection,
penetratingtrauma);lackofenhancementofbrainlesiononimaging.
ContributedbyVinceMarconi,M.D.

©2024InfectiousDiseaseBoardReview,LLC
185
45– PhotoOpportunityI:PhotosandQuestionstoTestYourBoardPreparation
Speaker:RajeshGandhi,MD

PE:T102.Nonblanching,nonpalpable,
Case4 purpuricpatchesonhead,trunk,thighs;
puncturewoundsondorsalaspectof
60yo Mwaswelluntildayofadmissionwhenhedevelopedlethargyand hand;edema,cyanosisofnose.
confusion.Overthecourseoftheday,hishandsandfeetgrewcoldand
numbandhedevelopedarash.
A. E.coli 0157:H7
B. Yersiniapestis
SH:Helivesinaruralarea(mountainͲlionterritory)anddrinkswellͲwater.
C. Pasteurella
Hehasahistoryofalcoholusedisorder.Herideshorsesandhasdogs,
oneofwhombithimafewdaysbefore.
D. Capnocytophaga
E. Leptospirosis

19 20

Capnocytophaga canimorsus Differentialdiagnosis

• BloodculturespositiveforC.canimorsus • E.coli 0157:H7:abdominalcramping,diarrhea;fevertypicallyabsent


• Facultative,fastidiousgramͲnegativebacillusfoundinthemouthof • Yersinia pestis:usuallypresentsasbubonicplague,withregional
dogs,cats. lymphadenitis
• Riskfactors:malesex,dogͲbite,alcoholabuse,asplenia,
• Pasteurella:mayfollowcatordogbit;usuallypresentswithcellulitis;
immunosuppression
septicemiauncommon
• Septicemia:20Ͳ40%havearash(maculopapular,progressingto
purpura fulminans) • Leptospirosis:contactwithurineortissueofinfectedanimals;in
acutephase,ptmayhaveconjunctival suffusion;purpura fulminans,
asinthiscase,wouldbeunusual
21 22

Case5 Whatisthediagnosis?
Culturefromliveraspirate
• AwomanfromChinainher40s
A. Entamoeba histolytica
developedfever,epigastricpain,and
B. E.coli
nausea.Oneweeklater,shewas
C. Streptococcusmilleri
broughttoEDwithconfusionand
D. Actinomyces
fever.
E. Klebsiella pneumoniae
• T101°F.Rightupperquadrant
abdominaltenderness
• AbdomenCT:10cmhypoattenuated
liverlesion ContributedbyDianaI.MercadoMD,DongH.LeeMD,ToddI.Braun,MD

©2024InfectiousDiseaseBoardReview,LLC
186
45– PhotoOpportunityI:PhotosandQuestionstoTestYourBoardPreparation
Speaker:RajeshGandhi,MD

Klebsiella liverabscess Case6


• 35yo manofEthiopiandescentcuthisleftthumbwithaknifewhile
• Hypermucoid strainofKlebsiellapneumoniaeassociatedwithadistinctive
slaughteringalambaspartofEasterfestivities.Hewashedthewound
clinicalsyndromeinSoutheastAsiathatincludesprimaryliverabscess,
withwaterandappliedlemonjuiceandalcohol.Oneweeklater,he
bacteremia,andmetastaticinfection
developedswellingandtendernessandafluctuantlesionatthesite.
• Riskfactors:diabetesandAsianancestry
• Coloniesexhibitextreme“stickiness”onagarplates(“hypermucoviscosity • Twoweeksaftertheinjury,heunderwentincisionanddrainage;
phenotype”) culturesgrewStaph.aureus (oxacillin sensitive).Treatedwith
• PositiveStringtest:“string”of>5mmwhenloopusedtostretcha cephalexin butdidnotimprove.
colonyonanagarplate

CreatinineandLFTsnormal.Glucose158.WBC4.2(normal
Afebrile.2x2x2cmfirmlesiononhisthumb,without
differential).
discoloration,purulentdischarge,fluctuance,orbleeding
XͲray:fungatingsofttissuelesionondorsalaspectofdistalthumb;no
underlyingboneorjointabnormality

Whatisthediagnosis? FollowͲup

• Lesionremovedsurgically.
A. BotryomycosisduetoS.
aureus • Pathology:hyperkeratosis,
B. Nocardia epidermalnecrosis,dermalinfiltrate
C. Brucella ofmixedinflammatorycells;surface
D. Orf keritonocytes witheosinophilic
E. Salmonella inclusions Appearanceconsistent
withecythma
• PCRtestingatCDC+fororf virus contagiousum
Contributors:Drs.IsaacBogoch,RajeshGandhi DNA.

©2024InfectiousDiseaseBoardReview,LLC
187
45– PhotoOpportunityI:PhotosandQuestionstoTestYourBoardPreparation
Speaker:RajeshGandhi,MD

Orf (contagiousecthyma) Orf (continued)

• Zoonoticinfectioncausedbyadermatropic parapox virus(ds • 3Ͳ7dincubationperiod.


DNA)ofgoatsandsheep • MaculeorpapuleÆ nodulewithredcenter,whitehaloand
peripheralerythema Æ ulcerativelesionÆ regenerative
• Transmittedbycontactwithinfectedanimalorfomites papilloma.
• Animalhandlers;childrenaftervisitingpettingzoos, • Mostresolvein4Ͳ8wk
livestockfairs • HumanͲtoͲhumantransmissionhasnotbeenreported
• ClustersreportedafterEid,otherfestivitiesinvolvinglamb • Protectiveimmunityincomplete;personscanbeinfected
sacrifice(Passover,Easter) multipletimes

MMWR(April13,2012)highlighted4casesoforf associated Case7


withhouseholdmeatprocessingoranimalslaughter
• Bullacausedbyorf virus • Nodulecausedbyorf virus • 50yo Fwaswelluntil7dayspriortoadmissionwhenshenoted“bite”
infectionafterpuncturebya infectionaftercontactwitha onleftthigh.Lesionenlargedoverseveraldays.Fourdayspriorto
boneofarecently lambbeingsacrificedfora
slaughteredgoat—PA,2009 holiday— MA,2010 admission,developedfatigue,arthralgias,myalgias,fever,headache.On
admission(July),developedgeneralizedrashonextremities,trunk,back.
• SH:LivedinNewEngland.Shehadseenmouseinherbasement.She
hadadog.Deniedsexualactivity.
• PE:appearedwell.T100.5.Noadenopathy.Lesionpresentonleft
thigh.Papular erythematousrashonherextremities,back,chest.

Rickettsialpox
Doesthispatientmostlikelyhave:
• CausedbyRickettsiaakari,memberofspottedfevergroupofrickettsiae.
A. Varicella • Transmittedtohumansbymousemite
B. Monkeypox
• NYCoutbreakin1980s;highseroprevalence(16%)inIDUsinBaltimore
C. Cutaneousanthrax
• Afterbiteofinfectedmite,R.akari proliferatesresultinginpapule,ulceratesto
D. Rickettsialpox
formeschar
E. Lyme
• 3Ͳ7dayslater,highfever,chillsandheadache.
• 2Ͳ3daysafteronsetoffever,generalizedpapulovesicularrash(notinvolving
palms,soles)
• Diagnosis:serologictesting.Treatment:doxycycline

©2024InfectiousDiseaseBoardReview,LLC
188
45– PhotoOpportunityI:PhotosandQuestionstoTestYourBoardPreparation
Speaker:RajeshGandhi,MD

Rickettsialpox Chickenpox
Rickettsialpoxvs.Chickenpox
Rickettsialpox Chickenpox
Eschar Yes No
Lesionsincrops No Yes
Numberoflesions Relativelysparse(20Ͳ40) Many

Maturelesion Papulovesicle Vesicle

CasecontributedbyKarenThomas,M.D.andLeenaGandhi,M.D.

Case8 T101.Diffuseerythematous,blanchingmaculopapular rashonface,trunkand


arms.Conjunctivalinjection.Examotherwisenormal.
• Maninhis40swaswelluntil5daysbeforepresentationwhen,inmidͲ WBC3.3.Plateletsnormal.
spring,hedevelopedheadache.Twodayslater,hedevelopednonͲ
productivecough,throatdiscomfortandhiseyesbecamewateryandred.
• On5th dayofillness,whiletravelingtoNewEnglandfromMidwest,he
developedarashonface,upperarms&chest.
• LivedinMidwestwithwife,teenagers,dog.Monogamous.Deniedillicit
druguse.TravelsthroughoutUSforwork.

Contributed by Drs. Jessica Haberer, Justin Chan, Rochelle Walensky

Rashinadifferentpatientwiththesame
DifferentialDiagnosis
infection
A. Syphilis
B. Scarletfever
C. Parvovirusinfection
D. Measles
E. Rockymountainspottedfever

©2024InfectiousDiseaseBoardReview,LLC
189
45– PhotoOpportunityI:PhotosandQuestionstoTestYourBoardPreparation
Speaker:RajeshGandhi,MD

• Placedonairborne Measles
precautions
• Acutefebrilerashillness
• Testingforinfluenzanegative
• Airbornevirus,contagiousfromseveraldaysbeforetoseveraldaysafter
• Nasalspecimenpositivefor appearanceofrash.
measlesvirusbydirect DFA • Incubationperiod:10Ͳ14dfromexposuretorash
fluorescentantibody(DFA) From Wendy Kallas and Bernie Collins
• Prodromalsx:fever,cough,coryza,conjunctivitis
• MeaslesIgMandIgG
antibodiespositive Person in airport he was in had • Koplik spotsmayappeartowardendofprodromalsymptoms,justbefore
been diagnosed with measles rash
of same genotype (imported • Rashtypicallybeginsonface;thenspreadsdownbodytoinvolvetrunk
case)
andthenextremities.Lasts5Ͳ6days.

Case9 Case(cont.)
PhysicalExamination
Previouslyhealthymaninhisseventiespresentedwith2weeksoffever,
• Mildrespiratorydistress
headaches,myalgiasand5daysofnonproductivecough,dyspnea,andfevers
• BP141/80.Pulse94.Temp.97.7ºF,RR20,oxygensat93%on6Loxygenbynasalcanula.
Epidemiologichistory • Respiratoryexam:rhonchiatthelungbases.
• LivesinSouthernCaliforniainmountainwilderness. • Examinationwasotherwisenormal.
• Leaveshisvehicleoutsidewiththewindowsdown;frequentlycleans Studies
dashboardandupholstery. • WBC19.3;10%atypicallymphocytes;noeosinophilia.
• Nodomesticpets,butsurroundedbyrodents,deer,sheep,raccoons,other • Hemoglobin18.4g/dL.Hematocrit52.6%.Platelets102,000
wildlife. • Chlamydiapneumoniae, Mycoplasma,HIVͲ1/2,Coxiellaserologieswerenegative.
• Priortosymptoms,hehadvisitedlocalzoo;nodirectanimalcontact • Legionellapneumophila urineantigenwerenegative.
• Noothertravelhistoryoutsidethecountry;noknownsickcontacts. • Respiratoryviralpanelnegative.

Studies
ClinicalCoursePriortoDiagnosis
• PatientwasadmittedwithdiagnosisofcommunityͲacquired
pneumonia.
• Hewasstartedonazithromycinandceftriaxone.
• Hewasinitiallyrequiringminimalsupplementaloxygen,however,his
respiratorystatusworsenedrequiringhighflownasalcanulaat20L
withfractionalinspiredoxygenof80%saturation(FiO2%)during
ChestXͲraydemonstrating ChestCT:Hazygroundglass
groundͲglassopacitiesinthe densitiesinthelowerlobes initialcourseofhospitalization.
upperandlowerlobesconsistent bilaterallywithbilateralpleural
withpneumonia. effusions.

©2024InfectiousDiseaseBoardReview,LLC
190
45– PhotoOpportunityI:PhotosandQuestionstoTestYourBoardPreparation
Speaker:RajeshGandhi,MD

Whatisthediagnosis? FollowͲup

A. Coccidioidomycosis
• HantavirusIgGandIgMserologieswerepositive.
B. Legionellapneumonia
• Patientimprovedandhissymptomsresolved.
C. HantavirusCardiopulmonarySyndrome
D. LeptospirosisPulmonaryHemorrhageSyn.
E. Tularemia

Hantaviruscardiopulmonarysyndrome(HCPS):Clues FinalDiagnosis

• MostcasesareinsouthwesternUS;firstrecognizedinFourCornersregion •HantavirusCardiopulmonarySyndrome(HCPS)
• Transmittedbyrodentreservoir,ofteninruralsettings
• Febrileillness,bilateralinterstitialinfiltrates,andrespiratorycompromise
requiringoxygenwithin72hoursofhospitalization.
• Cardiopulmonaryphasecharacterizedbycapillaryleak,hemoconcentration
(elevatedhemoglobin/hematocrit),shock,pulmonaryedema
• Diagnostictest:serologicassays
ContributedbyDr.DavePatel

©2024InfectiousDiseaseBoardReview,LLC
191
192
Tuesday, August 20, 2024

46

Antiretroviral Therapy for Special


Populations

Dr. Roy Gulick

©2024 Infectious Disease Board Review, LLC


COPYRIGHT NOTICE: The Copyright Act (Title 17 of U.S. Code) governs the rights attributed to owners of
copyrighted work. Under certain circumstances, educational institutions may provide copies of
copyrighted works to continuing education participants. The copies may not be copied nor used for any
other purpose besides private study, scholarship, or research. Participants should not provide electronic
or print copies of ant materials provided by the university to unauthorized users. If a participant fails to
comply with these restrictions, the participant may be held liable for copyright infringement. No further
transmission or electronic distribution is permitted.

193
194
46– WhatCouldBeontheExamAboutCOVID
Speaker:RoyGulick,MD

Disclosures of Financial Relationships with Relevant


What Could Be on the Exam About COVID Commercial Interests

• None
Roy Gulick, MD, MPH
Rochelle Belfer Professor in Medicine
Chief, Division of Infectious Diseases
Weill Cornell Medicine
7/1/2024

Outline – COVID-19
• Virology
• Clinical
• Treatment Virology
• Prevention

COVID-19 Structure SARS-CoV-2 Variants: Global (1/20Æ6/24)


(receptor binding domain) original alpha delta omicron

BA.2.86 2%

JN.1 49%

JN.1.11.1 49%

Krammer Nature 2020;586:516-527


Annavajhala Nature 2021;597:703-708 nextstrain.org

©2024InfectiousDiseaseBoardReview,LLC
195
46– WhatCouldBeontheExamAboutCOVID
Speaker:RoyGulick,MD

COVID-19: Clinical Course

Clinical

Marks and Gulick, Ann Intern Med 2023;176:ITC145-ITC160

What’s the strongest risk factor for progression What’s the strongest risk factor for progression
of COVID-19 to severe disease? of COVID-19 to severe disease?
1. Older age 1. Older age
2. Diabetes, heart disease, or other comorbidities 2. Diabetes, heart disease, or other comorbidities
3. Race/ethnicity 3. Race/ethnicity
4. Vaccine status 4. Vaccine status
5. Being infected with an omicron variant 5. Being infected with an omicron variant

U.S. CDC: Risk for Severe COVID-19


• Older age remains the strongest risk factor
• Compared with age 18-29, risk of death (vaccinated/unvaccinated
individuals in 2020-2022) is:
• 25X Ĺ for age 50-64
• 60X Ĺ for age 65-74
• 140X Ĺ for age 75-84 Treatment
• 340X Ĺ for age >85
• Comorbidities 1.3-2.9X Ĺ
• Racial/ethnic minorities, compared to Non-Hispanic Whites, have
Ĺ SARS-CoV-2 infections, hospitalizations, ICU admissions, death
• Unvaccinated or not up-to-date with vaccines www.cdc.gov (4/15/24)

• Risk Ļ with omicron variants

©2024InfectiousDiseaseBoardReview,LLC
196
46– WhatCouldBeontheExamAboutCOVID
Speaker:RoyGulick,MD

COVID-19: Clinical Course and Interventions


NIH COVID-19 Treatment Guidelines – Outpatients (2/29/24)
antivirals

Marks and Gulick, Ann Intern Med 2023;176:ITC145-ITC160 https://www.covid19treatmentguidelines.nih.gov/

Nirmatrelvir/ritonavir: Drug Drug Interactions What’s the treatment of choice for COVID-19
• Ritonavir inhibits CYP3A during rx (5 days) and 2-3 days after rx with hypoxia?
• Some medicines should not be coadministered: e.g. rivaroxaban, amiodarone,
rifampin, tadalafil (for pulmonary hypertension) 1. Nirmatrelvir-ritonavir
• Others may need to be dose-reduced or temporarily stopped: e.g., atorvastatin,
rosuvastatin
2. Remdesivir
3. Dexamethasone
• Useful resources: 4. 1 and 2
• NIH COVID-19 Treatment Guidelines
• IDSA Management of Drug Interactions: Resource for Clinicians
5. 2 and 3
• University of Liverpool COVID-19 Drug Interaction Checker
https://www.covid19treatmentguidelines.nih.gov/
https://www.idsociety.org/practiceͲguideline/covidͲ19ͲguidelineͲtreatmentͲandͲmanagement/managementͲofͲ
drugͲinteractionsͲwithͲnirmatrelvirritonavirͲpaxlovid/
https://www.covid19Ͳdruginteractions.org/

COVID-19: Clinical Course and Interventions


What’s the treatment of choice for COVID-19
antivirals immuno-
with hypoxia? modulators

1. Nirmatrelvir-ritonavir
2. Remdesivir
3. Dexamethasone
4. 1 and 2
5. 2 and 3

Marks and Gulick, Ann Intern Med 2023;176:ITC145-ITC160

©2024InfectiousDiseaseBoardReview,LLC
197
46– WhatCouldBeontheExamAboutCOVID
Speaker:RoyGulick,MD

NIH COVID-19 Treatment Guidelines – Inpatients (2/29/24) NIH COVID-19 Treatment Guidelines – Inpatients (2/29/24)

https://www.covid19treatmentguidelines.nih.gov/ https://www.covid19treatmentguidelines.nih.gov/

COVID-19 Vaccines
Krammer Nature 2020;586:516-527
RNA Vaccines Viral Vector Vaccines Protein Subunit Vaccines

Prevention

May 2023

COVID-19 Vaccines
COVID-19: 5 Questions They Could Ask
Billions of vaccine doses given globally
Benefits of vaccination outweigh risks; serious adverse events are rare
1. What leads to SARS-CoV-2 MUTATIONS IN THE SPIKE PROTEIN
Side Effects
variants?
• Most common: fever, HA, fatigue, myalgias, pain at injection site X 1-2 days
• Myocarditis / pericarditis: rare (~1/5000-1/100,000) 2. What are important risk factors ĹAGE and IMMUNOSUPPRESSION
• more common in men: late teens-early 20s for COVID-19 progression?
• mild; most recover fully 3. What characterizes severe
• Anaphylaxis: rare (1/200,000) COVID-19? HYPOXIA
• related to PEG/polysorbate(?)
4. Who should receive outpatient PEOPLE WITH RISK FACTORS FOR
• more common in women, 80-86% had history of allergies, 24% had history of
anaphylaxis treatment for COVID-19? SEVERE DISEASE
• most within 15 minutes (one outlier at 20 hours) www.CDC.gov 9/12/23 5. What is the preferred outpatient
regimen for COVID-19? NIRMATRELVIR-RITONAVIR
• Uptake remains suboptimal (2023-4 vaccine: 23% of US adults; 42% >65 yo as
of 5/24)

©2024InfectiousDiseaseBoardReview,LLC
198
46– WhatCouldBeontheExamAboutCOVID
Speaker:RoyGulick,MD

COVID-19: 5 MORE Questions They Could Ask Acknowledgments: Thanks to multiple colleagues who shared their ideas and slides.

6. What drugs interact with DRUGs METABOLIZED THROUGH


nirmatrelvir-ritonavir? CYTOCHROME P450 3A4 ENZYMES
7. What is the preferred (E.G. AMIODARONE, RIFAMPIN)
regimen for inpatients with DEXAMETHASONE + REMDESIVIR
COVID-19 and hypoxia?
8. How do you manage a
patient with rapidly DEXAMETHASONE + A SECOND
progressive hypoxia or IMMUNOMODULATOR (BARICITINIB
needing mechanical OR TOCILIZUMAB)
ventilation?
9. How do COVID-19 mRNA MRNA TRANSCRIBED TO SPIKE
vaccines work? PROTEIN THAT PROVOKES AN
10.What’s the most important EFFECTIVE IMMUNE RESPONSE
serious side effect of
COVID-19 mRNA [email protected]
MYOCARDITIS
vaccines?

©2024InfectiousDiseaseBoardReview,LLC
199
200
Agenda Day 5: Wednesday, August 21, 2024

AM Moderator: John Bennett, MD


# Start End Presentation Faculty
Endocarditis of Native and Prosthetic Devices,
8:00 AM 9:00 AM
47 - and Infections of Pacers and Ventricular Assist Henry Chambers, MD
EDT EDT
Devices
Photo Opportunities II You Should Know for
48 9:00 AM - 9:45 AM John Bennett, MD
Exam
Drs. Bennett (Moderator)
FC13 9:45 AM - 10:00 AM Faculty Q&A
and Chambers
49 10:00 AM - 10:45 AM Staphylococcus aureus Henry Chambers, MD
50 10:45 AM - 11:30 AM Bone and Joint Infections Sandra Nelson, MD
11:30 AM - 11:45 AM Lunch Break
PM Moderator: Henry Masur, MD
Drs. Masur (Moderator),
BR5 11:45 AM - 12:30 PM Board Review Day 5 Bennett, Chambers, Mitre,
Nelson, and Rose
51 12:30 PM - 1:30 PM Lots of Protozoa Edward Mitre, MD

Drs. Masur (Moderator),


FC14 1:30 PM - 1:45 PM Faculty Q&A
Mitre, Nelson, and Rose

52 1:45 PM - 2:15 PM Worms That Could Be on The Exam Edward Mitre, MD


53 2:15 PM - 2:30 PM Penicillin Allergies Sandra Nelson, MD
Kitchen Sink: Syndromes Not Covered
54 2:30 PM - 3:15 PM Stacey Rose, MD
Elsewhere

201
202
Wednesday, August 21, 2024

47

Endocarditis of Native and Prosthetic


Devices, and Infections of Pacers and
Ventricular Assist Devices

Dr. Henry Chambers

©2024 Infectious Disease Board Review, LLC


COPYRIGHT NOTICE: The Copyright Act (Title 17 of U.S. Code) governs the rights attributed to owners of
copyrighted work. Under certain circumstances, educational institutions may provide copies of
copyrighted works to continuing education participants. The copies may not be copied nor used for any
other purpose besides private study, scholarship, or research. Participants should not provide electronic
or print copies of ant materials provided by the university to unauthorized users. If a participant fails to
comply with these restrictions, the participant may be held liable for copyright infringement. No further
transmission or electronic distribution is permitted.

203
204
47– EndocarditisofNativeandProstheticDevices,and
InfectionsofPacersandVentricularAssistDevices
Speaker:HenryF.Chambers,MD

Lecture Title

Disclosures of Financial Relationships with Relevant Commercial


Endocarditis of Native and Prosthetic Devices, and Interests
Infections of Pacers and Ventricular Assist Devices
• Merck: Data Monitoring Committee (member); Stock
• Moderna: Stock
Henry F. Chambers, MD
Professor of Medicine, Emeritus
San Francisco General Hospital
University of California, San Francisco

7/1/2024

Topics for Discussion

• Diagnosis of endocarditis
• Native valve endocarditis Diagnosis of Endocarditis
• Culture-negative endocarditis
• Prosthetic valve and device-related infections

Clinical Signs and Symptoms Q1. Which one of the following statements is correct?
Finding Approximate
Prevalence, % 1. Staphylococcus aureus is the most common cause of
Fever 90 bacterial endocarditis
Murmur 70-85 2. Dental procedures carry a substantial risk for streptococcal
New murmur 50 endocarditis for patients with predisposing cardiac lesions
Worsening old murmur 20 3. Three-quarters of patients with endocarditis have a known
underlying cardiac predisposing condition
Peripheral stigmata (e.g., Osler’s) 20% or less
4. Fever and a new cardiac murmur are present in the majority
Heart failure, cardiac complications 20-50
of patients with endocarditis
CNS complications 20-40
Arch Intern Med. 2009;169:463-473

©2024InfectiousDiseaseBoardReview,LLC
205
47– EndocarditisofNativeandProstheticDevices,and
InfectionsofPacersandVentricularAssistDevices
Speaker:HenryF.Chambers,MD

Microbiology
Organisms Approximate % of Total
Staphylococci 40-50
S. aureus 30-40
Coag-neg 10
Streptococci 25-30
Viridans group 20
S. gallolyticus 5
Groups B, C, D 5
Enterococcus 10
HACEK 1-2 ClinInfectDis.2023;77:518andClinInfectDis.2024;78:964Ͳ967
Culture-negative 3-5
Arch Intern Med. 2009;169:463; Antimicrob Agents Chemother. 2015;60:1411;
Clin Infect Dis. 2018;66:104; Lancet 2016; 387: 882

2023 Duke-ISCVID Criteria for Diagnosis of Endocarditis


Weaknesses of Modified Duke Criteria Definite pathologic Definite Clinical Possible Clinical
diagnosis Diagnosis Diagnosis
Microorganisms identified on Two major criteria Three minor criteria
• Reduced sensitivity for diagnosis of PVE, CIED- cardiac tissue, vegetation,
related endocarditis graft, device OR OR
• Reduced sensitivity for culture-negative endocarditis OR Five minor criteria One major plus one
• Poorly validated in pediatric populations minor criteria
Vegetation, leaflet OR
• Newer imaging modalities and molecular diagnostics destruction, or adjacent
not included in criteria cardiac tissue showing One major plus three
inflammatory changes minor criteria
• Uncertainty about “possible” cases
Rejected endocarditis: criteria for definite or possible endocarditis are not met OR firm
alternative diagnosis established OR lack of recurrence with < 4 days antibiotic therapy

2023 Duke-ISCVID Major Criteria


Positive blood cultures Imaging Surgical
2023 Duke-ISCVID Minor Criteria
• Predisposition: previous IE, PV, h/o valve repair, CHD, more than mild valve
Typical microorganisms* from 2 + ECHO/Cardiac CT Evidence of IE by direct
separate blood cultures 1) Vegetation, leaflet inspection at surgery regurgitation or stenosis, CIED, hypertrophic cardiomyopathy, IVDU
OR perforation, aneurysm, • Fever, documented temperature >38.0oC (>100.4oF)
Non-typical organisms in 3 or abscess, pseudo-
aneurysm, fistula OR
• Vascular phenomena: systemic arterial emboli, septic pulmonary emboli,
more separate blood cultures mycotic aneurysm, intracranial hemorrhage, conjunctival hemorrhages, or
2) New regurgitation c/w
OR Janeway lesions, cerebral or splenic abscess
prior imaging OR
+ PCR for Coxiella burnetti, 3) NEW PVE dehiscence
Bartonella, T whipplei; Coxiella • Immunologic phenomena: glomerulonephritis, Osler nodes, Roth spots, or
phase I IgG antibody titer rheumatoid factor
+ PET/CT
>1:800, IFA IgG titer for PV, device, or graft • Positive blood cultures that do not meet major criteria, OR +PCR/NGS for
Bartonella > 1:800 typical organism from sterile body site
• + PET/CT of PV, graft, or device within 3 mo of implantation
*Staphylococcus aureus, viridans group streptococci, Streptococcus gallolyticus, HACEK species • New regurgitant murmur on exam and echocardiography unavailable
(Hemophilus species, Aggregatibacter, Cardiobacterium, Eikenella, Kingella), E. faecalis, S. lugdunensis,
Granulicatella, Gamella, Abiotphia and in addition for PVE CoNS, C. acnes, Corynebacterium, Serratia

©2024InfectiousDiseaseBoardReview,LLC
206
47– EndocarditisofNativeandProstheticDevices,and
InfectionsofPacersandVentricularAssistDevices
Speaker:HenryF.Chambers,MD

Performance of New vs Old Duke Criteria What about “Possible” IE Cases?


Sensitivity
2000 Criteria 2023 Criteria
True Positive Definition 2000 Criteria 2023 Criteria
Definite 76 84
% of all cases 18-38 15-34
Definite + Possible 93 99 classified as possible

Specificity % of all possible cases 41-52 30-36


True Negative Definition 2000 Criteria 2023 Criteria that were true IE
Rejected 74 60
Rejected + Possible 85 83
Chambers, et al. Duke Infective Endocarditis Criteria 3.0 for the Clinician: Defining What Is Possible.
Chambers, et al. Duke Infective Endocarditis Criteria 3.0 for the Clinician: Defining What Is Possible. Clin Infect Dis. 2024, in press
Clin Infect Dis. 2024, in press

Microvascular/Immunologic Phenomena Role of Echocardiograpy


Clinical Suspicion of IE

TTE

Petechiae Splinter hemorrhages Osler’s node Prosthetic Non-diagnostic


Valve, Intracardiac Positive Negative
TTE TTE TTE
Subconjunctival device
hemorrhages
High
Clinical Suspicion
Janeway lesion TEE
of IE
If negative, repeat Stop Low
If suspicion remains high
Roth spots
Roth spots
European Heart Journal (2015) 36, 3075–3123

Harmonic TTE vs TEE for Diagnosis of IE


- Conclusively negative harmonic
TTE useful to r/o native valve IE
- TTE less sensitive than TEE for
detecting prosthetic IE vegetations,
dehiscence, abscess
- Indeterminant TTE not useful to r/o Native Valve Endocarditis
endocarditis
- TTE insensitive for abscess, IE of
implantable cardiac device
- Specificity of TTE is ~95%

Bai, J Am Soc Echocardiogr 2017; 30:639-646.e8

©2024InfectiousDiseaseBoardReview,LLC
207
47– EndocarditisofNativeandProstheticDevices,and
InfectionsofPacersandVentricularAssistDevices
Speaker:HenryF.Chambers,MD

Q2. A 63 y/o. man with no significant past medical history


presents with a week of fever, rigors, and progressive Q2. What antibiotic regimen would you
dyspnea on exertion. recommend pending further information about
• Exam : BP 160/40 P110 , 39.5 Gram-positive cocci?
• Rales ½ way up bilaterally
• Loud diastolic decrescendo murmur, lower left sternal border
1. Nafcillin
• Labs and studies 2. Vancomycin
– WBC 23,000 90% PMNS, HCT 30. Platelets 110. 3. Vancomycin + nafcillin
– Creatinine 1.6 mg/dl 4. Vancomycin + gentamicin
– TTE 1.5 cm oscillating mass, on bicuspid AV with severe aortic 5. Vancomycin + gentamicin + rifampin
regurgitation
• 3/3 blood cultures: Gram positive cocci in clusters.

Native Valve Staph. aureus IE Q3. A 63 y/o woman with a history of mitral valve
prolapse presents with 3 weeks of low-grade fever,
Regimen Duration Comments fatigue, generalized weakness, weight loss, arthralgias.
MSSA She is first chair violinist for the local orchestra
Nafcillin or oxacillin 6 wk 2 wk uncomplicated R-sided IE (IDU)
• Exam: BP 135/90 P100 , 38.2oC
Cefazolin 6 wk Pen-allergic naf-intolerant patient
(equivalent to naf) • 3/6 holosystolic murmur, radiating the the axilla
MRSA • Lungs are clear, no peripheral stigmata of endocarditis
Vancomycin 6 wk For MSSA if beta-lactam • Serum creatinine 1.2 mg/dl
hypersensitivity • TTE: mitral valve prolapse with 0.5 cm vegetation on anterior
Daptomycin 6 wk > 8 mg/kg/day, vanco alternative leaflet, moderate regurgitation
• 3/3 blood cultures from admission positive for Streptococcus
No gentamicin, no rifampin mitis, penicillin MIC = 0.25 ߤg/ml, ceftriaxone MIC = 0.25 ߤg/ml.

Treatment of VGS and Strep. gallolyticus


Q3. What antibiotic regimen would you recommend for Native Valve Endocarditis
definitive therapy of this patient’s infection? • Pen MIC < 0.12 ȝg/ml
– Penicillin or ceftriaxone + gent x 2 weeks
– Penicillin, ceftriaxone, vancomycin x 4 weeks
1. Penicillin for 6 weeks • Pen MIC > 0.12 ȝg/ml, < 0.5 ȝg/ml
2. Penicillin + gentamicin for 4 weeks – Penicillin or ceftriaxone (4 wk) + gent (2 wk)
3. Ceftriaxone for 4 weeks – Ceftriaxone or vancomycin (4 wk)
4. Penicillin + gentamicin for 2 weeks then penicillin for 2 weeks • Pen MIC > 0.5 ȝg/ml (Gemella and nutritionally
5. Ceftriaxone + gentamicin for 2 weeks then ceftriaxone for 2 deficient species, Abiotrophia and Granulicatella)
weeks – Penicillin or ceftriaxone + gent
– Vancomycin
– Duration 4-6 weeks (two weeks of gent may be sufficient)

©2024InfectiousDiseaseBoardReview,LLC
208
47– EndocarditisofNativeandProstheticDevices,and
InfectionsofPacersandVentricularAssistDevices
Speaker:HenryF.Chambers,MD

Q4. A 72 y/o man type 2 diabetes mellitus, stage II chronic kidney


disease (CKD), and a history of mild aortic stenosis is admitted to Q4. What antibiotic regimen would you recommend for
the hospital with fever, dysuria, and urinary frequency.
definitive therapy of this patient’s infection?
• Exam: T38.9oC, Pulse 110 , BP 145/95 mm Hg.
– Lungs are clear 1. Ampicillin for 2 weeks
– 3/6 systolic ejection murmur at the right upper sternal boarder. 2. Penicillin + gentamicin for 4 weeks
• Lab results 3. Ampicillin + gentamicin for 4 weeks
– Serum glucose 340 mg/dl
4. Ampicillin + ceftriaxone for 6 weeks
– Serum creatinine 1.7 mg/dl, BMP otherwise normal
5. Daptomycin for 8 weeks
– UA: 3+ protein, 20-50 wbcs/high power field, 4+ glucose.
– Two blood cultures and a urine culture are positive for ampicillin-
susceptible Enterococcus faecalis.

Enterococcal Endocarditis
Regimen Duration Comments HACEK Organisms
Pen or amp + gent 4-6 wk Pen S, Gent 1 mg/kg q8h, 6 wk for PVE, • Haemophilus species
symptoms >3 mo*
• Aggregatibacter species
Amp + ceftriaxone 6 wk Pen S, aminoglycoside susceptible
or resistant, E. faecalis only! • Cardiobacterium hominis
Pen or amp + strep 4-6 wk Gent resistant, strep synergy, ClCr > 50 • Eikenella corrodens
Vanco + gent 6 wk Pen resistant or beta-lactam intolerant
(toxic!)
• Kingella species
Linezolid or dapto > 6 wk VRE: Dapto 10-12 mg/kg & combo with
amp or ceftaroline

*Limited data that 2 weeks of gent is sufficient

Antimicrobial Therapy of HACEK Empirical Therapy for Endocarditis While


Endocarditis Awaiting Culture Results
Regimen Comments
Ceftriaxone Regimen of choice • Vancomycin 60 mg/kg/d in divided doses + ceftriaxone
NO GENT: nephrotoxic 2 gm Q24h
Levofloxacin Levo or FQ as single agent OK as • Severe penicillin allergy: Vancomycin + aztreonam 2
alternative regimen gm q8h
NO GENT: nephrotoxic
Ampicillin Avoid: assume amp or pen resistant if
no reliable MIC
NO GENT: nephrotoxic

©2024InfectiousDiseaseBoardReview,LLC
209
47– EndocarditisofNativeandProstheticDevices,and
InfectionsofPacersandVentricularAssistDevices
Speaker:HenryF.Chambers,MD

Principles Of Antimicrobial Therapy

• The regimen should kill the pathogen


Oral Therapy of Endocarditis • A prolonged course of therapy (i.e., weeks not days)
• Intensive dosing to ensure adequate drug exposure
• Source control

POET Trial of Oral Therapy Outcomes: POET Trial of Oral Therapy


• Noninferiority trial, 10% margin, left-sided endocarditis, IV 1954 assessed
vs partial oral for eligibility Outcome IV PO
• Streptococci, Enterococcus faecalis, Staph. aureus, coag- (N=199) (N=201)
negative staphylococci Mortality 13 (6.5%) 7 (3.5%)
1554 excluded
• All patients given IV antibiotics for at least 10 days (428 no Duke criteria) Unplanned surgery 6 (3.0%) 6 (3.0%)
• Primary outcome: composite of all-cause mortality, Embolic event 3 (1.5%) 3 (1.5%)
unplanned cardiac surgery, embolic events, or relapse Relapse 5 (2.5%) 5 (2.5%)
within 6 mo. 400 randomized

N Engl J Med 2019;380:415 N Engl J Med 2019;380:415

Outcomes: POET Trial of Oral Therapy

Culture-Negative Endocarditis

N Engl J Med 2019;380:415 N Engl J Med 2019;380:1373

©2024InfectiousDiseaseBoardReview,LLC
210
47– EndocarditisofNativeandProstheticDevices,and
InfectionsofPacersandVentricularAssistDevices
Speaker:HenryF.Chambers,MD

Culture-Negative Scenarios
Culture-Negative Endocarditis
• Coxiella burnetii (Q fever): Direct or indirect animal contact,
• Prior antibiotics hepatosplenomegaly, abnormal or prosthetic valve.
• Fastidious organisms
Doxycycline + hydroxychloroquine >1 yr.
– HACEK
– Abiotrophia defectiva, et al • Bartonella: Homeless, indolent, valve normal or abnormal,
• 䇾Non-cultivatable䇿 organism louse vector. Rx: 6 wks doxycycline plus two wks gentamicin
– Bartonella quintana > henselae or plus 2 wks rifampin if valve resected (otherwise 3 months
– Coxiella burnetii, Tropheryma whipplei, Legionella spp. more of doxy)
• Fungi (molds) • Tropheryma whippeli: Indolent, protracted course with
• Not endocarditis arthralgias, diarrhea, malabsorption, weight loss, CNS
– Libman-Sacks, myxoma, APLS, marantic
involvement
38

Tools for Diagnosis of Culture-Negative


Endocarditis
Organism Clinical clues Serology Specific Universal
PCR 16s/18s
rRNA PCR Prosthetic Valve and Device-
Related Endocarditis
HACEK, strep, etc Prior antibiotics X
Legionella spp. Immunocompromise, PVE X X X
T. whipplei Chronic illness X X
Brucella spp. Travel X X
Bartonella spp. Cats, homeless, lice X X X
Mycoplasma X X
Q fever Animal contact, lab X X X
Yeast, molds Immunocompromised X X

Q5. A 72 y/o man s/p AV replacement with a bioprosthetic valve


for bicuspid AV with insufficiency. He reports sore throat, cough, Q5. What is the best approach for managing this patient?
congestion, fever, chills, sweats and malaise for 3 days

1. Obtain throat culture and prescribe Pen VK while awaiting


• Exam: T 100.2o F, Pulse 85 , BP 130/70mm Hg, RR 16
results
– HEENT: oral cavity and tonsils red and swollen, no
lymphadenopathy
2. Obtain throat culture and give a script for Pen VK to be filled
if culture is positive for GAS
– Lungs: clear
3. Prescribe azithromycin for treatment of acute URI
– Heart: No murmur
– Skin: no rash 4. Obtain blood cultures and await results
• Rapid rapid strep, rapid flu both negative 5. Obtain blood cultures and initiate therapy with vancomycin,
gentamicin, and rifampin

©2024InfectiousDiseaseBoardReview,LLC
211
47– EndocarditisofNativeandProstheticDevices,and
InfectionsofPacersandVentricularAssistDevices
Speaker:HenryF.Chambers,MD

Microbiology of PVE Diagnosis of PVE


Organisms 2 mo. 2-12 mo. > 12 mo
• Duke criteria and TEE less sensitive for PVE compared to
Post-op (%) Post-op (%) Post-op (%)
native valve endocarditis
S. aureus 30 13 22
• PET-CT (18F-fluorodeoxyglucose positron emission
Streptococci 2 13 30 tomography/computed tomography) plus mod Duke criteria*
Enterococci 8 11 11 – Increased sensitivity: 84% vs. 57%
HACEK 0 0 4 – Reduced specificity: 71% vs 96%
CoNS 28 36 12 • Multislice/Cardiac CT angiography similar to TEE in sensitivity
Gram-neg bacilli 10 4 5 and specificity, but added anatomic detail, useful if TEE non-
Fungi 9 8 1 diagnostic
Culture-negative 6 6 10 *J Am Coll Cardiol Img 2020;13:2605
Clin Infect Dis 2021; 72:1687; Journal of Cardiology 2019; 73:126
Adapted from Karchmer and Chu, UpToDate, 2020

Antimicrobial Therapy of PVE


Organism Regimen Duration
S. aureus, CoNS Naf (MS) or vanco (MR) + Gent x 2 wk, naf/vanco
gent + rif (add later) + rif x 6 weeks
Cardiac Implantable Device Infections
Streptococci, Pen or ceftriaxone + gent OR 6 weeks (optional gent, (permanent pacemakers, defibrillators)
MIC < 0.12 ߤg/ml Vancomycin 1st 2 wk)
6 weeks
Streptococci, Pen or ceftriaxone + gent OR 6 weeks
MIC > 0.12 ߤg/ml Vancomycin 6 weeks
Enterococci Same as for NVE 6 weeks J Am Coll Cardiol 2008;49:1851; Circulation 2010;121:458;
NEJM 2012;367:842; JAMA 2012;307:1727

Survival with and without Device Removal


Cardiac Implantable Device Infection Types
• Pocket site/generator only : ~ 60%
– Blood culture positive <50%
– Pocket infection or generator/lead erosion
• Occult bacteremia/fungemia: ~7-30%
• Lead infection +/- endocarditis: ~10-25%
• PET-CT may detect localized infection if work-up is
inconclusive

Athan, JAMA. 2012; 307:1727-1735

©2024InfectiousDiseaseBoardReview,LLC
212
47– EndocarditisofNativeandProstheticDevices,and
InfectionsofPacersandVentricularAssistDevices
Speaker:HenryF.Chambers,MD

Algorithm for Management of an Infected Cardiac Implantable Algorithm for Management of an Infected Cardiac Implantable
Device (CIED) Infection Device (CIED) Infection
Suspected CIED Infection
Blood Culture

Positive Remove Negative


Entire
Device

Baddour LM et al. N Engl J Med 2012;367:842-849 Baddour LM et al. N Engl J Med 2012;367:842-849

AHA Guidelines for Management of


Cardiac Implantable Device Infections AHA Guidelines for Device Reimplantation
• Blood cultures before antibiotics
– If positive, then TEE • Determine if reimplantation necessary
• Gram stain, culture of pocket tissue, lead tips
• New device on contralateral side
• Device removal for all infections and occult staphylococcal bacteremia
(conside for bacteremia with other endocarditis-causing organisms) • >72h negative BC before reimplantation
• Therapy (antibiotic based on susceptibility) • If IE: reimplant > 14d after original removal
– Pocket infection: 10-14 days • Antibiotic prophylaxis: 1h before implantation, none
– Bloodstream infection: > 14 days
thereafter
– Lead or valve vegetations/endocarditis: 4-6 weeks

Circulation 2010;121:458-77

Main Take-home Points Infection of Ventricular Assist Devices


• Duke-ISCVID criteria is a valuable tool for assessing the Heart Mate 3

likelihood of endocarditis
• TTE is acceptable to rule out endocarditis if of high quality and
in a low probability setting
• Use a tried-and-true regimen, avoid aminoglycoside
combination therapy for NVE
• Think prior antibiotics and Bartonella in culture-negative
endocarditis
• Any fever is a patient with a prosthetic valve is endocarditis
until proven otherwise

©2024InfectiousDiseaseBoardReview,LLC
213
47– EndocarditisofNativeandProstheticDevices,and
InfectionsofPacersandVentricularAssistDevices
Speaker:HenryF.Chambers,MD

Types of VAD Infections Microbiology


• VAD-specific infections – occurs only in LVAD patients of VAD-Specific Infections
– Pump pocket/cannula infections
– Pocket infections • S. aureus/coag-negative staphylococci
– Driveline exit site infections (superficial or deep) • Pseudomonas aeruginosa
• VAD-related infections- risk of LVAD infection increased • Enteric Gram-negatives
– Bloodstream infections (VAD-related, IV catheter/non-VAD • Enterococci
related)
– Endocarditis (pump or cannula, native valve)
• Candida
– Mediastinitis, sternal wound infections
Clinical Transplantation 2019;33:e13552.
• Non-VAD infections
Ann Cardiothorac Surg 2021;10:233; Clinical Transplantation 2019;33:e13552.

Antimicrobial Therapy
Management and Therapy Infection type Initial therapy Chronic suppressive therapy
(oral or IV)
BSI, non-L-VAD IV, 2 wk Probably not needed
• Initial empirical coverage for MRSA and Pseudomonas BSI, L-VAD-related IV, 6 wk Expected
aeruginosa Mediastinitis IV, 4-8 wk Expected
• Pathogen-directed therapy when possible Superficial driveline Oral or IV, 2 wk OK to stop, but may relapse
Deep driveline IV, 2-8 wk depending on Expected
• Chronic suppressive therapy to prevent relapse source control, BSI present
Pump pocket IV, 4-8 wk, source Expected unless device removed
control/device exchange
Pump/cannula IV, > 6 wk, device exchange Expected unless device removed
Clinical Transplantation 2019;33:e13552; Clinical Transplantation 2019;33:e13552; Open Forum Infect Dis. 2020 Nov 16;8(1):ofaa532
Open Forum Infect Dis. 2020 Nov 16;8(1):ofaa532 Ann Cardiothorac Surg 2021;10(2):233-239

IE Prophylaxis after Dental Procedures Which Dental Procedures?


YES NO YES

NO

Circulation. 2021;143:e963-e978

©2024InfectiousDiseaseBoardReview,LLC
214
47– EndocarditisofNativeandProstheticDevices,and
InfectionsofPacersandVentricularAssistDevices
Speaker:HenryF.Chambers,MD

IE Prophylaxis Regimens

Single dose
30-60 min
Other Stuff
before Procedure

Valve Surgery with Stroke Pan-Scanning


• Stroke is an independent risk factor for post-op mortality
• Early surgery with stroke or subclinical cerebral emboli may be • If done, perform prior to surgery
considered if intracranial hemorrhage is excluded by imaging • No recommendations for routine evaluation of patients with IE
and neurological damage is not severe for metastatic foci of infection
• For patients with major stroke or hemorrhage, delay valve • Cerebrovascular imaging may be considered in all patients
surgery 4 weeks (although more recent studies have called with L-sided IE
this into question)

Am Heart J 2019;216:102-112

Fever during Therapy of Endocarditis


• Very common, lasts into the second week, a concern in PVE
• Cause (if one is found, often it is not)
– Abscess: valve ring or elsewhere
– Septic pulmonary emboli, pleural effusion
– Another infection (e.g., IV site, fungal superinfection)
– Polymicrobial endocarditis
– Drug fever
• Work-up:
– Repeat blood cultures
– Imaging studies: TEE, abdominal CT, MRI of the spine, PET/CT, etc

©2024InfectiousDiseaseBoardReview,LLC
215
216
47 – Endocarditis of Native and Prosthetic Devices, and Infections of Pacers
and Ventricular Assist Devices
Speaker: Henry Chambers, MD
Enlarged Slides: 35, 49

Outcomes: POET Trial of Oral Therapy

N Engl J Med 2019;380:415 N Engl J Med 2019;380:1373


*

Algorithm for Management of an Infected Cardiac Implantable


Device (CIED) Infection

* Baddour LM et al. N Engl J Med 2012;367:842-849

©2023 Infectious Disease Board Review, LLC


217
47 – Endocarditis of Native and Prosthetic Devices, and Infections of Pacers
and Ventricular Assist Devices
Speaker: Henry Chambers, MD
Enlarged Slides: 50

Algorithm for Management of an Infected Cardiac Implantable


Device (CIED) Infection
Suspected CIED Infection
Blood Culture

Positive Remove Negative


Entire
Device

* Baddour LM et al. N Engl J Med 2012;367:842-849

©2023 Infectious Disease Board Review, LLC


218
Wednesday, August 21, 2024

48

Photo Opportunities II: Images You


Should Know for the Exam

Dr. John Bennett

©2024 Infectious Disease Board Review, LLC


COPYRIGHT NOTICE: The Copyright Act (Title 17 of U.S. Code) governs the rights attributed to owners of
copyrighted work. Under certain circumstances, educational institutions may provide copies of
copyrighted works to continuing education participants. The copies may not be copied nor used for any
other purpose besides private study, scholarship, or research. Participants should not provide electronic
or print copies of ant materials provided by the university to unauthorized users. If a participant fails to
comply with these restrictions, the participant may be held liable for copyright infringement. No further
transmission or electronic distribution is permitted.

219
220
48– PhotoOpportunitiesIIYouShouldKnowforExam
Speaker:JohnBennett,MD

Lecture Title

• Disclosures of Financial Relationships with Relevant


Photo Opportunities II: You Should Know for Exam Commercial Interests

- None

John E. Bennett, MD
Bethesda, Maryland

7/1/2024

Case 1. Indolent peritonitis .Themostsensitivediagnostic


testwouldbewhichofthe
courtesyofPrishanyaPillai,MD.GeorgetownUniversityHospital
following
A. Cultureandsmearofascitic
ʃ A37yr oldwomanwasadmittedwithathreemonthhistoryoffatigue,night
fluidformycobacteriaand
sweatsanda30poundweightloss.Shehadbeenpreviouslyhealthy,working
fungi
asanursenowandinthePhilippinespriortomovingtotheUSA5yearsago.
Shehadnohistoryofalcoholorillicitdruguseandlivedwithherhusband B. Cultureandpathologyof
andtwoyoungchildreninBaltimore.Herphysicalexaminationwasnormal peritonealtissuefor
exceptforatemperatureof38C,pallorandabdominaldistentionwith mycobacteriaandfungi
shiftingdullness.LabworkfoundHgb6.1gm/l,WBC8.7,creatinine1.5, C. Cultureandpathologyof
albumin3.3andnormalLFT.Pericentesis foundWBC2500with70% needleaspirateofenlarged
lymphs,albumin2.9g/l,negativecytologyandnegativeacidfaststain. abdominallymphnode
AbdominalͲpelvicCTfoundenlargedmesentericnodes,ascites,thickened
D. Laparoscopicbiopsyof
peritoneumwith“caking” (seephotoandarrow)anda6.5cmdiameter
ovarianmass
enlargedleftovary.Findingwereconsistentwithmetastatictumorinthe
peritoneum,likelyovarianinorigin. E. PCRforTB

Tuberculous peritonitis Case 2. Asymptomatic lung masses


ʃ Tuberculousperitonitisoftenresembleovariancancer.
ʃ A30yr oldwomanfromLosAngeleswasreferredtoyoubecausea
ʃ AbdominalCT canhaveperitonealgranulomatousinflammation(“caking”) chestxray donebecauseofcoughandfever,foundtobedueto
COVIDͲ19.AbnormalitieswereconfirmedonachestCTdonelater,
ʃ Cultureandsmearofascites haslowsensitivity.
afterthesymptomshadresolved.Sheiscurrentlyasymptomatic,
ʃ PCRvalueunsure.Maynotprovidesusceptibility livinginLosAngeleswithherhusbandandthreechildrenandworking
inretail.ShegrewupinruralPerubuthasnotreturnedsincemoving
ʃ Cultureofperitonealbiopsytissue=mostsensitivemethod
totheUSA15yearsgo.Sheistakingnomedications,hasnever
ʃ Needlebiopsyofthenodeorovarianmassappearunnecessary. smokedandhasonlytraveledaroundCaliforniainthelastdecade.
Routinelaboratoryworkisnormal.Abronchalveolar lavagewas
ʃ Empiricaltherapyfortuberculousperitonitisifbiopsyofperitonealtissue
negativeoncytopathologyandcultureforbacteria,fungiand
showsgranuloma mycobacteria

©2024InfectiousDiseaseBoardReview,LLC
221
48– PhotoOpportunitiesIIYouShouldKnowforExam
Speaker:JohnBennett,MD

Echinococcosis (Echinococcus
Thehistoryandimagesarehighlysuggestiveofwhatdiagnosis?
A. Cysticercosis
B. Echinococcosis
C. Paragonimiasis granulosis) Hydatic lung disease
D. Coccidioidomycosis
E. Paracoccidioidomycosis
ʃ Clinicalpictureishighlyconsistent.
ʃ EndemicInruralPeru
ʃ canprogressinthelungorliverwithoutsymptomsformanyyears.
ʃ Aspirationorbiopsymayreleaseprotoscolices intothepleura,
leadingtonumerousnewlesions.Referraltoamedicalcenter
familiarwithsurgicalmanagementofthediseaseisindicated.
ʃ Rounded,dense,wellcircumscribedlunglesionswouldnotbe
characteristicoftheotherlisteddiagnoses
Images courtesy of Adrienne Showler, MD, Georgetown University Hospital

Case 3, FEVER AND RASH


A 30 year-old man from El Salvador, living in the United States for 10 years, returned
to United States from visiting family in a residential area of San Salvador for two Themostlikelysourceofinfection

weeks. On the second day home, he had the onset of fever, headache, muscle A. Food

ache, and retrobulbar pain. He had some nausea but no abdominal pain, B. Mosquito

diarrhea or constipation. C. Flea


D. Anotherhuman
The symptoms persisted, but he did not seek medical attention until the third day of
E. Animalurine
illness, when a diffuse petecchial, non pruritic rash appeared on his arms and
upper chest. The home he stayed at in San Salvador was in the city and had no
pets. Children and adults in the family were healthy. Physical examination was
negative except for fever of 102F, rash and two tender occipital lymph nodes. No PhotocourtesyofGlennWortmann,Washington
nuchal rigidity was found. Labs revealed a WBC = 1.6 with a normal differential and HospitalCenter
no atypical lymphs, platelets 60,000, Normal blood chemistries and chest x-ray.

• CorrectanswerB.Mosquito(Dengue)
Case 4. Rapid visual loss one eye
ʃ Rashafterseveraldaysoffever,myalgia,headache.Thrombocytopenia,leukopenia common.
ʃ A20yr oldwomangraduatestudentfromWashington,DCpresentedin
Diagnosisearlyintheinfection byPCRorNS1antigen.Treatmentsupportive.
Januarywiththeacuteonsetofvisionlossinherrighteye,witha“black
ʃ Dengueismoreofanurbandiseasethanmalaria.Aedesaegyptimosquitobreeds insmallurban hole”inthemiddleandblurredimagesaroundthescotoma.Shehadno
poolsofwater,asinoldautotires,nearhumanhabitationandtobiteinthedaytime,particularlyin ocularpainandnormalvisioninherlefteye.Shewasnotsexually
theearlymorningandlateafternoon. Theincubationperiodisusually4Ͳ7daysbutcanbeupto14 active,takingnomedicationsandnorecenttraveloutsidethelocal
days.
area.Shedidsomehikinginlocalparksbutwasnotawareoftickbites.
Shelivedalonewithakittenandagoldfish.Sheoccasionallyateraw
ʃ Animalurine(leptospirosis):rashandleukopeniaareagainstthediagnosis sushiandbeeftartar.Routinelaboratoryworkwasnormal..
ʃ Ratfleas(murinetyphus)uncommoninCentralAmericaandtherashisusuallymoresubtle. Funduscopicexaminationfoundblurringofthediscandretinaledema
inthemacula.
ʃ Food(typhoid)Therashofrosespots,is muchlessextensive
ʃ Anotherhuman:(measles)rashisdifferent.Noconjunctivitis,cough,coryza

©2024InfectiousDiseaseBoardReview,LLC
222
48– PhotoOpportunitiesIIYouShouldKnowforExam
Speaker:JohnBennett,MD

Bartonella henselae
Whichofthefollowing
ʃ Bartonellahenselae:smallatenderswollendraininglymphnode.Also
pathogensismostlikely: encephalitis,neuroretinitis inpreviouslyhealthy
ʃ Bacillaryangiomatosismoreoftenimmunosuppressed
A.Toxoplasma gondii ʃ Diagnosisisusuallymadebyserology.
B.Bartonellahenselae ʃ Acuteoculartoxoplasmosisfromeatingpoorlycookedmeat.Fluffy
C.Treponemapallidum exudativelesionswithoverlyingvitritis
D.Toxocara cati ʃ Toxocara causessingleinflammatorymassfromalarvaembeddedinthe
E.Anisakis eye,sometimesmistakenforretinoblastoma.Youngchildrenaccidentally
ingestingcatfecesareatrisk.
ʃ Syphiliscanpresentintheeyeinmanywaysbuthersexuallyhistoryisnot
suggestive.
ʃ Anisakis(rawfish)causesstomachlesionsthatdonotspreadtotheeye.

Courtesy of TARA PALMORE, MD


Case 5. Pharyngitis and popular skin lesions

ʃ A23yr oldmanpresentedtotheemergencydepartmentwith18days
ofseveresorethroat,notimprovingdespiteinjectionofceftriaxoneand
acourseofazithromycingivenhiminemergencyroomvisits2and14
daysprior.Rapidstreptestsonathroatswabhadbeennegativeat
priorvisits.Inadditionfourpustularlesionshadappearedintheprior
twodays,scatteredoverhistrunkandextremities.Hehadfeltfeverish
atnightbutnottakenhistemperature.Helivedindowntown
WashingtonDC,workedinretail,hadsexwithmenandhadnorecent
travel,medications,orillicitdrugs.Onexam,hehadseveretonsilitis,
temperatureof38.5C,prominentsubmentallymphnodesandfourskin
lesionsliketheonetobeshown.Hisroutinelabswerenormal

Which of the following is likely to be


most helpful? Throat swab for Mpox PCR
A. throatswab ʃ AthroatswabforMpoxDNA.MSM.Pustularskinlesion.Possiblereceptiveoralsex.
SkinbxforPCRalsopossible
B. RapidHIVtest ʃ Tecovirimattreatment.
ʃ Notifyhealthdepartmentforcontacttracing andvaccinecandidates
C. UrineNAAT
ʃ Throatswabforherpessimplex?localizationtotheposteriororopharynxandthis
severityisunusual.
D. Serologyforsyphilis
ʃ ThroatswaborNAATforgonorrheaandChlamydiatrachomatis?Priorantibiotics.
E. Bloodculture
ʃ A4th generationtestforHIVisindicatedbutthereisnorashandseveretonsillitisis
unusualforacuteretroviralsyndrome.
ʃ Syphiliticchancrescanoccurinthemouthbutwouldnotbethispainful

©2024InfectiousDiseaseBoardReview,LLC
223
48– PhotoOpportunitiesIIYouShouldKnowforExam
Speaker:JohnBennett,MD

Case6.PostͲopcomplication
Clue: finger 2 weeks later
A64yearoldwomanpresentedintheemergencyroom
withfever,nausea,sorethroat,musclepain,headacheand
severalloosestoolsoverthepast24hours.Shehadbeen
ingoodhealthandwasrecoveringwellafterfunctional
endoscopicnasalsurgerydone9daysagoforchronic
sinusitis.ShelivedindowntownChicagowithherhusband,
adog,akittenandher5yearoldgranddaughter,whowas
justrecoveringfromseveraldaysofcoughandfever.The ʃ Themostlikelypathogenwaswhichofthefollowing:
patienthadnorecenttravelandwastakingno A. Streptococcuspyogenes
medications.Onexaminationshehadatemperatureof
38.9C,pulse109andBP86/45.Shehadadiffuse
B. Staphylococcusaureus
erythematousrash.Routinelabswerenotablefora C. Capnocytophaga canimorsus
creatinineof3.1mg/dl,WBC14,900andplateletsof D. Bartonellahenselae
112,000.Shewasgiventhreelitersofsalinewithlittle
improvementinherbloodpressure,admittedtointensive E. COVIDͲ19
careandbeganrequiringoxygensupport.

Staphylococcal toxic shock Case 7. Young man with a stroke


Ͳ Staphylococcaltoxicshock canfollownasalsurgery
Ͳ Postoperativenasalpackinghasbeenthoughttocontribute. A26yearoldmaleconstructionworkerfromtheDistrictof
Ͳ Symptomsoftenappearaweekorsoaftersurgerybutcanbedelayed. ColumbiapresentedwiththeacuteonsetofrightͲsidedweakness.
Ͳ Hypotension,fever,renalfailure,myalgias,abdominalpain,nausea,vomitingand Ventricular
diarrheaarecommon.SinuspainnotworsethanusualpostͲop.Desquamationlater. MRIconfirmedastrokeintheleftMCA.Echocardiographylooking aneurysm on
Ͳ Toxin1ͲproducingStaphylococcusaureusinnasaldischarge.Bloodcultureneg. forsourcethatmightembolizefoundaventricularaneurysmand echo
Ͳ Rx:antistaphylococcal betalactampluslinezolid. Clindamycinmaybeusefulbut
macrolideresistanceisaconcern. muralthrombusintheapexoftheleftventricle.CardiacMRI
Ͳ confirmedthepresenceofanapicalventricularaneurysm.EKG
ʃ Streptococcuspyogenestoxicshock.Infectionobvious.Acuterheumaticfever– noshock foundaleftanteriorfascicularblockandrightbundlebranchblock.
ʃ Capnocytophaga sepsis:nodogbite.Spleenintact. ReviewofanEKGtakenintheEmergencyroomtwoyearsprior
whenhewasseenafterfallingoffaladderfoundthatanEKGhad
ʃ Bartonellahenselae doesnotcausehypotension. shownthesameconductionblockbutthepatienthadnotreturned
ʃ COVIDͲ19multisysteminflammatorysyndrome(MISͲA)2Ͳ6weeksafterCOVIDͲ19butthe forascheduledcardiologyclinicvisit.Thepatient’spasthistorywas
patientnotinfected. Onlygranddaughter sick unremarkableexceptthathehadlivedinruralBoliviauntilcoming
theUSAatage12.HIVtestingwasnegative.

Trypanosomiasis (Chagas’ disease)


Whichofthefollowinginfectionsmayexplainhiscardiacdisorder? ʃ Chagas’disease)commoninmanyareasofCentralandSouthAmerica
ʃ Infectioncanprogressdecadesaftertheinitialinfection
a. Leishmaniasis ʃ Presentsascardiacorintestinaldisease(megacolon,megaesophagus)
b. Trypanosomiasis ʃ Cardiacconductionblockisanearlysignofcardiacdisease.
c. Cysticercosis ʃ Myocarditis.Congestivefailure.Apicalaneurysm,muralthrombus.
ʃ Toxoplasmosiscancausemyocarditisbutalmostalwaysinan
d. Toxoplasmosis
immunocompromisedpatient
e.Paracoccidioidomycosis ʃ Cystsofcysticercosisrarelyinmyocardium.Noaneurysm
ʃ Visceralleishmaniasisandparacoccidioidomycosis:nomyocarditis.

©2024InfectiousDiseaseBoardReview,LLC
224
48– PhotoOpportunitiesIIYouShouldKnowforExam
Speaker:JohnBennett,MD

Case 8. Back ache


This22ǦyearǦoldcollegestudentwholivedinIndiauntilimmigrating totheUnitedStatesatage18
Portal for vertebral osteo=Lung (Pott’s )
yearspresentedwithprogressivethoracicbackpainofthreeweeks'duration.

Noforeigntravelinthepastyear.Nounusualfoodhabits.Nopets.Liveswithhealthysisterinan ʃ Lung:HighriskforTB:foreignborn.Immigrationinpast5yrs
apartmentinDC.
ʃ GItract:Brucellosisnorecentexposure.ActinomycosisnoGIlesion
Thelikelyportalofentryofthisinfectionis: ʃ Skin:Staphylococcalinfection.NoIVdruguse.Noskinlesion.Nosepsis.
Staphaureus=overhalfofcasesintheUSA.Portalnotalwaysobvious
A.Lung
ʃ Urinarytract:organismsarerarecausesofspondylitis.
B.GItract
C.Skin
ʃ MRI:lesiononbothsidesofdiscsuggestsinfection,nottumor.
D.Urinarytract

Case 9. Managing an epidural abscess

A55ǦyearǦoldmanisbroughttotheemergencyroombecauseofincreasinglyseverebackpainoftwodays’duration,
Now what? MRI showing vertebral
osteomyelitis and cord
compression L3-L4
precipitatedbyloadingsomegrainsacksontohistruck.

Hehasbeenseeninthepastbecauseofobesity,poorlycontrolledtype2diabetesmellitusandhypertension. A. Surgical decompression of the


spinal cord
AdmissionbloodcultureshavegrownMSSA. B. Aspiration of the abcess
C.Nafcillin alone
NafcillinandaTTEhavebeenordered.
D. Vancomycin alone
MRIhasfoundosteomyelitisofvertebralbodiesT12andL1,withacontiguousepiduralabscessimpingingonthe E. Dexamethasone
spinalcord.

Onyourexamination,temperatureis39oC,pulse120andBP160/90.Thepatientisalertbuthasseverebackpain.He
isunabletowalkbecauseofpainbuthasweaknessinbothlegsandabsentdeeptendonreflexesinbothlegs.

Case10. Skin lesions


Anotherwisehealthy58ǦyearǦoldwomanwholivesin
ʃ Answer:surgicaldecompressionofthespinalcordbecauseofneurologicsigns Wisconsinpresentswithprogressivenodularlesionson
ʃ Aspirationoftheabscessisoftendiagnosticbutunabletopreventpermanent herrighthand. Shehasrecentlyacquiredakitten. The
paraparesisonceneurologicsigns,suchaslegweakness,arepresent. lesionshavebeenpresentforapproximatelysixweeks
andhaveincreasedinnumberastheyhaveprogressed
ʃ Dexamethasonemaydecreaseinflammationbuthasnoroleinthisscenario.
fromherfingertothebackofherhandSheworksata
localflowerstore.ShevisitedBrazil3monthsagoand
spent2weeksintheAmazonbasin.Shegivesnohistory
offeverorconstitutionalsymptoms.
Herexamisremarkableforseveralsubcutaneouslesions
fromtherighthandextendingtotheforearm.Thereisno
associatedlymphadenopathyorlymphangiticstreaking.
Thelesionsaresomewhatpainfultopalpation,andsome
ofthesenoduleshavespontaneouslysuppuratedand
drained.

©2024InfectiousDiseaseBoardReview,LLC
225
48– PhotoOpportunitiesIIYouShouldKnowforExam
Speaker:JohnBennett,MD

What is the most likely organism? Sporothrix schenckii


A. Leishmania brasiliensis Nodularlymphangitis
B. Prototheca wickerhamii • Sporotrichosis:thornyplants(flowerstore)
C. Bartonella henselae • Mycobacteriummarinum:water,fishtanks
D. Sporothrix schenckii • Cutaneousleishmaniasis:foreigntravel
E. Mycobacterium marinum • Nocardiabrasiliensis:soil(notlistedasapossibility)
Otherinoculationlesions
Ͳ Bartonellahenselae:localcatscratchthenlocallymphadenitis(axillary, epitrochlear,
inguinal)
Ͳ Prototheca wickerhamii:soil,water.localverrucousorulceratedlesion.
Onlycontiguousspread.

©2024InfectiousDiseaseBoardReview,LLC
226
Wednesday, August 21, 2024

49

Staphylococcal Disease

Dr. Henry Chambers

©2024 Infectious Disease Board Review, LLC


COPYRIGHT NOTICE: The Copyright Act (Title 17 of U.S. Code) governs the rights attributed to owners of
copyrighted work. Under certain circumstances, educational institutions may provide copies of
copyrighted works to continuing education participants. The copies may not be copied nor used for any
other purpose besides private study, scholarship, or research. Participants should not provide electronic
or print copies of ant materials provided by the university to unauthorized users. If a participant fails to
comply with these restrictions, the participant may be held liable for copyright infringement. No further
transmission or electronic distribution is permitted.

227
228
49 – Staphylococcal Aureus
Speaker: Henry F. Chambers, MD

Lecture Title

Disclosures of Financial Relationships with Relevant Commercial


Staphylococcal Aureus Interests

• Merck: Data Monitoring Committee (member); Stock


• Moderna: Stock
Henry F. Chambers, MD
Professor of Medicine, Emeritus
San Francisco General Hospital
University of California, San Francisco

7/1/2024

Outline of the Talk


Q1. Which one of the following risk factors
• Risk factors for poor outcome, complicated bacteremia is most predictive of complicated Staph.
• Echocardiography aureus bacteremia and mortality?
• Treatment of MSSA bacteremia
• Treatment of MRSA bacteremia A. MRSA infection
• Duration of Therapy B. Hospital-onset infection
• Oral Therapy C. Positive blood cultures on appropriate therapy
• Combination therapy D. Community-onset infection

Infective Endocarditis (IE) Prevalence in Community Associated (CA)


Predictors of Complicated/High Risk SAB* vs. Heathcare Associated (HCA) S. aureus bacteremia

Fowler, et al (OR) Liu, et al (IDSA MRSA) van der Vaart, et al (OR)


Persistent bacteremia (5.6) Persistent bacteremia Persistent bacteremia (6.8)

Skin findings (2.04) Skin findings Community onset (2.9)

Community onset (3.1) Prosthetic material (infected) Prosthetic material (2.3)

Persistent fever (2.2) Persistent fever

Complicated/High Risk = mortality, metastatic foci or complicated local infection, embolic stroke, recurrent bacteremia

Fowler, et al. Arch Intern Med. 2003; 163:2066;


Liu, et al. Clin Infect Dis. 2011; 52:e18-55;
van der Vaart, et al. Clin Infect Dis. 2023 Dec 29:ciad784. doi: 10.1093/cid/ciad784
OR = Odds ratio 5549 community cases, 7491 HCA cases in Denmark Registries 2010-18; OFID 2022; 9:ofac647

©2024 Infectious Disease Board Review, LLC


229
49 – Staphylococcal Aureus
Speaker: Henry F. Chambers, MD

ECHO and Mortality in S. aureus Bacteremia Role of Echocardiography for S. aureus


1.2
VA Study: JAMA Intern Med 177:1489, 2017 Bacteremia
1 18523
• Prevalence of endocarditis 12%-18% overall
Adjusted Odds Ratio

(50) • Depends on the pre-test probability


0.8
 Consider TTE (sensitivity 70%, specificity 95%) in all
0.6 12769 2054 patients with SAB
(29) (5.6)  Obtain TEE (sensitivity 90%, specificity 95%) in high risk
0.4 patients
5522 • Embolic events, intracardiac device, IVDU, prior IE
0.2 (15) • Suspected endocarditis, negative TTE
0
TEE TTE ECHO nos No ECHO
OFID Nov 24, 4:ofx261, 2017; Clin Micro Infect 23:900, 2017
Numbers on bars indicate number of patients (%)

Matched Cohort Study of FDG-PET/CT in Patients


with Staph. aureus Bacteremia
Detection of Infected Foci by PET/CT according to Clinically Suspicion

FDG-PET/CT in Patients with Clinically suspected sites (n=136) PET/CT + sites (n=217)
Staph. aureus Bacteremia PET/CT +,
confirmed
72 (53%) PET/CT +,
clinically
145 (69%)

unsuspected
PET/CT -, 64 (47%) PET/CT +, 72 (31%)
excluded clinically
suspected
Clin Infect Dis. 2021;73:e3859

Matched Cohort Study of FDG-PET/CT in Patients Q2. A single positive blood culture for
with Staph. aureus Bacteremia
Staph. aureus…….
Relapse rate 3% A. Represents contamination in a quarter or more of cases
both groups B. Is associated with a significantly lower relapse rate than
Issues: presence multiple positive blood cultures
--Availability C. Is associated with complicated bacteremia at a rate similar to
--Reimbursement multiple positive cultures
--Observation studies D. Excludes the need to perform echocardiography to rule out
only endocarditis
HR 0.42 (0.22-0.73) --Goals of care? E. Is associated with a lower 60-day mortality than multiple
positive blood cultures

Clin Infect Dis. 2021;73:e3859

©2024 Infectious Disease Board Review, LLC


230
49 – Staphylococcal Aureus
Speaker: Henry F. Chambers, MD

Single positive blood culture for S. aureus


• Represents contamination in < 10% of cases
• Follow-up blood cultures will be positive in ~15% of cases in
whom half will be afebrile
• Carries similar risks of mortality, relapse, and complicated
Treatment of MSSA Bacteremia
bacteremia as multiple positive cultures
• Although the risk of endocarditis is less than with multiple
positive cultures (~ 4% vs ~14%), an ECHO still should be
obtained
• Always obtain follow-up blood cultures

Infect Dis 2020;52:207, OFID. 2021;9(2):ofab642

FDA-approved Antibiotics for SAB AHA Guidelines for S. aureus Native Valve Endocarditis

• MSSA
• Penicillin • Nafcillin (or Oxacillin) 2 gm q4h x 6 weeks
• Nafcillin/Oxacillin • Cefazolin 2 gm q8h x 6 weeks, allergic or intolerant to naf
• No aminoglycoside
• Cefazolin
• Vancomycin • MRSA
• Daptomycin • Vancomycin 30-60 mg/kg/d divided q8-12h
• Daptomycin 6-10 mg/kg q24h x 6 weeks
• Ceftobiprole
• No aminoglycoside

Circulation. 2015; 132:1435

Q3. On day 9 of nafcillin therapy for complicated methicillin-


sensitive S. aureus bacteremia the patient has developed
new neutropenia (1,000 neutrophils). MICs (μg/ml) of the
blood isolate are penicillin 0.12 (S), cefazolin 0.5 (S),
vancomycin 1 (S), daptomycin 0.5 (S), ceftaroline 0.5 (S).
Which one of the alternative agents would you
recommend?
Switching to cefazolin after a
A. Penicillin non-IgE-mediated hypersensitivity reaction
B. Cefazolin to nafcillin is safe
C. Vancomycin
D. Daptomycin Clin Infect Dis 2021; 73:1650

©2024 Infectious Disease Board Review, LLC


231
49 – Staphylococcal Aureus
Speaker: Henry F. Chambers, MD

What about Penicillin G for Penicillin- Zone edge test for β-lactamase
susceptible SAB? Probably Yes
• Confirm susceptibility
• MIC < 0.025 µg/ml (J Antimicrob Chemother. 2021; PMID: 33615356)
Positive
• MIC < 0.25 µg/ml (CLSI breakpoint) and
• Negative PCR for beta-lactamase gene (blaZ) or
• Negative zone test
• References supporting efficacy
• J Antimicrob Chemother. 2023; PMID: 37596905
• Int J Antimicrob Agents. 2022; PMID: 35288257 Negative
• Int J Antimicrob Agents. 2019; PMID: 31181352

MSSA Bacteremia: Cefazolin Inoculum Effect (CzIE*) in


3 Hospitals in Argentina
Cefazolin vs. Antistaphylococcal Penicillins *Beta-lactamase-mediated increase in broth dilution MIC to > 16 µg/ml
at high inoculum (5 x 107 cfu/ml instead of 5 x 105 cfu/ml )
• Efficacy:
• Anti-staphylococcal penicillins are not available in Argentina
• Penicillinase inoculum effect on cefazolin MICs
– does it matter? • Cefazolin is the primary beta-lactam used to treat MSSA
• 54.5% prevalence (42/77 patients with SAB
• Safety :
• 30-day mortality CIE pos vs CIE neg: 40% vs 15% (p=0.03)
• Adverse events due to ASPs

Open Forum Infect Dis.018 May 23;5(6):ofy123

Summary: MSSA bacteremia


• An ASP and cefazolin overall preferred agents for definite therapy
• An ASP is first-line but less well tolerated than cefazolin
• Observational studies suggest mortality, relapse, and treatment failures rates
are similar with cefazolin
• Anxiety over the inoculum effect, which may adversely impact outcome in a
subset of cefazolin-treated patients
Treatment of MRSA Bacteremia
• Start with an ASP until source control established
• Vancomycin, daptomycin if serious beta-lactam allergy or intolerance
and possibly for OPAT ( daptomycin > vancomycin)
• Ceftriaxone not 1st or 2nd line, should be avoided in patients with
endocarditis, more serious infections, complicated/high risk SAB

*ASP = antistaphylococcal penicillin

©2024 Infectious Disease Board Review, LLC


232
49 – Staphylococcal Aureus
Speaker: Henry F. Chambers, MD

AHA guidelines for therapy of native valve


First-line choices for MRSA bacteremia S. aureus endocarditis
• Vancomycin • MSSA
• 30-60 mg/kg/d in 2-3 divided doses • Nafcillin (or Oxacillin) 2 gm q4h x 6 weeks
• Nephrotoxic at higher trough concentrations (15-20 μg/ml) • Cefazolin 2 gm q8h x 6 weeks, allergic or intolerant to naf
• Need for therapeutic drug monitoring • No aminoglycoside
• Daptomycin • MRSA
• Non-inferior to vancomycin, better tolerated • Vancomycin 30-60 mg/kg/d divided q8-12h to achieve trough
• Potential for emergence of resistance on therapy (mprF mutants), of 15-20 μg/ml AUC 400-600 x 6 weeks
especially in high inoculum infections, poor source control • Daptomycin 6-10 mg/kg q24h x 6 weeks
• Do not use for primary pneumonia (OK for septic emboli) • No aminoglycoside
• Some cross-resistance with VISA
Circulation 2015; 132:1435
Holland et al: JAMA 312:1330, 2014
AUC = Area under the concentration-time curve

Q4. A patient with complicated MRSA bacteremia on day 9


Vancomycin or Daptomycin? of therapy with daptomycin q48h develops myalgias with a
creatinine kinase of 1250 u/L (upper limit of normal 200).
• Meta-analysis, 24 studies, MRSA and MSSA, heavily weighted to The last positive blood culture was on day 3 of therapy.
retrospective studies MICs (μg/ml) of the isolate are as follows: vancomycin 2
• Microbiological cure (n=1036): favored daptomycin (S), daptomycin 0.5 (S), dalbavancin 0.25 (S), telavancin 0.5
(S), ceftaroline 1 (S). Which one of the following would
• Clinical cure (n=888): favored daptomycin
you recommend?
• Relapse (n=878): not significantly different
• Mortality (n=8845): not significantly different A. Ceftaroline
B. Dalbavancin
• Adverse events: favored daptomycin
C. Telavancin
D. Vancomycin
Int J Antimicrob Agents. 2023, 62:106946 E. Linezolid

Vancomycin MICs Vary by Method

But what about that


vancomycin MIC of 2 μg/ml?

Int J Antimicro Agent 32:378, 2008

©2024 Infectious Disease Board Review, LLC


233
49 – Staphylococcal Aureus
Speaker: Henry F. Chambers, MD

Vancomycin Dosing: Higher AUC Correlates with Worse Outcome


Lodise, et al Clinical Infectious Diseases 2020;70(8):1536–45 Highlights of Modern Vancomycin Dosing for
MRSA Infections
>757 • Use of troughs no longer recommended
• Target AUC/MICMBD to 400-600 mg*h/L(assume MICBMD = 1 𝜇g/ml)
621-757 • Bayesian-derived monitoring, 1-2 samples (Cmax, Cmin)
• 1st order PK equation with Cmax, Cmin at near steady-state
516-621 • Continuous infusion: multiply steady-state concentration x 24
• Consider loading dose for more seriously ill patients
390-516
• Intermittent infusion: 30-35 mg/kg, max 3000 mg (actual body weight), then
15-20 mg/kg q8-12h
< 389 • Continuous infusion: 15-20 mg/kg then 30-60 mg/kg, target steady state of
20-25 𝜇g/ml
• Pediatric doses higher: 60-80 mg/kg/d divided q6-8h
Am J Health-Syst Pharm. 2020;77:835-864

Duration of Therapy for S. aureus BSI


14 days • UNCOMPLICATED/LOW RISK (~10% of cases)
• Fever resolves by day 3
• Sterile blood culture after 2-3 days (DOCUMENT!)
• Easily removed focus of infection (no DVT)
• No metastatic infection (e.g., osteo)
• Negative echo, no evidence of endocarditis
• No predisposing valvular abnormalities
• (No implanted prosthetic devices, no DM, no
immunosuppression)
Endocar Epidural Septic Osteo +BC, MRSA
4-6 weeks + • COMPLICATED/HIGH RISK ditis* abscess Arthritis 5+ days*
• Failure to meet one or more of above criteria 65% 15% 24% 19% 32% 42%
• Osteomyelitis, endocarditis, epidural abscess, septic arthritis,
pneumonia, complicated UTI
Clin Infect Dis 2023; 76:487
Adapted from Fowler, Ann Intern Med 163:2066, 2003

Readmissions with Treatment Failure


Outcomes of 3 Treatment Strategies
Outcomes A: Standard of B: Partial IV C: Partial IV
care IV Discharged Discharged
N=122 No PO With PO
N=36 N=69
Median
Death, micro 15d IV
failure @ 90 11% 44% 13% Median
days of D/C 18d IV

Clin Infect Dis


Readmission @
2023; 76:487
90 days of D/C 31% 53% 26%

* Days of effective IV Rx
Clin Infect Dis 2023; 76:487

©2024 Infectious Disease Board Review, LLC


234
49 – Staphylococcal Aureus
Speaker: Henry F. Chambers, MD

SABATO Trial: Oral (PO) Step-down vs IV Therapy for Oral Therapy of S. aureus Bacteremia
“Low Risk” SAB • Only a single randomized clinical trial (RCT), somewhat low in quality
• Observation studies (Obs.) subject to selection bias, confounding by
Outcomes PO (n=108) IV (n=105) indication
• Mortality and relapse rates consistently higher with IV!! Really!?
SAB complication @ 90 days 14 (13%) 13 (12%)
• Role in treatment of and efficacy for endocarditis, endovascular
Relapse 3 (3%) 4(4%) infections, complicated bacteremia, MRSA in particular is emerging
• May be an option for treatment of “low risk” patients, but there is a
Deep-seated infection 5 (5%) 8 (8%) lack of standard definition
Death due to SAB 2(2%) 0 • Infectious disease consultation strongly recommended for all
SAB!
Missing/non-attributable death 8 (7%)/3 (3%) 5(5%)/1 (1%) • Prefer agents with good oral bioavailability: linezolid, TMP/SMX,
fluoroquinolone + rifampin, clindamycin, anti-staphylococcal beta-
Lancet ID. 2024; 2024 Jan 17:S1473-3099(23)00756-9 lactam (?)

Q5. Which one of the following combinations have


been shown to improve mortality of patients with S.
aureus bacteremia or native valve endocarditis?
Combination Therapy of S. aureus BSI
A. Anti-staphylococcal beta-lactam + gentamicin for MSSA
B. Anti-staphylococcal beta-lactam + rifampin for MSSA
C. Vancomycin + a beta-lactam for MRSA or MSSA, pending
cultures
D. Daptomycin + fosfomycin for MRSA
E. No combination regimen

Overview of Studies of Combination Therapy for SAB Overview of Studies of Combination Therapy for SAB
Regimen Study Population Comments PMID Regimen Study Population Comments PMID
Adjunctive rifampin RCT MRSA, No benefit 1929035 Adjunctive rifampin RCT MRSA, No benefit 1929035
MSSA 29249276 MSSA 29249276
Adjunctive Obs., MRSA, 1 d shorter Various Adjunctive Obs., MRSA, 1 d shorter Various
aminoglycoside RCT MSSA SAB, toxic aminoglycoside RCT MSSA SAB, toxic
Adjunctive dapto RCT MSSA No benefit 32667982 Adjunctive dapto RCT MSSA No benefit 32667982
Adjunctive β-lactam + RCT MRSA ↑↑ AKI, higher 32044943 Adjunctive β-lactam + RCT MRSA ↑↑ AKI, higher 32044943
vanco/dapto mortality vanco/dapto mortality
Dapto + ceftaroline Obs., MRSA Low quality 30858203, Dapto + ceftaroline Obs., MRSA Low quality 30858203,
aborted data 31640977, aborted data 31640977,
RCT 31404468 RCT 31404468
Dapto + fosfomycin RCT MRSA No mortality 32725216 Dapto + fosfomycin RCT MRSA No mortality 32725216
benefit, ↓ micro 32887985 benefit, ↓ micro 32887985
failure, ↑ AEs failure, ↑ AEs

©2024 Infectious Disease Board Review, LLC


235
49 – Staphylococcal Aureus
Speaker: Henry F. Chambers, MD

De-Escalation of Combo Therapy for Complicated


MRSA bacteremia Take-Home Points
• “Uncomplicated” Bacteremia is uncommon
Outcome Combo (n=66) Mono (n=74) P-value • 2 weeks of therapy for “uncomplicated” SAB, otherwise 4-6 weeks
Composite clinical failure 14 (21%) 8 (24%) 0.66 • Community and HCA SAB do not differ in early mortality rates,
Recurrent bacteremia, 60d 2 (3%) 5 (7%) 0.45
but the former has a 2-fold increased risk of endocarditis
• Parenteral drugs of choice
In-patient mortality 1 (2%) 4 (5%) 1
• MSSA: Nafcillin, cefazolin, penicillin
Readmission, 60d 13 (20%) 13 (18%) 0.75 • MRSA: Daptomycin, vancomycin
Duration of bacteremia, d 8 (IQR 6-11) 8 (IQR 5-12) 0.33 • Monotherapy is effective in most cases, reserve combination
therapy for MRSA salvage
Adverse drug event 2 (4%) 1 (1) 0.47
• Role of oral therapy is an evolving area
Length of stay, d 26 (IQR 20-41) 24 (IQR 16-33) 0.08

Open Forum Infect Dis. 2021 Jun 22;8(7):ofab327.

Thanks

©2024 Infectious Disease Board Review, LLC


236
Wednesday, August 21, 2024

50

Bone and Joint Infections

Dr. Sandra Nelson

©2024 Infectious Disease Board Review, LLC


COPYRIGHT NOTICE: The Copyright Act (Title 17 of U.S. Code) governs the rights attributed to owners of
copyrighted work. Under certain circumstances, educational institutions may provide copies of
copyrighted works to continuing education participants. The copies may not be copied nor used for any
other purpose besides private study, scholarship, or research. Participants should not provide electronic
or print copies of ant materials provided by the university to unauthorized users. If a participant fails to
comply with these restrictions, the participant may be held liable for copyright infringement. No further
transmission or electronic distribution is permitted.

237
238
50– BoneandJointInfections
Speaker:SandraNelson,MD

Lecture Title

• Disclosures of Financial Relationships with Relevant


Commercial Interests
Bone and Joint Infections
- None

Sandra B. Nelson, MD
Assistant Professor of Medicine
Harvard Medical School

7/1/2024

Osteomyelitis:UnifyingPrinciples
• Radiographicstudies:
Ё MRIisthemostsensitiveimagingstudyfordiagnosis
Ё SerialplainfilmsandCTarethemostusefulinsubacuteandchronicinfection
Ё Bonescanisanexcellent“ruleͲout”testwhennegative,butlacksspecificity
Ё
Osteomyelitis Noimagingtestcanconfirmthediagnosisofosteomyelitis,norconfirmcure

• Diagnosiscanonlybeconfirmedthroughbonehistopathologyandculture
Ё Swabculturesofdrainagehavepoorconcordancewithbonecultures

• Optimalrouteanddurationoftherapyareanevolvingtarget
Ё 6weeksofantimicrobialtherapycommonlyused
Ё Oraltherapyincreasinglysupported
Ё Longeroralsuppressioninsettingofretainedhardware

3 4

Case#1 Case#1:Vote
• 57ͲyearͲoldmalepresentedwith3monthsof Whatisthebestnextstepinmanagement?
progressivelowerbackpain.Hedeniedfeversor A. Repeat2setsofbloodcultures
chills,buthiswifenoticedweightloss B. Obtaininterferongammareleaseassay
• BorninCambodia,emigratedtoU.S.asachild C. Percutaneousbiopsyofdiscspace
• Employedataseafoodprocessingplant D. Initiatevancomycin;placePICCforsixͲweektreatmentcourse
• ESR84CRP16 E. Empirictreatmentwithrifampin,isoniazid,ethambutol,and
pyrazinamide
• MRIwithdiscitisandosteomyelitisatL5ͲS1
• BloodculturesgrewStaphepidermidisin2of4
bottles

5 76

©2024InfectiousDiseaseBoardReview,LLC
239
50– BoneandJointInfections
Speaker:SandraNelson,MD

VertebralOsteomyelitis:diagnosis Pott’sDisease
• Imagingpearls • Clinical:
Ё MRIbestforearlyinfection;plainfilmsandCTforsubacuteinfection Ё Moreindolentthanpyogenicosteomyelitis
Ё Findings:dischyperintensity,lossofdischeight,bonemarrowedema, Ё Constitutionalsymptomscommon
endplateerosions,paraspinaland/orepiduralcollections Ё Anteriorcollapsemayleadtogibbusdeformity
Ё Infectionalmostalwaysinvolvestwocontiguousvertebralbodies
• Radiographic:
• Bloodculturesareoftenpositiveinearlyinfection Ё Thoracic>lumbarwithanteriorinvolvement
Ё NofurtherdiagnosticsifStaphaureusorStaphlugdunensis Ё Relativesparingofthediscspaceuntillater
• Brucellaserologies,PPD/IGRAwhenappropriateepidemiology Ё MultiͲleveldisease,largeparaspinalabscesses

• Percutaneousbiopsywhenbloodculturesnegative • Treatment:
Ё Holdantibiotics1Ͳ2weekspriorifnosepsisorneurologiccompromise Ё ConventionalTBtherapy,6Ͳ12months
Ё Ifnegative,repeatpercutaneousbiopsyorconsideropenprocedure Ё Surgeryoftennotnecessary
Simpfendorfer InfectDis
ClinNAm2017;31:299

Reference:2015IDSAGuidelines 97 118

Brodie’sAbscess:
Subacutehematogenousosteomyelitis
• Morecommoninchildrenandyoungadults
• Bacteriadepositinmedullarycanalof
metaphysealbone,becomesurroundedby
rimofscleroƟcboneїintraosseousabscess
• “Penumbrasign”onMRI
SepticArthritis
Ё Granulationtissueliningabscesscavityinsidebone
givesappearanceofdoubleline
• Staphaureusmostcommon

Simpfendorfer InfectDisClin NAm2017;31:299

59 1210

SepticArthritis:ClinicalPearls Polyarthritis
• Synovialfluidcellcounts:Nodiagnosticthreshold • 10Ͳ20%ofsepticarthritisispolyarticular
Ё HigherprobabilityofSAifWBC>50,000/mm3 • Associatedwithbacteremia/sepsis
Ё Lowercellcountsdonotexcludesepticarthritis Ё Staphaureusmostcommon(lookforendocarditis)

• Moresubtlepresentationsinimmunocompromisedhostsandwith • Consideralso:
Ё gonococcal,viral,nonͲinfectious
indolentorganisms
Ё Subacutehistory • Ratbitefever
Ё Polyarthritis(usuallysymmetric),fever,maculopapular
Ё Lowersynovialfluidcellcounts and/orpustularrash
• Negativeculturesand/ordelayedculturepositivity: Ё Streptobacillusmoniliformis(orifbitteninAsia– Spirillum
minus)
Ё thinkGonococcus,HACEK,Lyme,Mycoplasma Ё Rx:penicillin
Giorgiutti NEJM2019:381:1762

13
11 1412

©2024InfectiousDiseaseBoardReview,LLC
240
50– BoneandJointInfections
Speaker:SandraNelson,MD

GonococcalArthritis Viralarthritides
• Tenosynovitis,arthralgias,skinlesions • Symmetricpolyarthritis,ofteninvolvingsmalljoints
Ё Especiallyextensorsurfacetenosynovitis • Oftenassociatedwithfeverandrash
Ё Migratoryarthralgias • Diagnoseserologically(+IgMor4ͲfoldriseinIgGtiter)
• Purulentarthritis
Ё Maybepolyarticular;kneesmostcommon Mostcommonvirusestocause arthritis Clinical andEpidemiologicClues
Ё Lowersynovialfluidcellcountsmorecommon ParvovirusB19 Morecommoninwomen.Historyofexposuretoyoungchildren,oftena
teacherorparent.Handsmostcommon;canbesevere.
• Asymptomaticmucosalphasepredisposes
Rubella NonͲimmune(nonUSborn). Seecervicallymphadenopathy,fever,rash.
Ё Disseminationmorecommoninwomen
HepatitisBVirus SerumͲsicknesslike reaction,resolveswithdevelopmentofjaundice;
• Dx:mucosalsitesampling(cervical,urethral)ishighestyield alsopolyarteritis nodosa (PAN)
Ё Blood(<30%)andsynovialfluid(<50%)culturesloweryield Hepatitis CVirus Immune complexarthritisassociatedwithcryoglobulinemia
Ё Compatibleclinicalsyndrome Alphaviruses(esp Chikungunya) Traveltoendemicareas

1513 16
14

Crystallinearthritis:clinicalpearls MasqueradingasInfection…
• Acutegoutflaremimicssepticarthritis • Othernoninfectiouscausesofarthritis:
Ё Fevercommon
Ё Reactivearthritis
Ё Monoarthritis andpolyarthritisforms
ƒ Followingentericorgenitourinaryinfection
Ё Clues:rapidonset(hours),historyofpriorgout,alcohol,CKD,
diuretics,elevateduricacid ƒ AsymmetricmonooroligoͲarthritisaffectingknees/ankles
Gout
Ё SynovialWBC10,000Ͳ100,000/mm3 ƒ Associatedfeatures:enthesitis (tendoninsertion),dactylitis(sausage
digits),mucosallesions,urethritis,conjunctivitis/uveitis,skinlesions
Ё NeedleͲshapedmonosodiumuratecrystals
(keratodermablennorhagica)
Ё Still’sdisease
• Crystallinediseaseandsepticarthritiscancoexist(esp.
CPPD) Ё Sarcoid(Lofgren’s)
Ё CPPDrarelyhascellcount>30,000 Ё Polymyalgiarheumatica
Ё CPPDrarelyassociatedwithhighfever Ё Manyothers….
Ё RhomboidͲshapedcalciumpyrophosphatedihydratecrystals CPPD
CoelhoBMJCaseReports2017Ͳ222475

Images:Taljanovic RadioGraphics 2015;35:2026 17


15 16
18

Case#2
• 44ͲyearͲoldhealthywomansufferedarightankle
closedpilonfractureandunderwentopenreduction
andinternalfixation(ORIF)
Osteofixation • Chronicallydischargingwounddespitecoursesof
cephalexinandtrimethoprimͲsulfamethoxazole
Infections • TwomonthsafterORIF,superficialwoundculture
growsmethicillinͲsusceptibleStaphaureus
• Plainfilms:Hardwareintact;fracturenotyet
consolidated

19
17 20
18

©2024InfectiousDiseaseBoardReview,LLC
241
50– BoneandJointInfections
Speaker:SandraNelson,MD

Case#2:Vote Osteofixation Infections


Whatareyournextsteps? Goals:BOTH fractureconsolidationandinfectioneradication
Removalofhardwaredependsuponfracturehealing
A. Nosurgicaldebridement;cefazolinfor6weeks Antibioticdurationnotwellstudied
B. Surgicaldebridementwithhardwareremoval;6weeksofcefazolin
Earlyordelayedinfections Latenonunion Late,healedfracture
C. Surgicaldebridementwithhardwareremoval;6weeksofcefazolinandrifampin priortofractureunion
D. Surgicaldebridementwithouthardwareremoval;6weeksofcefazolinandrifampin Microbiology Virulentorganisms Indolentorganisms(coagulaseͲ Oftenindolentorganisms,or
Staphaureusmostcommon negativeStaphylococcus, recurrenceofearlyinfection
E. Surgicaldebridementwithhardwareexchange;6weeksofcefazolinandrifampin Cutibacterium acnes)
Surgical Debrideandretain(assuming Hardwareremoval Hardwareremoval
Strategy implantswellfixed) Revisionorexternalfixation
Antimicrobial PathogenͲdirectedtherapy PathogenͲdirectedtherapy PathogenͲdirectedtherapy
Management AdditionofrifampinifStaph Durationoftensixweeks Durationoftentwoweeks
Durationoften12weeksor followinghardwareremoval
untilfractureheals

21
19 23
20

Oralantibioticsforboneandjointinfections Rifampininorthopedicinfections
• Nowsupportedbyalargebodyofliteratureforanytypeof
boneandjointinfection • Considereda“biofilmactive”agent
Ё CautionwithlifeͲ orlimbͲthreateninginfections • BeststudiedforStaphylococcalPJIinsettingofhardwareretention
• UsuallyafteranIVleadͲinandafterclinicalresponse Ё Dataextrapolatedforotherhardwareinfections(osteofixation,spinalimplant)
• Relativecontraindications/exclusions:
Ё LowertreatmentfailureinPJIwithimplantretention
Ё Lackofsuitableoraloption
Ё OtherindicationforIVtreatment(e.g.endocarditisandbacteremia) • Specifics
Ё NotwellstudiedfordrugͲresistantbacteria(e.g.MRSA) Ё Nevertobeusedinmonotherapyofestablishedinfection
Ё Concernformalabsorption Ё Shouldnotbeusedpriortosurgicaldebridementanduntilpartnerdrugtherapeutic
• Littledatatosupport“boneͲpenetratingantibiotics” Ё Multipledruginteractions(primarilyviaCyp 3A4pathway)
Ё Someadvantagetoquinolone+rifampininStaphylococcalPJI

21 24
22

PJI:Clinicalpresentations
Prosthetic • Earlysurgicalsiteinfection(<3months)
Ё Acuteonsetoffever,jointpain,swelling
Joint Ё Causedbyvirulentorganisms(Staphaureus)
• Delayed/Subacuteinfection(3– 24months)
Infection Ё Insidiousonsetofpain;feverisuncommon
(PJI) Ё Lessvirulentorganisms:e.g.CoagulaseͲnegativeStaph,Cutibacterium
• Acutehematogenousinfection(anytimeafterarthroplasty)
Ё Acuteonsetfever,jointpain,swellinginpreviouslywelljointreplacement
Ё Hematogenous seeding,virulentorganisms(Staphaureus,Streptococcus)

25
23 26
24

©2024InfectiousDiseaseBoardReview,LLC
242
50– BoneandJointInfections
Speaker:SandraNelson,MD

PJI:Diagnosticpearls PJI:Management
• DiagnosisofacutePJIusuallystraightforward
• MultiplediagnosticalgorithmshavebeendevelopedforchronicPJI
SurgicalProcedure Mostappropriatefor: AntimicrobialTherapy*
• DiagnosisofchronicPJIconfirmedif:
А Sinustracttothejoint Debridementandimplant Acuteinfections 1Ͳ6weeksIVantibiotics,then
retention(exchangeof Ͳ bothearlyandlate 3Ͳ6monthsoralantibiotics
А Twosynovialfluidortissueculturespositivewiththesameorganism polyethyleneliner) WellͲfixedcomponents RifampinifStaph
EarlyPJIand Delayed(chronic)PJI
1stageexchange Acuteandsubacuteinfections 1Ͳ6weeksIVantibiotics,then
Latehematogenous
withhealthysofttissues, 3Ͳ6monthsoralantibiotics
ESR/CRP High Normalormoderatelyelevated sensitiveorganisms RifampinifStaph
Plainfilms Maybenormalorshow Maybenormalorshow 2stageexchange Chronicinfections 6weeksIVorhighlybioavailable
effusion periprostheticlucency “Spacer”utilizingantibiotics Sinustracts oralantibiotics
Synovialfluid WBC>10,000/ʅL WBC>3000/ʅL incement Resistantorganisms
cellcounts %pmns >90 %pmns >70
*2012IDSAGuidelines;durationoftherapybasedonlimitedliterature
Synovialfluid Usuallypositive Usuallypositive
AlphaͲdefensin

2725 2826

Case#3 Case#3:Vote
• A57ͲyearͲoldwomanunderwenttotalhiparthroplasty Youareaskedtoproviderecommendationsaboutsystemicandlocal
Ё SheneverachievedapainͲfreestateaftersurgery antimicrobialtherapyforthespacer.Shehasnoantimicrobialallergies.
• Eighteenmonthspostoperatively,shewasdiagnosedwithdelayed
Youadvise:
periprostheticinfectionduetoEnterococcusfaecalis
Ё Sensitivetoampicillin,vancomycin,linezolid,daptomycin,gentamicin A. Ampicillininthecement;systemicvancomycin
• HerorthopedistplansatwoͲstageexchangeprocedureutilizingatemporary B. Ampicillininthecement;systemicampicillin
spacercomprisedofpolymethylmethacrylate(PMMA) C. Gentamicininthecement;systemicampicillin
D. Tobramycininthecement;systemicdaptomycin
E. Ceftriaxoneinthecement;systemiclinezolid

29
27 30
28

AntimicrobialCement(PMMA) Case#4
• Mechanicalfunction“spacer”:
Ё Jointstability,allowsmobility,preventscontractures,facilitates
reoperation A63ͲyearͲoldwomanwithrheumatoidarthritisisscheduledforknee
• Elution:highlevelswithinthefirstfewdays arthroplastyin2weeks.Shetakesmethotrexate,hydroxychloroquine
Ё Localtissueconcentrationexceedssystemicdelivery andlowdoseprednisone(2.5mgdaily).Shehasahistoryofrecurrent
Ё Mayeluteformonthsorlonger urinarytractinfections,lastonemonthago.Sheaskshowshemight
• Antimicrobialconsiderations preventinfectionafterkneereplacement.
Ё Knownorsuspectedorganisms
Ё Thermalstability(avoidmostɴͲlactams)
Ё Osteocytetoxicity(avoidquinolones)
Ё Vancomycinandaminoglycosidesmostcommon
Ё Toxicityandallergyreportedbutrare

32
29 33
30

©2024InfectiousDiseaseBoardReview,LLC
243
50– BoneandJointInfections
Speaker:SandraNelson,MD

Case#4:Vote PreventionofPJI
• Immunosuppressives:
Whatdoyouadvise? Ё Stopbiologics,noneedtostopDMARDsorlowdoseprednisone
A. Stopmethotrexateandprednisonenow(twoweekspreoperative) • Surgicalantibioticprophylaxis:onedosepriortosurgery
B. ScreenforStaphaureuscolonization;decolonizeifpresent • Urinarytractinfections:
C. ScreeningUAandurineculture,treatifpositive Ё DiagnoseandtreatsymptomaticUTI
Ё Donotscreenforasymptomaticbacteriuria
D. 48hoursperioperativeprophylaxiswithcefazolin
• Dentalprophylaxis:nomore!
E. Amoxicillinpriortodentalproceduresfor2yearspostoperatively
• Staphaureusdecolonizationreducessurgicalsiteinfection

34
31 36
32

Case#5
Microbiologyof
A56ͲyearͲoldmanwithpoorlycontrolleddiabetespresentstoEDwitha
Musculoskeletal oneͲweekhistoryoflowͲgradefeversandgraduallyprogressiverightknee
painandswelling.HetraveledtotheDominicanRepubliconemonthagoand
Infections hadnoillnesseswhiletraveling.Helastsawadentistsixmonthsagoand
deniestoothpain.Thereisnohistoryofinjectiondruguse.

Onexamhehasamoderateeffusionandpainwithpassiverangeofmotion
oftheknee.HisESR(68)andCRP(17mg/dL)areelevated,andsynovialfluid
isinflammatory(45,000WBCs,with82%neutrophils)withanegativegram
stain.

3733 38
34

Case#5:Vote GuesstheBug
Mycobacterium
abscessus
Culturegrowthat3daysincubation Whatisthemostlikelyorganism? MusculoskeletalEdition Nocardia Candida
A. Serratiamarcescens
B. Salmonellaheidelberg
C. Staphylococcusaureus
Neisseria
D. Kingella kingae Mycoplasma gonorrhoeae
E. Pasteurellamultocida
Borrelia
burgdorferi

35 36

©2024InfectiousDiseaseBoardReview,LLC
244
50– BoneandJointInfections
Speaker:SandraNelson,MD

SalmonellaSpecies SerratiaandPseudomonas
• Clinical
Ё Seeninsickle celldisease,immunocompromised,diabetes
• RiskFactors
Ё Hematogenousinfection(septicarthritis,spondylodiscitis,longboneinfection) Ё Injectiondruguse(tapwater)
Ё Immunocompromisedhost
• Epidemiology Ё Indwellinglines
Ё Reptileexposure
• Clinicalfactors
Ё Traveltodevelopingworld
Ё Unsafefoodhygiene Ё Usuallyhematogenous
Ё Predilectionforsacroiliacandsternoclavicularjointsininjectiondruguse

37 38

HACEKOrganisms Brucellaspecies
• Clinical
• Clinical
Ё Usuallyhematogenous Ё Feversoftenprecedemusculoskeletalsymptoms
• Epidemiology Ё SepticarthritiswithpredilectionforsacroͲiliacjoint
Ё Alsocausesspondylodiscitis
Ё Antecedentmouthtrauma,gumordental
infection,ordentalprocedure • Epidemiology
Ё Odontogenicinfectionmaybesilent Ё EndemicinLatinAmerica,Mediterranean,MiddleEast,parts
ofAsia
• Microbiology Ё Consumptionofunpasteurizeddairymostcommon
Ё Lategrowthinculture,maybeculturenegative • Microbiology
Ё SmallgramͲnegativecoccobacillus;growslateinculture
• Kingella kingae
Ё Laboratorybiohazard
Ё Mostcommoncauseofosteoarticularinfectionin Ё SerologieshelpfulinnonͲresidentsofendemicareas
youngchildren;diagnosedbypcr

39 40

Pasteurellaspecies Mycoplasmahominis
• Hostfactors
• Clinical Ё Immunodeficiency,especiallyhumoral(CVID,XLA)
Ё Directinoculation(bite) Ё Postpartumwomen
Ё Hematogenousspread • Clinicalfactors:hematogenousinfection
Ё Rapidclinicalonset
• Microbiology
• Epidemiology Ё Difficulttogrowinroutineculture
Ё Exposuretocats/dogs Ё “Friedegg”morphologyinculture
Ё Bitehistorynotalwayselicitedinhematogenous
infection

41 42

©2024InfectiousDiseaseBoardReview,LLC
245
50– BoneandJointInfections
Speaker:SandraNelson,MD

Borreliaburgdorferi(Lyme) NonͲtuberculousmycobacteria
• Clinical • Clinical
Ё Largeeffusions;someresolveoverweeksbutmayrecur Ё Slowlyprogressivetenosynovitis;canspreadtobonesand
joints
Ё Warmthandswellingoutofproportiontopain Ё Maybeaccompaniedbynodularlymphangitis
Ё MonoͲarthritisofthekneemostcommon Ё Maycausepolyarthritisinimmunocompromisedhosts

• Epidemiology • Epidemiology
Ё Environmentalsourcesofwater
Ё NortheastU.S.anduppermidͲwestwithtickexposure
Ё Marineinjury/trauma
• Micro:cultureͲnegative Ё FishͲtankexposure
Ё DiagnosedserologicallyorwithsynovialfluidBorreliapcr • Microbiology
Ё Someorganisms(marinum)growbetterincooler
temperatures

43 44

Yeastsandmolds Endemicmycoses
• Clinical • Coccidiodes andBlastomyces>Histoplasma
Ё Maybecontiguousinoculationorhematogenousspread
• Clinical
Ё Oftenmoreindolentthanbacterialorganisms
Ё Inthespinemaymimictuberculosis Ё Subacutesepticarthritisandlongboneosteomyelitis
Ё Mayseedrainingsinusesadjacenttoosteomyelitis
• Epidemiology Ё Inspine,mayalsomimictuberculosis
Ё Candida:injectiondruguse,indwellinglines, Ё Hostimmunocompromisemorecommonincoccidioides
immunocompromise,antibioticexposure
Ё Molds:soilcontamination(trauma),barefootwalking Ё Mayseeconcomitantpulmonaryinfection
(Madurafoot),immunocompromise(neutropenia),medical
tourism

Karrakchou BMCDermatology2020

45 46

Thankyou!

43
47

©2024InfectiousDiseaseBoardReview,LLC
246
Wednesday, August 21, 2024

BR5

Board Review Session 5

Drs. Masur (Moderator), Bennett, Chambers,


Mitre, Nelson, and Rose

©2024 Infectious Disease Board Review, LLC


COPYRIGHT NOTICE: The Copyright Act (Title 17 of U.S. Code) governs the rights attributed to owners of
copyrighted work. Under certain circumstances, educational institutions may provide copies of
copyrighted works to continuing education participants. The copies may not be copied nor used for any
other purpose besides private study, scholarship, or research. Participants should not provide electronic
or print copies of ant materials provided by the university to unauthorized users. If a participant fails to
comply with these restrictions, the participant may be held liable for copyright infringement. No further
transmission or electronic distribution is permitted. 247
248
BR5 –Board Review: Day 5
Moderator: Henry Masur, MD

BOARD REVIEW DAY 51 2024


#53 A 40-year-old white male was switched to a
darunavir+ritonavir+tenofovir (TDF)/emtricitabine
regimen, secondary to virologic failure and multiple PI
mutations including I50L.
He presents to the clinic 6 months after the switch for
a routine evaluation.
Board Review: Day 5 Current CD4 = 350 and VL = <20 copies
Moderator: Henry Masur, MD
Faculty: Drs. Bennett, Chambers, Mitre,
Nelson, and Rose 7/1/2024 1 of 4

BOARD REVIEW DAY 51 2024 BOARD REVIEW DAY 51 2024


#53 Lipids drawn during the evaluation reveal #53 What is the most appropriate management for his
hypertriglyceridemia?
• Total cholesterol of 260 mg/dL

• LDL 130 mg/dL


A) Switch patient to an atazanavir-based regimen
• HDL 35 mg/dL
B) Increase exercise regimen
• Triglycerides 1200 mg/dL
C) Fenofibrate
D) Fat-free diet

2 of 4 3 of 4

BOARD REVIEW DAY 51 2024 BOARD REVIEW DAY 51 2024


#54 A 77-year-old woman with insulin-dependent #54 On examination, she is afebrile.
diabetes is seen for 4 days of severe right ear
pain. The pinnae appear normal and symmetrical but
tugging on the right external ear produces pain.
The pain is worsened by chewing.
The right ear canal appears moist and is partially
She has no previous history of ear problems and occluded by heaped-up granulation tissue.
has not had fever.
The part of the tympanic membrane that can be
She says that the ear feels wet, and that there is a seen is normal.
yellow stain on her pillowcase in the morning.
1 of 5 2 of 5

©2024 Infectious Disease Board Review, LLC


249
BR5 –Board Review: Day 5
Moderator: Henry Masur, MD

BOARD REVIEW DAY 51 2024 BOARD REVIEW DAY 51 2024


#54 There is no mastoid tenderness and hearing is #54 Pending culture results, which one of the following
grossly normal. antimicrobials is most appropriate for this patient?
There is mild facial nerve palsy on the right side. A) Ciprofloxacin
The rest of the exam is unremarkable except for B) Amphotericin B
the sequelae of diabetes.
C) Clindamycin
D) Gentamicin ear drops
E) Amoxicillin-clavulanate
3 of 5 4 of 5

BOARD REVIEW DAY 51 2024 BOARD REVIEW DAY 51 2024


#55 A 42 yo female who injects drugs is admitted with fever #55 Chest x-ray shows multiple bilateral nodular infiltrates
and chest pain of 4 days duration. consistent with septic pulmonary emboli.

Past medical history is positive for several prior Three blood cultures are drawn, and she is empirically
episodes of cutaneous abscesses not requiring treated with vancomycin and ceftriaxone.
hospitalization.
The following day, hospital day 2, all three blood
She takes no medications and is allergic to sulfa drugs. cultures are reported positive for Gram-positive cocci in
clusters.
There is a 4 out of 6 systolic murmur at the lower left
sternal border. A transthoracic echocardiogram shows a 1.2 cm mobile
mass on the posterior leaflet of the tricuspid valve.
1 of 5 2 of 5

BOARD REVIEW DAY 51 2024 BOARD REVIEW DAY 51 2024


#55 Two blood cultures are drawn, and the ceftriaxone is #55 Which one of the following drug regimens is most
discontinued. appropriate for treatment of this patient?
The following day, the isolate from the admission blood
A) Cefazolin plus gentamicin
cultures is identified as methicillin-susceptible
Staphylococcus aureus (MSSA), resistant to penicillin, and B) Daptomycin
erythromycin.

Vancomycin is discontinued and cefazolin is administered.


C) Daptomycin plus ceftaroline

Serial daily blood cultures continue to be positive through D) Nafcillin


hospital day 5. You are asked to see the patient to
recommend salvage therapy. E) Vancomycin
3 of 5 4 of 5

©2024 Infectious Disease Board Review, LLC


250
BR5 –Board Review: Day 5
Moderator: Henry Masur, MD

BOARD REVIEW DAY 51 2024 BOARD REVIEW DAY 51 2024


#56 A 35-year-old male traveled outside of the United #56 CBC and chemistry panel are normal.
States for the first time, going on a safari to Botswana,
Africa for 3 weeks. The astute ER physician finds out the patient did not
take his malarone prophylaxis after the first week in
5 days after returning from Botswana, he developed a Botswana.
fever to 39.1°C for 12 hours and comes to the
Emergency Room. He also complains of malaise and The ER physician performs a blood smear for malaria:
headache. no parasites are seen by an experienced technician. A
rapid test is also negative.
His physical examination is normal. No splenomegaly
is detected by physical examination. You are asked if additional evaluation for malaria is
needed.
1 of 4 2 of 4

BOARD REVIEW DAY 51 2024 BOARD REVIEW DAY 51 2024


#56 You recommend? #57 A 62-year-old woman enters the hospital and is
A) The initial blood smear that is negative rules out malaria scheduled for multiple tests to evaluate masses in
as a cause of the fever her colon and liver that are suspected of being
B) The initial rapid malaria test that is negative rule out neoplastic.
malaria as a cause of fever
She has previously been well and takes no drugs
C) A rapid antibody test should be performed: together with other than a statin and every other day aspirin.
a negative rapid antigen test, malaria would be ruled out
as a cause of fever She is afebrile, has a normal WBC, and her
D) Further testing with malaria smears and/or rapid test
complaints and findings on exam are related to her
should be done every 12 hours for 2-3 days to rule of hepatic lesions.
malaria 3 of 4 1 of 4

BOARD REVIEW DAY 51 2024 BOARD REVIEW DAY 51 2024


#57 Six years prior to this admission the patient had a #57 For which of the following procedures should this patient
left shoulder replacement and two years ago she receive antimicrobial prophylaxis to avoid infecting one of
her prosthetic joints?
had a knee replacement.
A) Bronchoscopy with transbronchial biopsy

Both joint replacements were uncomplicated, and B) Extraction of a decayed wisdom tooth
she has not had any change in joint function over C) Colonoscopy with biopsy of a suspected carcinoma
the past year. D) Percutaneous liver biopsy
E) The patient does not need antimicrobial prophylaxis for
any procedure
2 of 4 3 of 4

©2024 Infectious Disease Board Review, LLC


251
BR5 –Board Review: Day 5
Moderator: Henry Masur, MD

BOARD REVIEW DAY 51 2024 BOARD REVIEW DAY 51 2024


#58 A 16 yr old male high school student from #58 He grew up in Iran, but his family has moved the
suburban Alexandria, Virginia presented with USA four years previously.
episodes during the past three months when he On exam, he was afebrile and appeared healthy.
felt like his heart was “bursting from his chest”
when he was doing push-ups in gym class. A grade 3 systolic and diastolic murmur was heard
at the left sternal border.
This went away promptly when he stopped
exercising. He said it didn’t feel like skipped beats Echocardiogram found mitral stenosis and
and was not associated with chest pain or regurgitation, with a thickened mitral valve without
dyspnea. vegetations and an enlarged left atrium.

1 of 5 2 of 5

BOARD REVIEW DAY 51 2024 BOARD REVIEW DAY 51 2024


#58 EKG showed first degree heart block with a PR #58 Which of these tests might be helpful in
interval of 300 msec and no extrasystoles. diagnosis?
Routine chemistries and CBC were normal but
CRP and ESR were elevated. A) Anticardiolipin IgG
B) Anti dsDNA
C) Anti Coxiella burnetii phase 2 IgG
D) Anti streptococcal DNase B
E) PCR on blood for Tropheryma whipplei
3 of 5 4 of 5

BOARD REVIEW DAY 51 2024 BOARD REVIEW DAY 51 2024


#59 A 56-year-old woman with poorly controlled type 2 #59 CT showed air in
diabetes mellitus presents with fever, rigors and her left atrium (see
near syncope. In the Emergency Room she is figure) and multiple
brain abscesses.
confused, hypotensive, and febrile to 38.5°C.
Her WBC is 18,000 with 90% neutrophils.
The left atrium
She is given bolus IV fluids for shock, started on shows air in the
vancomycin and piperacillin-tazobactam and atrium (black
admitted. arrow) and the
esophagus (white
arrow).
1 of 5 2 of 5

©2024 Infectious Disease Board Review, LLC


252
BR5 –Board Review: Day 5
Moderator: Henry Masur, MD

BOARD REVIEW DAY 51 2024 BOARD REVIEW DAY 51 2024


#59 Her past history is remarkable for a dental #59 The most likely cause of this syndrome is:
extraction one week prior.
No prophylaxis was given. A) Aortic valve endocarditis
B) Atriobronchial fistula
Four weeks prior she had her second catheter
radiofrequency ablation for atrial fibrillation. C) Atrioesophageal fistula
D) Mucormycosis of the left atrium
E) Atrial myxoma

3 of 5 4 of 5

BOARD REVIEW DAY 51 2024 BOARD REVIEW DAY 51 2024


#60 A woman in her 60s presented with three months #60 A Chest CT showed a left upper lobe lung nodule.
of non-productive cough, night sweats and a 10- Pathology from a lung biopsy is shown.
pound weight loss.
She had received empiric clarithromycin, but the
symptoms persisted.
The patient had traveled in the past year to South
America, Europe, Arizona, and Australia.

1 of 4 2 of 4

BOARD REVIEW DAY 51 2024 BOARD REVIEW DAY 51 2024


#60 The most likely diagnosis is which of the #61 A 25-year-old man who uses injection drugs presents
following? to the hospital with low back pain and tenderness.
These symptoms began 3 days earlier.

A) Histoplasmosis He denies lower extremity weakness.


B) Cryptococcosis On physical examination, his temperature is 102°F,
pulse is 120/minute, and his blood pressure is normal.
C) Pneumocystis jirovecii
D) Coccidioidomycosis The physical exam reveals tenderness on palpation of
his lumbosacral spine region. There is no evidence of
E) Paracoccidioidomycosis lower extremity weakness.
3 of 4 1 of 4

©2024 Infectious Disease Board Review, LLC


253
BR5 –Board Review: Day 5
Moderator: Henry Masur, MD

BOARD REVIEW DAY 51 2024 BOARD REVIEW DAY 51 2024


#61 He undergoes an MRI with contrast of his entire spine, #61 Which of the following is the most appropriate next step in
which reveals evidence of discitis at L1-L2 disc space, management?
and evidence of osteomyelitis of the inferior L1 and
superior L2 vertebrae, and the presence of a 1 cm A) Continue vancomycin and monitor his clinical
ventral epidural abscess. examination and blood culture results
He is started on therapy with vancomycin and B) Change therapy to ceftaroline and daptomycin
cefepime. C) Consult interventional radiology for aspiration of the
Blood cultures reveal gram-positive cocci, epidural abscess to obtain microbiologic data
subsequently identified as methicillin-resistant D) Consult neurosurgery for laminectomy and drainage of
Staphylococcus aureus. Cefepime is discontinued. the epidural abscess
2 of 4 3 of 4

BOARD REVIEW DAY 51 2024 BOARD REVIEW DAY 51 2024


#62 A 36-year-old healthy man presents with a 10-month #62 He works as a field biologist performing reptile
history of ulcerating skin lesions. These initially surveys. In the months preceding onset of the initial
started on his right ear following a minor trauma. skin lesion, he was working in Mexico and
Guatemala.
He subsequently developed similar lesions on his
scalp and right wrist. He was given a course of He denies known arthropod assault at the site of the
Keflex with no response. lesions but does note he is frequently bitten by
mosquitos and other insects while working in the
He denies fevers or systemic symptoms but is field.
bothered by the cosmetic appearance of these
lesions. He is afebrile and vital signs are within normal limits.
1 of 5 2 of 5

BOARD REVIEW DAY 51 2024 BOARD REVIEW DAY 51 2024


#62 His skin lesions are shown below. #62 What is the most likely cause of this patient’s
lesions?
A) Yaws
B) Leishmania mexicana
C) Paracoccidioides brasiliensis
D) Pyoderma gangrenosum

CBC with differential and CMP are within normal limits. E) Mycobacterium leprae
HIV antibody testing is negative. 3 of 5 4 of 5

©2024 Infectious Disease Board Review, LLC


254
BR5 –Board Review: Day 5
Moderator: Henry Masur, MD

BOARD REVIEW DAY 51 2024 BOARD REVIEW DAY 51 2024


#63 A 46-year-old male with poorly controlled diabetes and #63 He takes metformin, semaglutide, atorvastatin, and
obesity presents to the emergency department with fever valsartan.
and erythema involving his left forefoot.
He does not have a prior history of foot infections. He has
He has had diabetes mellitus for 24 years; his last A1C no history of MRSA infection.
was 10.4%. He lives in Delaware with his wife and two young children,
all of whom are well.
He works in construction and first noted skin breakdown
over the medial metatarsophalangeal joint three weeks In the ED, he was afebrile and normotensive. There was
ago that he attributed to irritation from a new pair of an ulcer involving the medial metatarsophalangeal joint
shoes. The day prior to presentation he noted increased that measured 1.5 wide and 0.6 cm deep. There was moist
pain, swelling, and drainage on his socks. drainage on the sock but no expressible purulence and
1 of 5 no malodor. 2 of 5

BOARD REVIEW DAY 51 2024 BOARD REVIEW DAY 51 2024


#63 The dorsalis pedis pulse was faint, and probe-to-bone test #63 What do you recommend as the best next step?
was negative. There was tender erythema that involved
the forefoot and midfoot. A) Discharge from the emergency department on oral
amoxicillin-clavulanate
Plain films demonstrated no cortical erosion and no soft B) Hospitalization and initiation of ampicillin-sulbactam after
tissue gas. wound cultures are collected
ESR and CRP are elevated at 42 mm/Hr and CRP 84 mg/L. C) Hospitalization and initiation of vancomycin and piperacillin-
tazobactam after wound cultures are collected
D) Addition of topical silver sulfadiazine to the wound in
addition to systemic antibiotic therapy
E) Surgical evaluation for debridement
3 of 5 4 of 5

BOARD REVIEW DAY 51 2024 BOARD REVIEW DAY 51 2024


#64 A 38-year-old male is evaluated in the emergency room #64 • WBC 9,000 cells/L
for onset of abdominal pain and vomiting starting 4 hours • Hemoglobin 14 g/dl
ago.
• amylase 450 (nl 23-85) U/L,
He does not have any significant past medical history but • lipase 643 (nl 0-160) U/L
has been drinking 4-5 shots of vodka daily for the past 2 • AST 45 (nl 10-40) U/L
years.
• ALT 65 (nl 7-56) U/L
On examination, his temperature is 100.1°F, pulse 100, • ALK 120 (nl 20-140) U/L
blood pressure is 140/80 and respiration of 18. Liver is • TBili 1.1 mg/dL (nl 0.3-1.3)
palpable 1 cm below right costal margin and there is pain
• CRP 23 (nl <0.5) mg/dL
on deep palpation at the epigastrium.
1 of 5 2 of 5

©2024 Infectious Disease Board Review, LLC


255
BR5 –Board Review: Day 5
Moderator: Henry Masur, MD

BOARD REVIEW DAY 51 2024 BOARD REVIEW DAY 51 2024


#64 Plain film of the abdomen revealed an ileus. #64 Which of the following is the most appropriate initial
management?
CT of the abdomen showed diffusely enlarged pancreas
with evidence of phlegmonous changes. A) Upper endoscopy
B) Surgical exploration
C) Hydration and pain control
D) IV antibiotics to cover Gram-negative and anaerobic
bacteria
E) ERCP or MRCP (endoscopic cholangiopancreatography or
magnetic resonance cholangiopancreatography)

3 of 5 4 of 5

BOARD REVIEW DAY 51 2024 BOARD REVIEW DAY 51 2024


#65 A 68-year-old diabetic woman is admitted for pneumonia. #65 She has been receiving vancomycin 1 gram once daily for
the past 3 days based on a calculated creatinine
• A rapid RT-PCR test is positive for influenza A clearance of 45 ml/min and has a trough concentration of
22 μg/ml.
• Sputum Gram-stain shows Gram-positive cocci in
clusters The primary physician is concerned because a
vancomycin susceptibility result has returned with MIC =
• The culture grows methicillin-resistant Staphylococcus
2 μg/ml and has asked you to recommend an alternative
aureus susceptible to vancomycin, daptomycin,
agent.
linezolid, and trimethoprim-sulfamethoxazole (TMP/SMX)

1 of 4 2 of 4

BOARD REVIEW DAY 51 2024


#65 Which one of the following alternative regimens would
you recommend?

A) Daptomycin 6 mg/kg IV once daily

B) Linezolid 600 mg PO twice daily

C) Telavancin 7.5 mg/kg IV once daily

D) TMP-SMX 10 mg/kg (TMP component) per day in 2


divided doses
3 of 4

©2024 Infectious Disease Board Review, LLC


256
Wednesday, August 21, 2024

51

Lots of Protozoa

Dr. Edward Mitre

©2024 Infectious Disease Board Review, LLC


COPYRIGHT NOTICE: The Copyright Act (Title 17 of U.S. Code) governs the rights attributed to owners of
copyrighted work. Under certain circumstances, educational institutions may provide copies of
copyrighted works to continuing education participants. The copies may not be copied nor used for any
other purpose besides private study, scholarship, or research. Participants should not provide electronic
or print copies of ant materials provided by the university to unauthorized users. If a participant fails to
comply with these restrictions, the participant may be held liable for copyright infringement. No further
transmission or electronic distribution is permitted.
257
258
51– LotsofProtozoa
Speaker:EdwardMitre,MD

Lecture Title

• Disclosures of Financial Relationships with Relevant


Lots of Protozoa Commercial Interests

- None
Edward Mitre, MD
Rockville, MD

Disclaimer: Dr. Mitre is giving this presentation in a personal capacity. The views expressed in this presentation
are the sole responsibility of the presenter and do not necessarily reflect the views, opinions, or policies of the
Uniformed Services University of the Health Sciences, the Department of Defense, or the United States
Government.

Protozoa
Protozoa - Extraintestinal Protozoa - Intestinal
Apicomplexa Apicomplexa
Plasmodium Cryptosporidium
(Babesia) Cyclospora
(Toxoplasma) Cystoisospora

Flagellates Flagellates
Leishmania Giardia
Trypanosomes Dientamoeba
(Trichomonas) Amoebae
Entamoeba
Amoebae
Naegleria Ciliates
Acanthamoeba Balantidium
Balamuthia

Maybe Not Protozoa Kingdom Fungi: Microsporidiosis agents


Domain SAR: Blastocystis

Protozoa Question 1: A 54 yo woman presents with fever, chills, and oliguria


Protozoa - Extraintestinal Protozoa - Intestinal one week after travel to Malaysia.
Apicomplexa Apicomplexa
Vitals: 39.0o C, HR 96/min, RR 24/min, BP 86/50
Plasmodium Cryptosporidium
(Babesia) Cyclospora
Labs: Hct 31%, platelets 14,000/ȝl, Cr of 3.2 mg/dL.
(Toxoplasma) Cystoisospora

Flagellates Flagellates Peripheral blood smear has intraerythrocytic forms that are
Leishmania Giardia
Dientamoeba morphologically consistent with Plasmodium malariae.
Trypanosomes
(Trichomonas) Amoebae
Entamoeba
The most likely infectious agent causing the patient’s illness is:
National Institutes
of Health
Amoebae
Naegleria Ciliates
Acanthamoeba Balantidium A. Plasmodium malariae
Balamuthia B. Plasmodium knowlesi
C. Plasmodium vivax
National Institute of D. Plasmodium
National Institute of falciparum
Maybe Not Protozoa Kingdom Fungi: Microsporidiosis agents
Allergy and Infectious
Diseases
Allergy and Infectious
E. Babesia microti
Diseases
Domain SAR: Blastocystis

©2024 Infectious Disease Board Review, LLC 259


51– LotsofProtozoa
Speaker:EdwardMitre,MD

MALARIA
P. knowlesi one of the most important pathogens in the history of
the world
morphologically similar to P. malariae

usually a parasite of long-tailed


macaques

increasingly recognized in Myanmar,


Philippines, Indonesia, and Thailand
In 1775 the Continental Congress purchased
causes high parasitemia National Malaria Elimination Program: 1947- 1951
Æ DDT spraying, drainage of wetlands quinine for George Washington’s troops
Æ Atlanta was chosen for the Office of Malaria Control in
War Areas (the predecessor agency of the CDC) in part
highly morbid and can be lethal because of its location in a malaria-endemic region

Disease Burden - WHO 2023 World Malaria Report MALARIA EPIDEMIOLOGY – CDC map from a few years ago…what’s missing?
249 million cases
608,000 deaths
U.S. ~ 2000 cases reported each year

https://www.cdc.gov/malaria/about/distribution.html

2023 U.S. Malaria Cases


1 P. falciparum

9
case
2023
In non-immune patients, falciparum
P. vivax cases
2023
MD malaria is a medical emergency!!
1- TX June
1 – AR Oct
7 – FL May - July
one of the most common causes of fever in a
First autochthonous returned traveler
cases in U.S. in 20
years! Last was 2003
when 8 cases of P.
vivax were reported
infected individuals can rapidly progress from
2023 7 P. vivax cases in Florida
• All within 4 miles of each other in
Sarasota county
in Palm Beach appearing well to being critically ill
• All with fever and low platelets
County, FL.
• 3 individuals were homeless
• April 20th there had been an imported P.
vivax case
• CDC testing of 407 Anopheles
mosquitoes Æ 3 A. crucians were PCR+
https://www.cdc.gov/mmwr/volumes/72/wr/mm7236a1.htm https://www.cdc.gov/malaria/about/distribution.html

©2024 Infectious Disease Board Review, LLC 260


51– LotsofProtozoa
Speaker:EdwardMitre,MD

---Some helpful heuristics--- ---Some helpful heuristics---


If patient has make sure patient doesn’t have If patient has make sure patient doesn’t have

Fever and freshwater contact-------------------> Fever and freshwater contact------------------->


Malaria
Fever and unpasteurized milk-------------------> Fever and unpasteurized milk------------------->
Fever and undercooked meat--------------------> Fever and undercooked meat--------------------> Malaria
Fever and raw vegetables-------------------------> Fever and raw vegetables-------------------------> Malaria
Fever and untreated water------------------------> Fever and untreated water------------------------> Malaria
Fever and wild dog bite----------------------------> Fever and wild dog bite----------------------------> Malaria
Fever and abdominal pain------------------------> Fever and abdominal pain------------------------> Malaria
Fever and headache---------------------------------> Fever and headache--------------------------------->
Malaria
Fever and diarrhea-----------------------------------> Fever and diarrhea----------------------------------->
Fever and cough--------------------------------------> Fever and cough--------------------------------------> Malaria
Fever and dysuria------------------------------------> Fever and dysuria------------------------------------>
Malaria

Sporozoites
• Infective stage
• Come from mosquito
characteristics of human malaria species
Liver schizont
• Asymptomatic replicative stage
• Become 10,000 to 30,000 merozoites
Hypnozoite
• Dormant liver stage in vivax and ovale
• Release merozoites weeks to months
after primary infection

Merozoites
• Infect RBCs and develop into ring-stage
trophozoites
• Mature into schizonts, which release
merozoites which infect more RBCs
Gametocytes
• Infective stage for mosquitoes

Possible evolutionary defenses against malaria


Uncomplicated (mild) malaria
Symptoms: fevers, chills, headache, fatigue
Duffy antigen negative (P. vivax uses Duffy Ag to enter RBCs) *NOTE: abdominal pain presenting symptom in 20%

Sickle cell trait (increases survival during P. falciparum infection, Æ periodicity of fevers not common when patients seen acutely
perhaps by selective sickling of infected RBCs)

Labs: thrombocytopenia in 50%


Glucose-6-phosphate dehydrogenase deficiency mild anemia in 30%
(malaria parasites grow poorly in G6PD deficient RBCs, perhaps b/c typically no leukocytosis
this results in an overall increase in reactive oxygen species in RBCs)
may see evidence of hemolysis with mild increase T bili and LDH

©2024 Infectious Disease Board Review, LLC 261


51– LotsofProtozoa
Speaker:EdwardMitre,MD

P. vivax or ovale
Complicated (severe) malaria
• intracellular Schüffner’s dots
• Cerebral malaria (altered mental status, seizures) Often seen in children of • enlarged infected cells
• Respiratory distress/pulmonary edema endemic countries.
• Severe anemia (hct <15% in children, <20% in adults) Adults more often get
multiorgan failure.
• Renal failure
• Hypoglycemia
• Shock (SBP < 80 mm Hg or capillary refill > 3 seconds)
• Acidosis (often lactic acidosis) P. ovale
• Jaundice (total bilirubin > 3 mg/dL)
• Bleeding disorder (spontaneous bleeding or evidence of DIC)
• elongated or oval
These complications primarily occur with • 6-12 merozoites (vs 12-24 for vivax)
Plasmodium falciparum, usually when parasitemia • 2%.

NOTE: in the absence of end organ damage, parasitemia • 5% is


often used as the cut-off to treat for severe malaria in the U.S.

P. malariae Malaria: Diagnosis


Rapid diagnostic antigen tests
Æ sensitivity > 90% for P. falciparum
• band form (about 85% for P. vivax, lower for P. knowlesi and P. ovale)
(also seen in P. knowlesi)

P. falciparum

for P. falciparum (T1) Æ tests for histidine-rich protein 2


• Banana shaped gametocyte for all species (T2) Æ tests for aldolase

*Most false-negative antigen tests are due to low parasite burden


Æ Retest suspected patients that initially test negative

*Increasing false negative cases occurring worldwide due to mutations in HRP2

Malaria Chemoprophylaxis (note: no vax for travelers)

CENTRAL AMERICA and MIDDLE EAST


Question 2: A 33-year-old woman is traveling Pre-Exposure During Post-Travel
Chloroquine 1 tab/wk x 2 wks 1 tab/wk 4 weeks
to Uganda to do field studies in anthropology. 500mg tabs

She is two months pregnant. Which of the EVERYWHERE


Atovaquone/proguanil 1 tab daily x 2 d 1 daily 7 days
following do you prescribe for malaria 250/100mg
Doxycycline none 1 daily 4 weeks
prophylaxis? 100mg tabs
Tafenoquine* 2 tab daily x 3 d 2 tab/wk 2 tab after 1 wk
A. Doxycycline 100mg tabs

B. Chloroquine Mefloquine (not SE Asia)** 1tab/wk x 2-3 wks 1 tab/wk 4 weeks


C. Mefloquine 250mg tabs
* Tafenoquine can precipitate severe hemolytic anemia in individuals that are G6PD deficient
D. Atovaquone/proguanil
** FDA black box warning mefloquine can cause neurologic symptoms, hallucinations, and
E. No prophylaxis feelings of anxiety, mistrust, and depression. Can also cause QT prolongation. Thus, many
U.S. practitioners now reserve mefloquine for pregnant travelers to areas with chloroquine
resistance

©2024 Infectious Disease Board Review, LLC 262


51– LotsofProtozoa
Speaker:EdwardMitre,MD

P. falciparum treatment P. vivax/P.ovale Treatment


Excellent review Æ 2022 JAMA, 328(5):460-47, PMID: 3591684
chloroquine x 3 days, or ACT (artemether/lumefrantrine in U.S.)
Uncomplicated P. falciparum malaria (no organ dysfunction, low parasitemia, able take po) note: PNG, Indonesia, Oceania have CLQ R P. vivax Æ use ACT
if chloroquine sensitive area Æ chloroquine or hydroxychloroquine
then ANTIRELAPSE THERAPY with primaquine or tafenoquine
if not chloroquine sensitive area (most cases) Æ artemether/lumefantrine (Coartem)
ACTs are treatment of choice, WHO 2022 guidelines Æ Need to check G6PD status before administering primaquine OR tafenoquine
alternatives if artemether/lumefrantrine not available: (as both can cause severe hemolysis in patients with G6PD deficiency)
atovaquone/proguanil (Malarone), quinine + doxycycline, mefloquine
Æ Both primaquine and tafenoquine contraindicated during pregnancy
Severe Malaria භ primaquine – weight based dosing and duration as determined by G6PD activity
• Æ IV artesunate (CDC malaria hotline: 770-488-7788) ***ALWAYS LOOK UP DOSING BEFORE ADMINISTERING***
-> usually 30 mg primaquine base per day x 14 days if normal G6PD activity
-> do not exceed 30 mg primaquine base per day
NOTES -> if over 70 kg, can calculate total dose 6 mg/kg and then extend duration of 30 mg daily doses until total goal met
1) Treatment failures can occur with artemether/lumefrantrine, especially when > 65 kg -> if G6PD deficient consider weekly chloroquine x 1 year
Sonden K. et al, Clinical Infectious Diseases 2017 PMID: 27986683
භ tafenoquine (two 150 mg tabs once, given on 1st or 2nd day of chloroquine therapy)
2) Artemisinin resistance reported in SE Asia (Cambodia, Laos, Myanmar, Thailand, Vietnam), parts of Africa (Uganda,
(Tafenoquine was approved for radical cure of P. vivax in 2018, P. ovale treatment is off-label)
Rwanda), and in S. America (Guyuna)
3) Delayed-onset anemia in 2.7% of U.S. patients after treatment with artesunate
Abanyie F. et al, Clinical Infectious Diseases 2022 PMID: 36052468
2020 : Company (GSK) reported some failures when tafenoquine was used after ACT treatment of P. vivax.
4) Hypoglycemia and ARDS are complications that can occur during treatment of malaria. ARDS sometimes develops NEW FDA LABELING: Tafenoquine now only approved and recommended after chloroquine treatment
even as pt improving from malaria.
https://www.cdc.gov/malaria/hcp/clinical-guidance/treatment-uncomplicated.html

Babesia
* Suggestions for all ID practitioners * Transmission
• Ixodes ticks in Northeast and upper midwest
Æ co-infection with Lyme and Anaplasma
• Transfusion
1) Make sure the facility where one works has the means to (Ab screening tests approved by FDA in 2018)
rapidly test for malaria
Symptoms: fever, headache, chills, myalgias
less common: nausea, dry cough, neck stiffness,
2) Ensure that hospital pharmacy has access to appropriate vomiting, diarrhea, arthralgias
medications for treatment of malaria Æ severe disease: in HIV, asplenia

Labs: anemia, thrombocytopenia, mild increase LFTs,


normal/low/high WBC

Diagnosis: small ring forms in RBCs, PCR, Ab


merozoites can make tetrad (“Maltese cross”)
Treatment: azithromycin + atovaquone
(clindamycin + quinine is alternative)
Æ Exchange transfusion for severe disease CDC DpDx

Protozoa Leishmaniasis
> obligate intracellular protozoan infection
Protozoa - Extraintestinal Protozoa - Intestinal
> transmitted by sand flies (noiseless, active in evenings)
Apicomplexa Apicomplexa
Plasmodium Cryptosporidium
Babesia Cyclospora
(Toxoplasma) Cystoisospora Lutzomyia Phlebotomus
New world leishmaniasis Old world leishmaniasis
Flagellates Flagellates
Leishmania Giardia
Trypanosomes Dientamoeba
(Trichomonas) Amoebae
Entamoeba
Amoebae
Naegleria Ciliates
Acanthamoeba Balantidium
Balamuthia

Maybe Not Protozoa Kingdom Fungi: Microsporidiosis agents


Domain SAR: Blastocystis

©2024 Infectious Disease Board Review, LLC 263


51– LotsofProtozoa
Speaker:EdwardMitre,MD

Leishmania life cycle – Two stages Question 3: A 42 yo man from Bolivia presents with
nasal stuffiness and is found to have nasal septal
Promastigote Amastigote perforation. Biopsy demonstrates intracellular
extracellular, in sand fly
2 ȝm wide x 20 ȝm long
Intracellular (macrophages)
Round or oval
amastigotes consistent with Leishmania.
• flagella Wright-Giemsa:
• large central nucleus
• dark-purple nucleus
• band shaped kinetoplast
• small rod shaped kinetoplast Which is the most likely species?
A. L. mexicana
B. L. braziliensis
C. L. peruviana
D. L. infantum chagasi
E. L. major

CDC DpDx

Cutaneous Leishmaniasis – Clinical Presentation

Leishmania taxonomy and disease simplified • papule Æ nodule Æ ulcerative lesion Æ atrophic scar
ulcerative lesion may have:
Cutaneous Mucosal Visceral
induration,
NEW WORLD
scaliness
L. mexicana complex X central depression
L. braziliensis X X raised border
L. infantum chagasi X
• takes weeks to months to develop
OLD WORLD
L. tropica X • usually painless, unless superinfected
L. major X
L. donovani X • most lesions will eventually resolve on their own
L. infantum chagasi X

*note: L. braziliensis is in the Viannia subgenus. L. V. guyanensis and L. V. panamensis


also cause mucosal disease. L. peruviana DOES NOT

©2024 Infectious Disease Board Review, LLC 264


51– LotsofProtozoa
Speaker:EdwardMitre,MD

Cutaneous Leishmaniasis – Diagnosis Cutaneous Leishmaniasis – Treatment Recommendations


Definitive diagnosis is very helpful because ¾ Treat systemically if L. (V.) braziliensis, guyanensis, panamensis
1. Allows you to rule out other possibilities
2. May help in deciding whether and how to treat ¾ If not, ok to observe if there are:
few lesions, they are < 5 cm, not on face/fingers/toes/genitals,
Diagnostic Tools (edge of ulcer skin: scraping, aspirate, punch) normal host, no subcutaneous nodules
Touch prep with examination under oil looking for amastigotes
Culture on triple N media (may take weeks to grow) Treatment Options
(Nicolle’s modification of Novy and MacNeal’s medium – biphasic) local: heat with radiotherapy (FDA approved), cryotherapy, intralesional therapy
systemic
Histology
oral: miltefosine for certain species, especially New World CL species
PCR ketoconazole, fluconazole (off-label)
IV: liposomal amphotericin B (off-label)
pentavalent antimony (meglumine antimoniate, ASTMH website has
instructions for obtaining on IND from Sanofi)

***2016 IDSA GUIDELINES FOR TREATMENT OF LEISHMANIA***


http://www.idsociety.org/Guidelines/Patient_Care/IDSA_Practice_Guidelines/Infections_by_Organism/Parasites/Leishmaniasis/

Visceral Leishmaniasis
Mucosal leishmaniasis L. donovani (South Asia, East Africa)
Leishmania (Viannia) braziliensis, L. infantum chagasi (Middle East, Central Asia,
Guyanensis, panemensis Mediterranean, Central and S. America)

amastigotes in macrophages go to local LNs then


• dissemination to nasal mucosa hematogenously to liver, spleen, bone marrow
• slow, progressive, destructive
• can occur months or years after A persistent disease that can reactivate
cutaneous ulcer TNF blockade, HIV CD4 < 200

Treatment: • wks/months of fevers, chills, hepatosplenomegaly


• oral miltefosine • pancytopenia & hypergammaglobulinemia
(FDA approved for L. braziliensis)
• IV lip. amphotericin (off-label)
• IV antimony (no longer Diagnosis: PCR, culture, or histopathology for intracellular amastigotes
commercially available) Æ bone marrow aspirate preferred, can also check LN, spleen, or buffy
coat antibody to rK39 recombinant Ag (dipstick test)
Miltefosine notes
side effects: nausea, vomiting, diarrhea, increased AST/ALT Treatment: liposomal ampho B (FDA approved)
contraindicated in pregnancy, use contraception for 5 months after treatment (t1/2 = 30 d) miltefosine (oral) FDA approved for L. donovani
(combination treatment for L. donovani in people living with HIV in SE Asia)

©2024 Infectious Disease Board Review, LLC 265


51– LotsofProtozoa
Speaker:EdwardMitre,MD

Question 4: A 41 yo woman presented to a local African Trypanosomiasis (sleeping sickness)

emergency department with a one day history of fever


Vector = tse tse fly (Glossina sp)
associated with swelling and redness in her groin four
days after returning from safari in Tanzania. Peripheral Trypanosoma brucei gambiense (W. Africa)
blood smear is obtained. • humans as reservoirs
• progression over many months

What is the most likely Trypanosoma brucei rhodesiense (E. Africa)


diagnosis? • cattle and game park animals as reservoirs
• progression over weeks

A. Leishmania donovani DISEASE


B. Plasmodium vivax within 5 days: chancre at Tse Tse fly bite
regional lymphadenopathy
C. Trypanosoma brucei
D. Wuchereria bancrofti for weeks: fever, hepatosplenomegaly,
lymphadenopathy, faint rash, headache
E. Leptospira interrogans
late: mental status changes, terminal somnolent state W.H.O.

African Trypanosomiasis – Lab findings


African Trypanosomiasis - Life Cycle
Non-specific lab findings
• anemia
• elevated IgM Q. Why are Trypanosoma brucei infections associated with persistently
• thrombocytopenia elevated IgM levels?
• hypergammaglobulinemia

Diagnostic lab findings


• detection of parasite in lymph node, circulating blood, or CSF
-->do FNA of lymph node while massaging node, then push out the aspirate onto a
slide and immediately inspect under 400x power. Trypanosomes can be seen moving for
15-20minutes, usually at edge of the coverslip

• a card agglutination test that detects T.b.gambiense sp. antibodies.


-->V. sensitive (94-98%), but poor specificity
--> can get false +s in pts with Schisto, filaria, toxo, malaria

African Trypanosomiasis –Treatment


African Trypanosomes – The Lady Gaga of the Microbial World
West African (T. gambiense)
If < 6 yo or < 20 kg: lumbar puncture
CSF < 5 WBC/ul Æ iv pentamidine
CSF > 5 WBC/ul Æ iv eflornithine + nifurtimox
If adult: confusion, ataxia, anxiety, abnl speech, motor weakness, abnl gait?
no suspicion of late disease Æ oral fexinidazole
if suspicion of CNS disease Æobtain lumbar puncture
CSF < 100 cells/ul (non-severe 2nd stage) Æoral fexinidazole
CSF > 100 cells/ul Æ iv eflornithine+ nifurtimox

East African (T. rhodesiense): Rx always guided by lumbar puncture


CSF < 5 WBC/ul Æ suramin
CSF > 5 WBC/ul Æ melarsoprol

July 16, 2021: Oral fexinidazole FDA approved for T. gambiense

Notes: 1) Melarsoprol associated with ~5% death rate due to reactive encephalopathy.
2) This is reduced by co-administration of corticosteroids.

©2024 Infectious Disease Board Review, LLC 266


51– LotsofProtozoa
Speaker:EdwardMitre,MD

Chagas disease Chagas – Clinical Disease


• transmitted by Trypanosoma cruzi Acute (starts 1 week after infection, can persist for 8 weeks)
(also blood transfusion and congenitally) • fever
• local lymphadenopathy
• vector: reduviid (triatomine) bugs
• unilateral, painless periorbital edema
• reservoirs: opossums, rats, armadillos, raccoons, dogs, cats
Indeterminate stage
• autochthonus cases in the U.S. • serology positive, no evidence of disease
Texas
Louisiana Chronic
Mississippi
Missouri
California

• oral ingestion of food and drinks


contaminated with reduviid bugs or the
feces of those bugs is a major route of
infection
(acai and sugar cane juice) dilated cardiomyopathy, R>L
megaesophagus
(CHF, syncope, arrythmia)

Chagas Diagnosis & Rx Chagas in immunosuppressed patients


Acute disease
• identification of parasites in blood T. cruzi and AIDS
Primarily reactivation neurologic disease
Chronic disease
• T. cruzi specific IgG antibodies in serum ¾ acute, diffuse, necrotic
Æ two antibody tests using different antigens meningoencephalitis
and different techniques recommended for dx
(research: xenodiagnosis, hemoculture, PCR) ¾ focal CNS lesions (similar to Toxo)**
NOTE: U.S. blood supply screened for 1st time donors
2008 Int J Infectious Diseases

T. cruzi and solid organ transplant


Treatment
> recipient of infected organ:
Benznidazole for 30 – 60 d, alternative: Nifurtimox (both FDA approved)
Benznidazole AEs: peripheral neuropathy, granulocytopenia, rash fevers, hepatosplenomegaly, myocarditis
Nifurtimox AEs: abdominal pain/vomiting, tremors, peripheral neuropathy > disease often does not occur until months after transplant

Always offer: acute infection, congenital, < 18 yo, reactivation disease ALSO…. reactivation myocarditis occurs in ~40% of patients
Usually offer: 19-50 years old and no advanced cardiac disease
that receive heart transplant because of Chagas cardiomyopathy
Individual decision: > 50 years old and no advanced cardiac disease

Protozoa Free-living amoebae


Protozoa - Extraintestinal Protozoa - Intestinal Naegleria fowleri
• warm freshwater exposure
Apicomplexa Apicomplexa • enters through olfactory neuroepithelium
Plasmodium Cryptosporidium • fulminant meningoencephalitis
Babesia Cyclospora • immunocompetent children/young adults
(Toxoplasma) Cystoisospora

Flagellates Flagellates Acanthamoeba


Leishmania Giardia • found in soil and water
Trypanosomes Dientamoeba • enter through lower respiratory tract or broken skin
(Trichomonas) • subacute granulomatous encephalitis
Amoebae • immunocompromised hosts
Entamoeba
Amoebae • chronic granulomatous keratitis (contact lens, LASIK)
Naegleria Ciliates
Acanthamoeba Balantidium Balamuthia mandrillaris
Balamuthia • likely enters through lower respiratory tract or broken skin
• transmission by solid organ transplantation has been reported
• subacute granulomatous encephalitis
• normal and immunocompromised hosts
Maybe Not Protozoa Kingdom Fungi: Microsporidiosis agents
Domain SAR: Blastocystis Outcome Æ often fatal (amphotericin B, azoles, pentamidine, others tried)

©2024 Infectious Disease Board Review, LLC 267


51– LotsofProtozoa
Speaker:EdwardMitre,MD

Protozoa When to suspect an intestinal protozoan infection:


Protozoa - Extraintestinal Protozoa - Intestinal
Apicomplexa Apicomplexa
Plasmodium Cryptosporidium Patient has: Protracted watery diarrhea
Babesia Cyclospora weeks to months)
(Toxoplasma) Cystoisospora
Flagellates AND/OR:
Flagellates • history of travel [domestic (esp. camping) or foreign]
Leishmania Giardia
Trypanosomes Dientamoeba • recreational water activities
(Trichomonas) Amoebae • altered immunity (HIV infection)
Entamoeba • exposure to group care (daycare)
Amoebae
Naegleria Ciliates
Acanthamoeba Balantidium Note: discussion will focus on intestinal protozoa as they occur
Balamuthia
in patients seen in the U.S. These are leading causes of
diarrhea, morbidity, and mortality worldwide, especially in
young children.
Maybe Not Protozoa Kingdom Fungi: Microsporidiosis agents
Domain SAR: Blastocystis

Intestinal Apicomplexa parasites Intestinal Apicomplexa: clinical clues

Cryptosporidium
Cryptosporidium • watery diarrhea of several weeks
• C. parvum: cows
• cattle workers and daycare outbreaks
• C. hominis: humans
• cysts are resistant to chlorine (water supply outbreaks)
Cyclospora cayetanensis --> #1 cause of water park/swimming pool outbreaks

Cystoisospora belli Cyclospora cayetanensis - self-limited immunocompetent BUT can last up to 10 weeks!
• abrupt onset with nausea, vomiting, and fever early
Cryptosporidium in enterocyte. CDC DpDx • anorexia, weight loss, fatigue late in course
• all have worldwide distribution
• food associated outbreaks: raspberries, lettuce, herbs
• all transmitted by water or food contaminated with oocysts • esp. Nepal, Peru, Guatemala
• organisms invade enterocytes
Cystoisospora belli
• all cause watery diarrhea that can be prolonged & severe in immunocompromised • no animal reservoirs known
• watery diarrhea
• may be associated with a peripheral eosinophilia!
(the ONLY intestinal protozoa that does this)

Intestinal Apicomplexa characteristics


Pathogen Size Stain Treatment
Cryptosporidium 4 ȝm m acid-fast (none) nitazoxanide or paromomycin
Cyclospora 10 ȝm m acid-fast TMP/SMX
Cystoisospora 20 ȝm m acid-fast TMP/SMX

CD
C
Molecular tests
most stool multiplex PCR assays detect cryptosporidium AND Cyclospora
but NOT Cystoisospora
stool Ag tests commercially available for cryptosporidium

©2024 Infectious Disease Board Review, LLC 268


51– LotsofProtozoa
Speaker:EdwardMitre,MD

“The number of reported outbreaks has increased an average of


approximately 13% per year.”

Question 5: A 28 year old woman returns after studying


Balantidium coli
mosquito breeding habits in Honduras for one year. She
reports intermittent abdominal pain and diarrhea for
several months. Stool ova and parasite exam is positive • the only ciliated pathogen of humans!
for the presence of a ciliated single cell organism.
• largest protozoan pathogen of humans! CDC DpDx
(about 70 ȝm wide and up to 200 ȝm long)
What is the most likely diagnosis?
• found worldwide, especially Central and S. America, S.E. Asia, and Papua New Guinea

A. Balantidium coli • associated with eating food/water contaminated with pig feces

B. Entamoeba histolytica • Symptoms: most people asymptomatic


C. Giardia lamblia can cause colitis with abdominal pain, weight loss, +/- diarrhea
(especially in malnourished and immunocompromised)
D. Dientamoeba fragilis
E. Endolimax nana • Treatment: tetracycline (!) or metronidazole

Entamoeba histolytica
Entamoeba histolytica Diagnosis
• strictly human pathogen Stool PCR (multiplex or single)
• fecal/oral (contaminated food/water) • close to 100% sensitivity and specificity
• cysts = infective stage
Stool O/P
• trophozoites = active form, tissue-destructive
• only 50% sensitive for colitis and abscess
E. histolytica
• poor specificity b/c unable to differentiate E.histolytica
clinical presentations trophozoites with
from non-pathogenic E. dispar and the diarrhea-only
• asymptomatic ingested RBCs.
causing E. moshkovskii
• traveler’s diarrhea (note: ingested RBCs suggestive of Eh, but not 100%)
• colitis Stool antigen testing > 85% sensitive for intestinal disease
sharp abdominal pain
bloody diarrhea Serology 95% sensitive for liver abscess, 85% sensitive for intestinal infection
fever
flask-shaped ulcerations Treatment
> onset can occurs weeks to months after travel asymptomatic: luminal agents such as paromomycin
• ameboma symptomatic: tissue agents such as metronidazole or tinidazole THEN luminal agent
liver abscess: medical therapy (tissue agent then luminal agent) usually sufficient!
• liver and brain abscesses, esp in young men, drainage if no response to medical therapy or dx unclear or v large abscess
usually 2-5 months after travel

©2024 Infectious Disease Board Review, LLC 269


51– LotsofProtozoa
Speaker:EdwardMitre,MD

Giardia duodenalis - described by Antony van Leeuwenhoek in 1681! Giardia


At risk populations
• international travelers
Flagellated protozoan • swimming in lakes/streams, outdoor survival/camping
• fecal/oral via ingestion of cyst form in food/water
• cyst is chlorine resistant • infants in daycare
• cysts from humans (beavers, muskrats) • child care workers
• immunoglobulin deficiencies (esp CVID)
Disease in U.S. • HIV when CD4 < 100
• most common parasitic infection in the U.S
(20k cases reported/year, likely 2M)
- U.S-acquired cases peak in the late summer/early fall Diagnosis
- a leading cause of traveler’s diarrhea • stool antigen test
• stool multiplex PCR
Symptoms
• intermittent watery diarrhea weeks to months
Treatment
• foul smelling stools, flatulence, “sulfur burps”
tinidazole (FDA approved)
metronidazole (off-label), nitazoxanide (FDA-approved), and albendazole (off label)

Other intestinal protozoa


Protozoa
Protozoa - Extraintestinal Protozoa - Intestinal
Apicomplexa Apicomplexa
Plasmodium Cryptosporidium
Non-pathogens Cyclospora
Babesia
amoebae flagellates (Toxoplasma) Cystoisospora
Entamoeba dispar Chilomastix mesnili Flagellates
Entamoeba hartmanni Trichomonas hominis Flagellates
Leishmania Giardia
Entamoeba coli Trypanosomes Dientamoeba
Endolimax nana (Trichomonas) Amoebae
Iodamoeba bütschlii Entamoeba
Amoebae
Naegleria Ciliates
Treat if symptomatic: Dientamoeba fragilis (implicated in IBS) Acanthamoeba Balantidium
Balamuthia

Maybe Not Protozoa Kingdom Fungi: Microsporidiosis agents


Domain SAR: Blastocystis

Microsporidia – obligate intracellular fungi!


Blastocystis
¾ Produce extracellular, 1-2 micron, infective spores
¾ Spores have a coiled organelle called a polar tubule What is it?
¾ After ingestion, the spore germinates and the polar tubule is used to
inject sporoplasm into a host cell Currently classified as a protozoa.

Enterocytozoon bieneusi Forms are 5-40 microns wide.


• watery diarrhea
Anaerobic. Eukaryotic.
• biliary disease (cholangitis, acalculous cholecystitis) Spores of E. hellem bursting out of a cell (CDC DpDx)
- cystic, ameboid, granular, and vacuolar forms Blastocystis cyst-like forms , trichrome (CDC DpDx)

Encephalitozoon intestinalis
• watery diarrhea Often the most common eukaryotic organism found in human stool samples
• biliary disease
• disseminated disease (liver, kidney, lung, sinuses)
Does it cause disease?
Encephalitozoon cuniculi, hellem Maybe.
• can cause disseminated disease of multiple organs, plus eye Associated with watery diarrhea, abdominal discomfort, nausea, and flatulence.
Polar tubule inserted into a eukaryotic cell (CDC DpDx)
Many species (including Vittaforma corneae): punctate keratoconjunctivitis Diagnosis: light microscopy of stool samples
(contact lens use, after eye surgery, bathing in hot springs)
Treatment?
DIAGNOSIS: modified trichrome stain, Calcofluor white, IFA metronidazole, tinidazole, TMP/SMX, or nitazoxanide (none FDA-approved)
TREATMENT: albendazole (not effective for E. bieneusi)

©2024 Infectious Disease Board Review, LLC 270


51– LotsofProtozoa
Speaker:EdwardMitre,MD

Protozoan infections that can reactivate in the severely immunocompromised


• Toxoplasmosis
encephalitis with mass lesions
pneumonitis
retinitis
• Leishmania
reactivation of visceral and cutaneous reported
visceral with fever, hepatosplenomegaly, pancytopenia
• Chagas
encephalitis with mass lesions
Good luck on the exam!
hepatosplenomegaly and fevers
myocarditis in 40% that receive heart transplant b/c Chagas disease
• Malaria

Some other protozoa that can cause severe disease in immunocompromised


• Cryptosporidium
• Giardia
• Microsporidia
• Babesia Edward Mitre, M.D.
• Acanthamoeba [email protected]

©2024 Infectious Disease Board Review, LLC 271


51 – Lots of Protozoa
Speaker: Edward Mitre, MD
Enlarged Slide: 15

Sporozoites
• Infective stage
• Come from mosquito
Liver schizont
• Asymptomatic replicative stage
• Become 10,000 to 30,000 merozoites
Hypnozoite
• Dormant liver stage in vivax and ovale
• Release merozoites weeks to months
after primary infection

Merozoites
• Infect RBCs and develop into ring-stage
trophozoites
• Mature into schizonts, which release
merozoites which infect more RBCs
Gametocytes
• Infective stage for mosquitoes
*

©2024 Infectious Disease Board Review, LLC 272


273
Wednesday, August 21, 2024

52

Worms That Could be on the


Exam
Dr. Edward Mitre

©2024 Infectious Disease Board Review, LLC


COPYRIGHT NOTICE: The Copyright Act (Title 17 of U.S. Code) governs the rights attributed to owners of
copyrighted work. Under certain circumstances, educational institutions may provide copies of
copyrighted works to continuing education participants. The copies may not be copied nor used for any
other purpose besides private study, scholarship, or research. Participants should not provide electronic
or print copies of ant materials provided by the university to unauthorized users. If a participant fails to
comply with these restrictions, the participant may be held liable for copyright infringement. No further
transmission or electronic distribution is permitted.

274
275
52– WormsThatCouldbeontheExam
Speaker:EdwardMitre,MD

Lecture Title

• Disclosures of Financial Relationships with Relevant


Worms and More Worms Commercial Interests

- None

Edward Mitre, MD
Rockville, MD

Disclaimer: Dr. Mitre is giving this presentation in a personal capacity. The views expressed in this presentation are the sole
responsibility of the presenter and do not necessarily reflect the views, opinions, or policies of the Uniformed Services University
of the Health Sciences, the Department of Defense, or the United States Government.

Question #1 Major Helminth Pathogens


TREMATODES CESTODES NEMATODES

28 yo F Blood flukes Intestinal tapeworms Intestinal


Ascaris lumbricoides
- recurrent abdominal cramps for several months Schistosoma mansoni Taenia solium
Ancylostoma duodenale
- just returned to the U.S. after living in Tanzania for one year Schistosoma japonicum Taenia saginata Necator americanus
Diphyllobothrium latum
- colonoscopy reveals small white papules Schistosoma haematobium
Hymenolepis nana
Trichuris trichiura
Strongyloides stercoralis
- biopsy reveals an egg with eosinophilic granulomatous inflammation Paracapillaria philippinensis
Liver flukes Enterobius vermicularis
Larval cysts
Fasciola hepatica
Most likely diagnosis? Clonorchis sinensis
Taenia solium
Tissue Invasive
Echinococcus granulosus
A. Entamoeba histolytica Opisthorchis viverrini Echinococcus multilocularis
Wuchereria bancrofti
Brugia malayi
B. Ascaris lumbricoides Onchocerca volvulus
Lung flukes
C. Wuchereria bancrofti Paragonimus westermani
Loa loa
Trichinella spiralis
D. Schistosoma mansoni Angiostrongylus cantonensis
Anisakis simplex
E. Paragonimus westermani Intestinal flukes Toxocara canis/cati
Fasciolopsis buski Baylisascaris procyonis
Metagonimus yokagawai Gnathostoma spinigerum

Schistosomiasis life cycle Acute Schistosomiasis


Cercarial dermatitis (Swimmer’s itch)
• urticarial plaques and pruritic papules
• occurs upon reexposure to cercariae penetrating skin in a
sensitized individual
• symptoms develop minutes to days after water exposure
• can occur with human or avian schistosomes

Katayama fever
• fever, myalgias, abdominal pain, headache, diarrhea, urticaria
• occurs in previously unexposed hosts.
• symptoms typically start 3 - 8 weeks after water exposure
• eosinophilia, elevated AST and alkaline phosphatase
• no reliable way to confirm diagnosis acutely as serology and stool O/P frequently negative

©2024InfectiousDiseaseBoardReview,LLC
276
52– WormsThatCouldbeontheExam
Speaker:EdwardMitre,MD

Chronic Schistosomiasis Schistosoma genital disease


Intestinal and hepatosplenic disease (Sm, Sj, Si, Smk, Sh/b) primarily due to S. haematobium
• granulomatous colitis
• portal hypertension
women
• vaginal and cervical lesions) sand grains
Genitourinary disease (Sh, Sh/b) sandy yellow patches
• granulomatous cystitis • pelvic pain
• bladder fibrosis and cancer • dysmenorrhea
• obstructive uropathy
Major Schistosoma species • dyspareunia
• S. mansoni (Sm) • post-coital bleeding
Pulmonary Disease (Sm, Sh, Sj) • S. japonicum (Sj)
• S. intercalatum (Si) • endometritis/salpingitis
CNS disease (any, most often Sj) • S. mekongi (Smk)
• eggs to spinal cord or brain • S. haematobium (Sh)
• S. haematobium/S. bovis hybrid (Sh/b)* men
• in spinal cord: often causes transverse myelitis
• in brain (less common): one or more intracerebral inflammatory lesion(s)
• epididymitis abnormal vessels rubbery papules
*primarily in W. Africa, with endemicity in
Corsica, France since 2013 • prostatitis
WHO Female Genital Schistosomiasis Pocket Atlas

Schistosome eggs Question #2


S. mansoni S. haematobium
(lateral spine) (terminal spine) A person that ingests food contaminated with Taenia solium
eggs is at risk of which of the following?

A. HTLV-1 infection
B. bladder cancer
C. vitamin B12 deficiency
D. seizures
CDC DPDx image library CDC DPDx image library
E. anemia

Major Helminth Pathogens Taenia lifecyle for tapeworm infection in humans


predator
TREMATODES CESTODES NEMATODES prey
(tapeworm)
Blood flukes Intestinal tapeworms Intestinal
Ascaris lumbricoides (larval cyst)
Schistosoma mansoni Taenia solium
Ancylostoma duodenale
Schistosoma japonicum Taenia saginata Necator americanus
Schistosoma haematobium Diphyllobothrium latum Trichuris trichiura
Hymenolepis nana Strongyloides stercoralis
Paracapillaria philippinensis
Liver flukes Enterobius vermicularis
Larval cysts
Fasciola hepatica
Taenia solium
Clonorchis sinensis Tissue Invasive
Echinococcus granulosus Wuchereria bancrofti
Opisthorchis viverrini Echinococcus multilocularis Brugia malayi
Onchocerca volvulus
Lung flukes Loa loa
Paragonimus westermani Trichinella spiralis
Angiostrongylus cantonensis
Anisakis simplex
Intestinal flukes Toxocara canis/cati
Fasciolopsis buski Baylisascaris procyonis
Metagonimus yokagawai Gnathostoma spinigerum

©2024InfectiousDiseaseBoardReview,LLC
277
52– WormsThatCouldbeontheExam
Speaker:EdwardMitre,MD

INTESTINAL TAPEWORMS

Taenia solium
tapeworm is acquired by eating larvae in pork
adult tapeworm causes few symptoms
For some cestodes, humans can
be infected by the larval stages
Taenia saginatum
acquired by eating larvae in undercooked beef
and this can cause severe
causes few symptoms pathology.
can grow to 10 m

Diphyllobothrium latum (can grow > 10 m)


acquired by ingesting fish with larvae
*B12 deficiency in up to 40% of patients

Dx: eggs/proglottids in stool Rx: praziquantel (not FDA-approved)

Neurocysticercosis

Neurocysticercosis Neurocysticercosis – single lesion disease is diagnostic challenge


Clinical manifestations Perilesional edema Multiple intracerebral
• seizures • Typically around dying cyst calcifications
• hydrocephalus • often seen with new seizure • sometimes a clinical clue
• headaches or terrible headache • non-contrast head CT scan
• focal neurologic deficits

©2024InfectiousDiseaseBoardReview,LLC
278
52– WormsThatCouldbeontheExam
Speaker:EdwardMitre,MD

Neurocysticercosis Echinococcus granulosus


Diagnosis: adult worms (3-6mm long)
Definitive = tissue biopsy
multiple cystic lesions each with scolex on imaging live in dog intestines
retinal cysticercus seen on fundoscopic exam

Presumptive = suggestive lesions on imaging humans = accidental


Cysticercosis serology Æ supportive (sensitive if high burden of disease) hosts

Treatment: •
Medical therapy decreases risk of future seizures, but has immediate risk of increasing
seizures/brain inflammation
infected by ingestion
of eggs in dog feces
If hydrocephalus or diffuse cerebral edema, treat with steroids and/or surgery, not anti-parasitic therapy

If no increased ICP: 1-2 viable cysts Æalbendazole for 1-2 viable cysts
> 2 viable cysts Æalbendazole + praziquantel

AND corticosteroids started before anti-parasitic therapy

**2017 IDSA Guidelines for Diagnosis and Treatment of Cysticercosis**

Echinococcus granulosus Echinococcus granulosus - presentation


hydatic cyst = “watery vessel”
Most cysts (65%) in the liver
25% in the lung, usually in the right lower lobe
Rest occur practically everywhere else in the body
surrounding inflammatory response of
fibrosis and chronic inflammation
Common presentations
• allergic symptoms/anaphylaxis due to cyst rupture after trauma
• cholangitis and biliary obstruction due to rupture into biliary tree
outer acellular laminated layer
• peritonitis b/c intraperitoneal rupture
• pneumonia symptoms due to rupture into the bronchial tree

inner, nucleated germinal layer


Uncommon presentations
(PLURIPOTENTIAL TISSUE!)
• bone fracture due to bone cysts
• mechanical rupture of heart with pericardial tampanode
internal cystic fluid and daughter cysts Echinococcus and Hydatid Disease 1995.
• hematuria or flank pain due to renal cysts

Echinococcus granulosus - diagnosis Echinococcus granulosus – treatment


Radiology Microscopy
Risks of cyst rupture
1. Anaphylaxis may occur
2. Spilled protoscoleces can reestablish infection

Typically treat with albendazole for a few days


before surgery or PAIR (usually 3-4 days before and 1-3 months after)

Serology
• IgG ELISA about 85% sensitive for liver cysts of E. granulosus
• only 50% sensitive in cases of single pulmonary cyst

©2024InfectiousDiseaseBoardReview,LLC
279
52– WormsThatCouldbeontheExam
Speaker:EdwardMitre,MD

Cystic Echinococcus Treatment – depends on cyst stage


Major Helminth Pathogens
TREMATODES CESTODES NEMATODES

Blood flukes Intestinal tapeworms Intestinal


Ascaris lumbricoides
Schistosoma mansoni Taenia solium
Ancylostoma duodenale
Schistosoma japonicum Taenia saginata Necator americanus
Schistosoma haematobium Diphyllobothrium latum Trichuris trichiura
Hymenolepis nana Strongyloides stercoralis
Paracapillaria philippinensis
Liver flukes Enterobius vermicularis
Larval cysts
Fasciola hepatica
Taenia solium
Clonorchis sinensis Tissue Invasive
Echinococcus granulosus Wuchereria bancrofti
Opisthorchis viverrini Echinococcus multilocularis Brugia malayi
Onchocerca volvulus
ACTIVE TRANSITIONAL INACTIVE Lung flukes Loa loa
Anechoic content Heterogenous, hypoechoic or Paragonimus westermani Trichinella spiralis
Unilocular Multivesicular Angiostrongylus cantonensis
Simply cyst Multiseptated Detached membrane hyperechoic Anisakis simplex
Solid matrix No daughter cysts Intestinal flukes Toxocara canis/cati
Cyst wall visible cysts
CE5 with thick calfied wall Fasciolopsis buski Baylisascaris procyonis
---ALB +/- PAIR ---SURGERY--- ---SURGERY--- Metagonimus yokagawai Gnathostoma spinigerum
or Surgery--- ---PAIR if no solid matrix--- ---NO TREATMENT--

Intestinal Helminths - Lifecycles Ascaris lumbricoides


Three intestinal nematodes transit through the lungs and can cause up to 40 cm long and 6 mm wide!
Loeffler’s syndrome
Æ transient cough, wheeze, shortness of breath, shifting pulmonary infiltrates, eosinophilia Symptoms
Æ usually starts within a week after exposure and typically lasts for about two weeks • abdominal distention, pain, & intestinal obstruction
with large worm burdens
• Ascaris • cholangitis and/or pancreatitis b/c aberrant migration
• Hookworms
• Strongyloides Diagnosis
• stool o/p exam
Ascaris (ingestion of eggs, larvae released in intestinal tract) • note: will not find eggs until 2-3 months after
GUT Æ PORTAL CIRCULATION Æ LUNGS Æ GUT
pulmonary symptoms occur

Strongyloides and Hookworms (skin penetration by larvae) Treatment


SKIN Æ SYSTEMIC CIRCULATION Æ LUNGS Æ GUT • albendazole or mebendazole

CDC DPDx

HOOKWORMS
Ancylostoma duodenale and Necator americanus Trichuris trichiura (whipworm)
(also Ancylostoma ceylanicum - zoonotic from dogs/cats in Asia)
4cm long nematode
• abdominal pain
Life cycle: Fecal-oral
• MAJOR cause of ANEMIA and protein loss (b/c plasma loss)
• Loeffler’s syndrome In heavy infections:
• ground itch (if previously sensitized, dermatitis at entry site) - loose and frequent stools
- tenesmus
If worms migrate laterally Æ cutaneous larva migrans Rockefeller Foundation Archive Center
- occ blood to frank blood
(especially dog and cat hookworms, as late as 2-8 wks after exposure to A. braziliense) - in heavily infected children:
rectal prolapse

Still endemic in the U.S. 35% of individuals from a rural community in Alabama had N. Dx: eggs are football shaped with two polar plugs
americanus in their stool samples
CDC DPDx

©2024InfectiousDiseaseBoardReview,LLC
280
52– WormsThatCouldbeontheExam
Speaker:EdwardMitre,MD

Question #3 Strongyloides stercoralis


(can complete lifecycle in host!)
Usual manifestations
A 25 yo F from rural Peru presents with shortness of breath, bilateral interstitial infiltrates, fever, GI: mild abdominal/epigrastric pain
loose stools, hypotension, and E. coli bacteremia. She has received 1 month of high dose Pulm: wheezing, transient infiltrates
corticosteroids and cyclophosphamide for a recent diagnosis of lupus nephritis. Skin: urticarial rashes, larva currens

Which of the following anthelmintic agents should be included in her empiric treatment regimen? Hyperinfection syndrome
immunocompromised state
(steroids, TNF-inhibitors, HTLV-1, malignancy, malnutrition….not HIV)
A. Albendazole large burden of parasites
B. Ivermectin
GI: Nausea, vomiting, abdominal pain, diarrhea, erosions
C. Praziquantel
b/c millions of larvae in intestinal mucosa
D. Pyrantel pamoate
E. Diethylcarbamazine Pulmonary: diffuse infiltrates, wheezing, dyspnea, cough

Systemic: fever and hypotension due to gram negative sepsis

-- Often do not see eosinophilia in hyperinfection --

Strongyloides stercoralis Ivermectin


activates nematode glutamate-gated chloride channels causing muscle paralysis
Diagnosis
Drug of choice
• stool o/p (sensitivity is low - 30-60%) • Strongyloides
• Onchocerca volvulus (microfilaricidal only)
• serology
• Also has activity against Ascaris, whipworm, cutaneous larva migrans, gnathostomiasis AND
ectoparasites such as scabies and lice

ADVERSE EFFECTS
Treatment of choice: ivermectin Æ reports of seizures, ataxia, and confusion after ingestion of large veterinary doses
N Engl J Med 2021; 385:2197-2198

Æ altered mental status in 13 yo boy given standard dose for scabies


Prevention in pts from endemic countries who are about to due to a mutation in ABCB1 (aka P glycoprotein 1 and MDR1)
be immunosuppressed N Engl J Med 2020; 383:787-789

• Empirically treat, or check serology and treat if positive

Filariae: tissue-invasive, thread-like nematodes, transmitted by arthropod


Question #4 vectors
Adults Microfilariae
A 32 yo M from Cameroon reports intermittently experiencing a
worm crawling across his eye. Which of the following tests can Wuchereria bancrofti
be used to confirm the most likely diagnosis? Brugia malayi lymphatics blood (night)
(lymphatic filariasis)
--mosquitoes--
A. Brain MRI scan
B. Midnight blood draw Loa loa SQ tissues (moving) blood (day)
(eyeworm)
C. Noon blood draw
--Chrysops flies--
D. Skin snip
E. Scrotal ultrasound Onchocerciasis SQ tissues (nodules) skin
(river blindness)
--blackflies--

©2024InfectiousDiseaseBoardReview,LLC
281
52– WormsThatCouldbeontheExam
Speaker:EdwardMitre,MD

Treatment of Filariasis W. bancrofti and B. malayi


Treatment Avoid
Lymphatic filariasis DEC -----

Loa Loa DEC DEC and Ivermectin


if high microfilaria level
• Asymptomatic microfilaremia
Onchocerciasis ivermectin DEC • Lymphangitis
• retrograde (filarial lymphangitis)
ADVERSE EFFECTS
• bacterial skin/soft tissue infections (dermatolymphangioadenitis)
Loa with high microfilaremia Æ encephalopathy and death
Onchocerciasis Æ severe skin inflammation and blindness • Lymphatic dysfunction
• Lymphedema, elephantiasis, hydrocele, chyluria

Tropical pulmonary eosinophilia Lymphatic filariasis: diagnosis


•Paroxysmal nocturnal asthma Definitive diagnosis

•Pulmonary infiltrates • Identification of microfilariae in nighttime blood


• Detection of circulating antigen in blood (only Wb)
•Peripheral blood eosinophilia
(>3,000/mm3) • Identification of adult worm (by tissue biopsy or ultrasound “filaria dance sign”)

•Elevated serum IgE Presumptive diagnosis


•Rapid response to anti-filarial • Compatible clinical picture + positive antifilarial antibodies
therapy
Treatment
• DEC, doxycycline
Likely due to excessive immune
response to microfilariae in lung • NOTE: Triple drug single dose therapy (DEC/albendazole/ivermectin) is now
recommended by W.H.O. for mass drug administration eradication campaigns in areas
vasculature
that are NOT co-endemic for Loa loa or Onchocerca

Manifestations of Onchocerciasis Manifestations of Onchocerciasis


Skin: nodules, pruritus, rash, depigmentation, lichenification
• Eye: punctate keratitis, sclerosing keratitis, chorioretinitis

©2024InfectiousDiseaseBoardReview,LLC
282
52– WormsThatCouldbeontheExam
Speaker:EdwardMitre,MD

Onchocerciasis Loiasis: clinical manifestations


Diagnosis • Eyeworm
• Serology
• anti-filarial • Asymptomatic microfilaremia
• onchocerca-specific
• Parasitologic: skin snips, nodulectomy
• Non-specific symptoms
Treatment (fatigue, urticaria, arthralgias, myalgias)
Ivermectin
Moxidectin (FDA approved in 2018…has much longer half-life) • Calabar swellings
Æ both are primarily microfilaricidal (transient, migratory, subcutaneous swellings)
Æ therefore need repeated treatments for many years

(alternative: doxycycline for 6 weeks, which kills endosymbiotic Wolbachia • End organ complications (rare)
bacteria, kills adult worms) (endomyocardial fibrosis, encephalopathy, renal failure)

Possible question hints


Loiasis: Diagnosis Freshwater exposure + eosinophilia Æ Schistosomiasis
Crab/crayfish + pulmonary sxs + eosinophilia Æ Paragonimus
Cysticercosis Æ ANY food contaminated with tapeworm eggs
Definitive diagnosis Allergic symptoms after trauma Æ Echinococcus
• Identification of adult worm in itchy feet return to tropics Æ ground itch due to hookworms
subconjunctiva Gram- sepsis after corticosteroids or TNF inhibitor Æ Strongyloides hyperinfection
• Detection of Loa microfilariae in noon blood Subcutaneous nodules Æ Onchocerca volvulus
CDC DpDx
Blood microfilaria night Æ lymphatic filariasis (day = Loa loa, skin = Ov)
Muscle pain + eosinophilia Æ Trichinella
Presumptive diagnosis
Eosinophilic meningitis Æ Angiostrongylus
Compatible clinical picture + positive antifilarial antibodies
Abdominal pain after sushi Æ Anisakis
Eosinophilia + F + Ĺ AST/ALT in child Æ visceral larva migrans

Good Luck!
Ed Mitre
[email protected]

©2024InfectiousDiseaseBoardReview,LLC
283
Wednesday, August 21, 2024

53

Penicillin Allergies

Dr. Sandra Nelson

©2024 Infectious Disease Board Review, LLC


COPYRIGHT NOTICE: The Copyright Act (Title 17 of U.S. Code) governs the rights attributed to owners of
copyrighted work. Under certain circumstances, educational institutions may provide copies of
copyrighted works to continuing education participants. The copies may not be copied nor used for any
other purpose besides private study, scholarship, or research. Participants should not provide electronic
or print copies of ant materials provided by the university to unauthorized users. If a participant fails to
comply with these restrictions, the participant may be held liable for copyright infringement. No further
transmission or electronic distribution is permitted.

284
285
53– PenicillinAllergies
Speaker:SandraNelson,MD

Lecture Title

• Disclosures of Financial Relationships with Relevant


Commercial Interests
Penicillin Allergies
- None

Sandra B. Nelson, MD
Assistant Professor of Medicine
Harvard Medical School
7/1/2024

Case#1 Case#1:Vote
A73ͲyearͲoldwomanundergoingchemotherapyfor Youareaskedaboutoptimalantibiotictreatment.Whatdoyou
cholangiocarcinomaishospitalizedforbacteremiaandsepsis advise?
duetoEnterococcusfaecalis.SheiscurrentlyreceivingIV A. AdministerIVampicillinwithoutpriortesting
vancomycinbuthashadprogressiverenalinjury.Shehasa
historyofallergytopenicillinthatislistedintherecordsas B. Skintestforpenicillinreaction;ifnegativethenadministerfulldose
ampicillin
rash;thefamilyrecallsthatshewenttotheEDwhentherash
occurredseveralyearsearlier.Sheisdeliriousandnotableto C. Skintestforpenicillinreaction;ifnegativethenadministertestdose
corroboratethehistory;noadditionaldocumentationofthe ampicillinfollowedbyfulldoseampicillin
reactionisavailable.Twoofherdaughtershaveallergiesto D. Desensitizetoampicillin
penicillin. E. Continuevancomycin;thereisnosafepathfortransitiontoampicillin.

3 4

Penicillin(PCN)Allergy:Premise Likelihoodoftruepenicillinallergy
• Highestwith:
• 10%oftheUSpopulationhavereportedpenicillinallergy
Ё Fiveorfeweryearssincethereaction
• MajoritywithhistoryofPCNallergycansafelyreceivepenicillins
(withappropriateevaluationandtesting) Ё Anaphylaxisorangioedema
Ё Reactionsdonotalwaysrecur Ё Severecutaneousadversereaction
Ё Trueallergiesoftenwanewithtime Ё Treatmentrequiredforreaction
Ё Somereactionsarenotallergic
• PCNallergyisassociatedwithimportantmorbidity
Ё HigherriskofMRSAandVRE,Cdifficilecolitis,surgicalsiteinfection
Ё Greaterassociatedantimicrobialcostsandtoxicities

5 PENͲFASTDecisionTool:https://qxmd.com/calculate/calculator_752/penͲfastͲpenicillinͲallergyͲriskͲtool 6

©2024InfectiousDiseaseBoardReview,LLC
286
53– PenicillinAllergies
Speaker:SandraNelson,MD

OptionsforApproachingPCNAllergy OptionsforApproachingPCNAllergy
1. Monitoredoralchallenge 3. Gradedchallenge(alsocalledtestdoseprocedure)
Ё UsewithlowͲriskreactions(e.g.remoterash,pruritus) Ё Procedure:1/4th to1/10th dose,followedbyfulldose30Ͳ60minuteslater
Ё Canbeusedasafirststepifsuspicionforimmediatereactionislow
Ё AlsousedafternegativePCNskintesting
2. Penicillinskintesting
4. Desensitization
Ё Procedure:epicutaneous andintradermaladministrationof
PPL(penicilloyl polylysine,PreͲPen)andpenicillinG Ё Administrationofincreasingdosesevery15Ͳ30minutesuntiltherapeuticdosereached
Ё UsewithhistoryoforsuspectedIgE mediatedreaction Ё Usedforpositiveskintestand/orconfirmedimmediatereactionwhenapenicillinisthe
besttherapyforanimportantinfection
Ё ConsiderforunknownhistorywhenotherhighͲriskfeatures
Ё Desensitizationwaneswithmisseddoses(3halfͲlives)
Ё Ifnegative,followedbytestdoseofimplicatedordesired
drug ShenoyJAMA2019;321:188 5. Useofalternatetherapy

7 8

DecipheringCutaneousReactions DecipheringCutaneousReactions
• IgE MediatedReactions(hives) • Severecutaneousreactions
Ё Occurwithinminutestohours,resolvewithin24hours Ё DRESS,AGEPandSJS/TEN
Ё Oftenrecurswithrepeatexposure Ё Usualonsetdaystoweeks
Ё Blistering,mucosalinvolvement,severeskin
desquamation,organinvolvement
• BenignTͲcellmediated
Ё Morbilliformormaculopapular
Ё Mayhaveassociatedeosinophilia • Vagueorunknownskinreaction
Ё Usualonsetdaystoweeks Ё Evaluateriskofseverecutaneousreaction
Ё Persistslongerthan24hoursandresolvesoverdaystoweeks Ё AssumepossiblyIgE mediated
Ё Maynotrecurwithsubsequentexposure

ShenoyJAMA2019;321:188 9 SternNEJM2012;366:2492ShenoyJAMA2019;321:188 10

Puttingitalltogether:penicillinskinreactions WhataboutnonͲcutaneousreactions?
PenicillinSkintesting PenicillinSkintesting
IgE Mediated IgE Mediated
Ifpositive:desensitizationoralternativetherapy Ifpositive:desensitizationoralternativetherapy
(Urticaria) Angioedemaand
Ifnegative:testdoseamoxicillin Ifnegative:testdoseamoxicillin
Anaphylaxis

Antibodymediated
Ifapenicillin:amoxicillinoralchallenge (Hemolyticanemia,neutropenia,
BenignSkinRash
Anycephalosporinsafe thrombocytopenia)
ImmuneͲcomplex
(vasculitis,serumsickness) Noavailabletesting
SevereCutaneousAdverse CellͲmediated Ingeneral,whensevereavoidbetaͲlactams
Reaction AvoidbetaͲlactams (Interstitialnephritis,drugͲinduced
SJS/TEN,DRESS,AGEP liverinjury)

11 12

©2024InfectiousDiseaseBoardReview,LLC
287
53– PenicillinAllergies
Speaker:SandraNelson,MD

Case#2 Case#2:Vote
A43ͲyearͲoldmanwithdiabetesishospitalizedwithaclosed
Whatdoyoudocounsel?
tibialfracture.Threeyearsago,whenhewasbeingtreatedfor
afootinfectionwithpiperacillinͲtazobactamhedevelopeda
veryitchyrashafterseveralweeksoftreatment.The A. Administerclindamycin
anesthesiologistcallstoaskadviceaboutsurgicalantibiotic B. Administercefazolin
prophylaxispriortooperativefixation. C. Administercefazolinafterintraoperativetestdose
D. Administerceftriaxone
E. Administervancomycin

13 14

PCNAllergyanduseofcephalosporins CephalosporinAllergy
• Significantcrossreactivityrare • Allergyoftenarisesfromsidechains
Ё higherwithearliergenerationcephalosporins Ё MorecommonthanbetaͲlactamring

• ForIgE mediatedPCNallergy: • Probabilityofreactionhigherwhencephalosporinswith


similarsidechainsused(R1>R2)
Ё usestructurallydissimilar(3rd/4thgen) Penicillin
cephalosporinwithoutpriortesting • SidechaintablesareavailabletoguidecrossͲreactivity
Ё usestructurallysimilar(1st/2nd gen)afterPCNskin SimilarSideChainGroups(R1)
testingandamoxicillinchallenge
Amoxicillin,Cefadroxil,Cefprozil
• Milddelayeddrugrash: Ampicillin,Cefaclor,Cephalexin
Cephalosporin
Ё anycephalosporinOK Cefepime,Ceftriaxone,Cefotaxime,Cefpodoxime
• AvoidifseverereactiontoPCN CeftazidimeandAztreonam

15 16

Afewmoretestablepoints Thankyouandgoodluck!
• Selectiveallergytotheaminopenicillinsoccurs
Ё ApatientthattoleratesPCNmaystillbeallergictoaminopenicillins
Ё ApatientthattoleratesaminopenicillinsisnotallergictoPCN.
• Cefazolinhasdifferentsidechainsfromallothercephalosporins
• Ceftazidimedoesnotsharesidechainswithceftriaxoneorcefepime
• AztreonamcanbesafelyusedinindividualswithbetaͲlactamallergy
exceptforthoseallergictoceftazidime

17 18

©2024InfectiousDiseaseBoardReview,LLC
288
289
Wednesday, August 21, 2024

54

Kitchen Sink: Syndromes Not Covered


Elsewhere

Dr. Stacey Rose

©2023 Infectious Disease Board Review, LLC


COPYRIGHT NOTICE: The Copyright Act (Title 17 of U.S. Code) governs the rights attributed to owners of
copyrighted work. Under certain circumstances, educational institutions may provide copies of
copyrighted works to continuing education participants. The copies may not be copied nor used for any
other purpose besides private study, scholarship, or research. Participants should not provide electronic
or print copies of ant materials provided by the university to unauthorized users. If a participant fails to
comply with these restrictions, the participant may be held liable for copyright infringement. No further
transmission or electronic distribution is permitted.

290
291
54– KitchenSink:SyndromesNotCoveredElsewhere
Speaker:StaceyRose,MD

Lecture Title

• Disclosures of Financial Relationships with Relevant


Kitchen Sink: Syndromes Not Covered Elsewhere Commercial Interests

- None

Stacey R. Rose, MD, FACP, FIDSA


Associate Professor of Medicine, Infectious Diseases
Associate Director, Center for Professionalism
Baylor College of Medicine
7/1/2024

• A51yearͲoldmalewithpast
medicalhistorysignificantfor
Question1 insulindependentdiabetes
presentswithasixͲmonth
historyofprogressiveathralgias,
abdominalpain,diarrhea,
Sessionplan weightloss,andlowͲgrade
fevers.
• CaseͲbaseddiscussionsof
topicsnotextensivelycovered • Workupthusfar:
inothersessions Negativebloodculturesx2
• Highlightpointslikelytobe NegativeRheumatoidfactor
assessedonIDBoards(rather Normalmetabolicpanels
thancomprehensiveoverview)
Mildnormocyticanemia

3 4

• Whichofthefollowingtests Whipple’sdisease
Question1 willmostlikelyyieldthe
diagnosis?
• CausedbyTrophyrema whipplei (gram
variablebacterium,difficulttocultivate)
a) AntiͲstreptolysinOAntibody • Morecommoninmiddleaged,
Caucasianmen
b) AntiͲnuclearAntibody • Diagnosisoftendelayedduetoindolent
c) Stoolovaandparasite clinicalpresentation
• Mostcommonlydiagnosedviaduodenal
d) Duodenalbiopsy biopsy,stainedwithPAS
• PCRincreasinglyused

PeriodicacidͲSchiffͲdiastase(PASͲD)Ͳstained
duodenalbiopsyspecimenswithPASͲDͲpositive
DolmansRAV,Boel CHE,Lacle MM,KustersJG.2017.Clinicalmanifestations,treatment,anddiagnosisofTropheryma granulesinthefoamymacrophages(arrows).
5 whipplei infections.ClinMicrobiolRev30:529–555. 6

©2024InfectiousDiseaseBoardReview,LLC
292
54– KitchenSink:SyndromesNotCoveredElsewhere
Speaker:StaceyRose,MD

Whipple’s:clinicalpresentations Whipple’sendocarditis– increasinglydiagnosed


• Considerinpatients
Increaseinreportedcases witharthralgiasplus
ofT.whipplei endocarditis “culturenegative”
withmoleculardiagnostics endocarditis

• T.whipplei PCRfrom
bloodaddedtoDuke’s
criteria(2023)for
diagnosisof
endocarditis
Totalpublishedcases
Casesfromthisarticle
7 Fenollar F,Célard M,Lagier JC,Lepidi H,FournierPE,Raoult D.Tropheryma whipplei endocarditis.Emerg InfectDis.2013 8
DolmansRAV,Boel CHE,Lacle MM,KustersJG.2017.Clinicalmanifestations,treatment,anddiagnosisofTropheryma whipplei infections.ClinMicrobiolRev30:529–555.

Whipple’s:treatment
• Cause:Trophyrema Whipplei
Nogoldstandard • Epidemiology:middleaged,Caucasianmales
• Clinicalpresentation:classic– arthralgia,diarrhea,weightloss
Options: • Localizedinfectione.g.endocarditis (increasinglyrecognized)
• Diagnosiswithduodenalbiopsy(PAS stain;foamymacrophages)
• Ceftriaxoneormeropenemplus Symptomsimprove,but
orPCR ofinfectedtissueorblood
prolongedtrimethoprimͲsulfamethoxazole(~1year) relapseiscommonwithout
prolongedtreatment/ • Prolongedtreatmentneededtopreventrelapse

OR suppression

• Doxycyclineplus
hydroxychloroquine(12Ͳ18mos) Whipple’sdisease
Takehomepoints
Clinicalmanifestations,treatment,anddiagnosisofTropheryma whipplei infections.ClinMicrobiolRev2017.
Whipple'sdiseaseandTropherymawhippleiinfections:frombenchtobedside.LancetInfectDis.2022
PrinciplesandPracticeofInfectiousDiseases,9th ed 9 10

•A20yearͲoldfemaleschoolteacher
presentswitha1Ͳweekhistoryoffever
Question2 andpain/swellinginknees,elbows
andwrists.Shenotesthatthepain
Question2 Whichofthefollowingisthe
bestexplanationforher
movesfromjointtojoint.
symptoms?
•Shereportsbeingill~3weeksprior
withsorethroatandheadachewhich
resolvedwithoutspecifictreatment. a. AcuteHIVinfection
•Shehasnorashorlymphadenopathy.
b. Mononucleosisdueto
•Shedeniestravel.Sheissexuallyactive EpsteinBarrVirus
withonemalepartner,usingbarrier
protection(condoms). c. Acuterheumaticfever
•LabsarenotableforelevatedESRand d. Lemierre’s syndrome
CRPand+ASOandAntiͲDNaseBtiters;
pregnancyandHIVtests(4th generation
Ag/Ab)arenegative. 11 12

©2024InfectiousDiseaseBoardReview,LLC
293
54– KitchenSink:SyndromesNotCoveredElsewhere
Speaker:StaceyRose,MD

Explanation AcuteRheumaticFever
• RareinUS(0.5per100Kper
year),butcommonworldwide
(0.5millionperyear)
• Affectschildren/youngadults
• Recurrencecommon
• Pathogenesis:immune
responsefollowing
Streptococcuspyogenes
infection(pharyngitis;
impetigo)
• Leadstosystemic
manifestations(aarthritis,
carditis,chorea,skin)

13 Acuterheumaticfeverandrheumaticheartdisease.NatRevDisPrimers.2016 14

REVISEDJONES REVISEDJONES
Major Minor Major Minor
CRITERIA Arthritis(usually Arthralgia
CRITERIA Arthritis(usually Arthralgia
migratorypolyarthritis) polyarthritis)
Forpatientswithevidenceof Carditis(clinicalor Fever Forpatientswithevidenceof Carditis(clinicalor Fever
priorGASinfection*, subclinical) priorGASinfection*, subclinical)
AcuteRheumaticfever= Chorea ElevatedESRorCRP AcuteRheumaticfever= Chorea ElevatedESRorCRP
2MAJOR Erythemamarginatum ProlongedPRinterval 2MAJOR Erythemamarginatum ProlongedPRinterval
OR (unlesscarditisisa OR (unlesscarditisisa
1MAJORplus2MINOR majorcriterion) 1MAJORplus2MINOR majorcriterion)
Subcutaneousnodules Subcutaneousnodules

*e.g.rapidstreptest;culture;antiͲstreptolysinͲOtiter(ASO) *e.g.rapidstreptest;culture;antiͲstreptolysinͲOtiter(ASO)
RevisionoftheJonesCriteriaforthediagnosisofacuterheumaticfeverintheeraofDoppler
orantiͲDNaseB(ADB) RevisionoftheJonesCriteriaforthediagnosisofacuterheumaticfeverintheeraofDoppler
orantiͲDNaseB(ADB)
echocardiography:ascientificstatementfromtheAmericanHeartAssociation.Circulation.2015 echocardiography:ascientificstatementfromtheAmericanHeartAssociation.Circulation.2015

RecognizingAcuteRheumaticFever TreatmentandprophylaxisofAcuteRheumaticFever

• Timing:average19dafterGAS Secondary Goal:toprevent


infection Primaryepisode
prophylaxis rheumaticheart
• Arthritis:migratory,polyarthritis
involvinglargejoints(knees, IMbenzathine IMbenzathine disease
ankles,elbows,wrists)
penicillinx1 penicillinq4weeks
• Carditis:widerangeofeffects–
or
e.g.pericarditis,systolic
dysfunction,valvulardisease
Durationofppx
Oralpenicillinx10d
• Chorea:latemanifestation; variesbyseverityof
involuntarymovements
primaryillness
• Skin:Subcutaneousnodules;
erythemamarginatum(blanches;
transient)– rarebutspecific https://www.cdc.gov/groupastrep/diseasesͲ
public/rheumaticͲfever.html

KarthikeyanG,GuilhermeL.Acuterheumaticfever.Lancet.2018. ContemporaryDiagnosisandManagementofRheumaticHeartDisease:ImplicationsforClosingtheGap:AScientificStatementFromtheAmericanHeartAssociation.Circulation.2020
PrinciplesandPracticeofInfectiousDisease,9th ed. 17 PrinciplesandPracticeofInfectiousDiseases,9th ed. 18

©2024InfectiousDiseaseBoardReview,LLC
294
54– KitchenSink:SyndromesNotCoveredElsewhere
Speaker:StaceyRose,MD

• Cause:immunedysregulationfollowingS.pyogenesinfection
• Epidemiology:children/youngadults;rareinUS
• Clinicalpresentation:~3weeksfollowingGASinfection
• Major:migratorypolyarthritis,carditis,chorea,subcutaneous
nodules,erythemamarginatum
• Minor:fever,arthralgia,elevatedESR/CRP;PRprolongation
• DiagnosisbasedonJonescriteria=2majorOR1major+2minor
(pluse/opriorGASinfectione.g.ASOtiter)
• Treatmentandsecondaryppx withIMPenicillin;durationbasedon
carditis(10yr ortoage40ifcarditis+residualvalvulardisease)

Durationofsecondaryprophylaxisfollowingacute
rheumaticfever:
AcuteRheumaticFever
longestifcarditisandresidualvalvulardisease Takehomepoints

PrinciplesandPracticeofInfectiousDiseases,9th ed. 20

Question3 • A34yearͲoldmalewitha
historyofinjectiondruguse
Question3 • Whichofthefollowing
treatmentsare
presentstotheemergency recommended?
roomwithtwodaysofblurry
visionanddifficulty
swallowing.Heisalso A. Plasmapheresis
beginningtofeelweakinhis
armmuscles. B. Naloxone
• Onexamination,vitalsigns C. Tetanusantitoxin
arenormal,butthepatientis D. Botulinumantitoxin
notedtohaveptosisand
sluggishpupillaryresponses
aswellasslurredspeech.
21 22

Explanation
Plasmapheresis– forLambertͲEatonsyndrome,immune Botulism
attackofneuromuscularjunction(chronic;associated
Tetanus: withlungcancer) • Causedby*Clostridium
sardonicsmile botulinum(grampositive,strict
Naloxone– foropioidintoxication(respiratory
anaerobewithsubterminal
suppression,constricted pupils)
spore;foundinsoil)
• SymptomsduetoTOXINSwhich
preventreleaseofacetylcholine
inneuromuscularjunction
Tetanusantitoxin– fortetanus(rigidparalysis)
• Leadstoflaccidparalysisof
motorandautonomicnerves,
Botulism: beginningwiththecranialnerves
Botulinumantitoxin – forbotulism(flaccidparalysis) (descending weakness)
ptosis
https://phil.cdc.gov/details.aspx?pid=2107
• DX:cultureordetectionoftoxin
*otherneurotoxinproducingspeciesofClostridium:
https://www.thelancet.com/journals/lancet/article/PIIS0140Ͳ6736(19)31137Ͳ7/fulltex
https://www.nejm.org/doi/pdf/10.1056%2FNEJMicm1003352 23
C.butyricum,or C.baratii 24

©2024InfectiousDiseaseBoardReview,LLC
295
54– KitchenSink:SyndromesNotCoveredElsewhere
Speaker:StaceyRose,MD

Botulism Bioterrorism potential (aerosolization) REDFLAGS:symmetricCNpalsiesanddescending/symmetricflaccid


paralysisshouldraisesuspicionforbotulism

Infant Iatrogenic

Foodborne Wound(blackͲtar
heroin)

PeakCM,RosenH,Kamali A,etal.WoundBotulismOutbreakAmongPersonsWhoUseBlackTarHeroin— SanDiegoCounty,California,2017–2018.MMWRMorb MortalWkly Rep2019.


https://www.cdc.gov/botulism/;PrinciplesandPracticeofInfectiousDiseases,9th ed. 25 https://emergency.cdc.gov/han/2024/han00507.asp 26

Botulismtreatment • Cause:Clostridiumbotulinumtoxin impedesacetylcholine


releasefromneuromuscularjunction
• Epidemiology:food (homecannedveggies/fruits/fish);
Supportivecare Antitoxin infant (honey);wound (blackͲtarheroin);iatrogenic (rare)
• Clinicalpresentation:symmetric,descendingflaccidparalysis,
• Ventilatorysupport • BotulinumantiͲtoxin
startingwithcranialnerves (ptosis,blurredvision,slurred
forrespiratory (BAT)toprevent speech)
compromise progression • Diagnosis:clinical;confirmedbycultureordetectionoftoxin
• Wounddebridement • Treatment:antitoxin &supportivecare;wounddebridement

(forinfantbotulism
syndrome,useBotulinum
immuneglobulin(BabyBIG) Botulism
Takehomepoints
RaoAK,SobelJ,ChathamͲStephensK,Luquez C.ClinicalGuidelinesforDiagnosisandTreatmentofBotulism,2021.MMWRRecomm Rep.2021.
https://www.cdc.gov/botulism/;PrinciplesandPracticeofInfectiousDiseases,9th ed.
27 28

• A23ͲyearͲoldfemalepresentswith
anonͲproductivecoughfor2 • Whichofthefollowingwould
Question4 weeks.Shedescribesspellsduring
whichshecoughsrepeatedlyfor
Question4 yourecommendforthis
severalminutes.Ontwooccasions patient?
shevomitedaftercoughing.
• Shereportsepisodesofsweating
buthashadnofeverorother A.Azithromycin,withreturnto
constitutionalsymptoms. workafter5days
• Shehastriedseveralcough B.Azithromycin,withreturnto
medicines,butnothingseemsto
help. workafterfirstdose
• PCRrespiratorypanelwaspositive C.Notreatment,withreturn
forBordatella pertussis. toworkafter5days
• Sheworksasanurseinapediatric
intensivecareunit,andwouldlike https://www.youtube.com/watch?v=31tnXPlhA7w (NEJMvideo ) C.Notreatment;canreturnto
guidanceforwhenshecanreturnto workimmediately
work.
29 30

©2024InfectiousDiseaseBoardReview,LLC
296
54– KitchenSink:SyndromesNotCoveredElsewhere
Speaker:StaceyRose,MD

Pertussis Clinicalcasecriteria(inabsenceofalternatedx):
diagnosis– • coughillnesslastingш2weeks,withatleastone
ofthefollowing:
Catarrhal requires • Paroxysmsofcoughing; OR
• Inspiratorywhoop; OR
Paroxysmal
clinical • PostͲtussivevomiting; OR
suspicion • Apnea(withorwithoutcyanosis)
Convalescent

Polymerase chain reaction (PCR) is most sensitive and specific


• Nasopharyngeal swab / aspirate
• Best if sent within first 3 weeks of illness

Pertussis:clinicalstages https://wwwn.cdc.gov/nndss/conditions/pertussis/caseͲdefinition/2020/;https://www.cdc.gov/pertussis/clinical/diagnosticͲtesting/diagnosisͲpcrͲbestpractices.html
32
https://www.cdc.gov/pertussis/signsͲsymptoms/index.html ClinicalevaluationandvalidationoflaboratorymethodsforthediagnosisofBordetellapertussisinfection:Culture,polymerasechainreaction(PCR)andantiͲpertussistoxinIgGserology(IgGͲPT).PLoS One.2018

Treatmentandpostexposureprophylaxis Pertussis:recommendationsforhealthcare
workers(HCW)
• TREATwith • POSTEXPOSURE
macrolide (e.g. PROPHYLAXIS Symptomatic infection: exclude from
azithromycin)if (PEP)givento
work for 21 days from onset of cough
within3weeksof household
onset membersand OR until 5 days after the start of
contactsatrisk effective antimicrobial therapy
ofsevere
• Treatwithin6 infection(within
weeksofonsetfor 3weeksof Exposure: regardless of vaccination
infantsorpregnant exposure) status, administer post-exposure
women
prophylaxis if likely to interact with
persons at high risk of complications

https://www.cdc.gov/pertussis/ Recommendedantimicrobialagentsforthetreatmentandpostexposureprophylaxisofpertussis:2005CDCGuidelines.MMWRRecomm Rep.2005.


DeckerMD,EdwardsKM.Pertussis(WhoopingCough).JInfectDis.2021. 33 34
https://www.cdc.gov/infectioncontrol/guidelines/healthcareͲpersonnel/selectedͲinfections/pertussis.html

Pertussisinthenews

Pertussis
Vaccination https://www.cidrap.umn.edu/pertussis/ecdc-warns-surge-pertussis-cases-europe

https://www.cdc.gov/vaccines/vpd/dtap-tdap-td/hcp/recommendations.html

Guidance:useTdaP inlieuofTdwhereavailable
36

©2024InfectiousDiseaseBoardReview,LLC
297
54– KitchenSink:SyndromesNotCoveredElsewhere
Speaker:StaceyRose,MD

• Epidemiology:infants>adolescents
• Highriskforseveredisease:infants,pregnantwomen,lungdisease
• Clinicalpresentation:cough lasting2+weeksplusparoxysmalcough,
inspiratorywhoop,postͲtussivevomitingorapnea
Question5
• Diagnosis:clinical;PCR
• Treatwithmacrolide within3wks ofonset(6wks ifhighrisk) • A34yearͲoldmotorcyclistis
• PostͲexposureprophylaxis: (within3wks ofexposure)forhousehold involvedinaseveremotor
contacts/highrisk/HCW likelytointeractwithhighriskpatients vehicleaccident,resultingin
• HCWcanreturntoworkafter5dofeffectivetreatmentor21d lacerationofthespleenand
aftercoughonset
requiringsplenectomy.

Bordetellapertussis
Takehomepoints

37 38

• PostͲsplenectomy,thepatient Splenectomyandinfectionrisk
Question5 isatincreasedriskofsevere
diseaseduetowhichofthe Why:reducedclearanceofencapsulated
followingmicroorganisms? organisms;impairedhumoralimmunity

A. Helicobacterpylori Ontheboards,lookfor…
• Streptococcuspneumonia
B. Capnocytophaga canimorsus • HemophilusinfluenzatypeB
C. Candidaglabrata • Neisseriameningitidis
https://www.nationalgeographic.com/animals/mammals/facts/prairieͲdogs
• Capnocytophaga canimorsus (dogbite)
D. Clostridiumdifficile Skov Sørensen etal.(1988)InfectImmun 56:1890Ͳ1896
• Babesiamicroti(tickborne)
• Bordetellaholmesii
• Salmonellatyphi

39 RubinLG,SchaffnerW.Clinicalpractice.Careoftheasplenic patient.NEnglJMed.2014 40

• Increasedriskforinfectionwithencapsulatedorganisms(and
others)…
• S.pneumoniae;N.meningitidis;HIB;Capnocytophaga;Babesia;
Salmonellatyphi
Strategiesto • Reduceriskofinfectionvia:
Vaccinationfor Penicillin
reduceinfection encapsulated prophylaxis • Immunizations
riskinasplenia organisms • PCNppx if<5yrs old;recentsplenectomy;h/osepsis

• Pneumococcus • Children<5years
• Meningococcus • Olderchildren/adults
• Hemophilus within1Ͳ2yearsof
influenzatypeB splenectomy
• Anyage:secondary
prevention(lifelong)
Infectioninasplenia
followingsepsis
Takehomepoints
RubinLG,SchaffnerW.Clinicalpractice.Careoftheasplenic patient.NEnglJMed.2014 41 42

©2024InfectiousDiseaseBoardReview,LLC
298
54– KitchenSink:SyndromesNotCoveredElsewhere
Speaker:StaceyRose,MD

• A19yearͲoldmalewithnopast Whichofthefollowingisa
medicalhistorypresentswith recognizeddisadvantage ofthis
Question6 acuteonsetofpainthatstarted
intheperiumbilicalregionand
Question6 approach,whencomparedto
movedtothelowerregion. immediatesurgery?
• Physicalexamisnotableforpoint Younotethatantibiotic
tendernessintherightlower
quadrant. therapyforuncomplicated
A. RiskofC.difficilewithin30days
• Appendicitisisdiagnosedbased appendicitishasbecome
onclinicalfindingsandimaging B. Riskofbowelobstructionin1year
acceptedpracticebysome
results,withnoevidenceof C. 20%riskofintraͲabdominal
periappendiceal abscess. physicians,andofferto abscesswithin30days
• Thepatientwantstoavoid counselhimregardingrisks D. 30Ͳ50%riskofsubsequent
surgeryifatallpossible. andbenefits. appendectomywithin4years

43 44

Risksandbenefits
Appendicitis: Inseveralstudies,nonͲoperative
management(antibioticsalone)was

tocutornot “nonͲinferior”tooperativemanagement
foracute,uncomplicatedappendicitis 30Ͳ50%ofpatientsinitiallymanagedwith
antibioticsrequiredappendectomywithin5years
tocut… FeaturesthatmaypromptOPERATIVE
Longtermfollowupsuggestsoverallequivalent
management: patientsatisfaction
• Appendicolith(+/Ͳ)
• Perforation
• Abscess FortheIDboards:
• Suspicionoftumor knowwhentorecommendsurgery
• Peritonitis
• Serioussystemicillness

QualityofLifeandPatientSatisfactionat7ͲYearFollowͲupofAntibioticTherapyvsAppendectomyfor
CODA:NEnglJMed.2020; APPAC:JAMA.2018;Pediatr SurgInt.2020 45 UncomplicatedAcuteAppendicitis:ASecondaryAnalysisofaRandomizedClinicalTrial.JAMASurg.2020 46

• NonͲoperativemanagementofacuteappendicitismaybe
consideredifuncomplicated Question7 • A44yearͲoldmalewitha
• Featureswhichshouldpromptimmediatesurgery:
perforation;abscess;suspectedtumor;peritonitis; historyofcirrhosisdueto
systemicillness HepatitisBandalcoholism
• Upto50%willrequiresubsequentappendectomy presentswithfever,lethargy
• IDboardpotential– recognizewhenanoperationisNEEDED andlegswelling.Onexam,he
isfebrile,hypotensiveand
tachycardic.Skinexamisas
pictured.

Appendicitis
Takehomepoints
LancetInfectDis.2008Jun;8(6):399.
47 48

©2024InfectiousDiseaseBoardReview,LLC
299
54– KitchenSink:SyndromesNotCoveredElsewhere
Speaker:StaceyRose,MD

Explanation
Question7 • Thepatient’sclinical
Petechialrashfrom
Streptobacillus
moniliformis(ratbite
fever);fever,rash,
syndromewasmostlikely Hemorrhagic migratoryarthritis
causedbywhichofthe bullaefrom
followingexposures? Vibrio
CMAJ.2006Aug15;175(4):354.
vulnificus

A. Ratbite AmJTropMedHyg.2017;97(1):1Ͳ2.

B. Tickbite
Erythema
C. Consumptionofrawoysters Rosespots migrans dueto
from Borrelia
D. Consumptionofrawegg Salmonella burgdorferi(tick
typhi borne)
https://www.cdc.gov/lyme/signs_symptoms/rashes.html
LancetInfectDis.2008Jun;8(6):399.
49 https://www.cdc.gov/vaccines/vpd/typhoid/public/photos.html 50

Vibriovulnificus
• GramͲnegative,curvedbacillus • Abruptonset
• Halophilic(saltloving)– brackish Clinical • Fever,hypotension
water
• Cause:consumptionofrawseafood
presentation • Rapidlyprogressiveskinlesions:
erythemaÆ hemorrhagicbullae Æ
(oysters)orcontaminationofopen and necrosis
wound
• Atrisk:liverdisease(cirrhosis);iron
treatment • Bacteremiacommon
• Treatment:
overload;renaldisease; • 3rd generationcephaloporin
immunosuppression plus doxycyclineORfluoroquinolone
• Highmortality • Debridement(fornecrotizing
fasciitis)
Skin Manifestations of Primary Vibrio
vulnificus Septicemia. Am J Trop Med Hyg. 2017.

51 PrinciplesandPracticeofInfectiousDiseases,9th ed. 52

• Epidemiology:consumptionofrawoysters;contaminationof • A38yearͲoldfemaletravelsto
wound(organismlivesinwarm,brackishwater) Bangladeshforafriend’s(outdoor)
• Atrisk:liverdisease,ironoverloadstates(alsochronickidney Question8 wedding.
disease;diabetesorotherimmunesuppression) • Shehasnevertraveledtothis
region.Inpreparationforthetrip,
• Clinicalpresentation:rapidlyprogressiveskinlesionswith shereceivedTyphoidvaccineand
hemorrhagicbullae;fever,hypotension,sepsis wasstartedonmalariaprophylaxis
• Diagnosis:clinical;bloodculturesusuallypositive withdoxycycline.
• Treatment:3rd generationcephalosporinplusdoxycyclineor • Fivedaysafterreturninghome,she
developsfever,headache and
fluoroquinolone;debridement diffusemuscleandjointpain.
• Overthenextfewdays,arash
develops– beginningonthe
VibrioVulnificus dorsumofherhandsandfeetwith
spreadtoherarms,legsandtorso.
Takehomepoints • Shepresentstourgentcarefor
evaluation.
53 54

©2024InfectiousDiseaseBoardReview,LLC
300
54– KitchenSink:SyndromesNotCoveredElsewhere
Speaker:StaceyRose,MD

• Physicalexamisnotablefor Whichofthefollowingtestsis
Question8 fever(101.2degrees Question8 mostlikelytoyieldthe
diagnosis?
Fahrenheit)andadiffuse,
morbilliformrash.
• CBCisasfollows: A. Dengue realͲtimePCR
• WBC3.26x109 /L(normal) B. Bloodculture
• Hgb12.9g/dL(normal) C. Lymeenzymeimmunoassay
• Platelets113,000/mcL (low) (EIA)
• Acomprehensivemetabolic
https://www.nationalgeographic.com/animals/mammals/facts/prairieͲdogs

D. Malariarapiddiagnostictest
profileisnormalincluding (RDT)
renalandliverfunctiontests.

Indian J Dermatol. 2010;55(1):79-85. 55 56

Question8Ͳ explained Dengueiscommonworldwide…andrising

Dengue – characteristicsymptomsand
epidemiology;PCRorNS1antigentest
Fever, recommendedwithinfirst7days

headache,body Bloodculture – presumablylookingfor


pain,rashand Typhoidfever,butrashisnotcharacteristic
andnogastrointestinalsymptoms
lowplateletsin
Lyme – wrongepidemiology(noknown
areturning exposuretoticks)andrashnottypicalfor
Lyme;lowplateletsdoesnotfit
traveler
Malaria – RDTwouldbediagnostic,BUTno https://www.who.int/newsͲroom/factͲsheets/detail/dengueͲandͲsevereͲdengue
anemiaandrashnottypicalwithmalaria;
alsowastakingprophylaxis • Tropicalandsubtropicalclimates,outbreaksinurbanandsemiͲurbanareas
57
• 100Ͳ400millioninfectionseachyearworldwide 58

Dengue:diagnostictesting SevereDengue
•Symptomaticऺinfectionऺtypicallyऺ
• Earlyindisease improvesऺafterऺ1-2ऺweeks
course:nucleicacid •MayऺprogressऺtoऺsevereऺDengue;ऺ
testing(PCR)or riskऺincreasedऺifऺpriorऺinfectionऺ
NS1antigen (withऺanotherऺserovar)
• IgM ismore •Signsऺofऺsevereऺdengue:
sensitiveafter7 •Vomiting
days
•Tachypnea
• IgG not helpfulin •Mucosalऺbleedingऺ(gums;ऺ
acutephase epistaxis) https://www.who.int/newsͲroom/factͲsheets/detail/dengueͲandͲsevereͲdengue
https://www.cdc.gov/dengue/hcp/clinicalͲsigns/index.html
•Bloodऺinऺvomitऺorऺstool
Guzman,M.G.etal.Dengue:Acontinuingglobalthreat.NatureReviewsMicrobiology8,S7–S16(2010). •Hypotensionऺ/ऺshock
https://www.cdc.gov/dengue/hcp/diagnosisͲtesting/index.html
59 60

©2024InfectiousDiseaseBoardReview,LLC
301
54– KitchenSink:SyndromesNotCoveredElsewhere
Speaker:StaceyRose,MD

KeyfeaturesofmosquitoͲborneillnesses
MosquitoͲborne
illnessesina Epidemiology
Africa,theAmericas,Asia,Europe,
Vector Clinicalfeatures
Aedes aegypti
returningtraveler islandsinIndianandPacific
Chikungunya Oceans;prominentoutbreak (A.albopticusin
Europe)
Feverandjointpain;rashless
common.Canhavechronicsx’s
Caribbean2013
Americas,Africa,Caribbean,Middle
Fever,headache,rash,muscleand
East,Asia,PacificIslands
Fortheboards,know: Dengue
Aedes aegypti(or
A.albopticus)
jointpain

Ͳtypicalepidemiology 4serotypes;infectionwitha2nd
serotypeÆ severeillness
Severe:hemorrhagicfever/shock

Ͳclinicalpresentation
Aedesaegyptimosquito,imagefrom
https://www.cdc.gov/mosquitoes/gallery/aedes/index.html
Aedes aegypti Oftenasx;fever;rash (startson
ProminentinAmericas~2017,then
Ͳvector
face);conjunctivitis
Zika morewidespread(Caribbean,
Alsosexual IfinfectedduringpregnancyÆ
Africa,India)
transmission fetalanomalies(microcephaly)

CDC;PPID9th edition
61 62

DiagnosticapproachformosquitoͲborneillnesses Comparingsx’s ofDengue,Zika,Chikungunya


>7dafter
Within7dofsymptomonset
symptomonset
Chikungunya Nucleicacidtesting(RTͲPCR) IgM

Nucleicacidtesting(RTͲPCR)or
Dengue NS1(nonstructuralprotein1)immunoassay
IgM

Zika Nucleicacidtesting(RTͲPCR) IgM

• TestingavailablethroughhealthdepartmentorCDC
• IgMforZika andDengue crossͲreact;ifPCRnegative,positiveIgMshouldpromptPRNTtodifferentiate
• IgGnothelpfulasremainspositivelifelong
https://www.cdc.gov/dengue/healthcareͲproviders/diagnosis.html
https://www.cdc.gov/chikungunya/hc/diagnostic.html
Dengueinfection. NatRevDisPrimers (2016).https://doi.org/10.1038/nrdp.2016.55 PRNT=plaquereductionneutralizationtest 63 Kharwadkar S,HerathN.Clinicalmanifestationsofdengue,ZikaandchikungunyainthePacificIslands:AsystematicreviewandmetaͲanalysis.RevMedVirol.2024Mar.

https://cdc.gov/malaria/about/distribution.html

Malaria Malaria
• Epidemiology:worldwide,tropicsand • Epidemiology:worldwide,tropicsand
subtropics subtropics

• Vector:Anopheles mosquito • Vector:Anopheles mosquito

• Symptoms:Fever,headache,N/V,diarrhea; • Symptoms:Fever,headache,N/V,diarrhea;
severe:anemia,jaundice,splenomegaly, severe:anemia,jaundice,splenomegaly,
neurologic neurologic

• SpeciesͲspecificfeatures • SpeciesͲspecificfeatures

• Microscopy(bloodsmear);RDTif
microscopynotavailable

https://www.cdc.gov/malaria/diagnosis_treatment/diagnostic_tools.html 65 https://www.cdc.gov/malaria/diagnosis_treatment/diagnostic_tools.html 66

©2024InfectiousDiseaseBoardReview,LLC
302
54– KitchenSink:SyndromesNotCoveredElsewhere
Speaker:StaceyRose,MD

Mosquitoborneillnesseshaveoverlappingfeatures;lookforkeywords
• Dengue,Zika,ChikungunyaallspreadviaAedesmosquitos
• Dengue:headache,rash,“boneͲbreak”pain,lowplatelets;infxn
w/2nd serotypeÆ severedengue
• Zika:maybeasx;rash/conjunctivitiscommon;birthdefects
• Chikungunya:prominentjointpain;maybecomechronic
• Diagnosis:
• PCRif<7d(plusNS1antigenforDengue)
• IgMif>7dbutDengue/Zika crossͲreact
• Malaria:Anophelesmosquito;fever,anemia,speciesͲspecific
presentations(P.falciparumͲ severe;P.vivax/ovale Ͳ relapsing)
• Diagnosis:bloodsmearorrapiddetectiontest(RDT)

MosquitoͲborneillnessina
returningtraveler
Takehomepoints
https://emergency.cdc.gov/han/2023/han00496.asp#print Takehomepoints 68

KitchenSinksummaryͲ 1
KitchenSink
summary Whipple’s: AcuteRheumaticfever:
• Classic:arthralgia, • Kids/youngadults
diarrhea,weightloss withmigratory
polyarthritis,carditis,
• Dxwithduodenalbx chorea,subcutaneous
(PAS+,foamy nodules,erythema
macrophages) marginatumfollowing
GASpharyngitis
• orPCRoftissue(heart
valveforendocarditis) • MonthlyIMpenicillin
prophylaxisfor10
yearsortoage40if
carditis+residual
valvulardisease

https://www.cdc.gov/groupastrep/diseasesͲpublic/rheumaticͲfever.html
69 70

KitchenSinksummaryͲ 2 KitchenSinksummaryͲ 3

Botulism: Pertussis: Appendicitis Asplenia


• Nonoperative • Increasedriskofinfectionwith
• DuetoC.botulinum • Clinicaldiagnosis:2+ encapsulatedorganisms
toxin managementmaybe
weeksofcoughplus reasonablefor • Ifpromptsaysasplenia,think…
• Food;infant;wound paroxysms,inspiratory uncomplicatedcases o S.pneumoniae
(blackͲtarheroin); whoop,postͲtussive o N.meningitidis
• Identifyfeaturesthat o H.InfluenzaetypeB
iatrogenic emesis,apnea shouldpromptsurgery:
o Capnocytophaga
• Descendingflaccid • Macrolideifwithin3 • Appendicolith+/Ͳ o Babesia
paralysis(startswith • perforation o Salmonellatyphi
cranialnerves) weeksofonsetoras
• Abscess • Preventinfectionwith
PEPforcontactsatrisk • Suspicionoftumor
• Antitoxin/supportive ofseveredisease
immunizationsand
care • Peritonitis • PCNprophylaxis(if<5yrs old;
• Systemicillness recentsplenectomy;prior
episodeofsepsis)

71 72

©2024InfectiousDiseaseBoardReview,LLC
303
54– KitchenSink:SyndromesNotCoveredElsewhere
Speaker:StaceyRose,MD

KitchenSinksummaryͲ 4

MosquitoͲborneillnesses
Vibriovulnificus:
Chikungunya,Dengue,Zikaallspreadvia
Aedes mosquitosandcanpresentwithfever
• Liverdiseaseatrisk plus…

• Exposuretorawseafood
• Chikungunya – jointpain
• Dengue – headache,rash,muscleand
Questions?
orcontaminatedwound
jointpain;higherriskofsevereDengue
(brackishwater) with2ndinfection

• Rapidlyprogressive, • Zika – rash,conjunctivitis;fetalanomalies; StaceyRose,MD,FACP,FIDSA


sexualtransmission
hemorrhagicbullae/ [email protected]
sepsis • PCRif<7d(plusNS1antigenforDengue)
• IgMif>7dbutDengue/ZikacrossͲreact
• Fluoroquinolone, Malaria:Anopheles mosquito;fever,
ceftriaxone,debridement anemia;speciesͲspecificpresentations;DX:
smearorRDT
73

©2024InfectiousDiseaseBoardReview,LLC
304

You might also like